You are on page 1of 515

मू ल अिधकार एवं रा के नीित िनदे शक त : टे 5

मू ल अिधकार : टे 4

मू ल अिधकार : टे 3

सं िवधान का िनमाण, ावना, सं घ एवं उसका रा े , नाग रकता तथा मू ल अिधकार : टे 2

सं वैधािनक िवकास : टे 1

POLITY PT QUIZ
[1] रे ू लेिटं ग ए -1773 के सं दभ म िन िल खत म से कौन-सा कथन सही
नही ं है ?

इस अिधिनयम के अं तगत कलक ा म 1773 म एक सव ायालय की


A)
थापना की गई थी।

इस अिधिनयम के ारा बं गाल के गवनर को बं गाल के गवनर जनरल पद नाम


B)
िदया गया।

C) लॉड वॉरे न हे ं बं गाल के थम गवनर जनरल बने ।

इस अिधिनयम के ारा भारत म ई इं िडया कंपनी पर िनयं ण रखने का


D)
थम यास िकया गया था।

Hide Answer -
उ रः (a)

ा ाः कथन (a) स नहीं है , ोंिक रे ू लेिटं ग ए -1773 के ारा भारत म एक सव

ायालय थािपत करने सं बंधी ावधान िकया गया था, परं तु इसकी थापना वष 1774 म

कलक ा म की गई थी न िक 1773 म। इस सव ायालय म एक मु ायाधीश एवं तीन

अ ायाधीश थे । सर एिलजा इ े को मु ायाधीश बनाया गया था।

इस अिधिनयम के ारा बं गाल के गवनर को बं गाल का गवनर जनरल पद का नाम िदया

गया। म ास एवं बं बई के गवनरों को बं गाल के गवनर जनरल के अं तगत कर िदया गया।

गवनर जनरल की सहायता के िलये चार सद ों वाली एक कायकारी प रषद का गठन

िकया गया। बं गाल का पहला गवनर जनरल वॉरे न हे ं को बनाया गया।

इस अिधिनयम के ारा ई इं िडया कंपनी पर िनयं ण रखने के िलये थम बार यास

िकया गया था। इस अिधिनयम म पहली बार भारत म कंपनी के राज के िलये िल खत

सं िवधान ु त िकया गया तथा कंपनी के राजनीितक एवं शासिनक उ रदािय ों को

ीकार िकया गया।

रे ू लेिटं ग ए 1773 की किमयों को दू र करने के िलये ि िटश सं सद ारा 1781 म

एक सं शोधन अिधिनयम पा रत िकया गया, िजसे ‘ए ऑफ सै टलमट’ के नाम से जाना

जाता है ।

[2] िपट् स इं िडया ए , 1784 के सं दभ म िन िल खत कथनों पर िवचार


कीिजये :
1. इस अिधिनयम ारा ै ध शासन व था का ारं भ आ।
2. इस अिधिनयम के ारा िनयं ण मं डल (Board of Control) नाम से एक
नए िनकाय का गठन िकया गया। िजसे ापार सं बंधी बं धन का काय सौंपा गया।
3. राजनीित सं बंधी बं धन का काय सं चालक-मं डल (Court of Directors)
के हाथों म सौंपा गया।
4. कायकारी प रषद के सद ों की सं ा चार से घटाकर तीन कर दी गई।
उपयु कथनों म से कौन-सा/से सही नही ं है /ह?
A) केवल 1

B) केवल 2 और 3

C) केवल 1 और 4

D) केवल 1, 2, 3 और 4

Hide Answer -
उ रः (b)

ा ाः कथन 2 एवं 3 सही नहीं ह, ोंिक इस अिधिनयम के ारा ि िटश सं सद ारा कंपनी

के राजनीितक एवं ापा रक काय को पृ थक् िकया गया। ापार सं बंधी काय का बं धन कंपनी

के ितिनिध सं चालक मं डल (Court of Directors) के हाथों म सौंपा गया, जबिक राजनीितक

काय के बं धन के िलये ि िटश सं सद ारा ‘िनयं ण-मं डल’ (Board of Control) नामक

एक नए िनकाय का गठन िकया गया।

कंपनी के काय को दो भागों म िवभािजत िकया गया: ापा रक एवं राजनीितक सं बंिधत

काय, िज मशः कोट ऑफ डायरे स एवं बोड ऑफ कंटोल के हाथों म सौंपा गया।

इस कार इस अिधिनयम ारा ै ध शासन व था का ारं भ आ।

बं गाल के गवनर जनरल की सहायता के िलये रे ू लेिटं ग ए , 1773 म चार सद ीय

एक कायकारी प रषद की थापना की गई थी। इस अिधिनयम ारा इसकी सद सं ा

चार से घटाकर तीन कर दी गई ।

इस ए से सं बंिधत िवधे यक ि िटश सं सद म त ालीन ि िटश धानमं ी िपट द यं गर ने

ु त िकया था, इसिलये इस िवधे यक को ‘िपट् स इं िडया ए ’ के नाम से जाना जाता है ।

[3] िन िल खत अिधिनयमों पर िवचार कीिजयेः

1. 1793 का अिधिनयम
2. 1813 का अिधिनयम
3. 1833 का अिधिनयम
4. 1853 का अिधिनयम
उपयु म से कौन-से अिधिनयम चाटर अिधिनयम के नाम से जाने जाते ह?

A) केवल 1 और 2

B) केवल 1, 2 और 3

C) केवल 2, 3 और 4

D) 1, 2, 3 और 4
Hide Answer -
उ रः (d)

ा ाः उपयु सभी अिधिनयम चाटर अिधिनयम के नाम से जाने जाते ह। 1786 एवं 1857

के बीच ि िटश सं सद ने कंपनी के मामले म चार मु ख चाटर अिधिनयम पा रत िकये । उपरो

चाटर अिधिनयम 20-20 वष के अं तराल पर पा रत िकये गए। इ चाटर अिधिनयम इसिलये

कहा जाता है , ोंिक इन अिधिनयमों के ारा कंपनी की आ ा-प अविध बढ़ाने के साथ-साथ

कंपनी के अिधकार, कंपनी एवं सरकार के बीच सं बंध, कंपनी के भारतीय शासन म समय-

समय पर मह पू ण बदलाव िकये गए।

[4] िन िल खत म से कौन-सा कथन अस है ?

1813 के चाटर ए के तहत ईसाई िमशन रयों को भारत म काय करने की


A)
अनु मित दान कर दी गई।

1813 के चाटर ए के ारा भारत म कंपनी के ापा रक एकािधकार को पू णतः


B)
समा कर िदया गया।

1833 के चाटर ए के ारा गवनर जनरल की प रषद म चौथे सद के


C)
प म िविध सद िनयु िकया गया।

1833 के चाटर ए के ारा बं गाल के गवनर जनरल को भारत का गवनर


D)
जनरल बना िदया गया।

Hide Answer -
उ रः (b)

ा ाः कथन (b) गलत है , ोंिक 1813 के चाटर ए ारा भारत म कंपनी के ापा रक

एकािधकार को समा कर िदया गया, परं तु चाय एवं चीन पर कंपनी का ापा रक एकािधकार

बना रहा। 1813 के चाटर ए ारा भारत म ईसाई धम के चार की अनु मित दान कर दी

गई।

1833 के चाटर ए के तहत गवनर जनरल की प रषद म िविध सद के प म चौथे

सद की िनयु की गई। लॉड मै काले को िविध सद के प म िनयु िकया गया।

लॉड मै काले की अ ता म भारत म पहला ‘िविध आयोग’ गिठत आ।

1833 के चाटर ए के तहत बं गाल के गवनर जनरल को भारत का गवनर जनरल बना

िदया गया। गवनर जनरल की सरकार भारत की सरकार एवं उसकी प रषद भारतीय

प रषद कहलाई। लॉड िविलयम बिटक भारत के थम गवनर जनरल बने ।

1833 के चाटर ए ारा भारत म दास था को गै र-कानू नी घोिषत कर िदया गया।

1833 का चाटर ए म यह ावधान िकया गया िक कंपनी के दे शों म रहने वाले िकसी

भारतीय को धम, वं श, रं ग या ज आिद के आधार पर कंपनी म िकसी पद के िलये

अयो नहीं ठहराया जा सकता, परं तु कोट ऑफ डायरे स के िवरोध के कारण इस

ावधान को समा कर िदया गया। आगे चलकर यह ावधान शासन म भागीदारी का

मु आधार बना।
[5] चाटर ए , 1853 के िवषय म िन िल खत म से कौन-सा कथन सही नही ं
है ?

A) चाटर अिधिनयमों की ृं खला म यह अं ितम अिधिनयम था।

इस अिधिनयम के ारा िसिवल से वकों की भत एवं चयन हे तु खु ली


B)
ितयोिगता का ारं भ िकया गया।

इस अिधिनयम के ारा गवनर जनरल के िलये एक िवधान प रषद का गठन


C)
िकया गया।

इस अिधिनयम ने भारतीय िवधान प रषद को बजट पर बहस करने की श


D)
दान की।

Hide Answer -
उ रः (d)

ा ाः कथन (d) अस है , ोंिक िवधान प रषद को बजट पर बहस करने की श

भारतीय प रषद अिधिनयम, 1892 के ारा ा ई।

1793 से 1853 के दौरान पा रत िकये गए चाटर अिधिनयमों की ृं खला म यह अं ितम

अिधिनयम था।

इस अिधिनयम ारा िसिवल से वकों की भत एवं चयन हे तु खु ली ितयोिगता की व था

ारं भ की गई। 1854 म भारतीय िसिवल से वा के सं बंध म ‘मै काले सिमित’ का गठन

िकया गया।

गवनर जनरल की प रषद म कानू न-िनमाण म सहायता दे ने के िलये छः नए सद ों की

िनयु की गई। इस कार गवनर जनरल के िलये िवधान प रषद अ म आया,

िजसे भारतीय (क ीय) िवधान प रषद कहा गया। इस िवधान प रषद ने ‘लघु सं सद’ की

तरह काय िकया। इस कार पहली बार गवनर जनरल की प रषद के िवधायी एवं

शासिनक काय को अलग-अलग िकया गया।

इस अिधिनयम के तहत पहली बार भारतीय िवधान प रषद म थानीय ितिनिध ारं भ

िकया गया।

इस अिधिनयम ारा बं गाल के िलये एक पृ थक् ले नट-गवनर की िनयु की गई एवं

गवनर जनरल को बं गाल के शासन भार से मु कर िदया गया।

[6] भारत शासन अिधिनयम, 1858 के सं दभ म िन िल खत कथनों पर िवचार

कीिजयेः
1. इस अिधिनयम के ारा भारत के रा सिचव नामक एक नए पद का सृ जन
िकया गया।
2. कंपनी म कोट ऑफ डायरे स एवं बोड ऑफ कंटोल की व था बनी रही।
3. गवनर जनरल पद का नाम बदलकर ‘वायसराय’ कर िदया गया।
4. लॉड िविलयम बिटक भारत के थम वायसराय बने ।
उपरो कथनों म से कौन-से सही ह?
A) केवल 1 और 2

B) केवल 1 और 3

C) केवल 1, 3 और 4

D) 1, 2, 3 और 4

Hide Answer -
उ रः (b)

ा ाः कथन 2 एवं 4 गलत ह, ोंिक भारत शासन अिधिनयम, 1858 के ारा कंपनी के

कोट ऑफ डायरे स एवं बोड ऑफ कंटोल को समा कर िदया गया एवं भारत म ै ध शासन

णाली का अं त हो गया। भारत का थम वायसराय लॉड िविलयम बिटक को नहीं ब लॉड

कैिनं ग को बनाया गया।

इस अिधिनयम के ारा भारत के रा सिचव नामक एक नए पद का सृ जन िकया गया।

यह सिचव ि िटश मं ि मं डल का सद होता था एवं भारतीय मामलों म ि िटश सं सद के

ित उ रदायी होता था। भारतीय शासन पर िनयं ण की सं पूण श रा सिचव म

िनिहत थी।

भारत सिचव की सहायता के िलये 15 सद ीय एक प रषद (Council of India) का

गठन िकया गया। यह प रषद एक सलाहकार सिमित थी। प रषद का अ भारत सिचव

को बनाया गया था।

गवनर जनरल का पद ‘वायसराय’ कहलाने लगा, ोंिक इस अिधिनयम के तहत कंपनी

के शासन को समा कर िदया गया। भारत का शासन सीधे महारानी िव ो रया के अधीन

चला गया । गवनर जनरल अब स ाट के ितिनिध की है िसयत से काय करने लगा था।

सं चालक मं डल, िनयं ण मं डल एवं गवनर जनरल की सभी श याँ प रषद सिहत भारत

सिचव म कि त कर दी गई, जो सं सद के ित उ रदायी था। भारत सिचव की प रषद को

िनगिमत िनकाय (Corporate Body) घोिषत िकया गया, िजस पर इं ड एवं भारत म

दावा िकया जा सकता था।

नई व था का ारं भ महारानी िव ो रया की घोषणा के साथ आ, िजसकी घोषणा

इलाहाबाद म आयोिजत एक िवशे ष दरबार म 1 नवं बर, 1858 म की गई थी।अतः कथन

1 और 3 सही है ।

[7] भारत प रषद अिधिनयम, 1861 के िवषय म िन िल खत म से कौन-सा कथन


अस है ?

इस अिधिनयम के ारा पहली बार भारतीय ितिनिधयों को कानू न बनाने की


A)
ि या म शािमल िकया गया।

B) इस अिधिनयम ने िवक ीकरण ि या का ारं भ िकया।

इस अिधिनयम ने वायसराय को अ ादे श िनकालने की श दान की,


C)
िजसकी अविध एक वष थी।

इस अिधिनयम ने कैिनं ग के ारा ारं भ की गई पोटफोिलयो णाली को


D)
मा ता दान की।
Hide Answer -
उ रः (c)

ा ाः कथन (c) अस है , ोंिक यह अिधिनयम वायसराय को आपातकाल म प रषद की

सं ु ित के िबना अ ादे श जारी करने की श दान करता है , परं तु इसकी अविध एक वष

नहीं, ब छः माह होती थी।

इस अिधिनयम के ारा पहली बार भारतीय ितिनिधयों को कानू न बनाने की ि या म

शािमल िकया गया। सन् 1862 म लॉड कैिनं ग ारा तीन भारतीयों: बनारस के राजा,

पिटयाला के महाराज एवं सर िदनकर राव को गै र-सरकारी सद के प म िवधान

प रषद म मनोनीत िकया गया।

इस अिधिनयम ारा रे ू लेिटं ग ए , 1773 म ारं भ की गई क ीकरण की वृ ि को

बदल िदया गया एवं िवके ीकरण की ि या ारं भ की गई। म ास एवं बं बई े सीडिसयों

को िवधायी श याँ पु नः दान की गईं ।

इस अिधिनयम के आधार पर बं गाल (1862), उ र-पि म सीमा ां त (1866) एवं

पं जाब (1897) म िवधान प रषदों का गठन आ।

इस अिधिनयम ने लॉड कैिनं ग ारा 1859 म ारं भ की गई पोटफोिलयो णाली को

मा ता दान की गई।

[8] भारतीय प रषद अिधिनयम, 1909 के सं दभ म िन िल खत म से कौन-सा/से


कथन स है /ह?
1. इस अिधिनयम के ारा पहली बार िवधान प रषद को बजट पर बहस करने की
श ा ई।
2. इस अिधिनयम के ारा सां दाियक िनवाचन णाली की शु आत ई।
3. यह अिधिनयम मोंटे ू -चे फोड सु धार के नाम से जाना जाता है ।
नीचे िदये गए कूट का योग कर सही उ र का चयन कीिजये :

A) केवल 1 और 2

B) केवल 2

C) केवल 1 और 3

D) 1, 2 और 3
Hide Answer -
उ रः (b)
ा ाः कथन 1 और 3 अस ह, ोंिक िवधान प रषद को बजट पर बहस करने का
अिधकार पहली बार भारत प रषद अिधिनयम, 1892 ारा दान िकया गया था, परं तु
बजट पर मतदान करने एवं पू रक पू छने की श दान नहीं की गई थी। बजट पर
मतदान करने एवं पू रक पू छने तथा सावजिनक िहत के िवषयों पर ाव पे श करने
की श िवधान प रषद को भारतीय प रषद अिधिनयम, 1909 के ारा ा ई।

इस अिधिनयम के ारा सां दाियक िनवाचन णाली अथवा पृ थक िनवाचन की


व था की गई अथात् मु म सद ों का चु नाव केवल मु म मतदाता ही कर
सकते थे ।
इस अिधिनयम को मॉल-िमं टो सु धार के नाम से जाना जाता है , न िक मोंटे ू -
चे फोड सु धार। माल भारत-सिचव एवं लॉड िमं टो भारत के वायसराय थे ।
लॉड िमं टो को सां दाियक िनवाचन के जनक के प म जाना जाता है ।
स िस ा वायसराय की कायपािलका प रषद के थम भारतीय सद बने । इ
िविध सद बनाया गया था।

[9] भारत शासन अिधिनयम, 1919 के सं दभ म िन िल खत म से कौन-सा कथन


अस है ?

इस अिधिनयम ने भारत म आं िशक प से उ रदायी शासन व था की थापना


A)
की।

B) इस अिधिनयम के ारा क म ि सदनीय व था ारं भ की गई।

C) इस अिधिनयम ारा क म ै ध शासन की थापना की गई।

D) इस अिधिनयम ारा पृ थक् िनवाचन णाली का िव ार िकया गया।


Hide Answer -
उ रः (c)
ा ाः कथन (c) गलत है , ोंिक भारत शासन अिधिनयम, 1919 के ारा क म
नहीं, ब ां तों म ै ध शासन णाली की थापना ई। ां तीय िवषयों को दो भागों म
िवभािजत िकया गया था: ह ां त रत एवं आरि त िवषय।

ह ां त रत िवषय पर गवनर का शासन होता था और इन काय म वह उन मं ि यों


की सहायता ले ता था, जो िवधान प रषद के ित उ रदायी थे । आरि त िवषयों पर
गवनर, कायपािलका प रषद की सहायता से काय करता था और कायपािलका
प रषद िवधान प रषद के ित उ रदायी नहीं थी। िश ा, ा आिद ह ां त रत
िवषय थे , जबिक पु िलस, जे ल, ाय, िव , राज आिद आरि त िवषय थे ।
भारत शासन अिधिनयम, 1919 ने भारत म आं िशक प से (Partially)
उ रदायी शासन की थापना की। इस अिधिनयम के ारा ि िटश सं सद ने पहली
बार भारत म उ रदायी शासन की िदशा म कदम बढ़ाया। इस अिधिनयम म
ां तीय र पर सं सदीय शासन व था की झलक िमलती है ।
इस अिधिनयम ारा क म पहली बार ि सदनीय व था ारं भ की गई-

(i) रा प रषद (Council of State)- ऊपरी सदन (5 वष के िलये )


(ii) िवधान सभा (Legislative Assembly)- िनचला सदन (3 वष के िलये )

इस अिधिनयम ारा पृ थक् िनवाचन णाली का िव ार मु मों के अलावा िसखों,


भारतीय ईसाइयों, आं -भारतीयों एवं यू रोिपयों तक िकया गया।
इस अिधिनयम ारा क एवं ां तीय िवषयों की सू ची पहचान कर पृ थक् िकया गया
तथा क ीय एवं ां तीय िवधान प रषदों को अपनी सू िचयों के िवषयों पर िवधान बनाने
की श दान की गई। यातायात, डाकतार, सु र ा एवं वै देिशक मामले क ीय
िवषय के अं तगत तथा कृिष, िश ा, ा , थानीय शासन ां तीय िवषयों के
अं तगत शािमल िकये गए।
सं पि , कर या िश ा के आधार पर सीिमत सं ा म लोगों को मतािधकार दान
िकया गया।
इस अिधिनयम के ारा एक लोकसे वा आयोग का गठन िकया गया तथा ली आयोग
की िसफा रश पर सन् 1926 म िसिवल से वकों की भत के िलये क ीय लोक से वा
आयोग का गठन िकया गया।
इस अिधिनयम ारा क ीय बजट को रा ों के बजट से अलग िकया गया।
इस अिधिनयम ारा एक वै धािनक आयोग का गठन िकया गया।
इस अिधिनयम ारा लं दन म भारत के उ ायु कायालय का सृ जन िकया गया।
भारत शासन अिधिनयम, 1919 मोंटे ू -चे फोड सु धार पर आधा रत, िजसकी
घोषणा 20 अग , 1917 म की गई तथा यह अिधिनयम सन् 1921 म लागू आ
एवं 1937 तक भारत म लागू रहा।

[10]िन िल खत पर िवचार कीिजयेः

1. ां तीय ाय ता
2. ां तों म ै ध शासन
3. सं घीय व था
4. सं घीय ायालय की थापना
5. भारतीय रज़व बक की थापना
उपरो म से कौन-से भारत शासन अिधिनयम, 1935 के सं दभ म सही ह?

A) केवल 1, 2 और 3

B) केवल 1, 2, 3 और 4

C) केवल 1, 3, 4 और 5

D) उपरो सभी

Hide Answer -
उ रः (c)
ा ाः कथन 2 अस है , ोंिक भारत शासन अिधिनयम, 1935 के ारा अिधिनयम,
1919 म लागू की गई ां तों म ै ध शासन व था समा करके क म ै ध शासन की
थापना की गई। क ीय िवषयों को आरि त एवं ह ां त रत भागों म िवभािजत िकया गया।
ितर ा, वै देिशक मामले , धािमक मामले (ईसाई धम सं बंधी) तथा कबाइली े आरि त
िवषय थे ।

इस अिधिनयम के ारा ां तीय ाय ता लागू की गई, ोंिक ेक ां त म एक


कायपािलका एवं िवधानमं डल की थापना की गई थी। गवनर को उन मं ि यों की
सलाह पर काय करना पड़ता था, जो िवधान प रषद के ित उ रदायी होते थे ।
इस अिधिनयम के ारा एक अ खल भारतीय सं घ की थापना का ावधान िकया
गया था, जो ि िटश शासन के ां तों एवं उन दे शी रयासतों, जो े ा से शािमल
होना चाहते थे , से िमलकर बनने वाला था। तीन सू िचयों-सं घ सू ची, रा सू ची एवं
समवत सू ची का बँ टवारा क एवं रा ों के बीच होना था। अविश श याँ
वायसराय को दी गई थीं, परं तु यह अ खल भारतीय सं घ अ म आ न सका,
ोंिक दे शी रयासतों ने इसम शािमल होने से इनकार कर िदया था।
इस अिधिनयम ारा सं घ की राजधानी िद ी म एक सं घीय ायालय का ावधान
िकया गया था। इसम एक मु ायाधीश, तीन अ ायाधीश एवं दो अित र
ायाधीशों का ावधान था। स ाट ारा ायाधीश का चु नाव िकया जाना था, जो
65 वष की आयु तक अपने पद पर रह सकता था। सन् 1937 म एक सं घीय
ायालय की थापना की गई।
इस अिधिनयम के तहत एक भारतीय रज़व बक की थापना की गई।
इस अिधिनयम के ारा सं घीय लोक से वा के अित र ां तीय से वा आयोग एवं दो
या अिधक रा ों के िलये सं यु से वा आयोग की थापना की गई।
गवनर जनरल आपातकाल की घोषणा कर छः माह के िलये क ीय सरकार के सभी
काय को सीधे अपने उ रदािय म ले सकता था।

[11]िन िल खत म से कौन-सा कथन सही नही ं है ?


भारत शासन अिधिनयम, 1935 के ावधानों के अनु प बमा को भारत से अलग
A)
कर िदया गया।

वतमान म भारत म रा पित को अ ादे श िनगत करने की श भारत शासन


B)
अिधिनयम,1935 से े रत है ।

भारत शासन अिधिनयम, 1935 म भारत के िलये एक िल खत सं िवधान का


C)
ावधान िकया गया था।

भारत शासन अिधिनयम 1935 म पहली बार ां तीय र पर ि सदनीय


D)
िवधानमं डल की थापना की गई।

Hide Answer -
उ रः (c)
ा ाः भारत शासन अिधिनयम,1935 म भारत के िलये एक िल खत सं िवधान का
उ े ख नहीं िकया गया था। भारत म सं िवधान का सव थम उ ेख ि िमशन म िकया
गया था। अतः कथन (c) अस है ।

भारत शासन अिधिनयम, 1935 के ावधानों के अनु प सन् 1937 म बमा को


भारत से अलग कर िदया गया।
वतमान म भारत म रा पित को जो अ ादे श (अनु े द-123)िनकालने की श
है ,वह भारत शासन अिधिनयम,1935 से े रत है ।
भारत शासन अिधिनयम,1935 म ां तीय ाय ता का ावधान िकया गया
था,इसके तहत एक ां तीय कायपािलका एवं ां तीय िवधानमं डल का ावधान िकया
गया था। इस अिधिनयम ारा ां तीय र पर पहली बार ि सदनीय िवधान मं डल
की थापना की गई थी। छः ां तों म दो सदनों की व था थी:म ास, बं बई, बं गाल,
िबहार, सं यु ां त एवं असम।
भारत शासन अिधिनयम 1935 ‘ े त प ’ पर आधा रत है , िजसे िविभ गोलमे ज
स े लन के प रणाम प तै यार िकया गया था।

[12]सं िवधान सभा के िवषय म िन िल खत म से कौन-से कथन स ह?


1. इसकी रचना कैिबने ट िमशन के ाव पर आधा रत थी।
2. इसके सद ों का चु नाव सीिमत मतािधकार के आधार पर िकया गया था।
3. इसके सद ों का िनवाचन सावभौम वय मतािधकार के आधार पर िकया गया
था।
4. यह एक ब दलीय िनकाय थी।
कूटः

A) केवल 1 और 2

B) केवल 1 और 3

C) केवल 1, 2 और 4

D) केवल 1, 3 और 4
Hide Answer -
उ रः (c)
ा ाः कथन 3 गलत है , ोंिक सं िवधान सभा के सद ों का िनवाचन जनता ारा
प से नहीं िकया गया था, ब 1935 म थािपत ां तीय िवधानसभाओं के
सद ों ारा अ िविध से िकया गया था।

सं िवधान सभा की रचना वै सी थी,जै सा िक कैिबने ट िमशन,1946 म ािवत था।


इसकी सीट जनसं ा के अनु पात म िनधा रत की गई थीं। मोटे तौर पर 10 लाख की
जनसं ा पर एक सीट का अनु पात रखा गया था।
इसके चु नाव म सीिमत मतािधकार का योग िकया गया था,जै से:कर,सं पि एवं
शै िणक यो ता के आधार पर सीिमत िकया गया था।
इसम कां े स एवं मु म लीग के अलावा अ छोटे -छोटे दल एवं समू ह शािमल
थे ,जै से-कृषक जा पाट , क ू िन पाट , यू िनयिन पाट आिद। अतः सं िवधान
सभा एक ब दलीय िनकाय थी।

[13]िन िल खत कथनों म से कौन-सा अस है ?

A) सं िवधान सभा ने अने क सिमितयों के मा म से काय िकया था।

B) सं िवधान सभा के थम अ डॉ. राज साद बने थे ।

C) उ े ाव पं . जवाहरलाल ने ह के ारा ु त िकया गया था।

D) सं िवधान सभा म सभी समु दायों को पया ितिनिध दान िकया गया था।
Hide Answer -
उ रः (b)
ा ाः कथन (b) गलत है , ोंिक सं िवधान सभा की थम बै ठक 9 िदसं बर, 1946
म ई थी, िजसम डॉ. स दानं द िस ा को अ थायी अ बनाया गया था। 11
िदसं बर, 1946 को डॉ. राज साद को सं िवधान सभा का थायी अ बनाया गया।

सं िवधान सभा म आठ बड़ी सिमितयाँ तथा अ छोटी सिमितयाँ थीं। सं िवधान सभा
इन सभी सिमितयों के मा म से अपना काय करती थी।
सं िवधान सभा म ‘उ े ाव’ 13 िदसं बर, 1946 म पं . जवाहरलाल ने ह के
ारा ु त िकया गया था, िजसे सं िवधान सभा ने 22 जनवरी, 1947 म पा रत
िकया था।
सं िवधान सभा म सभी समु दायों को उिचत ितिनिध दान िकया गया था, जै से-
िहं दू, मु म, िसख, पारसी, आं -भारतीय, भारतीय ईसाई, अनु सूिचत जाित एवं
जनजाितयाँ , मिहलाएँ आिद।
सं िवधान सभा म 15 मिहला सद एवं 28 अनु सूिचत जाित के सद थे ( ोत-
NCERT, Class-XI, Page-16)।
सं िवधान सभा से सं बंिधत कुछ अ बात: कैिबने ट िमशन के तहत सं िवधान सभा
की कुल सद सं ा-389 होनी थी, परं तु भारत िवभाजन के प ात् सं िवधान सभा
की सद सं ा घटकर 299 रह गई । 26 नवं बर, 1949 को 284 सद
उप थत ए और अं ितम प से पा रत सं िवधान पर ह ा र िकये थे ।

[14]िन िल खत कथनों पर िवचार कीिजयेः

1. सं िवधान सभा ारा रा ीय ज के ा प को 22 जु लाई,1947 को अपनाया गया


था।
2. रा ीय ज के बीच के च म कुल तीिलयाँ 24 ह।
3. रा ीय ज की लं बाई-चौड़ाई का अनु पात 4:3 है ।
नीचे िदये गए कूट का योग कर सही उ र का चयन कीिजयेः

A) केवल 1 और 2

B) केवल 2

C) केवल 2 और 3

D) 1, 2, 3 और 4
Hide Answer -
उ रः (a)
ा ाः कथन 3 गलत है , ोंिक रा ीय ज की लं बाई-चौड़ाई का अनु पात 3:2 है ।

सं िवधान सभा की कुल 12 बै ठक आयोिजत की गई थीं। अं ितम बै ठक 24 जनवरी,


1950 को आयोिजत की गई, जब इसके सद ों ने सं िवधान पर ह ा र िकये थे ।
सं िवधान सभा ने अपने चौथे अिधवे शन म 22 जु लाई 1947 को रा ीय ज के
ा प को ीकार िकया था।
रा ीय ज के बीच के च म कुल तीिलयों की सं ा 24 है ।
सं िवधान सभा से सं बंिधत कुछ अ त ः
- े निवले ऑ न ने अपनी पु क ‘द इं िडयन कॉ ी ू शन’ म िलखा है िक
‘सं िवधान सभा एकदलीय िनकाय है । सभा ही कां े स है और कां े स ही भारत है ।’
- लॉड िवसकाउ ने सं िवधान सभा को िहं दुओं का िनकाय कहा है ।
- िवं न चिचल के अनु सार सं िवधान सभा ने भारत के केवल एक बड़े समु दाय का
ितिनिध िकया है ।
- आइवर जे िनं के अनु सार भारतीय सं िवधान िनमाताओं ने भारतीय सं घ म
अ सं क िहतों एवं भावनाओं को ू नतम रखने का यास िकया है ।

[15]िन िल खत म से कौन-सा कथन अस है ?

A) भारतीय सं िवधान बनने म 2 वष 11 महीने एवं 18 िदन लगे ।

भारतीय सं िवधान के िनमाण होने तक सं िवधान सभा की कुल 11 बै ठक आयोिजत


B)
की गईं ।

C) सं िवधान सभा ने रा गान को 26 जनवरी, 1950 को अपनाया।

D) सं िवधान सभा िनमाण सं बंधी सव थम िवचार 1934 म ु त िकया गया।


Hide Answer -
उ रः (c)
ा ाः कथन (c) अस है , ोंिक सं िवधान सभा ारा रा गान एवं रा गीत 24
जनवरी,1950 म अपनाया गया। 24 जनवरी, 1950 को डॉ. राज साद को भारत का
थम रा पित चु ना गया।

भारतीय सं िवधान को बनने म 2 वष 11 महीने और 18 िदन लगे । इसम सं िवधान


सभा की कुल 11 बै ठक आयोिजत की गई थीं। थम बै ठक 9-23 िदसं बर, 1946
म एवं 11वीं बै ठक 14-26 नवं बर, 1949 म आयोिजत की गई थी। सं िवधान सभा
की अं ितम बै ठक 24 जनवरी, 1950 को आयोिजत की गई। इस बै ठक म सं िवधान
सभा के सद ों ने सं िवधान पर ह ा र िकये ।
नई सं सद के िनमाण तक सं िवधान सभा अं त रम सं सद के प म काय करती रही।
भारत म सं िवधान सभा के गठन का िवचार वष 1934 म पहली बार भारत म
वामपं थी आं दोलन के ने ता एम.एन.राय ारा रखा गया था, परं तु भारतीयों की ओर
से सं िवधान सभा की मां ग रखने का थम यास वष 1934 म राज पाट ारा
िकया गया। उसने एक ाव पा रत कर कहा िक वय मतािधकार के आधार पर
िनवािचत भारतीय ितिनिधयों की एक सं िवधान सभा होगी, जो सं िवधान का िनमाण
करे गी। इसके बाद वष 1935 म भारतीय रा ीय कां े स ने पहली बार भारत के
सं िवधान के िनमाण के िलये आिधका रक प से सं िवधान सभा के गठन की मां ग
की।

[16]िन िल खत म से कौन सं िवधान ा प सिमित के सद नही ं थे ?


1. जे .बी. कृपलानी
2. सरदार पटे ल
3. गोपाला ामी आयं गर
4. अ ादी कृ ामी अ र
5. टी.टी. कृ ामचारी
कूटः

A) केवल 1 और 2

B) केवल 1, 2 और 3

C) केवल 1, 2 और 5

D) केवल 1, 4 और 5
Hide Answer -
उ रः (a)

ा ाः सरदार पटे ल एवं जे .बी. कृपलानी सं िवधान ा प सिमित के सद नहीं थे , इसके

अित र सभी इसके सद थे ।

सं िवधान ा प सिमित का गठन 29 अग , 1947 म िकया गया था,िजसम अ

सिहत िन िल खत सात सद थे , जो िन िल खत थे -

1. डॉ. बी.आर. अं बेडकर-अ

2. एन. गोपाला ामी आयं गर

3. अ ादी कृ ामी अ र

4. डॉ. के.एम. मुं शी

5. सै द मोह द सादु ा

6. एन. माधव राव (बी.एल. िम के थान पर)

7. टी.टी. कृ ामाचारी (डी.पी. खे तान की मृ ु के प ात् )

सं िवधान सभा एवं सं िवधान ा प के सं बंध म कुछ अ बात:

-बी.एन. राव (जो सं िवधान सभा के सं वैधािनक सलाहकार थे ) ारा सं िवधान का पहला

ा प तै यार िकया गया था। इसके मू ल ा प म 243 अनु े द एवं 13 अनु सूिचयाँ

थीं।

-के.एम. मुं शी सं िवधान ा प सिमित के मू लतः एकमा कां े सी सद थे ।

-डॉ. अं बेडकर सं िवधान सभा म बं बई े सीडसी से िनवािचत ए थे ।

[17]सू ची -I को सू ची- II से सु मेिलत कीिजयेः

सू ची-I सू ची-II

A. सं चालन सिमित 1. पं . जवाहरलाल ने ह

B. ां तीय सं िवधान सिमित 2. जे .बी. कृपलानी

C. सं घीय सं िवधान सिमित 3. सरदार पटे ल

D. मू ल अिधकार उप-सिमित 4. डॉ. राज साद

कूटः
A B C D

A) 1 2 3 4

B) 1 2 4 3

C) 4 3 1 2

D) 4 3 2 1
Hide Answer -
उ रः (c)
ा ाः सं िवधान सभा के ारा सं िवधान िनमाण से सं बंिधत िविभ काय को करने के
िलये कई सिमितयों का गठन िकया गया, िजनम 8 बड़ी सिमितयाँ तथा अ छोटी
सिमितयाँ थीं िजसम से कुछ मह पू ण सिमितयों की सू ची इस कार है -
सिमितयाँ अ
1. सं घ श सिमित पं . जवाहरलाल ने ह
2. सं घीय सं िवधान सिमित पं . जवाहरलाल ने ह
3. ां तीय सं िवधान सिमित सरदार पटे ल
4. सं चालन सिमित डॉ. राज साद
5. ि या िनयम सिमित डॉ. राज साद
6. दे शी रयासतों से वाता के िलये बनी सिमित डॉ. राज साद
7. मौिलक अिधकार उपसिमित जे .बी. कृपलानी
8. अ सं क उपसिमित एच.सी. मु खज
9. सलाहकार सिमित (परामशदाता सिमित) सरदार पटे ल

नोट: - मौिलक अिधकार उप सिमित एवं अ सं क उप सिमित सलाहकार सिमित की


उप-सिमितयाँ थीं।
10. ा प सिमित डॉ. अं बेडकर
11. रा ज सं बंधी तदथ सिमित डॉ. राज साद
12. रा ों से वाता के िलये बनी सिमित पं . जवाहरलाल ने ह

[18]भारत एवं अमे रका के सं िवधान के िवषय म िन िल खत म से कौन-सा ावधान


एक-समान है ?

A) दोनों म एकल नाग रकता का ावधान है ।

B) दोनों म इकहरी ायपािलका पाई जाती है ।

C) दोनों के सं िवधान म तीन सू िचयाँ ह।

D) सं िवधान के िनवचन ( ा ा) के िलये एक तं ायपािलका पाई जाती है ।


Hide Answer -
उ रः (d)

ा ाः उपरो म से िवक (d) स है , ोंिक अमे रका एवं भारत म सं िवधान के िनवचन

के िलये एक तं ायपािलका पाई जाती है , िजसे ाियक पु नरावलोकन की श ा है ।

भारत म एकल नाग रकता का ावधान िकया गया है , जबिक सं यु रा अमे रका म

के एवं रा के िलये अलग-अलग नाग रकता का ावधान है ।

भारत म इकहरी ायपािलका है , जबिक अमे रका म ायपािलका की ै धता है , अथात

के एवं रा की अपनी ायपािलका है ।

भारत के सं िवधान म तीन कार की सू ची ह- सं घ सू ची, रा सू ची एवं समवत सू ची, वहीं

अमे रका म सं घ से सं बंिधत िवषयों को िलख िदया गया है एवं शे ष श रा ों को सौंप

दी गई है । भारत म अविश श याँ के के पास ह, जबिक अमे रका म रा ों के पास

ह।

िल खत सं िवधान का ारं भ अमे रका से आ, पर ु अमे रका का सं िवधान िव का सबसे

छोटा िल खत सं िवधान है , िजसके मू ल सं िवधान म केवल सात अनु े द ह। वहीं भारत का

सं िवधान िव का सबसे बड़ा िल खत सं िवधान है । भारत के मू ल सं िवधान म 395

अनु े द, 22 भाग एवं 8 अनु सूिचयाँ (अब 12 अनु सूिचयाँ ) ह।

[19]भारत एवं ि िटश सं िवधान के सं दभ म िन िल खत म से कौन-सा कथन स है ?

A) दोनों दे शों म सं सदीय णाली का ावधान है ।

B) दोनों दे शों म एक तं ायपािलका का ावधान है ।

C) दोनों दे शों ने सं घा क व था को अपनाया है ।

D) दोनों दे शों का सं िवधान न (लचीला) है ।


Hide Answer -
उ रः (a)

ा ाः कथन ( a) सही है , ोंिक भारत एवं ि टे न म सं सदीय णाली को अपनाया गया है ।

इसम वा िवक एवं नाममा कायपािलका का ावधान है । सामू िहक उ रदािय णाली को

अपनाया गया है ,अथात कायपािलका िवधानमं डल के ित उ रदायी रहती है । अमे रका म

अ ा क णाली को अपनाया गया है ।

भारत एवं अमे रका म सं घा क णाली को अपनाया गया है , जबिक ि टे न म एका क

णाली को अपनाया गया है ।

भारत एवं अमे रका म सं िवधान के िनवचन ( ा ा) के िलये एक तं ायालय का

ावधान िकया गया है , जबिक ि टे न म तं ायपािलका का अभाव है । इसिलये भारत

एवं अमे रका म ाियक पु नरावलोकन का ावधान है , जबिक ि टे न म इसका अभाव है ।

ि टे न म सं सद सबसे अिधक भावशाली है ।

अमे रकी सं िवधान कठोर सं िवधान का उदाहरण है , जबिक ि टे न का सं िवधान न

सं िवधान का उदाहरण है , वहीं भारत का सं िवधान कठोर एवं न सं िवधान का िम ण है ।

भारत म गणतं ा क णाली को अपनाया गया है , ोंिक भारत म रा पित का चु नाव

जनता ारा अ िविध से िकया जाता है , जबिक ि टे न म राजतं णाली को अपनाया

गया है , ोंिक वहाँ स ाट का चु नाव वं श पर रा के आधार पर िकया जाता है ।

[20]सू ची- I को सू ची- II से सु मेिलत कीिजयेः

सू ची-I सू ची-II

(भारतीय सं िवधान के ल ण) (सं बंिधत दे श )

A. समवत सू ची 1. जमनी

B. सं घा क व था 2. कनाडा

C. आपात उपबं ध 3. स (पू व सोिवयत सं घ)

D. मू ल क 4. ऑ े िलया

कूटः
A B C D

A) 1 2 3 4

B) 1 2 4 3

C) 4 2 1 3

D) 3 2 4 1
Hide Answer -
उ रः (c)

ा ाः

समवत सू ची- ऑ े िलया के सं िवधान से समवत सू ची (िव ृ त उभयिन सू ची)एवं दोनों

सदनों की सं यु बै ठक को अपनाया गया है ।

सं घा क व था- कनाडा के सं िवधान से सश के के साथ सं घीय व था, अविश

श याँ तथा के ारा रा पाल की िनयु के ावधानों को अपनाया गया है ।

आपात उपबं ध- जमनी के सं िवधान (वीमर) से आपात उपबं ध िलया गया है ।

मू ल क - स (पू व सोिवयत सं घ) के सं िवधान से िलया गया है ।

सं िवधान म सं शोधन की ि या- दि ण अ ीका के सं िवधान से सं िवधान म सं शोधन की

ि या एवं रा सभा के सद ों का िनवाचन को िलया गया है ।

गणतं ा क व था- ाँ स के सं िवधान से िलया गया है । इसके अलावा तं ता,

समानता एवं बं धु का िस ां त िलया गया है ।

रा के नीित िनदे शक त - आयरलै के सं िवधान से रा के नीित िनदे शक त ,

रा सभा म ितभा, अनु भव एवं से वा के आधार पर मनोनयन एवं रा पित की िनवाचन

प ित को अपनाया गया है ।

सं सदीय शासन णाली- ि टे न के सं िवधान से सं सदीय शासन णाली, िविध का शासन,

िवधायी ि या, मं ि मं डल णाली, एकल नाग रकता आिद अपनाया गया है ।

उ े िशका- सं यु रा अमे रका के सं िवधान से उ े िशका, तं ायपािलका, ाियक

पु नरावलोकन, मू ल अिधकार, उप-रा पित का पद, रा पित पर महािभयोग आिद िलया

गया है ।

िविध ारा थािपत ि या-इसे जापान के सं िवधान से िलया गया है ।


[1] िन िल खत कथनों पर िवचार कीिजयेः

1. डॉ.भीमराव अ े डकर ने ावना को सं िवधान की आ ा कहा है ।


2. एस.आर.बो ई बनाम यू िनयन ऑफ इ या म उ तम ायालय ने ावना को
सं िवधान का अिभ अं ग माना है ।
उपयु म से कौन-सा/से कथन स है /ह?

A) केवल 1

B) केवल 2

C) 1 और 2 दोनों

D) न तो 1 और न ही 2

Hide Answer -
उ रः (b)
ा ाः एस.आर.बो ई बनाम यू िनयन ऑफ इ या,1994 मामले म भारत के सव
ायालय ने ावना को सं िवधान का अिभ अं ग माना है । केशवान भारती
बनाम केरल रा , 1973 मामले म भी सव ायालय ने ावना को भारतीय
सं िवधान की मौिलक सं रचना का भाग ीकार िकया है ।

सं िवधान की आ ा ‘ ावना’ को माना जाता है , पर ु डॉ. अ े डकर ने सं िवधान


की आ ा ‘ ावना’ को नहीं, ब ‘सं वैधािनक उपचारों के
अिधकार’(अनु े द-32) को माना है । अतः कथन-1 गलत है ।
भारतीय सं िवधान की ावना म िजन आदश एवं उ े ों की परे खा दी गई
है ,उसकी ा ा मू ल अिधकारों,रा के नीित िनदे शक िस ा ों एवं मौिलक
क ों म की गई है ।
सं िवधान सभा की ा प सिमित के सद के.एम.मुं शी के अनु सार ावना‘हमारे
स भु लोकतां ि क गणरा का भिव फल/ज कु ली है ।’
राजनीितशा ी अन बाकर ने अपनी पु क ि ं िसप ऑफ सोशल ए
पॉिलिटकल ोरी की शु आत भारतीय सं िवधान की ावना से की है ।
डॉ.अ े डकर ारा सं िवधान सभा म िदये गए अपने अ म भाषण म तं ता,
समानता एवं बं धु श का उ े ख िकया गया है ।
डॉ.अ े डकर को भारतीय सं िवधान के िपता एवं आधु िनक मनु की उपािध दी जाती
है ।

[2] भारतीय सं िवधान के ावना पर िवचार कीिजयेः


1. चार कार के ाय की चचा की गई है ।
2. चार कार की तं ता की चचा की गई है ।
3. तीन कार की समानता की चचा की गई है ।
4. सं िवधान को अं गीकृत,अिधिनयिमत और आ ािपत करने की तारीख 26
नव र,1949 है ।
उपयु म से कौन-सा/से कथन स है /ह?
A) केवल 1 एवं 2

B) केवल 2 एवं 3

C) केवल 4

D) केवल 2 एवं 4

Hide Answer -
उ रः (c)

ा ाः उपयु म से केवल कथन 4 सही है । सं िवधान सभा म सं िवधान को अं गीकृत,

अिधिनयिमत एवं आ ािपत करने की तारीख 26 नव र, 1949 ई.(िमित मागशीष शु

स मी,सं वत् दो हज़ार छह िव मी) है ।

ावना म ाय तीन कार के ह: सामािजक, आिथक एवं राजनीितक ाय।

तं ता पाँ च कार की है ः िवचार, अिभ , िव ास, धम एवं उपासना की तं ता।

समानता दो कार की है ः ित ा एवं अवसर की समानता।

नाग रकता, चु नाव, तदथ सं सद आिद 26 नव र, 1949 को तः लागू हो गए एवं

सं िवधान का शे ष ावधान 26 जनवरी, 1950 को लागू आ।

े क वष 26 जनवरी को गणतं िदवस मनाते ह।

े क वष 26 नव र को सं िवधान िदवस मनाते ह।

े क वष 15 अग को तं ता िदवस मनाते ह।

डॉ. अ े डकर ने सं िवधान को एक पिव द ावे ज़ कहा है ।

[3] भारतीय जातं के सं दभ म िन िल खत म से कौन सा/से सही है /ह?

1. जनमत सं ह (Referendum)
2. पहल (Initiative)

3. ावतन (Recall)
4. जनमत िनणय (Plebiscite)
नीचे िदये गए कूट का योग कर सही उ र का चयन कर:

A) केवल 2

B) केवल 1 एवं 2

C) केवल 2, 3 एवं 4

D) उपयु म से कोई नहीं।


Hide Answer -
उ रः (d)
ा ाः भारतीय जातं /लोकतं के सं दभ म उपयु म से कोई भी सही नहीं है ,
ोंिक उपयु सभी लोकतं से सं बंिधत ह, जबिक भारत म अ लोकतं या
ितिनिध लोकतं है ।
भारतीय सं िवधान के ावना म लोकतं का योग ापक प म िकया गया है ।
राजनीितक तं ता के साथ-साथ सामािजक एवं आिथक तं ता को शािमल िकया गया
है ।

[4] िन िल खत म से कौन-सा कथन ‘अस ’ है ?

एस.के.धर आयोग ने भारत म भाषायी आधार पर रा ों के पु नगठन की ीकृित


A)
दी थी।

जवाहरलाल ने ह ,व भ भाई पटे ल एवं प ािभ सीतारमै या की सिमित ने भारत म


B)
भाषायी आधार पर रा ों के पु नगठन को अ ीकार कर िदया था।

फजल अली आयोग ने भारत म रा ों के पु नगठन के भाषायी आधार को ीकार


C)
कर िलया।

रा पु नगठन अिधिनयम,1956 के तहत भारत म 14 रा ों एवं 6 के शािसत


D)
दे शों का गठन िकया गया।

Hide Answer -
उ रः (a)

ा ाः कथन (a) गलत है । जू न 1948 म एस.के.धर की अ ता म ‘भाषायी ां त आयोग’

का गठन िकया गया। आयोग ने अपनी रपोट िदसं बर 1948 म पे श की एवं भारत म भाषायी

आधार पर रा ों के पु नगठन के बदले शासिनक सु िवधा को वरीयता दी।

िदसं बर 1948 म जवाहरलाल ने ह , व भ भाई पटे ल एवं प ािभ सीतारमै या (JVP

सिमित) को िमलाकर एक अ भाषायी ां त सिमित का गठन िकया गया। सिमित ने

अ ै ल, 1949 म अपनी रपोट पे श की एवं भारत म रा ों के पु नगठन के भाषायी आधार

को अ ीकार कर िदया।

िदसं बर 1953 म फजल अली की अ ता म तीन सद ीय रा पु नगठन आयोग

(फजल अली आयोग) का गठन िकया गया। इसके दो अ सद - के.एम. पिण र एवं

एच.एन. कुंज थे । इसने अपनी रपोट 1955 म पे श की एवं कहा िक भारत म रा ों

का पु नगठन भाषायी आधार पर होना चािहये ।

रा पु नगठन अिधिनयम एवं 7व सं िवधान सं शोधन अिधिनयम, 1956 के तहत 1 नवं बर,

1956 को 14 रा ों एवं 6 क -शािसत दे शों का गठन िकया गया।

भाषायी आधार पर वष 1953 म आं दे श का गठन िकया गया। ते लुगू भाषायी े ों को

ले कर म ास म एक ल ा आं दोलन चला। इस आं दोलन म 56 िदन की भू ख हड़ताल के

प ात् एक कां े सी ने ता पो ी ी रामु लु का िनधन हो गया, प रणाम प आं दे श का

गठन आ।
[5] िन िल खत कथनों पर िवचार कीिजयेः

1. भारतीय सं िवधान म ‘फेडरे शन’श का योग कहीं नहीं िकया गया है ।


2. भारत अथात् इं िडया श का योग भारतीय सं िवधान के पहले अनु े द म िकया
गया है ।
उपयु म से कौन-सा/से कथन सही है /ह?

A) केवल 1

B) केवल 2

C) 1 और 2 दोनों

D) न तो 1 और न ही 2

Hide Answer -
उ रः (c)
ा ाः उपयु दोनों कथन सही ह, ोंिक भारतीय सं िवधान म
फेडरे शन(सं घा क)श का योग कहीं नहीं िकया गया है । भारतीय सं िवधान म ‘यू िनयन
ऑफ े ट्स’(रा ों का सं घ)श का योग िकया गया है । हमारा सं िवधान िकसी
समझौते का प रणाम नहीं है जै सा िक अमे रका का सं िवधान है । हमारे सं िवधान म िकसी
भी रा को अलग होने की तं ता नहीं है ।

हमारा दे श िवभाजी रा ों का अिवभाजी सं घ है ,जबिक अमे रका अिवभाजी रा ों


का अिवभाजी सं घ है ।
भारतीय सं िवधान के पहले भाग म सं घ एवं उसका रा े है एवं पहले अनु ेद
म भारत अथात् इं िडया,रा ों का सं घ होगा,कहा गया है ।

[6] भारतीय रा एवं इसके े के सं बंध म िन िल खत म से कौन-सा कथन सही नहीं है ।

नए रा ों के िनमाण सं बंधी िवधे यक सं सद म पे श करने से पू व रा पित की


A)
िसफा रश आव क है ।

नए रा ों के िनमाण सं बंधी िवधे यक सं सद म तब तक पे श नहीं िकये जा सकते ,


B) जब तक िक उस रा के िवधानमं डल को िविनिद नहीं िकया गया हो, िजसके
े , नाम एवं सीमा पर भाव पड़ता है ।

सं सद,सं िवधान के अनु े द 2 एवं 3 म उ खत ावधानों म बदलाव के िलये


C) सं िवधान की पहली एवं चौथी अनु सूची म कोई प रवतन करता है तो इसे सं िवधान
के अनु े द 368 के तहत सं शोधन माना जाएगा।

भारत िकसी दू सरे दे श को अपना भू -भाग दे ता है ,तो सं िवधान के अनु े द 368 के


D)
तहत सं िवधान म सं शोधन करना आव क है ।
Hide Answer -
उ रः (c)
ा ाः कथन (c) गलत है , ोंिक सं सद सं िवधान के अनु े द-2 एवं 3 म िदये गए
ावधानों म प रवतन के िलये सं िवधान के अनु सूची-1 एवं 4 म कोई सं शोधन या प रवतन
करता है ,तो उसे सं िवधान के अनु े द-368 के तहत सं िवधान म सं शोधन नहीं माना
जाएगा। सं सद सं िवधान के अनु े द-2 एवं 3 से सं बंिधत िवधे यक साधारण ब मत एवं
सामा िवधायी ि या के तहत पा रत करता है ।

नए रा ों के िनमाण सं बंधी िवधे यक सं सद के िकसी भी सदन म पे श


(पु रः थािपत)करने से पू व रा पित की िसफा रश आव क है ।
नए रा ों के िनमाण से सं बंिधत िवधे यक सं सद के िकसी भी सदन म पे श करने से
पू व उस रा के िवधानमं डल को िविनिद करना आव क है ,िजस रा के
नाम, े एवं सीमा पर भाव पड़ता है । इसके िलये समय-सीमा िनि त होती है ।
रा पित, रा िवधानमं डल के मत को मानने के िलये बा नहीं है ।
यिद भारत िकसी दू सरे दे श को अपना भू -भाग सौंपता है या िकसी रा की सीमा
समा करता है , तो इसके िलये सं िवधान के अनु े द-368 के तहत सं िवधान म
सं शोधन करना आव क है । जै सा िक 9व सं िवधान सं शोधन अिधिनयम, 1960
ारा भारत ने अपना े पािक ान को ह ां त रत िकया एवं 100व सं िवधान
सं शोधन अिधिनयम, 2015 ारा भारत 111 इन े व (अं तः े ) बां ादे श को
सौंपा गया एवं बां ादे श 51 इन े व (अं तः े ) भारत को सौंपा।
भारत एवं अ दे शों के बीच सीमा िनधारण सं बंधी िववाद को हल करने के िलये
सं वैधािनक सं शोधन की आव कता नहीं है ,यह काय कायपािलका ारा िकया जा
सकता है (उ तम ायालय,1969)।

[7] िन िल खत पर िवचार कीिजयेः


1. ह रयाणा
2. महारा
3. नागालै
4. िस म
5. गोवा
भारत सं घ के पू ण रा का दज़ा ा करने का सही काल म कौन-सा है ?

A) 1-2-3-4-5

B) 1-2-3-5-4

C) 2-3-1-4-5

D) 2-1-3-4-5
Hide Answer -
उ रः (c)
ा ाः उपयु म से िवक (b) सही है ।
महारा ः वष 1960 म ब ई पु नगठन अिधिनयम, 1960 के तहत महारा एवं गु जरात
रा का गठन आ।
नागालै ः नागालै रा अिधिनयम, 1962 के तहत वष 1963 म नागालै रा का
गठन आ।
ह रयाणाः वष 1966 म पं जाब रा से ह रयाणा रा का गठन िकया गया।
िहमाचल दे शः वष 1971 म िहमाचल दे श को पू ण रा का दज़ा िमला।
मिणपु र, ि पु रा एवं मे घालयः वष 1972 म इ पू ण रा का दज़ा िमला।
िस मः 35व सं िवधान सं शोधन अिधिनयम, 1974 के ारा इसे ‘सं ब रा ’ का दजा
िदया गया एवं 36व सं िवधान सं शोधन अिधिनयम, 1975 के तहत इसे पू ण रा का दज़ा
िमला।
िमज़ोरम, अ णाचल दे श एवं गोवाः वष 1987 म इ रा का दज़ा िमला।
छ ीसगढ़, उ राखं ड एवं झारखं डः सन् 2000 म म दे श से छ ीसगढ़, उ र दे श
से उ राखं ड एवं िबहार से झारखं ड रा का गठन िकया गया।
ते लंगानाः 2 जू न, 2014 को आं दे श से एक नए रा ते लंगाना का गठन िकया गया।
वतमान म भारत म 29 रा एवं 7 क -शािसत दे श ह।

[8] सू ची-I को सू ची-II से सु मेिलत कीिजयेः

सू ची-I(रा ) सू ची-II( म)

A. छ ीसगढ़ 1. 28वाँ

B. उ राखं ड 2. 27वाँ

C. झारखं ड 3. 26वाँ

D. ते लंगाना 4. 29वाँ

कूटः
A B C D

A) 1 2 3 4

B) 1 3 2 4

C) 3 2 1 4

D) 3 1 2 4
Hide Answer -
उ रः (c)
ा ाः भारत के रा ों एवं उसके थान की सही सू ची इस कार है -

1. छ ीसगढ़ 26वाँ (1 नव र, 2000)

2. उ राखं ड 27वाँ (9 नव र, 2000)

3. झारखं ड 28वाँ (15 नव र, 2000)

4. ते लंगाना 29वाँ (2 जू न, 2014)

5. गु जरात 15वाँ

6. नागालै 16वाँ

7. ह रयाणा 17वाँ

8. िहमाचल दे श 18वाँ

9. मिणपु र 19वाँ

10. ि पु रा 20वाँ

11. मे घालय 21वाँ

12. िस म 22वाँ

13. िमज़ोरम 23वाँ

14. अ णाचल दे श 24वाँ

15. गोवा 25वाँ

[9] िन िल खत पर िवचार कीिजयेः


1. दादरा एवं नागर हवे ली म ाँ सीिसयों का शासन था।
2. पु डुचे री पु तगाल के अधीन था।
3. पु डुचे री तीन रा ों म अव थत है ।
उपयु म से कौन-सा/से कथन सही है /ह?

A) केवल 1

B) केवल 2 और 3

C) केवल 3

D) 1, 2 और 3
Hide Answer -
उ रः (a)
ा ाः केवल कथन-3 सही है , ोंिक सं घ-शािसत दे श पु डुचे री तीन रा ों म
अव थत है ः
1. तिमलनाडु ः कराइकल एवं पु डुचे री
2. आं दे शः यनम
3. केरलः माहे
दादरा एवं नागर हवे ली 1954 तक पु तगाल के अधीन था न िक ाँ सीिसयों के अधीन।
10व सं िवधान सं शोधन अिधिनयम,1961 ारा इसे सं घ-शािसत े म प रवितत कर
िदया गया।
पु डुचे री 1954 तक ाँ स के अधीन था न की पु तगाल के अधीन। 1954 म ाँ स ने इसे
भारत को सौंप िदया। 14व सं िवधान सं शोधन अिधिनयम ारा इसे सं घ-शािसत दे श
घोिषत िकया गया।
गोवा,दमन एवं दीव पर पु तगािलयों का अिधप था। सन् 1961 म भारत ने पु िलस
कायवाही के ारा इन े ों को अिधगृ हीत कर िलया। 12व सं िवधान सं शोधन
अिधिनयम,1962 ारा इन े ों को सं घ-शािसत दे शों म बदल िदया गया। 56व
सं िवधान सं शोधन अिधिनयम ारा दमन एवं दीव को गोवा से अलग कर िदया गया एवं
गोवा को पू ण रा का दज़ा दे िदया गया।

[10]7व सं िवधान सं शोधन अिधिनयम,1956 से पू व भारतीय रा ों को चार भागों म


िवभािजत िकया गया था। अं डमान एवं िनकोबार ीप समू ह को िनमनिल खत म से िकस
समू ह/भाग म रखा गया था?

A) भाग-(क)

B) भाग-(ख)

C) भाग-(ग)

D) भाग-(घ)

Hide Answer -
उ रः (d)
ा ाः अं डमान एवं िनकोबार ीप समू ह को भाग-घ म रखा गया था। भाग-घ म केवल
अं डमान एवं िनकोबार ीप समू ह को ही शािमल िकया गया था। रा पु नगठन अिधिनयम
एवं 7व सं िवधान सं शोधन, 1956 के ारा भाग-क एवं ख के रा ों के बीच की दू री
समा कर दी गई। भाग-ग को समा कर िदया गया।

[11]भारत का सं िवधान सं सार का सबसे बड़ा िल खत सं िवधान है । इस िवषय म िन िल खत

म से कौन-सा कथन सही नहीं है ?


A) भारत का सं िवधान सं घ एवं रा ों का एकल सं िवधान है ।

B) सं िवधान सभा म कानू न िवशे ष ों का भु था।

C) इसम िविभ दे शी रयासतों म चिलत कानू नों को भी थान िदया गया है ।

D) भारत के सं िवधान पर भारत शासन अिधिनयम 1935 का भाव रहा है ।

Hide Answer -
उ रः (c)

ा ाः भारतीय सं िवधान के िव ृ त होने के पीछे िन िल खत कारण िज़ े दार ह-

भारत का िव ार एवं िविवधता।

भारत शासन अिधिनयम, 1935 का भाव, ोंिक यह अिधिनयम ब त अिधक िव ृत

था।

सं िवधान सभा म कानू न िवशे ष ों का भु था।

[12] िन िल खत कथनों पर िवचार कीिजयेः


1. भारत की शासन णाली सं सदीय एवं अ ीय शासन णाली का सम य है ।
2. अ ीय शासन णाली का आधारभू त त सं िवधान की कठोरता है ।
3. सं सदीय शासन णाली म सामू िहक उ रदािय का िस ा पाया जाता है ।
उपरो म से कौन-सा/से कथन स है /ह?

A) केवल 1

B) केवल 1 और 2

C) केवल 3

D) केवल 2 और 3

Hide Answer -
उ रः (c)
ा ाः कथन 1 और 2 स नहीं ह, ोंिक भारत म सं सदीय शासन णाली (ि टे न के
समान) को अपनाया गया है । मं ि मं डल के सद सं सद के सद होते ह एवं मं ि मं डल
के सद सामू िहक प से लोकसभा के ित उ रदायी होते ह। अतः सं सदीय शासन
णाली म सामू िहक उ रदािय का िस ा पाया जाता है । इसम िवधाियका का
कायपािलका पर िनयं ण रहता है ।

अ ा क शासन णाली अमे रका से अपनाई गई है । इस शासन णाली का


आधारभू त त एकल कायपािलका का होना है । इसम रा एवं सरकार का मु ख
रा पित ही होता है । रा पित अपनी मं ि प रषद का चयन यं अपने िववे क के
आधार पर करता है । इस णाली म श पृ थ रण का िस ा पाया जाता है ।
अथात् कायपािलका, िवधाियका एवं ायपािलका का अिधकार एवं े अलग-
अलग होता है । इस णाली म रा पित का कायकाल िनि त होता है और इससे
पहले रा पित को नहीं हटाया जा सकता है ।
[13]गणतं ा क णाली के िवषय म िन िल खत म से कौन-सा कथन अस है ?

A) ि टे न म गणतं ा क व था को नहीं अपनाया गया है ।

B) गणतं ा क णाली म रा का मु ख वं शानु गत नहीं होता है ।

C) भारत म रा पित का चु नाव जनता ारा अ िविध से िकया जाता है ।

D) उपरो म से कोई नहीं।

Hide Answer -
उ रः (d)
ा ाः उपरो म से कोई भी कथन गलत नहीं है । गणतं ा क णाली म रा का
मु ख वं शानु गत नहीं होता है , ब जनता ारा या अ प से चु ना जाता है ।

ि टे न म राजतं व था को अपनाया गया है । वहाँ रा का मु ख स ाट/महारानी


होता है , िजसका चयन वं शानु गत िविध से िकया जाता है । अतः ि टे न म
गणतं ा क णाली को नहीं अपनाया गया है ।
भारत म रा मु ख (रा पित) का चु नाव जनता ारा अ तरीके से िकया
जाता है । इसिलये भारत म गणतं ा क णाली को अपनाया गया है ।
भारत एवं अमे रका एक लोकतां ि क गणतं ह, जबिक ि टे न लोकतां ि क राजतं ।

[14]भारत म राजनै ितक स ा का मु ख ोत िन िल खत म से कौन है ?

A) जनता

B) सं सद

C) सं िवधान

D) कायपािलका

Hide Answer -
उ रः (a)
ा ाः भारत म राजनै ितक स ा का मु ख ोत जनता है , ोंिक भारत का सं िवधान
"हम भारत के लोग.....।" से ारं भ होता है । अतः भारत म राजस ा जनता म िनिहत
होती है ।

[15]सं वैधािनक शासन व था के सं बंध म िन िल खत कथनों पर िवचार कीिजयेः


1. ऐसी शासन व था की तं ता के िहत म रा की स ा पर भावकारी
ितबं ध लगाती है ।
2. ऐसी शासन व था रा की स ा के िहत म की तं ता पर भावकारी
ितबं ध लगाती है ।
उपरो कथनों म से कौन-सा/से सही है /ह?
A) केवल 1

B) केवल 2

C) 1 और 2 दोनों

D) न तो 1 और न ही 2

Hide Answer -
उ रः (a)
ा ाः सं वैधािनक शासन व था म सरकार की श का ोत सं िवधान होता है ।
इसम सीिमत शासन व था को अपनाया जाता है । की तं ता के िहत म रा
की स ा पर भावकारी ितबं ध लगाती है ,न िक रा की स ा के िहत म की
तं ता पर। ि टे न एवं अमे रका म सं िवधानवाद का उदय आ।

[16]भारतीय शासन णाली के सं बंध म िन िल खत म से कौन-सा कथन ‘अस ’ है ?

A) सव स ा भारत की जनता म िनिहत है ।

B) एकीकृत श का ावधान िकया गया है ।

C) सं सदीय णाली को गणतं वाद से जोड़ा गया है ।

D) लोकतां ि क गणतं को अपनाया गया है ।

Hide Answer -
उ रः (b)
ा ाः भारतीय राजनीितक व था/शासन णाली के सं बंध म कथन(b)गलत
है , ोंिक भारतीय शासन णाली म एकीकृत श को नहीं अपनाया गया है ,ब
सं घा क णाली के तहत स ा के एवं रा ों के बीच िवभािजत है । स ा का एकीकृत
प ि टे न म अपनाया गया है ।

भारत का सं िवधान "हम भारत के लोग....." से ारं भ होता है । इसिलये सव


स ा भारत की जनता म िनिहत है अथात् राजनीितक स ा का मु ख ोत जनता है ।

भारत म सं सदीय णाली के साथ-साथ गणतं ा क णाली को भी अपनाया गया


है , ोंिक गणतं ा क णाली म रा मु ख (रा पित) का चु नाव जनता ारा
या अ तरीके से िकया जाता है । भारत म रा पित का चु नाव जनता ारा
अ िविध से िकया जाता है ।
भारतीय शासन णाली म लोकतं के साथ-साथ गणतं ीय णाली को अपनाया गया
है , ोंिक लोकतां ि क णाली म जनता को सरकार को चु नने एवं बदलने का
अिधकार होता है । भारत म भी चु नाव के मा म से जनता को सरकार चु नने एवं
बदलने का अिधकार है ।

[17]भारत म पं थिनरपे ता के सं दभ म िन िल खत कथनों पर िवचार कीिजयेः

1. भारत म पं थिनरपे ता की नकारा क अवधारणा को ीकार िकया गया है ।


2. मू ल ावना म पं थिनरपे ता की चचा की गई है ।
3. एस.आर.बो ई मामले म सव ायालय ने पं थिनरपे ता को सं िवधान के
आधारभू त सं रचना के प म मा ठहराया है ।
4. रा का अपना कोई धम नहीं है ।
नीचे िदये गए कूट का योग कर सही उ र का चयन कीिजये :
कूटः

A) केवल 1 और 2

B) केवल 3 और 4

C) केवल 1, 3 और 4

D) 1, 2, 3 और 4

Hide Answer -
उ रः (b)
ा ाः उपयु कथनों म से केवल 3 एवं 4 सही ह। एस.आर.बो ई बनाम भारत सं घ
(1994) म सव ायालय ने पं थिनरपे ता को सं िवधान के आधारभू त ढाँ चे के प म
मा ठहराया है । केशवान भारती बनाम केरल रा , 1973 के मामले म सव
ायालय ने पं थिनरपे ता को सं िवधान का आधारभू त ढाँ चा माना है । सं िवधान के
आधारभू त ढाँ चे का िवचार केशवान भारती मामले म ही िदया गया है ।

रा का अपना कोई धम नहीं है । रा सभी धम को समान मह दे ता है एवं


सभी धम का सं र ण करता है । भारत का समाज ब धमवादी है , इसिलये भारत का
सं िवधान सभी धम का समान आदर/स ान करता है ।
भारत म पं थिनरपे ता की पि मी अवधारणा को नहीं अपनाया गया है , ोंिक
पं थिनरपे ता की पि मी अवधारणा नकारा क है । धम एवं राजनीित को पू णतः
पृ थक् मानता है , जबिक भारत म पं थिनरपे ता की सकारा क अवधारणा को
ीकार िकया गया है ।
मू ल ावना/उ े िशका म पं थिनरपे ता की चचा नहीं की गई है , ब 42व
सं िवधान सं शोधन अिधिनयम, 1976 के ारा इसे जोड़ा गया है ।

[18]िन िल खत कथनों पर िवचार कीिजयेः

1. भारत म मतदान का अिधकार एक सं वैधािनक अिधकार है ।


2. 61व सं िवधान सं शोधन अिधिनयम के ारा भारत म मतदान करने की ू नतम आयु
25 वष से घटाकर 18 वष कर दी गई है ।
उपयु कथनों म से कौन सा/से कथन सही है /ह?

A) केवल 1

B) केवल 2

C) 1 और 2 दोनों

D) न तो 1 और न ही 2
Hide Answer -
उ रः (a)
ा ाः उपयु कथनों म से केवल कथन 1 सही है , ोंिक भारतीय सं िवधान के
अनु े द-326 म मतदान के अिधकार का उपबं ध िकया गया है । लोक सभा एवं रा की
िवधानसभाओं के चु नाव म मतदान करने की ू नतम आयु 18 वष है ।

61व सं िवधान सं शोधन अिधिनयम,1988 के तहत सन् 1989 म मतदान के


अिधकार की ू नतम आयु 21 वष से घटाकर 18 वष कर दी गई, न िक 25 वष
से घटाकर 18 वष।
ाम पं चायत के चु नाव म भी मतदान करने की ू नतम आयु 18 वष िनधा रत की
गई है ।

[19]भारतीय सं िवधान के ावना के सं दभ म िन िल खत पर िवचार कीिजयेः


A. भु -स B. लोकतां ि क
C. गणरा
D. पं थिनरपे
E. समाजवादी
उपयु का सही म िन िल खत म से कौन-सा है ?

A) A, B, C, D, E

B) A, B, C, E, D

C) A, D, E ,B, C

D) A, E, D, B, C

Hide Answer -
उ र: (d)
ा ाः उपयु म से िवक (d) सही है ।

भारतीय सं िवधान की ावना/उ े िशका हम, भारत के लोग... से ारं भ होती है ,


जो यह बतलाती है िक सं िवधान की श का ोत भारत की जनता है ।
स ूण भु -स ,समाजवादी,पं थिनरपे एवं लोकतां ि क गणरा रा की
कृित को बतलाता है ।
ाय, तं ता,समानता एवं बं धु सं िवधान का उ े है ।

[20]भारतीय सं िवधान की ावना के स भ म िन िल खत म से कौन-सा कथन सही नहीं


है ?
A) यह पं िडत ने ह के उ े ाव पर आधा रत है ।

B) इसकी कृित ाय-यो नहीं है ।

C) यह सं िवधान का अं ग नहीं है ।

D) इसका सं शोधन केवल एक बार िकया गया है ।

Hide Answer -
उ रः (c)
ा ाः कथन-C गलत है , ोंिक ावना को भारतीय सं िवधान का अं ग माना गया है ।
उ तम ायालय ने बे बाड़ी सं घ मामले (1960) म अपने ारा िदये गए िनणय को
अ ीकार कर केशवान भारती बनाम केरल रा , 1973 म व था दी िक ावना
सं िवधान का अं ग है ।

पं . ने ह ारा 13 िदस र, 1946 को सं िवधान सभा म एक उ े ाव रखा,


िजसे सं िवधान के ारा 22 जनवरी, 1947 को ीकार कर िलया गया, जो सं िवधान
की उ े िशका बनी।
उ े िशका गै र- ाियक है अथात् इसकी कृित ाययो नहीं है । इसे ायालय म
चु नौती नहीं दी जा सकती।
केशवान भारती मामले म उ तम ायालय ने यह िनणय िदया िक उ े िशका म
सं शोधन िकया जा सकता है , पर ु इसम िनिहत मू ल िवशे षताओं म सं शोधन नहीं
िकया जा सकता। अभी तक ावना म केवल एक बार सं शोधन आ है । 42व
सं िवधान सं शोधन अिधिनयम, 1976 के ारा ‘समाजवादी’ ‘पं थिनरपे ’ एवं
‘अख ता’ श को जोड़ा गया है ।
सव थम अमे रकी सं िवधान म ावना को स िलत िकया गया था।
भारतीय सं िवधान की ावना म तं ता, समानता एवं बं धुता का आदश ाँ स के
सं िवधान से िलया गया है ।
[1] िन िल खत पर िवचार कीिजयेः

1. नए रा ों का िनमाण।
2. नए रा ों का वे श।
3. रा ों के नामों म प रवतन।
उपयु म से िकस-िकस का सं बंध सं िवधान के अनु े द-3 से है ?

A) केवल 1 और 2

B) केवल 1 और 3

C) केवल 2 और 3

D) 1, 2 और 3

Hide Answer -
उ रः (b)

ा ाः कथन (1) और (3) का सं बंध भारतीय सं िवधान के अनु े द-3 से है । इस अनु ेद म

ावधान है िक सं सद िविध ारा नए रा ों का िनमाण और वतमान रा ों के े ों, सीमाओं या

नामों म प रवतन कर सकेगा। सं सद िविध ारा िकसी रा का े बढ़ा या घटा सकेगा।

अनु े द-3 का सं बंध रा ों के पु नस मन से है ।

सं िवधान का अनु े द-2 यह ावधान करता है िक सं सद िविध ारा नए रा ों का वे श

या थापना कर सकेगी।

सं िवधान का अनु े द-4 यह ावधान करता है िक सं सद अनु े द-2 एवं 3 के ावधानों म

कोई प रवतन करती है तो उसे अनु े द-368 के तहत सं शोधन नहीं माना जाएगा।

अनु े द-1 म कहा गया है िक भारत, अथात् इं िडया, रा ों का सं घ होगा। इस अनु ेद

म भारत के रा े को तीन े िणयों म बाँ टा गया है -

(क) रा ों के रा े

(ख) पहली अनु सूची म शािमल सं घ रा े

(ग) ऐसे अ रा े , जो भारत सरकार ारा िकसी भी समय अिजत िकये जाये ।

[2] िन िल खत म से कौन-सा कथन सही है ?

A) े फल की ि से भारत का सबसे छोटा रा िस म है ।

B) जनसं ा की ि से भारत का सबसे छोटा रा गोवा है ।

C) भारत म गाँ वों का सबसे अिधक ितशत िहमाचल दे श म है ।

D) ते लंगाना की राजधानी अमरावती को बनाया गया है ।


Hide Answer -
उ रः (c)

ा ाः केवल कथन (c) सही है , ोंिक भारत म गाँ वों का सबसे अिधक ितशत िहमाचल

दे श म है ।

े फल की ि से भारत का सबसे छोटा रा गोवा है , न िक िस म।

जनसं ा की ि से भारत का सबसे छोटा रा िस म है , न िक गोवा।

अमरावती को आं दे श की राजधानी बनाया गया है , न िक ते लंगाना की। ते लंगाना की

राजधानी है दराबाद है ।

[3] िन िल खत कथनों पर िवचार कीिजयेः

1. सं सद िकसी रा की सीमा, े या नाम म प रवतन उसके सहमित के िबना


नहीं कर सकती है ।
2. सं सद रा ों की सीमा, े या नाम म प रवतन िवशे ष िवधायी ि या ारा ही
कर सकती है ।
नीचे िदये गए कूट का योग कर सही उ र का चयन कर।

A) केवल 1

B) केवल 2

C) 1 और 2 दोनों

D) न तो 1 और न ही 2

Hide Answer -
उ रः (d)

ा ाः सं िवधान के अनु े द-4 म यह ावधान िकया गया है िक सं सद नए रा ों के वे श या

गठन अथवा नए रा ों के िनमाण या िकसी रा की सीमा, े या नाम म प रवतन िबना िकसी

रा की सहमित से कर सकती है । सं सद यह प रवतन साधारण िवधायी ि या/साधारण ब मत

ारा करती है । केवल सं सद ही इस सं बंध म िवधे यक बना सकती है । अतः उपयु दोंनों कथन

गलत ह ।

[4] िन िल खत कथनों पर िवचार कीिजयेः

1. गोवा
2. पु डुचे री
3. दमन एवं दीव
4. िस म
5. दादरा एवं नागर हवे ली
उपयु म से कौन-से े भारत ारा अिजत (Acquire) े ह?
A) केवल 1 और 2

B) केवल 1, 2 और 3

C) केवल 2, 4 और 5

D) 1,2,3,4 और 5

Hide Answer -
उ रः (d)

ा ाः गोवा और दमन एवं दीव म सन् 1961 से पू व पु तगािलयों का अिधकार था। भारत ने

सन् 1961 म पु िलस कायवाही के ारा इन े ों को अिधगृ हीत कर िलया।

पु डुचे रीः 1954 तक इस पर ाँ स का अिधकार था। 1954 म ाँ स ने इसे भारत को

सौंप िदया।

िस मः 1947 म ि िटश शासन समा होने के प ात् भारत ने इसे रि त े घोिषत

िकया। 1947 तक इस पर चो ाल का शासन था। 35व सं िवधान सं शोधन अिधिनयम

ारा सहयु एवं 36व सं शोधन अिधिनयम ारा पू ण रा का दज़ा दान िकया गया।

दादर एवं नागर हवे लीः 1954 तक यहाँ पु तगािलयों का शासन था। भारत ारा 1961 म

इसे सं घ रा े का दज़ा िदया गया।

[5] मू ल अिधकारों के सं दभ म िन िल खत कथनों पर िवचार कीिजये :


1. धम, मू लवं श, जाित, िलं ग या ज थान के आधार पर िवभे द का ितषे ध
2. लोक िनयोजन के िवषय म अवसर की समानता
3. िवचार, अिभ , शा पू ण स े लन, िनबाध िवचरण, िनवास एवं सं घ बनाने की
तं ता
4. ाण एवं दै िहक तं ता का सं र ण
उपयु म कौन-से मू ल अिधकार केवल भारतीय नाग रकों को ा ह?

A) केवल 1 एवं 2

B) केवल 1, 2 एवं 3

C) केवल 2, 3 एवं 4

D) 1, 2, 3 और 4
Hide Answer -
उ रः (b)

ा ाः भारतीय सं िवधान के पाँ च मू ल अिधकार केवल भारतीय नाग रकों को ा ह। ये मू ल

अिधकार, िवदे शी नाग रकों को नहीं िदये गए ह, जो िन िल खत ह:

अनु े द-15: धम, मू लवं श, जाित, िलं ग या ज थान के आधार पर िवभे द का ितषे ध।

अनु े द-16: लोक िनयोजन के िवषय म अवसर की समानता।

अनु े द-19: िवचार एवं अिभ , शा पू ण स े लन, िनबाध िवचरण, िनवास एवं

सं घ बनाने की तं ता।

अनु े द-29: अ सं क वग के िहतों का सं र ण।

अनु े द-30: िश ण सं थाओं की थापना एवं शासन करने का अ सं क वग का

अिधकार।

अनु े द-21: ाण एवं दै िहक तं ता का सं र ण भारतीय एवं िवदे शी दोनों

नाग रकों को ा है ।

[6] िन िल खत कथनों पर िवचार कीिजयेः

1. भारतीय सं िवधान के भाग-2 म ावधान िकया गया है िक सं सद एवं रा के


िवधानमं डल नाग रकता सं बंधी िविध बना सकते ह।
2. भारत एवं अमे रका म केवल ज जात नाग रक ही रा पित बन सकते ह।
3. भारत, ि टे न एवं अमे रका के सं िवधान एकल नाग रकता का ावधान करते
ह।
उपरो म से कौन-सा/से कथन सही है /ह?

A) केवल 1

B) केवल 2

C) 1, 2 और 3

D) उपयु म से कोई नहीं

Hide Answer -
उ रः (d)

ा ाः उपरो तीनों कथन सही नहीं ह, ोंिक-

भारतीय सं िवधान के भाग-2 (अनु े द 5-11) म नाग रकता सं बंधी ावधान िकया गया।

इसम नाग रकता सं बंधी कानू न बनाने की श केवल सं सद को दी गई है ।

अमे रका म केवल ज जात नाग रक ही रा पित बन सकता है , पर ु भारत म ज जात

या ाकृितक प से नाग रकता ा भी रा पित बनने की यो ता रखता है ।

भारत एवं ि टे न म एकल नाग रकता, जबिक अमे रका एवं ट् ज़रलड म दोहरी

नाग रकता सं बंधी ावधान है ।

[7] सू ची-I को सू ची-II से सु मेिलत कीिजये तथा नीचे िदए गये कूट का योग कर सही

उ र चु िनये :
सू ची-I(नाग रकता) सू ची-
II(अनु े द)

A. वह जो भारत का मू ल िनवासी हो 1. अनु े द-5

B. वह जो भारत से पािक ान थाना रत आ हो,


2. अनु े द-6
पर ु बाद म भारत लौट आया हो

C. वह जो पािक ान से भारत थाना रत आ हो 3. अनु े द-7

D. भारतीय मू ल का वह जो भारत से बाहर रह रहा हो 4. अनु े द-8

कूटः
A B C D

A) 1 2 3 4

B) 1 3 2 4

C) 4 2 3 1

D) 4 3 2 1
Hide Answer -
उ रः (b)

ा ाः भारत म नाग रकता के सं बंध म एक सं वैधािनक ावधान है , िजसकी चचा अनु े द-5

से 11 तक की गई है इसम नाग रकता की ा , समा एवं सं सद ारा िविध बनाने सं बंधी

ावधान शािमल ह एवं दू सरी सं सद ारा नाग रकता अिधिनयम, 1955 पा रत िकया गया है ।

सं वैधािनक ावधान:

अनु े द-5 म ावधान िकया गया है िक सं िवधान के ारं भ होने पर िजस का

अिधवास (domicile) भारत के रा े म है एवं कुछ शत का पालन करता है

अथात् वह जो भारत का मू ल िनवासी हो।

अनु े द-6 म उस की चचा है , िजसका पािक ान से भारत म जन/

थानां तरण आ है । इसे दो भागों म िवभािजत िकया गया है ;

(a) जो 19 जु लाई, 1948 से पहले भारत आए थे ।

(a) जो 19 जु लाई, 1948 को या उसके बाद भारत आए है ।

अनु े द-7 म उस की चचा की गई है , जो भारत से पािक ान थाना रण/

जन कर गए थे , पर ु बाद म भारत लौट आए।

अनु े द-8 म उस के सं बंध म चचा की गई है , जो भारतीय मू ल/उ व के ह,

परं तु वह भारत के बाहर रहते ह।

अनु े द-9 म ावधान िकया गया है िक कोई भारतीय नाग रक े ा से िकसी िवदे शी

रा की नाग रकता अिजत कर ले ता है , तो वह अनु े द-5, 6 या 8 के आधार

पर भारत का नाग रक नहीं समझा जाएगा।

अनु े द-10 सं सद को नाग रकता सं बंधी िविध बनाने का अिधकार दे ता है ।

अनु े द-11 म ावधान िकया गया है िक नाग रकता के अजन या समा या

नाग रकता सं बंधी अ िवषयों के सं बंध म सं सद की श कम नहीं होगी।

[8] नाग रकता अिधिनयम,1955 नाग रकता ा करने सं बंधी कुछ ावधान करता
है । िन िल खत म से कौन-सा ावधान इसम शािमल नही ं है ?

A) ज के आधार पर (by birth)

B) वं श के आधार पर (by descent)

C) पं जीकरण के आधार पर (by Registration)

D) रा ीयकरण के आधार पर (by nationalisation)


Hide Answer -

उ रः (d)

ा ाः उपयु म से िवक -d गलत है , ोिक रा ीयकरण (Nationalisation) के थान

पर दे शीयकरण/ ाकृितकरण (Naturalisation) होगा।

नाग रकता अिधिनयम, 1955 म नाग रकता ा करने सं बंधी पाँ च ावधान िकये गए ह:

(1) ज से (by birth)

(2) वं श के ारा (by descent)

(3) पं जीकरण ारा (by registration)

(4) दे शीयकरण/ ाकृितकरण ारा (by naturalisation)

(5) रा े के समावे शन ारा (by incorporation of territory)

[9] सू ची-I को सू ची-II से सु मेिलत कीिजये तथा नीचे िदये गए कूट का योग कर
सही उ र का चयन कीिजये :

सू ची-I
(नाग रकता सू ची-II (यो )
का अजन)

1. िजस का ज 26 जनवरी, 1950 को या


A. ज के
उसके बाद, पर ु 10 िदस र, 1992 से पू व भारत के
ारा
बाहर आ हो।

B. वं श के 2. िजस का ज 26 जनवरी, 1950 को या उसके


आधार पर बाद, पर ु 1 जु लाई, 1947 से पू व भारत म आ हो।
C. दे शीयकरण 3. सं िवधान की 8वीं अनु सूची म उ खत भाषाओं म से
के आधार पर िकसी एक का अ ा ाता हो।

D. पं जीकरण 4. भारतीय मू ल का जो नाग रकता ा के िलये


के ारा आवे दन दे ने से ठीक पहले 7 वष तक भारत म रह चु का हो।

कूटः
A B C D

A) 2 1 3 4

B) 2 3 1 4

C) 4 1 3 2

D) 4 3 1 2

Hide Answer -
उ रः (a)

ज के ारा - िजस का ज 26 जनवरी, 1950 को या उसके


बाद, पर ु 1 जु लाई, 1947 से पू व भारत म आ हो।
वं श के आधार पर - िजस का ज 26 जनवरी, 1950 को या
उसके बाद, पर ु 10 िदस र, 1992 से पू व भारत के बाहर आ हो।
दे शीयकरण के आधार पर - सं िवधान की 8वीं अनु सूची म उ खत भाषाओं म से
िकसी एक का अ ा ाता हो।
पं जीकरण के ारा - भारतीय मू ल का जो नाग रकता ा के िलये
आवे दन दे ने से ठीक पहले 7 वष तक भारत म रह चु का हो।

[10]िन िल खत म से कौन-सा कथन सही है ?

कोई जो दे शीयकरण ारा भारत का नाग रक है , उसे कभी भी नाग रकता


A)
से वं िचत नहीं िकया जा सकता है ।

जो भारतीय नाग रकता का े ा से ाग करता है , उस का ेक


B)
नाबािलग ब ा भारत का नाग रक नहीं रह जाएगा।

पं जीकरण या ाकृितक नाग रकता के पाँ च वष के दौरान नाग रक को िकसी अ


C)
दे श म 6 माह की कैद ई हो तो भारतीय नाग रकता से वं िचत कर िदया जाएगा।

यिद कोई भारतीय नाग रक े ा से िकसी अ दे श की नाग रकता हण कर


D) ले ता है तो उसकी भारतीय नाग रकता तब तक समा नहीं होगी, जब तक भारत
सरकार उसे बखा न करे ।
Hide Answer -
उ रः (b)

ा ाः कथन-b सही है । जब कोई भारतीय नाग रक े ा से भारतीय नाग रकता का ाग

करता है , तो उस का े क नाबािलग ब ा भारतीय नाग रक नहीं रहे गा।

ाकृितकरण/दे शीयकरण के ारा ा नाग रक को भी भारतीय नाग रकता से वं िचत

िकया जा सकता है ।

पं जीकरण या ाकृितक नाग रकता के पाँ च वष के दौरान नाग रक को िकसी अ दे श म

2 वष की कैद ई हो तो उसे भारतीय नाग रकता से वं िचत कर िदया जाएगा। अतः छः

माह के थान पर 2 वष होगा।

यिद कोई भारतीय नाग रक े ा से िकसी दू सरे दे श की नाग रकता हण कर ले ता है तो

उसकी भारतीय नाग रकता तः समा हो जाएगी।

ग रकता खोने के तीन कारण बताए गए ह:

(1) ै क ाग (Renunciation of Citizenship)

(2) नाग रकता की बखा गी (Termination of Citizenship)

(3) नाग रकता से वं िचत (Deprivation of Citizenship)

[11]भारतीय िवदे शी नाग रकता काडहो र के िवषय म िन िल खत म से कौन-सा कथन


सही है ?

इसम भारतीय मू ल के काड योजना एवं समु पारीय भारतीय नाग रक काड
A)
योजना का िवलय िकया गया है ।

B) इसे भारत म मतदान करने का अिधकार नहीं होगा।

C) इसे भारत म सं वैधािनक एवं सावजिनक पद ा करने का अिधकार नहीं होगा।

D) उपयु सभी।
Hide Answer -
उ रः (d)
ा ाः उपयु सभी कथन सही ह।
भारतीय िवदे शी नाग रक काडहो र एक नई योजना है , िजसम पी.आई.ओ. एवं
ओ.सी.आई. काड योजना का िवलय िकया गया है । यह नई काड योजना नाग रक
(सं शोधन) अिधिनयम, 2015 म ारं भ िकया गया है ।
पी.आई.ओ. काड योजना 2002 म एवं ओ.सी.आई. काड योजना 2005 म शु की
गई थी। अब पी.आई.ओ. काड योजना को समा कर िदया गया है ।
यह भारत म िन िल खत के िलये यो नहीं होंगे:
- मतदान के िलये ।
- रा पित, उप-रा पित चु ने जाने के िलये ।
- लोकसभा, रा सभा, िवधानसभा या िवधान प रषद के सद चु ने जाने
के िलये ।
- सव ायालय या उ ायालय के ायाधीश बनने के िलये ।
- सावजिनक पद ा करने के िलये आिद।

[12]िन िल खत कथनों पर िवचार कीिजये -


1. नाग रकता अिधिनयम, 1955 म अब तक चार बार सं शोधन िकया जा चु का है ।
2. भारतीय मू ल के यों को दोहरी नाग रकता दान करने के िलये भारत म वष
2003 म एक उ रीय सिमित का गठन िकया गया था।
नीचे िदये गए कूट का योग कर सही उ र का चयन कर।

A) केवल 1

B) केवल 2

C) 1 और 2 दोनों

D) न तो 1 और न ही 2
Hide Answer -
उ रः (d)
ा ाः उपयु दोनों कथन गलत ह, ोंिक नाग रकता अिधिनयम, 1955 म 2015
तक कुल आठ बार सं शोधन िकया जा चु का है , जो िन िल खत है :
(1) वष 1957
(2) वष 1960
(3) वष 1985
(4) वष 1986
(5) वष 1992
(6) वष 2003
(7) वष 2005
(8) वष 2015

भारतीय मू ल के यों को दोहरी नाग रकता दान करने के िलये िसत र


2000 म एल.एम. िसं घवी की अ ता म एक उ रीय सिमित का गठन िकया
गया न िक वष 2003 म। इस सिमित ारा जनवरी 2002 म अपनी रपोट सौपी
गई एवं नाग रकता (सं शोधन) अिधिनयम, 2003 म 16 िनिद दे शों म (पािक ान
एवं बां ादे श को छोड़कर) भारतीय मू ल के यों के िलये िवदे शी भारतीय
नाग रकता का ावधान िकया गया।
नाग रकता (सं शोधन) अिधिनयम, 2005 म सभी दे शों म (पािक ान एवं
बां ादे श को छोड़कर) भारतीय मू ल के यों के िलये िवदे शी भारतीय
नाग रकता का ावधान िकया गया।

[13]मू ल अिधकार के सं दभ म िन िल खत कथनों पर िवचार कीिजयेः


1. वाद-यो नहीं ह।
2. ाय-यो नहीं ह।
3. सं शोधन-यो नहीं ह।
4. उ तम ायालय के मू ल े ािधकार के अं तगत नहीं आते ह।
नीचे िदये गए कूट का योग कर सही उ र का चयन कर।

A) केवल 1 और 2

B) केवल 1 और 3

C) केवल 4

D) केवल 1, 2 और 4
Hide Answer -
उ रः (c)
ा ाः उपयु म से केवल कथन 4 सही है ।

मू ल अिधकार वाद-यो ह, जबिक रा के नीित िनदे शक त अवाद-यो ह।


मू ल अिधकार ाय-यो / ायोिचत ह, ोंिक इनके उ ं घन पर उ
ायालय या उ तम ायालय म जा सकता है ।
सं सद, सं िवधान के अनु े द-368 के तहत मू ल अिधकारों म सं शोधन कर सकती
है ,परं तु साधारण िविध से नहीं।
मू ल अिधकार के उ ं घन पर कोई सीधे उ तम ायालय म जा सकता है ,
परं तु यह उ तम ायालय के मू ल (Original) े ािधकार के अं तगत नहीं आता
है । उ तम ायालय के मू ल (Original) े ािधकार को अन े ािधकार भी
कहते ह।
मौिलक अिधकार असीिमत नहीं ह। रा इन पर यु यु ितबं ध लगा सकता है ।
आपातकाल के समय मू ल अिधकार को िनलं िबत िकया जा सकता है , अनु े द-20
एवं 21 को छोड़कर।

[14]मू ल अिधकार के िवषय म िन िल खत म से कौन-सा कथन सही नहीं है ?

वाक् एवं अिभ की तं ता भारतीय नाग रकों के साथ-साथ िवदे िशयों को भी


A)
ा है ।

धम, मू लवं श, जाित, िलं ग या ज थान के आधार पर िवभे द का ितषे ध िवदे िशयों
B)
को ा नहीं है ।

ाण एवं दै िहक तं ता का सं र ण भारतीय नाग रकों के साथ-साथ िवदे िशयों को


C)
भी ा है ।

D) उपरो सभी।
Hide Answer -
उ रः (a)
ा ाः उपयु कथनों म से कथन (a) गलत है , ोंिक कुछ ऐसे मू ल अिधकार ह, जो
केवल भारतीय नाग रकों को ा ह, जै से:
i. अनु े द-15
ii. अनु े द-16
iii. अनु े द-19
iv. अनु े द-29
v. अनु े द-30
वाक् एवं अिभ की तं ता सं िवधान के अनु े द-19(1) (क) म विणत है । यह
मू ल अिधकार केवल भारतीय नाग रकों को ा है ।
धम, मू लवं श, जाित, िलं ग या ज थान के आधार पर िवभे द का ितबं ध सं िवधान के
अनु े द-15 म विणत है । यह अिधकार केवल भारतीय नाग रकों को ा है ।
ाण एवं दै िहक तं ता का सं र ण सं िवधान के अनु े द-21 म विणत है । यह मू ल
अिधकार भारतीय नाग रकों एवं िवदे िशयों दोनों को ा है ।

[15]िन िल खत कथनों पर िवचार कीिजये :

1. भारतीय सं िवधान म मू ल अिधकारों को शािमल करने के िलये ने ह रपोट (1928)


म समथन िकया गया था।
2. भारतीय सं िवधान के भाग-3 म रा को प रभािषत िकया गया है ।
नीचे िदये गए कूट का योग कर सही उ र का चयन कर।

A) केवल 1

B) केवल 2

C) 1 और 2 दोनों

D) न तो 1 और न ही 2

Hide Answer -
उ रः (c)
ा ाः उपयु दोनों कथन सही ह।

ने ह रपोट, 1928 म मू ल अिधकारों को सं िवधान म शािमल करने का समथन


िकया गया था।
भारतीय सं िवधान के भाग-3 के अनु े द-12 से 35 तक मू ल अिधकारों की चचा
की गई है । अनु े द-12 म रा की िव ृ त प रभाषा दी गई है ।

[16]भारत म मू ल अिधकारों के सं दभ म िन ां िकत म से कौन-सा कथन ‘अस ’ है ?


A) यह रा के काय के िव एक गारं टी है ।

B) यह सामािजक, आिथक एवं राजनीितक ाय सु िनि त करता है ।

C) यह अमे रका म अिधकारों के िबल की भाँ ित है ।

D) इसे आपातकालीन थित म िनलं िबत िकया जा सकता है ।

Hide Answer -
उ रः (b)

ा ाः कथन (b) अस है , ोंिक सामािजक, आिथक एवं राजनीितक ाय सं बंधी ावधान

भारतीय सं िवधान की ावना एवं रा के नीित िनदे शक िस ां त म िकया गया है , न िक मू ल

अिधकार म।

मू ल अिधकार रा के काय के िव गारं टी दान करते ह। मू ल अिधकारों के उ ं घन

पर रा के िव ायालय म जा सकता है ।

भारत म मू ल अिधकार अमे रका म ‘िबल ऑफ राइट् स’ के समान है ।

आपातकालीन थित म अनु े द-20 एवं 21 को छोड़कर अित र मू ल अिधकारों को

िनलं िबत िकया जा सकता है ।

[17]िन िल खत कथनों पर िवचार कीिजये :


1. सं िवधान का अनु े द-13 सं वैधािनक ावधानों को सं सद या रा के िवधानमं डलों
ारा बनाए गए िनयमों/कानू नों पर ाथिमकता दान करता है ।
2. भारत म मू ल अिधकारों को लागू करने की श केवल उ तम ायालय एवं उ
ायालयों को ा है ।
नीचे िदये गए कूट का योग कर सही उ र का चयन कीिजये :

A) केवल 1

B) केवल 2

C) 1 और 2 दोनों

D) न तो 1 और न ही 2

Hide Answer -
उ रः (c)
ा ाः उपयु दोनों कथन सही ह, ोंिक सं िवधान के अनु े द 13 म यह ावधान
िकया गया है िक सं सद या रा के िवधानमं डलों ारा ऐसी कोई िविध/कानू न नहीं बनाई
जाएगी, जो भाग-3 म द मू ल अिधकारों को छीनता या ू न करता हो। यिद कोई िविध
मौिलक अिधकारों का उ ं घन करता है तो उस सीमा तक िविध शू /अवै ध होगी।

भारत म केवल उ तम ायालय एवं उ ायालयों को मू ल अिधकारों को वतन


करने की श ा है । वे सं िवधान के अनु े द-32 एवं 226 के तहत रट
िनकाल सकते ह।

[18]
िन िल खत कथनों पर िवचार कीिजयेः
1. समानता का अिधकार एक मू ल अिधकार है , िजसकी चचा भारतीय सं िवधान के चार
अनु े दों के अं तगत की गई है ।
2. धम आिद के आधार पर िवभे द का ितषे ध एक मू ल अिधकार है , िजसका उ ेख
समानता के अिधकार के अं तगत िकया गया है ।
नीचे िदये गए कूट का योग कर सही उ र का चयन कीिजये :

A) केवल 1

B) केवल 2

C) 1 और 2 दोनों

D) न तो 1 और न ही 2

Hide Answer -
उ रः (b)
ा ाः उपयु म से कथन 1 गलत ह, ोंिक समानता का अिधकार एक मू ल
अिधकार है । इनकी चचा सं िवधान के पाँ च अनु े दों (अनु े द-14-18) म की गई है न
िक चार अनु े दों म। धम आिद के आधार पर िवभे द का ितषे ध एक मू ल अिधकार है ।
इसकी चचा समानता के अिधकार (अनु े द-15) के अं तगत की गई है ।

[19]भारतीय सं िवधान के मू ल अिधकार के सं दभ म िन िल खत कथनों पर िवचार कीिजयेः


1. िविध के सम समता ि टे न के सं िवधान से िलया गया है । यह एक सकारा क
अिधकार है ।
2. िविधयों का समान सं र ण अमे रका के सं िवधान से िलया गया है । यह एक
नकारा क अिधकार है ।
नीचे िदये गए कूटों का योग कर सही उ र का चयन कीिजये :

A) केवल 1

B) केवल 2

C) 1 और 2 दोनों

D) न तो 1 और न ही 2
Hide Answer -
उ रः (d)
ा ाः उपयु दोनों कथन गलत ह।

सं िवधान के अनु े द-14 म कहा गया है िक रा , भारत के रा े म िकसी


को िविध के सम समता से या िविधयों के समान सं र ण से वं िचत नहीं
करे गा।
िविध के सम समता ि टे न के सं िवधान से िलया गया है । यह एक नकारा क
अिधकार है , जबिक िविधयों का समान सं र ण अमे रका के सं िवधान से िलया गया
है । यह एक सकारा क अिधकार है ।

[20]भारतीय सं िवधान के अनु े द-16 के अं तगत िनयोजन के िवषय म अवसर की समता

के अपवाद के अं तगत िन िल खत म से कौन शािमल नहीं है ?

अनु सूिचत जाित एवं जनजाित के िलये रा से वाओं म पदो ित म आर ण की


A)
व था।

B) रा के अधीन से वाओं म िपछड़ा वग के िलये आर ण की व था।

अनु सूिचत जाित, जनजाित एवं िपछड़ा वग के िलये सरकारी, गै र-सरकारी िश ण


C)
सं थानों म आर ण की व था।

धािमक या सां दाियक सं था के ि याकलाप से सं बंिधत िकसी िविश धम/सं दाय


D)
के पदधारी सं बंधी ावधान।

Hide Answer -
उ रः (c)
ा ाः कथन (c) सं बंधी ावधान भारतीय सं िवधान के अनु े द 15(5) के अं तगत
िकया गया है । 93व सं िवधान सं शोधन अिधिनयम के ारा यह ावधान जोड़ा गया।
अ सं क िश ण सं थानों को छोड़कर सभी सरकारी एवं गै र-सरकारी िश ण सं थानों
म अनु सूिचत जाित, जनजाित एवं िपछड़ा वग के आर ण सं बंधी ावधान है ।

अनु े द 15(3) यों एवं ब ों के िलये िवशे ष ावधान की व था करता है ।


कथन (a) सं बंधी ावधान अनु े द-16(4) (क) म िकया गया है । 77व सं िवधान
सं शोधन अिधिनयम 1995 ारा यह ावधान जोड़ा गया है । इसके िलये 82व
सं िवधान सं शोधन अिधिनयम, 2000 भी िकया गया एवं सं िवधान के अनु े द-335
म सं शोधन िकया गया। 85व सं िवधान सं शोधन अिधिनयम, 2001 के ारा पदो ित
म आर ण के साथ पा रमािणक े ता (Consequential Seniority) जोड़ा
गया और इसे भू तल ी भाव (retrospective effect) से लागू िकया गया।
कथन (b) सं बंधी ावधान अनु े द 16(4) म िकया गया है ।
कथन (d) सं बंधी ावधान अनु े द 16(5) म िकया गया है ।
[1] भारतीय सं िवधान के अनु े द-15 म िन िल खत म से िकस कार के िवभे द का
ितषे ध शािमल िकया गया है ?
1. धम

2. मू लवं श
3. जाित
4. िलं ग
5. ज थान
कूटः

A) केवल 1, 2, 3 और 4

B) केवल 1, 2, और 4

C) केवल 2, 3 और 5

D) 1, 2, 3, 4, और 5

Hide Answer -
उ रः (d)
ा ाः

भारतीय सं िवधान के अनु े द-15 म िवभे द का ितषे ध के अं तगत पाँ च आधारों को


शािमल िकया गया है :

1. धम
2. मू लवं श
3. जाित
4. िलं ग
5. ज थान

भारतीय सं िवधान के अनु े द-16 म रा के िनयोजन या पद से सं बंिधत िवभे द


नहीं िदये जाने सं बंधी सात आधार ह:

1. धम
2. मू लवं श
3. जाित
4. िलं ग
5. उ व (descent)
6. ज थान (place of birth)
7. िनवास (residence)

[2] भारतीय सं िवधान के अनु े द-16(4) म िपछड़ा वग के िलये रा की से वाओं म


आर ण का ावधान िकया गया है । इस सं दभ म िन िल खत कथनों पर िवचार कीिजये :
1. िपछड़ा वग का आधार सामािजक, शै िणक एवं आिथक है ।
2. रा की से वाओं म उस वग का ितिनिध पया नहीं है ।
उपयु कथनों म कौन- सा/ से कथन सही है /ह ?
A) केवल 1

B) केवल 2

C) 1 और 2 दोनों

D) न तो 1 और न ही 2

Hide Answer -
उ रः (b)
ा ाः कथन 1 गलत है , ोंिक िपछड़ा वग का आधार शै िणक एवं सामािजक माना
गया है । आिथक आधार को शािमल नहीं िकया गया है । रा की राय म यिद रा की
से वाओं म उस वग का ितिनिध पया नहीं है तो आर ण का ावधान कर सकता
है ।

[3] ीमीले यर के सं दभ म िन िल खत कथनों पर िवचार कीिजये :


1. इसके अं तगत शािमल िपछड़ा वग आर ण का लाभ उठा सकता है ।
2. िपछड़ा वग म ीमीले यर की पहचान के िलये रामनं दन सिमित का गठन िकया गया।
3. िजसके पास एक छूट सीमा से अिधक की सं पि है , ीमीले यर के अं तगत नहीं आते
ह।
उपयु म से कौन-सा/से कथन सही है /ह?

A) केवल 1

B) केवल 2

C) केवल 1 और 3

D) 1, 2 और 3

Hide Answer -
उ रः (b)

ा ाः कथन 1 और कथन 3 गलत ह, ोंिक जो िपछड़ा वग ीमीले यर के अं तगत आते ह,

उ रा की से वाओं म आर ण का लाभ नहीं िमले गा।

िपछड़े वग म ीमीले यर की पहचान के िलये रामनं दन सिमित का गठन िकया गया,

िजसने वष 1993 म अपनी रपोट पे श की िजसे ीकार कर िलया गया।

िजनके पास एक छूट सीमा से अिधक की सं पि है , वे ीमीले यर के अं तगत आएं गे । छूट

की सीमा का िनधारण रा करत ह।

[4] लोक िनयोजन (िनवास सं बंधी शत) अिधिनयम, 1957 के सं दभ म िन िल खत

कथनों पर िवचार कीिजये :


1. यह अिधिनयम भारतीय सं िवधान के अनु े द-16 (2) का अपवाद है ।
2. वतमान म यह व था िकसी रा /क -शािसत दे श म लागू नहीं है ।
उपयु म से कौन-सा/से कथन सही है /ह?
A) केवल 1

B) केवल 2

C) 1 और 2 दोंनों

D) न तो 1 और न ही 2

Hide Answer -
उ रः (a)
ा ाः

लोक िनयोजन (िनवास सं बंधी शत) अिधिनयम, 1957 भारतीय सं िवधान के


अनु े द-16 (2) का अपवाद है । अतः कथन 1 सही है ।
सं सद ारा यह अिधिनयम, 1957 म पा रत िकया गया एवं 1974 म यह
अिधिनयम समा हो गया, परं तु आं दे श एवं ते लंगाना म यह ावधान वतमान म
चल रहा है । 32व सं िवधान सं शोधन अिधिनयम, 1973 के ारा सं िवधान म
अनु े द-371(घ) जोड़ा गया एवं रा पु नगठन अिधिनयम, 2014 ारा इसका
िव ार ते लंगाना तक िकया गया। अतः कथन 2 गलत है ।

[5] ‘समान काय के िलये समान वे तन’ को एक मौिलक अिधकार बनाने के िलये भारतीय
सव ायालय को आधार िमलता है -

A) सं िवधान की ावना म यु समाजवादी श और अनु े द-18 से ।

B) ावना म यु समाजवादी श को सं िवधान के अनु े द-15 से ।

C) ावना म यु समाजवादी श को सं िवधान के अनु े द-17 से ।

D) ावना म यु समाजवादी श , अनु े द-14, 16 एवं 39(d) से ।

Hide Answer -
उ रः (d)
ा ाः उपयु म से िवक (d) सही है ।

ावना म यु समाजवादी श एवं अनु े द-14, 16 एवं 39(d) ने एक साथ


िमलकर समान काय के िलये समान वे तन को मौिलक अिधकार बनाने हे तु भारतीय
सव ायालय को आधार िमलता है ।

[6] िसिवल अिधकार सं र ण अिधिनयम, 1955 के सं दभ म िन िल खत कथनों पर िवचार

कीिजये :
1. इस अिधिनयम का सं बंध भारतीय सं िवधान के भाग-3 म विणत अ ृ ता के अं त
से है ।
2. भारतीय सं िवधान का अनु े द-35 सं सद एवं रा के िवधानमं डल को यह अिधकार
दे ता है िक वह इस कार के अिधिनयम बनाए।
3. अ ृ ता के आचरण से दोषिस सं सद या िकसी रा के िवधानमं डल की
सद ता के िलये यो नहीं रह जाता है ।
उपयु म से कौन-सा/से कथन सही है /ह?

A) केवल 1

B) केवल 1 और 2

C) केवल 1 और 3

D) 1, 2 और 3

Hide Answer -
उ रः (c)
ा ाः उपयु म से कथन 2 गलत है , ोंिक ‘अ ृ ता का अं त’ भारतीय सं िवधान
के अनु े द- 17 (समता का अिधकार) म विणत है तथा इस सं बध म केवल सं सद ही सं िवधान
के अनु े द-35 के तहत िविध बना सकती है । रा के िवधानमं डलों को यह अिधकार नहीं है ।

सं सद ने इस सं बंध म वष 1955 म अ ृ ता (अपराध) अिधिनयम, 1955 पा रत

िकया। वष 1976 म इसका नाम बदलकर ‘िसिवल अिधकार सं र ण अिधिनयम, 1955’

कर िदया गया, िजसम छः माह कैद या 500 पया का आिथक दं ड या िफर दोनों हो

सकता है । वतमान म कैद की सीमा 6 माह से बढ़ाकर 1-2 वष कर दी गई है ।

इस आचरण से दोषिस सं सद या रा िवधानमं डल की सद ता के िलये यो

नहीं रह जाता है ।

सं सद ने वष 1989 म अनु सूिचत जाित एवं जनजाित (उ ीड़न िनवारण) अिधिनयम,

1989 पा रत िकया, जो उपरो दोनों अिधिनयमों से ादा ापक एवं कठोर है । इस

अिधिनयम को एटोिसटीज ए (Atrocities Act) भी कहा जाता है । इस अिधिनयम म

6 माह से 5 वष की सज़ा एवं पीिड़त को ितपू ित का ावधान है ।

अ ृ ता श को न तो सं िवधान म और न ही इस अिधिनयम म प रभािषत िकया गया

है । मै सूर उ ायालय ने व था दी िक इसका योग ऐितहािसक है ।

[7] भारतीय सं िवधान के अनु े द-18 म विणत उपािधयों के अं त के सं दभ म िन िल खत


म से कौन-सा कथन सही नहीं है ?

A) रा , से ना या िव ा सं बंधी स ान के िसवाय और कोई उपािध दान नहीं करे गा।

B) भारत का कोई नाग रक िकसी िवदे शी रा से कोई उपािध ीकार नहीं करे गा।

C) भारत र , प िवभू षण, प भू षण एवं प ी उपािधयाँ नहीं है ।

इसके उ ं घन के िवषय म सं सद ने सं िवधान के अनु े द-35 के अं तगत


D)
अिधिनयम बनाया है ।
Hide Answer -
उ रः (d)

ा ाः उपयु म से िवक (d) गलत है , ोंिक सं सद ने अभी तक इस िवषय म कोई

अिधिनयम नही बनाया है ।

भारतीय सं िवधान के अनु े द-18 के अं तगत ‘उपािधयों का अं त’ (समता के अिधकार)

सं बंधी ावधान िकया गया है ।

वष 1954 म भारत सरकार ने चार अलं कार दे ने की व था शु की, ये अलं कार ह:

भारत र , प िवभू षण, प भू षण एवं प ी। ायालय के अनु सार ये पु र ार/उपािधयाँ

नहीं ह। वष 1977 म जनता दल की सरकार ने इन पु र ारों पर ितबं ध लगा िदया था।

पु नः 1980 म इं िदरा गां धी की सरकार ने इसे लागू कर िदया।

[8] भारतीय सं िवधान के अनु े द-19(i) म वाक्- तं ता आिद िवषय से सं बंिधत


अिधकार िदये गए ह। िन िल खत म से कौन सा/से अिधकार इसके अ गत शािमल
नहीं है /ह?
1. िवदे श या ा करने का अिधकार।
2. सं गम या सं घ या सहकारी सिमित बनाने का अिधकार।
3. िश ा का अिधकार।
नीचे िदये गए कूट का योग कर सही उ र का चयन कीिजये :

A) केवल 1

B) केवल 3

C) केवल 1 एवं 3

D) केवल 2 एवं 3

Hide Answer -
उ र: (c)
ा ाः उपयु म से कथन 1 एवं 3 गलत ह।
िवदे श या ा करने का अिधकार भारतीय सं िवधान के अनु े द-21 एवं िश ा का अिधकार
अनु े द-21(क) म ावधान िकये गए ह न िक अनु े द-19(i) म।
अनु े द-19(i) म वतमान म छः अिधकार ह:
1. वाक् एवं अिभ की तं ता का अिधकार।
2. शा पू वक एवं िनरायु स े लन का अिधकार।
3. सं गम या सं घ या सहकारी सिमित बनाने का अिधकार।
4. भारत के रा े म सव अबाध सं चरण (घू मने ) का अिधकार।
5. भारत के रा े के िकसी भी भाग म िनवास करने एवं बस जाने का अिधकार।
6. कोई वृ ि (Profession), उपजीिवका (Occupation), ापार (Trade) या
कारोबार (Business) करने का अिधकार।
Note:- अनु े द 19(i) (च) म स ि को खरीदने , अिध हण करने या बे च दे ने का
था, िजसे 44व सं िवधान सं शोधन 1978 ारा समा कर िदया गया।
[9] भारतीय सं िवधान के अनु े द- 19(i) म प से शािमल तो नहीं है , पर ु
अ िनिहत है । िन िल खत म से कौन सा/से अिधकार इसम अ िनिहत है /ह?
1. े स की तं ता।
2. ावसाियक िव ापन की तं ता।
3. हड़ताल करने का अिधकार।
नीचे िदये गए कूट का योग कर सही उ र का चयन कीिजये :

A) केवल 1

B) केवल 1 एवं 2

C) केवल 1 एवं 3

D) 1, 2 एवं 3

Hide Answer -
उ र: (b)
ा ाः उपयु म से कथन 3 गलत है ।
भारतीय सं िवधान के अनु े द-19(i) (क) के अ गत दशन एवं िवरोध करने का
अिधकार तो अं तिनिहत है , पर ु हड़ताल करने का अिधकार अं तिनिहत नहीं है ।
अनु े द-19(i) (क) के अ गत िन िल खत अिधकार अं तिनिहत ह:
1. े स की तं ता/समाचार प ों की तं ता
2. ावसाियक िव ापन की तं ता आिद।

[10]भारत के सं िवधान म मू ल अिधकार के सं दभ म िन िल खत कथनों पर िवचार

कीिजएः
1. भारत का सं िवधान अनु े द- 19 (i) पर यु यु ितबं ध लगाने का
ावधान करता है ।
2. सहकारी सिमित बनाने का अिधकार सं िवधान के अनु े द-19 (i) म मू ल
प से नहीं था।
नीचे िदये गए कूट का योग कर सही उ र का चयन कीिजये :

A) केवल 1

B) केवल 2

C) 1 और 2 दोनों

D) न तो 1 और न ही 2
Hide Answer -
उ रः (c)

ा ाः उपयु दोनों कथन सही ह। भारतीय सं िवधान के अनु े द- 19(i) म छः मू ल

अिधकार िदये गए ह, पर ु इन पर यु यु ितबं ध भी लगाया गया है ः

अनु े द-19 (i) के अ गत िदये गए छः मू ल अिधकार यु यु ितबं ध सं बंधी अनु ेद

अनु े द-19 (i) (क) अनु े द-19(2)

अनु े द-19 (i) (ख) अनु े द-19(3)

अनु े द-19 (i) (ग) अनु े द-19(4)

अनु े द-19 (i) (घ) अनु े द-19(5)

अनु े द-19 (i) (ड॰) अनु े द-19(5)

अनु े द-19 (i) (छ) अनु े द-19(6)

सहकारी सिमित बनाने का अिधकार अनु े द-19(i) (ग) म मू ल प से नहीं था, िजसे बाद म

97वाँ सं िवधान सं शोधन 2011 के ारा जोड़ िदया गया।

[11]नाग रक के िवचार एवं अिभ की तं ता पर यु यु ितबं ध लगाया गया है ।


िन िल खत म से कौन इसके अ गत शािमल ह?
1. भारत की स भु ता एवं अख ता
2. िवदे शी रा ों के साथ मै ीपू ण सं बंध
3. ायालय की अवमानना
4. अवां छनीय आलोचना
5. लोक व था
नीचे िदये गए कूट का योग कर सही उ र का चयन कीिजये :

A) केवल 1

B) केवल 1 एवं 2

C) केवल 1, 3, 4 एवं 5

D) केवल 1, 2, 3 एवं 5

Hide Answer -
उ रः (d)
ा ाः िवचार एवं अिभ की तं ता पर यु यु ितबं ध म अवां छनीय
आलोचना शािमल नहीं है । इसके अ गत 1, 2, 3 एवं 5 के अलावा रा की सु र ा,
अपराध-उ ीपन, सदाचार/िश ाचार, आिद शािमल ह।
[12]भारतीय सं िवधान के अनु े द 19 (i) (ख) म शा पू वक एवं िनरायु स े लन करने
सं बंधी अिधकार िदया गया है । इस सं दभ म िकसे शािमल नहीं िकया जा सकता है ?

A) एकि त जनता, िजसे हिथयार नहीं रखना चािहये ।

B) शा पू ण इक ा होना।

C) केवल सावजिनक भू िम पर होना चािहये ।

D) इसम हड़ताल का अिधकार शािमल है ।

Hide Answer -
उ रः (d)

ा ाः शा पू वक एवं िनरायु स े लन के अिधकार म हड़ताल करने का अिधकार शािमल

नहीं है । इसके अ गत िबना हिथयार के शा पू वक सं गिठत होने का नाग रकों को अिधकार

िदया गया है । जै सेः- सावजिनक बै ठकों म भाग ले ने का अिधकार एवं दशन, केवल सावजिनक

भू िम पर िबना हिथयार के स े लन आिद। रा , अनु े द 19(3) के तहत इन पर यु यु

ितबं ध लगा सकता है ।

[13] भारतीय सं िवधान के भाग-3 म ाण एवं दै िहक तं ता के सं र ण सं बंधी ावधान


िकया गया है । इस सं दभ म िन िल खत म से कौन-सा कथन सही है ?

A) इस मू ल अिधकार म िविध की स क ि या के िस ा को अपनाया गया है ।

B) यह मू ल अिधकार सभी यों को ा नहीं है ।

िविध ारा थािपत ि या के आधार पर भी िकसी को उसके ाण या


C)
दै िहक तं ता से वं िचत नहीं िकया जा सकता है ।

मा एक मू ल अिधकार है , िजसे आपातकाल म भी िनल त नहीं िकया जा सकता


D)
है ।

Hide Answer -
उ रः (a)
ा ाः उपयु म से िवक (a) सही है , ोंिक ाण एवं दै िहक तं ता का सं र ण
(अनु े द-21) म िविध की स क ि या के िस ा को अपनाया गया है । मे िनका
गाँ धी बनाम भारत सं घ, 1978 म ायालय ने िनणय िदया िक उिचत एवं ायपू ण आधार
पर ही इससे वं िचत िकया जा सकता है ।
यह मू ल अिधकार सभी यों (नाग रकों एवं िवदे िशयों) को ा है ।
केवल िविध ारा थािपत ि या के आधार पर ही िकसी को उसके ाण एवं
दै िहक तं ता से वं िचत िकया जा सकता है । अनु े द-20 एवं 21 दो ऐसे मू ल अिधकार
ह, िज आपातकाल म भी िनलं िबत नहीं िकया जा सकता है ।

[14]
िश ा के अिधकार के िवषय म िन िल खत कथनों पर िवचार कीिजएः
1. यह एक मू ल अिधकार है ।
2. इसे सं िवधान के अनु े द-19 के साथ जोड़ा गया है ।
3. रा 8-14 वष की आयु के ब ों को िनःशु एवं अिनवाय िश ा उपल
कराएगा।
4. 93व सं िवधान सं शोधन 2002 के ारा इसे मू ल अिधकार बनाया गया।
उपयु म से कौन-सा/से कथन सही है /ह?

A) केवल 1

B) केवल 1 और 2

C) केवल 1 और 3

D) केवल 1 एवं 4

Hide Answer -
उ रः (a)
ा ाः उपयु म से केवल कथन-1 सही है ।

िश ा के अिधकार को अनु े द-21(क) के साथ जोड़ा गया है ।


िश ा के अिधकार म 6-14 वष की आयु वग के ब ों के िलये िनःशु एवं
अिनवाय िश ा का ावधान िकया गया है न िक 8-14 वष की आयु वग के ब ों
के िलये ।
86व सं िवधान सं शोधन 2002 के ारा इसे मौिलक अिधकार घोिषत िकया गया न
िक 93व सं िवधान सं शोधन के ारा।

[15]िन िल खत म से कौन-सा कथन सही नहीं है ?

भारतीय सं िवधान के अनु े द-45 म कहा गया है िक रा छः वष से कम आु य


A)
के सभी ब ों के िलये िश ा दान करने का यास करे गा।

भारतीय सं िवधान के अनु े द-51(क) म ावधान िकया गया है िक माता-िपता या


B)
सं र क 6-14 वष की आयु वग के अपने ब े को िश ा के अवसर दान करे गा।

िनःशु एवं अिनवाय िश ा दान करने के िलये सं सद ारा िश ा का अिधकार


C)
सं बंधी अिधिनयम पा रत िकया गया।

D) िश ा का अिधकार अिधिनयम 1 अ ै ल 2009 से भावी हो गया है ।


Hide Answer -
उ रः (d)
ा ाः उपयु म से िवक (d) गलत है , ोंिक िश ा का अिधकार (RTE)
अिधिनयम 1 अ ै ल 2010 से भावी आ है न िक 1 अ ै ल 2009 से ।

िवक (a) सं बंधी उपबं ध 86व सं िवधान सं शोधन 2002 के ारा सं िवधान के

अनु े द-45 म जोड़ा गया है ।

िवक (b) सं बंधी उपबं ध 86व सं िवधान सं शोधन के ारा भारतीय सं िवधान के
अनु े द-51(क) (मू ल क म) म 11व मू ल क के प म जोड़ा गया है ।
6-14 वष की आयु के सभी ब ों को िनःशु एवं अिनवाय िश ा दान करने के
िलये िश ा का अिधकार अिधिनयम, 2009 सं सद ारा पा रत िकया गया है ।
िश ा का अिधकार एक मानव अिधकार भी है ।

[16]िन िल खत म से कौन-सा कथन सही नहीं है ?

भारतीय सं िवधान के अनु े द-45 म कहा गया है िक रा छः वष से कम आु य


A)
के सभी ब ों के िलये िश ा दान करने का यास करे गा।

भारतीय सं िवधान के अनु े द-51(क) म ावधान िकया गया है िक माता-िपता या


B)
सं र क 6-14 वष की आयु वग के अपने ब े को िश ा के अवसर दान करे गा।

िनःशु एवं अिनवाय िश ा दान करने के िलये सं सद ारा िश ा का अिधकार


C)
सं बंधी अिधिनयम पा रत िकया गया।

D) िश ा का अिधकार अिधिनयम 1 अ ै ल 2009 से भावी हो गया है ।

Hide Answer -
उ रः (d)
ा ाः उपयु म से िवक (d) गलत है , ोंिक िश ा का अिधकार (RTE)
अिधिनयम 1 अ ै ल 2010 से भावी आ है न िक 1 अ ै ल 2009 से ।

िवक (a) सं बंधी उपबं ध 86व सं िवधान सं शोधन 2002 के ारा सं िवधान के
अनु े द-45 म जोड़ा गया है ।
िवक (b) सं बंधी उपबं ध 86व सं िवधान सं शोधन के ारा भारतीय सं िवधान के
अनु े द-51(क) (मू ल क म) म 11व मू ल क के प म जोड़ा गया है ।
6-14 वष की आयु के सभी ब ों को िनःशु एवं अिनवाय िश ा दान करने के
िलये िश ा का अिधकार अिधिनयम, 2009 सं सद ारा पा रत िकया गया है ।
िश ा का अिधकार एक मानव अिधकार भी है ।

[17]भारतीय सं िवधान के अनु े द-22 यों को कुछ दशाओं म िगर ारी एवं िनरोध से
सं र ण का अिधकार दान करता है । इस सं दभ म िन िल खत म से कौन-सा कथन
सही है ?
िगर ारी के 24 घं टे की अविध म िनकटतम मिज े ट के सम पे श िकया
A)
जाएगा, िजसम या ा म लगा आव क समय भी शािमल है ।

िनवारक िनरोध िवषय सं बंधी िवधे यक बनाने की श रा के िवधानमं डलों को


B)
ा नहीं है ।

इस सं बध म सं सद ारा आतं कवाद िनवारण अिधिनयम (POTA), 2002 पा रत


C)
िकया गया, पर ु वतमान म यह लागू नहीं है ।

िनवारक िनरोध िवषय सं बंधी िवधे यक के तहत िगर ार िकये गए िकसी को


D)
एक माह की अविध से अिधक िहरासत म नहीं रखा जा सकता है ।

Hide Answer -
उ रः (c)
ा ाः िवक (c) सही है , ोंिक सं सद ारा िनवारक िनरोध अिधिनयम के तहत
आतं कवाद िनवारण अिधिनयम (POTA), 2002 पा रत िकया, पर ु 2004 म इसे
समा कर िदया गया। अतः वतमान म यह कानू न लागू नहीं है ।

सं िवधान के अनु े द-22 के तहत िगर ार िकये गए िकसी को 24 घं टे के


अ र िनकटतम मिज े ट के सम पे श िकया जाएगा, िजसम या ा म लगे
आव क समय को शािमल नहीं िकया जाता है ।
िनवारक िनरोध िवषय सं बंधी िवधे यक सं सद एवं रा के िवधानमं डल दोनों बना
सकते ह, पर ु दोनों के बीच श यों का िवभाजन िकया गया है ।
िनवारक िनरोध िवधे यक के तहत िगर ार िकये गए िकसी को अिधकतम
तीन माह िहरासत म रखा जा सकता है । इससे अिधक के िलये ‘सलाहकार बोड’ का
िनणय आव क है । सलाहकार बोड म एक अ एवं दो सद होंगे। अ
उ ायालय का से वारत ायाधीश होगा एवं सद उ ायालय के से वारत या
से वािनवृ त ायाधीश होंगे।

[18]भारतीय सं िवधान के अनु े द-19(i) (घ) को अनु े द-21 के साथ िमलाकर पढ़ने पर
िन िल खत म से कौन सा अिधकार ा होता है ?

A) सू चना का अिधकार

B) िवदे श या ा करने का अिधकार

C) एकां तता का अिधकार

D) आ य पाने का अिधकार

Hide Answer -
उ रः (c)
ा ाः खड़क िसं ह बनाम उ र दे श एवं गोिव बनाम म दे श मामले म सव
ायालय ने व था दी िक अनु े द-19(i) (घ) (भारत के रा े म सव अबाध
सं चरण का अिधकार) को अनु े द-21 (जीवन का अिधकार) से िमलाकर पढ़ने पर
एकां तता/िनजता का अिधकार (Right of privacy) ा होता है ।
[19] सू चना का अिधकार (Right to Information) िविधक अिधकार के साथ-साथ मू ल
अिधकार भी है । भारतीय सं िवधान के िकस अनु े द के अ गत इसे मू ल अिधकार माना
गया है ?

A) अनु े द-20

B) अनु े छ-22

C) अनु े द-19

D) अनु े द-21(क)

Hide Answer -
उ रः (c)
ा ाः उपयु म से िवक c सही है ।
इ यन ए े स बनाम भारत सं घ मामले म ायमू ित वकटचे लैया ने ीकार िकया था
िक िवचार एवं अिभ की तं ता [अनु े द-19(i) (क)] म सू चना ा का
अिधकार शािमल है ।
सू चना अिधकार अिधिनयम-2005 के अ गत कहा गया है िक े क नाग रक का यह
िविधक अिधकार है िक वह सावजिनक सं थानों से आव क सू चना मां ग सके।

[20]मानव दु ापार एवं बलात- म का ितषे ध के सं दभ म िन िल खत म से कौन-सा कथन


सही नहीं है ?

A) यह एक मू ल अिधकार है , िजसकी चचा शोषण के िव अिधकार म की गई है ।

B) यह अिधकार नाग रक एवं गै र-नाग रक दोनों को ा है ।

C) भारतीय सं िवधान के अनु े द-24 म इसकी चचा की गई है ।

इसका एक अपवाद है िक रा सावजिनक उ े ों के िलये अिनवाय से वा


D)
आरोिपत कर सकता है ।
Hide Answer -
उ रः (c)
ा ाः िवक (c) सही नहीं है , ोंिक मानव दु ापार एवं बला म की चचा भारतीय
सं िवधान के अनु े द-23 के अं तगत की गई है न िक अनु े द-24 के अं तगत।
कथन (a) सही है , ोंिक शोषण के िव अिधकार एक मू ल अिधकार है , िजसकी
चचा भारतीय सं िवधान के अनु े द 23-24 म की गई है । अनु े द 23(1) म मानव
दु ापार एवं बला म का ितषे ध िकया गया है ।

मानव दु ापार के सं बंध म सं सद ने अनै ितक दु ापार (िनवारण) अिधिनयम,


1956 पा रत िकया है ।
मानव दु ापार के अं तगत पु ष, मिहला एवं ब ों को शािमल िकया गया है ।
यह अिधकार नाग रक एवं गै र-नाग रक दोनों को ा है ।
अनु े द- 23(2) बे गार/बला म का अपवाद है । इसम ावधान िकया गया है िक
रा सावजिनक उ े ों के िलये अिनवाय से वा आरोिपत कर सकता है ।
बलात म को ितबं ध करने के िलये सं सद ने कई कानू न बनाए ह:
- बं धुआ मज़दू री व था (िनरसन) अिधिनयम, 1976
- ू नतम मज़दू री अिधिनयम, 1948
- ठे का िमक अिधिनयम, 1970
- समान पा र िमक अिधिनयम, 1976
[1] भारतीय सं िवधान का अनु े द- 23(2) मानव दु ापार एवं बलात् म का
अपवाद है । रा सावजिनक उ े ों के िलये अिनवाय से वा आरोिपत कर सकता
है , परं तु ऐसा करते समय रा िन िल खत आधारों पर िवभे द नही ं करे गा:
1. धम
2. मू लवं श
3. जाित
4. वग
5. िनवास थान
6. िलं ग
उपयु म से कौन से आधार को इसम शािमल नही ं िकया गया है ?

A) केवल 1, 4 और 5

B) केवल 2, 4 और 5

C) केवल 5 और 6

D) केवल 4, 5 और 6

Hide Answer -
उ रः (c)

ा ाः िनवास थान एवं िलं ग को अनु े द 23(2) म शािमल नहीं िकया गया है ।

इस अनु े द म कहा गया है िक रा सावजिनक उ े ों के िलये अिनवाय से वा आरोिपत कर

सकता है , परं तु रा केवल धम, जाित, मू लवं श या वग के आधार पर िवभे द नहीं करे गा।

[2] कारखानों म बालकों के िनयोजन का ितषे ध िकया गया है । इस सं दभ म िन िल खत


कथनों पर िवचार कीिजयेः
1. 18 वष से कम आयु वग के िकसी बालक को िकसी कारखाने या खान म िनयोिजत
नहीं िकया जा सकता।
2. भारतीय सं िवधान के अनु े द-23 म इसकी चचा की गई है ।
3. इसे शोषण के िव अिधकार म शािमल िकया गया है ।
4. शोषण के िव अिधकार गै र-नाग रक को ा नहीं है ।
उपयु म से कौन सा/से कथन सही है /ह?

A) केवल 1

B) केवल 1, 2 और 3

C) केवल 3

D) केवल 3 और 4
Hide Answer -
उ रः (c)
ा ाः केवल कथन (3) सही है , ोंिक शोषण के िव अिधकार की चचा भारतीय
सं िवधान के अनु े द 23-24 म की गई है ।
अनु े द- 24 म कारखानों आिद म बालकों के िनयोजन का ितषे ध िकया गया है ,
जबिक अनु े द-23 म मानव दु ापार एवं बला म का िनषे ध है ।
कथन (1) गलत है , ोंिक 18 वष से कम आयु के थान पर 14 वष से कम आयु
होगी।
कथन (4) गलत है , ोंिक शोषण के िव अिधकार नाग रक एवं गै र-नाग रक दोनों
के ा है ।

[3] बाल म (िनषे ध एवं िविनयमन) सं शोधन अिधिनयम, 2016 के सं दभ म


िन िल खत कथनों पर िवचार कीिजयेः
1. यह अिधिनयम 14 वष से कम आयु के सभी ब ों को सभी वसायों एवं
ि याओं म रोज़गार िनिष करता है ।
2. इस अिधिनयम के उ ं घन पर छः माह से ले कर दो वष तक के कैद का
ावधान है ।
उपयु म से कौन सा/से कथन सही है /ह?

A) केवल 1

B) केवल 2

C) 1 और 2 दोनों

D) न तो 1 और न ही 2

Hide Answer -
उ रः (c)

ा ाः उपयु दोनों कथन सही ह।

बाल म (िनषे ध एवं िविनयमन) सं शोधन अिधिनयम, 2016 पा रत िकया गया। इसके

ारा बाल म (िनषे ध एवं िविनयमन) अिधिनयम, 1986 म सं शोधन िकया गया। इस

सं शोधन ारा इसका नाम बदलकर बाल एवं िकशोर म (िनषे ध एवं िविनयमन)

अिधिनयम, 1986 कर िदया गया है ।

इसम ावधान िकया गया है िक 14 वष से कम आयु के सभी ब ों का सभी वसायों एवं

ि याओं म रोज़गार िनषे ध है । इसके पहले यह िनषे ध 18 वसायों एवं 65 ि याओं

पर लागू था।

14-18 वष की आयु वग के ब ों को खतरनाक वसायों एवं ि याओं म रोज़गार

िनषे ध का ावधान िकया गया है ।

इसका उ ं घन करने पर 6 माह से 2 वष तक की कैद अथवा 20,000 से 50,000

तक के जु माने का ावधान िकया गया है । अपराध दोहराने पर कैद की अविध 1-3 वष

की हो सकती है ।
[4] भारतीय सं िवधान के अनु े द-25 म यु ‘िह दू’ श के अ गत िन िल खत
म से िकसे शािमल िकया गया है ?
1. बौ
2. जै न
3. िस
4. पारसी
नीचे िदये गए कूट का योग कर सही उ र का चयन कीिजये :
कूटः

A) केवल 1 और 3

B) केवल 1, 2 और 3

C) केवल 1, 3 और 4

D) उपयु सभी

Hide Answer -
उ रः (b)

ा ाः भारतीय सं िवधान के अनु े द-25 के अ गत यु ‘िह दू’ श के अ गत िस ,

जै न या बौ धम को शािमल िकया गया है । इसके अ गत पारसी को शािमल नहीं िकया गया

है ।

[5] भारतीय सं िवधान के अनु े द-25 के अं तगत िन िल खत म से कौन-सा/से कथन


शािमल है /ह?
1. अ ःकरण की तं ता एवं धम को अबाध प से मानने , आचरण करने एवं चार
करने का अिधकार।
2. यह अिधकार नाग रक एवं गै र नाग रक दोनों को ा है ।
3. कृपाण धारण करना एवं ले कर चलना िस धम को मानने का अं ग समझा जाएगा।
नीचे िदये गए कूट का योग कर सही उ र का चयन कीिजये :

A) केवल 1

B) केवल 2

C) केवल 1 एवं 2

D) 1, 2 एवं 3

Hide Answer -
उ रः (d)

ा ाः उपयु सभी कथन भारतीय सं िवधान के अनु े द-25 के अ गत शािमल ह।

अनु े द-25(2) के अ गत रा को इस सं बंध म िविध बनाने का अिधकार िदया गया

है ।
[6] भारतीय सं िवधान का अनु े द-25 अ ःकरण की तं ता, धम को अबाध प से
मानने , आचरण और चार करने की तं ता दान करता है , ऐसी धािमक तं ता
िन िल खत म िकसके अधीन रहते ए ा होगीः
1. लोक व था
2. ा
3. सदाचार
4. मानववाद
नीचे िदये गए कूट का योग कर सही उ र का चयन कीिजये :

A) केवल 1

B) केवल 1 और 3

C) केवल 1, 2 और 3

D) केवल 1, 3 और 4

Hide Answer -
उ रः (c)

ा ाः सं िवधान के अनु े द-25 म कहा गया है िक लोक व था, सदाचार एवं ा के

अधीन रहते ए सभी यों को अ ःकरण की तं ता और धम के अबाध प से मानने ,

आचरण करने एवं चार करने का समान अिधकार होगा। मानववाद इसम शािमल नहीं है ।

[7] िन िल खत कथनों पर िवचार कीिजयेः

1. जं गम (Movable) और थावर (Immovable) स ि के अजन एवं


ािम का अिधकार।
2. धािमक सं थाओं की थापना एवं पोषण का अिधकार।
3. रा से मा ता ा शै ि क सं थान म धािमक िश ा या उपासना म
उप थत होने से तं ता।
4. रा -िनिध से पू णतः पोिषत िकसी शै ि क सं थान म धािमक िश ा दे ने पर
रोक।
उपयु कथनों म से कौन-सा/से कथन भारतीय सं िवधान के अनु े द-26 के
सं दभ म सही है /ह?

A) केवल 1 और 2

B) केवल 2

C) केवल 1, 2 और 3

D) केवल 2, 3 और 4
Hide Answer -
उ रः (a)
ा ाः उपयु म से कथन (1) और (2) का भारतीय सं िवधान के अनु े द-26 म
ावधान िकया गया ह। अनु े द-26 धािमक काय के बं धन की तं ता दान करता
है । इसम िन अिधकार शािमल ह:

► धािमक सं थाओं की थापना एवं पोषण।


► अपने धम िवषय काय का बं धन।
► जं गम एवं थावर स ि का अजन एवं ािम ।
► ऐसी स ि का िविध के अनु सार शासन।

कथन (3) एवं (4) सं बंधी ावधान भारतीय सं िवधान के अनु े द-28 म िकये गए
ह। इस अनु े द म िन ावधान ह-

► रा -िनिध से पू णतः पोिषत िकसी िश ा सं था,म कोई धािमक िश ा नही दी


जाएगी।
► ऐसी िश ा सं था, िजसका शासन रा करता है ,पर ु ऐसे िव ास या ास
के अधीन थािपत ई है ,िजसके अनु सार उस सं था म धािमक िश ा दे ना
आव क है , धािमक िश ा दी जा सकती है ।
► रा से मा ता ा या रा -िनिध से सहायता ा िश ा सं था म धािमक
िश ा या उपासना म उप थत होने के िलये बा नहीं िकया जा सकता, जब तक
िक उसने या उसके सं र क (अवय होने की थित म) ने सहमित नहीं दे दी
है ।
►भारतीय सं िवधान का अनु े द-27 ावधान करता है िक िकसी भी को
िकसी िविश धम या धािमक सं दाय की अिभवृ या पोषण म य करने के
िलये करों के सं दाय (Payment of taxes) हे तु बा नहीं िकया जाएगा।

[8] धािमक तं ता के अिधकार के ावधानों के अं तगत िन िल खत म कौन स िलत ह:


1. धम चार करने का अिधकार।
2. िस ों को कृपाण धारण करने एवं रखने का अिधकार।
3. रा ों को समाज-सु धारक िविध िनमाण का अिधकार।
4. धािमक िनकायों को लोगों का धम प रवतन कराने का अिधकार।
नीचे िदये गए कूट का योग कर सही उ र का चयन कर।

A) केवल 1, 2 और 3

B) केवल 2, 3 और 4

C) केवल 3 और 4

D) उपयु सभी
Hide Answer -
उ रः (a)

ा ाः धम की तं ता का अिधकार का वणन भारतीय सं िवधान के अनु े द 25-28 तक

िकया गया है । अनु े द-25 के अ गत धम चार करने , िस ों को कृपाण धारण करने एवं

रखने एवं रा ों को समाज-सु धारक िविध िनमाण का अिधकार शािमल है ।

यह अनु े द िकसी भी धािमक िनकाय को लोगों का धम प रवतन कराने का अिधकार नहीं

दे ता है ।

[9] भारतीय सं िवधान के अनु े द-29 म ावधान िकया गया है िक रा ारा पोिषत
या रा -िनिध से सहायता ा िकसी िश ण सं था म वे श से िकसी भी
नाग रक को िन िल खत आधार पर वं िचत नहीं िकया जाएगा। इसम शािमल ह:
1. धम
2.जाित
3. मू लवं श
4. भाषा
5. िलं ग
नीचे िदये गए कूट का योग कर सही उ र का चयन कीिजये :

A) केवल 1, 2 और 3

B) केवल 1, 2, 3 और 4

C) केवल 1, 2, 3 और 5

D) उपरो सभी

Hide Answer -
उ रः (b)

ा ाः भारतीय सं िवधान के अनु े द-29(2)म ावधान िकया गया है िक रा ारा पोिषत या

रा -िनिध से सहायता ा िकसी िश ा सं था म वे श से िन आधार पर वं िचत नहीं िकया

जाएगा:

मू लवं श

जाित

धम

भाषा

इसम िलं ग शािमल नहीं है ।

[10]भारतीय सं िवधान मा ता दे ता है :
A) केवल धािमक अ सं कों को।

B) केवल भाषायी अ सं कों को।

C) धािमक एवं भाषायी अ सं कों को।

D) धािमक, भाषायी एवं नृ जातीय अ सं कों को।

Hide Answer -
उ रः (c)

ा ाः भारतीय सं िवधान के अनु े द-30 म भाषायी एवं धािमक अ सं क वग की चचा की

गई है , हालां िक अ सं क श को सं िवधान म कहीं भी प रभािषत नहीं िकया गया है ।

इसम नृ जातीय अ सं क-वग की चचा नहीं है ।

[11]िन िल खत सू ची-I को सू ची-II से सु मेिलत कीिजये :

सू ची-II
सू ची-I (भारतीय सं िवधान का ावधान ) (अनु े द)

A. रा ारा पोिषत या रा -िनिध से सहायता ा िश ा


सं था म िकसी भी नाग रक को केवल धम, मू लवं श, जाित 1. अनु ेद
भाषा या उनम से िकसी एक के आधार पर वं िचत नहीं िकया 15(1)
जाएगा।

B. रा के अधीन िकसी िनयोजन या पद के सं बंध म केवल


2. अनु ेद
धम, मू लवं श, जाित, िलं ग, उ व, ज थान िनवास या उनम
23(2)
से िकसी एक के आधार पर िवभे द नहीं िकया जाएगा।

C. रा , साविजनक उ े के िलए अिनवाय से वा आरोिपत 3. (4)


करते समय केवल धम, मू लवं श, जाित या वग या इनम से अनु ेद
िकसी एक के आधार पर िवभे द नहीं करे गा। 29(2)

D. रा , िकसी नाग रक के िव केवल धम, मू लवं श, 4. (4)


जाित, िलं ग, ज थान या इनम से िकसी के आधार पर कोई अनु ेद
िवभे द नहीं करे गा। 16(2)

नीचे िदये गए कूट का योग कर सही उ र का चयन कीिजये :


कूटः
A B C D

A) 3 4 2 1

B) 3 2 4 1

C) 1 2 3 4

D) 1 3 2 4
Hide Answer -
उ रः (a)

ा ा: उपयु म से िवक a सही है ।

उपयु सू ची का सही सु मेलन िन कार है :

रा ारा पोिषत या रा -िनिध से सहायता ा िश ा सं था म िकसी भी नाग रक को

केवल धम, मू लवं श, जाित भाषा या उनम से िकसी एक के आधार पर वं िचत नहीं िकया

जाएगा।- अनु े द 29(2)

रा के अधीन िकसी िनयोजन या पद के सं बंध म केवल धम, मू लवं श, जाित, िलं ग,

उ व, ज थान िनवास या उनम से िकसी एक के आधार पर िवभे द नहीं िकया जाएगा।-

अनु े द 16(2)

रा , साविजनक उ े के िलए अिनवाय से वा आरोिपत करते समय केवल धम, मू लवं श,

जाित या वग या इनम से िकसी एक के आधार पर िवभे द नहीं करे गा।- अनु े द 23(2)

रा , िकसी नाग रक के िव केवल धम, मू लवं श, जाित, िलं ग, ज थान या इनम से

िकसी के आधार पर कोई िवभे द नहीं करे गा।- अनु े द 15(1)

[12]िन िल खत सू ची-I को सू ची-II से सु मेिलत कीिजये तथा नीचे िदये गये कूट का

योग कर सही उ र का चयन कीिजये :

सू ची-II(अनु े द)
सू ची-I(भारतीय सं िवधान का ावधान)

A. अ सं क वग को अपनी िवशे ष भाषा, िलिप या


1. अनु े द 30(1)
सं ृ ित को बनाए रखने का अिधकार।

B. धम या भाषा पर आधा रत सभी अ सं क-वग को


अपनी िच की िश ा सं थाओं की थापना एवं शासन 2. अनु े द 29(1)
का अिधकार।

C. माशल लॉ/ से ना िविध लागू होने पर मू ल अिधकारों


3. अनु े द 33
पर ितबं ध सं बंधी ावधान।

D. सश बलों, अधसै िनक बलों, पु िलस बलों, खु िफया


एजिसयों आिद के मू ल अिधकारों पर यु यु ितबं ध 4. अनु े द 34
सं बंधी ावधान।

कूटः
A B C D

A) 1 2 3 4

B) 1 3 2 4

C) 2 1 4 3

D) 2 4 1 3
Hide Answer -
उ रः (c)
ा ाः ा ा:उपयु म से िवक c सही है ।

भारतीय सं िवधान का अनु े द-29 अ सं क-वग के िहतों का सं र ण सं बंधी


ावधान करता है । अनु े द 29(1) म ावधान है िक इसे अपनी िवशे ष भाषा,
िलिप या सं ृ ित को बनाए रखने का अिधकार होगा।
अनु े द-30 अ सं क-वग को िश ण सं थाओं की थापना एवं शासन करने
का अिधकार दान करता है । अनु े द-30(i) म ावधान है िक धम या भाषा पर
आधा रत सभी अ सं क वग को अपनी िच की िश ण सं थाओं की थापना
एवं शासन का अिधकार होगा।
अनु े द 30(2) म ावधान है िक रा िश ा सं थाओं को सहायता दे ने म कोई
िवभे द नहीं करे गा।
अनु े द-33 सं सद को यह अिधकार दे ता है िक वह सश बलों, अधसै िनक बलों,
पु िलस बलों, खु िफया एजिसयों आिद के मू ल अिधकारों पर यु यु ितबं ध लगा
सकेगा।
इस सं बंध म सं सद ारा िन िल खत कानू न बनाए गए ह:

-सै अिधिनयम, 1950


-नौसे ना अिधिनयम, 1950
-वायु सेना अिधिनयम, 1950
-पु िलसबल (अिधकारों पर िनषे ध) अिधिनयम, 1966
-सीमा सु र ा बल अिधिनयम आिद।

अनु े द-34 सं सद को यह अिधकार दे ता है िक दे श म जहाँ कहीं भी माशल लॉ/


से ना िविध लागू है , मू ल अिधकारों पर ितबं ध लगा सकता है ।

[13]भारतीय सं िवधान के भाग-3 म मू ल अिधकार का ावधान है । इस सं दभ म िन िल खत

म से कौन-सा कथन सही है ?

A) सं ृ ित एवं िश ा सं बंधी अिधकार सभी यों को ा है ।

अनु े द-35 सं सद एवं रा के िवधानमं डलों को इस सं बंध म कानू न बनाने


B)
का अिधकार दे ता है ।

अनु े द-33 के अ गत सं सद ारा बनाई गई िविध को िकसी ायालय म


C)
चु नौती नहीं दी जा सकती है ।

अनु े द-34 के अ गत सं सद ारा बनाए गए ितपू ित अिधिनयम को इस


D) आधार पर ायालय म चु नौती दी जा सकती है िक यह अिधिनयम िकसी मू ल
अिधकार का उ ं घन करता है ।
Hide Answer -
उ रः (c)

ा ाः उपयु म से िवक c सही है ।

भारतीय सं िवधान के अनु े द 29-30 तक सं ृ ित एवं िश ा सं बंधी अिधकार िदये गए

ह। यह अिधकार केवल नाग रकों को ा ह, गै र-नाग रकों को नहीं। यह अनु ेद

अ सं क वग के िहतों का सं र ण करता है ।

अनु े द-35 के अधीन केवल सं सद िविध बना सकता है । रा के िवधानमं डलों को यह

अिधकार ा नहीं है ।

अनु े द-33 के अ गत सं सद ारा बनाए गए अिधिनयम को िकसी ायालय म चु नौती

नहीं दी जा सकती है । यह अनु े द मू ल अिधकारों का अपवाद है ।

अनु े द-34 के अ गत सं सद ारा बनाए गए ितपू ित अिधिनयम को िकसी ायालय म

केवल इस अवसर पर चु नौती नहीं दी जा सकती है िक वह िकसी मू ल अिधकारों का

उ ं घन है ।

अनु े द-32 के अ गत िनिमत सं सदीय िविध कोट माशल को उ तम एवं उ

ायालयों के रट े ािधकार से अपविजत करती है ।

[14]भारतीय सं िवधान के सं दभ म िन िल खत म से कौन-सा कथन सही नही ं है ?

A) ावना/उ े िशका को भारतीय सं िवधान का दय कहा जाता है ।

डॉ. अ े डकर ने सं वैधािनक उपचारों के अिधकार को सं िवधान का दय


B)
कहा है ।

उ तम ायालय की तु लना म उ ायालयों को रट िनकालने की श


C)
अिधक है ।

उ तम ायालय एवं उ ायालय अनु े द-32 के अ गत रट िनकालते


D)
ह।
Hide Answer -
उ रः (d)

ा ाः उपयु म से िवक (d) सही नहीं है , ोंिक उ तम ायालय अनु े द-32 एवं

उ ायालय अनु े द-226 के ारा रट िनकालते ह।

ावना/उ े िशका को भारतीय सं िवधान की आ ा/ दय कहा जाता है ।

डॉ. अ े डकर ने सं वैधािनक उपचारों के अिधकार को सं िवधान का दय/आ ा कहा है ।

उसके अनु सार ‘‘एक अनु े द िजसके िबना सं िवधान अथहीन है , यह सं िवधान की आ ा

एवं दय है ।’’

उ तम ायालय की तु लना म उ ायालयों की रट िनकालने की श अिधक

ापक है । उ तम ायालय केवल पाँ च कार के रट िनकाल सकते ह, जबिक उ

ायालय इन पाँ च के अित र अ रट भी िनकाल सकते ह।

सं सद को यह श है िक वह िविध बनाकर िकसी ायालय को रट िनकालने की श

दान कर सकती है , पर ु सं सद ने अभी तक ऐसी कोई िविध नहीं बनाई है ।

मू ल अिधकार भारतीय सव ायालय का अन /मू ल े ािधकार नहीं है , पर ु कोई

इसका उ ं घन होने पर सीधे उ तम ायालय म जा सकता है ।

[15]भारतीय सं िवधान म सं पि के अिधकार के सं दभ म िन िल खत कथनों पर िवचार


कीिजयेः
1. यह एक सं वैधािनक अिधकार है ।
2. यह एक वै धािनक अिधकार है ।
3. यह मौिलक अिधकार नहीं है ।
4. 42व सं िवधान सं शोधन, 1976 के बाद से यह मौिलक अिधकार नहीं रहा।
नीचे िदये गए कूट का योग कर सही उ र का चयन कीिजये :

A) केवल 2 और 3

B) केवल 1, 2 और 3

C) केवल 2, 3 और 4

D) 1, 2, 3 और 4

Hide Answer -
उ रः (b)

ा ाः उपयु म से कथन-4 गलत है , ोंिक 44व सं िवधान सं शोधन, 1978 के ारा

स ि का अिधकार को मौिलक अिधकार से बाहर कर िदया गया।

स ि का अिधकार एक सं वैधािनक अिधकार है ; िजसकी चचा सं िवधान के

अनु े द-300(क) म की गई है ।

44व सं िवधान सं शोधन, 1978 के ारा इसे एक वै धािनक अिधकार बना िदया गया।

[16]भारतीय सं िवधान म सं पि का अिधकार एक वै धािनक अिधकार है । इस सं दभ म


िन िल खत म से कौन-सा कथन सही है ?
A) इसकी चचा सं िवधान के भाग-XIII म की गई है ।

B) इसे सं िवधान के अनु े द-301(क) से सं ब िकया गया है ।

44व सं िवधान सं शोधन, 1978 म कां े स की सरकार ने इसे अनु ेद


C)
300(क) से सं ब िकया।

44व सं िवधान सं शोधन 1978 के ारा अनु े द 19(i) (च) एवं


D)
अनु े द-31 को मू ल अिधकार से हटा िदया गया।

Hide Answer -
उ रः (d)

ा ाः उपयु म से िवक (d) सही है ।

44व सं िवधान सं शोधन, 1978 ारा मू ल अिधकारों म से अनु े द-19(i) (च), िजसम

ावधान िकया गया था िक े क नाग रक को स ि को अिध हण करने , उसे रखने एवं

िनपटाने का अिधकार होगा एवं अनु े द-31 म स ि का अिधकार था, हटा िदया गया

है ।

44व सं िवधान सं शोधन ारा भाग-XII म एक नया अ ाय-IV जोड़ा गया है । अतः

सं िवधान के भाग-XIII के थान पर भाग-XII होगा।

44व सं िवधान सं शोधन ारा स ि के अिधकार को अनु े द-300(क) से सं ब िकया

गया एवं इसे एक वै धािनक अिधकार बना िदया गया है ।

44वां सं िवधान सं शोधन कां े स की सरकार नहीं, ब जनता पाट की सरकार ारा

िकया गया।

44व सं िवधान सं शोधन के बाद मू ल अिधकारों की सं ा 7 से घटकर 6 हो गई है ।

[17]भारतीय सं िवधान के अनु े द-31(क) के सं दभ म िन िल खत कथनों पर िवचार


कीिजयेः
1. इसे पहले सं िवधान सं शोधन अिधिनयम, 1951 के ारा जोड़ा गया।
2. इसे अनु े द-14 एवं 19 पर वरीयता दी जाती है ।
उपयु कथनों म कौन–सा/से कथन सही है /ह?

A) केवल 1

B) केवल 2

C) 1 और 2 दोनों

D) न तो 1 और न ही 2
Hide Answer -
उ रः (c)
ा ाः उपयु दोनों कथन सही ह।

अनु े द 31(क) मू लतः स ि के अिधकार के अपवाद के प म अिधिनयिमत


िकया गया। इसे पहले सं िवधान सं शोधन अिधिनयम, 1951 ारा अं तः थािपत िकया
गया। 4थे , 17व एवं 44व सं शोधन के ारा इसका िव ार िकया गया है । इसे
भू तल ी भाव (26 जनवरी, 1950 से ) िदया गया।
अनु े द 31(क) ायिक समी ा के अ गत आता है ।
यह अनु ेद स दाओं आिद के अजन के िलए उपब करने वाली िविधयों की
ावृ ि (saving) सं बंधी ावधान करता है ।
इस अनु े द के अ गत बनाई गई िविधयों को अनु े द-14 एवं 19 ारा द
मू ल अिधकारों के उ ं घन के आधार पर अवै ध नहीं ठहराया जा सकता अथात इसे
अनु े द-14 एवं 19 पर वरीयता दी जाती है ।

[18]भारतीय सं िवधान के अनु े द (31)(ख) के सं दभ म िन िल खत म से कौन-सा कथन

सही नहीं है ?

A) इस अनु े द को पहले सं िवधान सं शोधन अिधिनयम, 1951 के ारा जोड़ा गया।

B) इस अनु े द के साथ सं िवधान म 9वीं अनु सूची जोड़ी गई।

यह अनु े द 9वीं अनु सूची म उ खत अिधिनयमों एवं िनयमों को ावृ ि


C)
(saving) दान करता है ।

9वीं अनु सूची म डाली गई अिधिनयमों एवं िनयमों की ाियक समी ा नहीं हो
D)
सकती है ।
Hide Answer -
उ रः (d)
ा ाः उपयु म से िवक (d) सही नहीं है , ोंिक आई.आर. कोए ो बनाम
तिमलनाडु , 2007 म उ तम ायालय ने िनणय िदया िक 9वीं अनु सूची म स िलत
िविधयों को ाियक समी ा से उ ु ा नहीं हो सकती है । इस िनणय म कहा गया
िक 24 अ ै ल 1973 के बाद 9वीं अनु सूची म स िलत िविधयों की ाियक समी ा हो
सकती है ।

24 अ ै ल, 1973 म ही उ तम ायालय ने पहली बार केशवान भारती बनाम


केरल रा , 1973 मामले म सं िवधान के मौिलक ढ़ाँ चे के िस ा को ितपािदत
िकया।
अनु े द-31(ख) को पहले सं िवधान सं शोधन अिधिनयम 1951 के ारा सं िवधान
म जोड़ा गया। 4थे , 17व, 39व, 42व, 44व, 66व, 75व एवं 78व सं शोधन ारा
इसका िव ार िकया गया।
इस अनु े द को भू तल ी भाव िदया गया।
इस अनु े द के साथ 9वीं अनु सूची जोड़ी गई और इसम स िलत सभी िविधयों को
सभी मू ल अिधकारों से उ ु दान की गई।
मू ल प से (1951 म) नौवीं अनु सूची म 13 अिधिनयमों एवं िविनयमों को रखा
गया था, पर ु वतमान म (वष 2016 तक) इसकी सं ा 282 हो गई है ।

[19]भारतीय सं िवधान के अनु े द-31(ग) के सं दभ म िन िल खत कथनों पर िवचार


कीिजयेः
1. इस अनु े द को 24व सं िवधान सं शोधन अिधिनमय ारा जोड़ा गया।
2. यह अनु े द नीित िनदे शक िस ा के अनु े द-39(क) से (च) तक के सभी
ावधानों को अनु े द-14 एवं 19 पर थािमकता दे ता है ।
उपयु कथनों म से कौन–सा/से कथन सही है /है ?

A) केवल 1

B) केवल 2

C) 1 और 2 दोनों

D) न तो 1 और न ही 2
Hide Answer -
उ रः (d)
ा ाः उपयु दोनों कथन सही नहीं ह, ोंिक अनु े द 31(ग) को 25व सं िवधान
सं शोधन अिधिनयम, 1971 के ारा जोड़ा गया, न िक 24व सं िवधान सं शोधन अिधिनयम
के ारा।

यह अनु े द नीित िनदे शक िस ां त के अनु े द- 39(ख) या (ग) को अनु े द-


14, 19 एवं 31 पर ाथिमकता दे ता है न िक अनु े द-39 के सभी ावधानों को।
इसके अ गत यह ावधान िकया गया िक अनु े द-39(ख) अथवा (ग) सं बंधी
ावधानों को इस आधार पर अमा घोिषत नहीं िकया जा सकता िक ये
अनु े द-14 या 19 से मे ल नही खाता है ।
केशवान भारती मामले म ायालय ने उपरो को सं वैधािनक एवं वै ध माना।
42व सं िवधान सं शोधन अिधिनयम, 1976 के ारा सरकार ने भाग-4 म विणत
सभी िनदे शक िस ां तों को मू ल अिधकारों पर वरीयता दे दी गई, िक ु िमनवा िम
बनाम भारत सं घ, 1980 मामले म ायालय ने 42व सं िवधान सं शोधन अिधिनयम
को असं वैधािनक एवं अवै ध माना, िक ु केशवान भारती मामले म िदये गए
ाियक िनणय को सं वैधािनक एवं वै ध माना।

[20]भारतीय सं िवधान के अ गत मू ल अिधकारों का सं र क िन िल खत म से िकसे माना


जाता है ?

A) ायालय

B) सं सद

C) रा पित

D) (a) और (b)

Hide Answer -
उ रः (a)
ा ाः भारतीय सं िवधान के अ गत मू ल अिधकारों का सं र क ायालय को बनाया
गया है । अनु े द-32 सव ायालय को केवल मू ल अिधकारों के सं बंध म रट
िनकालने की श दान करता है । अतः सव ायालय को मू ल अिधकारों का सं र क
एवं गार र बनाया गया है । उ ायालय अनु े द-226 के तहत मू ल अिधकारों के
साथ-साथ अ अिधकारों के िलये रट िनकाल सकता है ।
[1] ‘बं ी ीकरण’ रट के सं दभ म िन िल खत कथनों पर िवचार कीिजयेः
1. यह रट सावजिनक ािधकरण या गत दोनों के िव जारी की जा सकती है ।
2. इस रट को की तं ता का महानतम र क माना जाता है ।
उपयु कथनों म कौन-सा /से कथन सही है /ह?

A) केवल 1

B) केवल 2

C) 1 और 2 दोनों

D) न तो 1 और न ही 2

Hide Answer -
उ रः (c)
ा ाः ब ी ीकरण (है िबयस कापस) के सं बंध म उपयु दोनों कथन सही ह।

सव ायालय अनु े द-32 एवं उ ायालय अनु े द-226 के ारा यह रट


िनकाल सकता है । उ तम ायालय अनु े द-32 के तहत केवल मू ल अिधकारों
के उ ं घन पर यह रट िनकाल सकता है , जबिक उ ायालय मू ल अिधकारों के
अित र अ मामले म ब ी बनाए गए के िलये भी जारी कर सकता है ।
इस रट को की तं ता का महानतम र क माना जाता है , ोंिक यह रट
ब ीकरण अिधकारी को एक िनि त समय सीमा (24 घं टे के अ र) के अ र
ब ी को ायालय के सम ु त करने का आदे श दे ती है ।
ब ीकरण अिधिनयम के अ गत एक को िबना मु कदमा चलाए 3
माह के िलये ब ी बनाया जा सकता है । 3 माह से अिधक के िलये सलाहकार बोड
की राय आव क है ।
ब ी ीकरण लै िटन भाषा से िलया गया है , िजसका शा क अथ है ‘को ुत
िकया जाए’।

[2] सू ची-I को सू ची-II से सु मेिलत कीिजये तथा नीचे िदए गये कूट की सहायता से सही
उ र चु िनये :

सू ची-I( ायालय ारा


सू ची-II(िविभ प र थितयाँ )
िनकाली गई रट)

1. जब अधीन थ ायालय ने अपना िनणय नहीं


A. उ े षण
िदया है ।

2. जब अधीन थ ायालय ने अपना िनणय दे िदया


B. ितषे ध
है ।

3. जब कोई अवै ध प से िकसी लोक पद


C. परमादे श
को हण कर ले ता है ।

4. जब कोई सावजिनक पदािधकारी अपने क ों


D. अिधकारपृ ा
का पालन नहीं करता है ।
कूटः
A B C D

A) 1 2 3 4

B) 2 1 4 3

C) 2 1 3 4

D) 1 2 4 3

Hide Answer -
उ रः (b)
ा ाः उपयु म से िवक b सही है ।

उ े षण (Certiorari): इसे िकसी व र ायालय ारा िकसी अधीन थ


ायालय या ाियक िनकाय को तब जारी िकया जाता है , जब अधीन थ ायालय
ने अपने अिधकार े से बाहर जाकर िनणय दे िदया है ।
ितषे ध (Prohibition): िकसी व र ायालय ारा िकसी अधीन थ ायालय
या ाियक िनकाय को तब जारी िकया जाता है , जब अधीन थ ायायल/ ाियक
िनकाय ने अपने अिधकार े से बाहर जाकर मामले को अपने पास रखा आ है ,
िक ु अभी अ म िनणय नहीं िदया है ।
परमादे श (Mandamus): ायालय ारा यह रट सावजिनक अिधका रयों को
जारी िकया जाता है , यिद कोई अिधकारी अपने क ों का पालन नहीं करता है ।
अिधकारपृ ा (Qua warranto): ायालय ारा यह रट उस के
िव जारी की जाती है , िजसने अवै ध प से सावजिनक पद को हण कर िलया
है । इसका शा क अथ है ‘आपकी िनयु का आधार ा है ?’

[3] िन िल खत कथनों पर िवचार कीिजयेः


1. भारत के सव ायालय को अनु े द-32 के अ गत केवल मू ल अिधकारों के
सं बंध म रट िनकालने की श है ।
2. भारत के उ ायालयों को सं िवधान के अनु े द-226 के अ गत मू ल अिधकारों
के अित र अ मामलों म रट िनकालने की श है ।
3. सव ायालय को यह श है िक अनु े द-32 के तहत िकसी अ ायालय
को भी रट िनकालने अिधकार दे सकता है ।
उपरो कथनों म कौन-सा/से कथन सही है /ह ?

A) केवल 1

B) केवल 1 और 2

C) केवल 1 और 3

D) 1, 2 और 3
Hide Answer -
उ रः (d)
ा ाः उपयु तीनों कथन सही ह।

सव ायालय को अनु े द-32 के तहत केवल मू ल अिधकारों के सं बंध म रट


जारी करने का अिधकार है ।
ायालयों को अनु े द-226 के तहत मू ल अिधकार के अित र अ मामलों
म भी रट िनकालने की श है ।
अनु े द-32 अपने -आप म एक मू ल अिधकार है । इसिलये सव ायालय अपने
रट ाय े को नकार नहीं सकता, जबिक अनु े द-226 के तहत उ
ायालय रट िनकालने से मना कर सकता है , ोंिक रट िनकालना उ
ायालयों का िववे कािधकार है ।

[4] मू ल अिधकार के सं र ण के िलये जारी की जाने वाली रट के सं दभ म िन िल खत मे से

कौन-सा कथन सही नहीं है ?

ितषे ध रट केवल ाियक एवं अध- ाियक ािधकरणों के िव जारी की जा


A)
सकती है ।

ब ी ीकरण रट सावजिनक ािधकरण एवं गत दोनों के िव जारी


B)
की जा सकती है ।

परमादे श रट सावजिनक अिधकारों एवं िनजी या इकाई के िव जारी की


C)
जा सकती है ।

उ े षण रट िविधक िनकायों एवं िनजी यों या इकाईयों के िव जारी नहीं


D)
की जा सकती है ।
Hide Answer -
उ रः (c)
ा ाः उपयु म से िवक (c) सही नहीं है , ोंिक परमादे श केवल सावजनिक
अिधका रयों, इकाईयों, िनगमों, अधीन थ ायालयों, ािधकरणों आिद के िव जारी
की जा सकती है । िनजी यों या इकाईयों, गै र-सं वैधािनक िवभाग, िववे कानु सार वाला
क , सं िवदा क दािय के िव इसे जारी नहीं िकया जा सकता है ।

परमादे श का शा क अथ है ; ‘हम आदे श दे ते ह’।


ितषे ध रट का शा क अथ ‘रोकना’ है । इसे शासिनक अिधकरणों, िवधायी
िनकायों एवं िनजी यों या िनकायों के िव जारी नहीं िकया जा सकता है ।
ब ी ीकरण रट िन िल खत दशाओं म नहीं िनकाली जा सकती है ः

- िगर ारी िकसी िवधानमं डल का ायालय की अवमानना के तहत ई है ।


- ायालय के ारा िहरासत म िलया गया है ।
- िगर ारी ायालय के ाय े से बाहर ई हो।

िहरासत कानू न स त है अथात िगर ारी िविध ारा थािपत ि या के अनु सार
ई है ।
उ े षण रट पहले केवल ाियक या अध- ाियक ािधकरणों के िव जारी की
जा सकती थी, पर ु सन् 1991 म ायालय ने व था दी की उ े षण शासिनक
पदािधका रयों के िव भी जारी की जा सकती है ।
उ े षण का शा क अथ है ‘ मािणत होना या सू चना दे ना’।

[5] ‘परमादे श’ ले ख िन िल खत म से िकसके िव नहीं िनकाला जा सकता है ?


1. भारत के रा पित
2. रा ों के रा पालों
3. उ ायालय के मु ायाधीश जो ाियक मता म कायरत ह।
नीचे िदये गए कूट का योग कर सही उ र का चयन कीिजये :

A) केवल 1

B) केवल 3

C) केवल 1 और 3

D) 1, 2 और 3

Hide Answer -
उ रः (d)
ा ाः उपयु म से िकसी के भी िव ‘परमादे श रट’ नहीं िनकाली जा सकती है ।
यह रट केवल सावजिनक अिधका रयों को जारी की जा सकती है ।

[6] िन िल खत कथनों पर िवचार कीिजयेः


1. बं धुआ मज़दू र (उ ू लन) अिधिनयम, 1976 भारतीय सं िवधान के अनु े द-23 को
भावी बनाता है ।
2. सं िवधान के अनु े द-35 सं सद एवं रा के िवधानमं डलों को यह श दान
करता है िक वे मानव दु ापार एवं बलात् - म को रोकने के िलये िविध बनाए।
उपयु कथनों म कौन-सा/से कथन सही है /ह ?

A) केवल 1

B) केवल 2

C) 1 और 2 दोनों

D) न तो 1 और न ही 2

Hide Answer -
उ रः (a)
ा ाः कथन 2 गलत है , ोंिक भारतीय सं िवधान के अनु े द-35 केवल सं सद को यह

श दे ता है िक वह उपयु के सं बंध म िविध बनाए।

सं सद ने अनु े द-23 के तहत मानव दु ापार एवं बला म पर रोक लगाने के िलये
िन िल खत कानू न बनाए ह:

- बं धुआ मज़दू री (उ ू लन) अिधिनयम, 1976


- ू नतम मज़दू री अिधिनयम, 1948
- ठे का िमक अिधिनयम, 1970
- समान पा र िमक अिधिनयम, 1976
- अनै ितक दु ापार (िनवारण) अिधिनयम, 1956 । इस अिधिनयम को मू लतः
ी तथा बािलका अनै ितक ापार दमन (सं शोधन) अिधिनयम, 1956 के पम
जाना जाता है ।

[7] भारत के सं िवधान म अनु े द-23 मानव दु ापार एवं बलात् - म के िनषे ध का
ावधान करता है । िन िल खत म से कौन-सा अनु े द-23 से सं बंध नहीं रखता है ?

A) पीप यू िनयन फॉर डे मो े िटक राइट् स बनाम भारत सं घ, 1982

B) अनै ितक ापार (िनवारण) अिधिनयम, 1956

C) बाल म ( ितषे ध और िविनयमन) अिधिनयम, 1986

D) बं िधत म प ित (उ ादन) अिधिनयम, 1976


Hide Answer -
उ रः (c)
ा ाः िवक (c) का सं बंध मानव दु ापार एवं बलात् म का िनषे ध से नहीं है ।
इसका सं बंध सं िवधान के अनु े द-24 म कारखानों आिद म बालकों के िनयोजन के
ितषे ध से है ।

पीप यू िनयन फॉर डे मो े िटक राइट् स बनाम भारत सं घ, 1982, (िजसे एिशयाई
खे ल मामले , 1982 के नाम से भी जाना जाता है ) मामले म ायालय ने ‘बल’ को
शारी रक बल या िविधक बल के साथ-साथ आिथक प र थितयों के कारण उ
बल को ही शािमल िकया।
बं धुआ मु मोचा बनाम भारत सं घ, 1983 का सं बंध भी सं िवधान के इसी
अनु े द से है ।

[8] भारत म 14 वष से कम आयु वग के बालकों को िकसी कारखाने या खान म काम

करने के िलये िनयोजन पर रोक है । यह रोक लगाई गई है :


1. भारत के सं िवधान ारा
2. उ तम ायालय के िनणय ारा
नीचे िदये गए कूट का योग कर सही उ र का चयन कीिजये :

A) केवल 1

B) केवल 2

C) 1 और 2 दोनों

D) न तो 1 और न ही 2

Hide Answer -
उ रः (c)
ा ाः उपयु दोनों कथन सही ह।

सं िवधान के अनु े द-24 म यह ावधान है िक 14 वष से कम आयु के िकसी


बालक का िकसी कारखाने या खान म काम करने के िलये िनयोजन नहीं िकया
जाएगा या िकसी अ जो खम भरे उ ोग म नहीं लगाया जाएगा।
एम.सी. मे हता बनाम तिमलनाडु रा , 1996 म उ तम ायालय ने िनणय िदया
िक 14 वष से कम आयु के बालकों को िकसी जो खम भरे उ ोग म िनयोिजत नहीं
िकया जाएगा।
सं यु रा सं घ के चाटर म भी बाल म का िनषे ध िकया गया है ।
भारतीय सं सद ने अनु े द-35 के तहत इस सं बंध म िन कानू न बनाए ह-
- बाल िनयोजन अिधिनयम, 1938
- बालक म ( ितषे ध एवं िविनयमन) अिधिनयम, 1986
- कारखाना अिधिनयम, 1948
- खान अिधिनयम, 1952
- िशशु अिधिनयम, 1960 आिद।
[9] अनु सूिचत जाितयाँ एवं अनु सूिचत जनजाितयाँ (अ ाचार िनवारण) अिधिनयम, 1989

को िन िल खत म से कौन-से मू ल अिधकारों को भावी करने वाले के प म दे खा जा


सकता है ?
1. िविध के सम समानता
2. िवभे द के िव अिधकार
3. अ ृ ता का अं त
4. धम की तं ता का अिधकार
नीचे िदये गए कूट का योग कर सही उ र का चयन कीिजये :

A) केवल 1, 2 और 3

B) केवल 2 और 3

C) केवल 2, 3 और 4

D) 1, 2, 3 और 4

Hide Answer -
उ रः (a)
ा ाः कथन 4 गलत है , ोंिक अिधिनयम, 1989 धम की तं ता के अिधकार
(अनु े द-25-28) को भावी नहीं बनाता है ।
भारत के सं िवधान का अनु े द-35 सं सद को यह श दे ता है िक वह अनु े द-17 म
अ ृ ता के अं त के सं बंध म कानू न बना सकती ह।
सं सद ारा अ ृ ता (अपराध) अिधिनयम, 1955 पा रत िकया गया। 1976 म इसका
सं शोधन िकया गया एवं नाम बदलकर िसिवल अिधकार सं र ण अिधिनयम, 1955 कर
िदया गया।
इस सं बंध म 1989 म अनु सूिचत जाितयाँ एवं अनु सूिचत जनजाितयाँ (अ ाचार िनवारण)
अिधिनयम, 1989 पा रत िकया गया। इस अिधिनयम को िन िल खत मू ल अिधकारों को
भावी बनाने के प म दे खा जा सकता है ;
i. िविध के सम समानता - अनु े द-14
ii. िवभे द के िव अिधकार - अनु े द-15(2)
iii. अ ृ ता का अं त - अनु े द-17

[10]‘अ ृ ता’ के सं दभ म िन िल खत कथनों पर िवचार कीिजये :


1. भारत के सं िवधान म ‘अ ृ ता’ श को प रभािषत नहीं िकया गया है ।
2. अ ृ ता (अपराध) अिधिनयम, 1955 या िसिवल अिधकार सं र ण अिधिनयम,
1955 म अ ृ ता श को प रभािषत िकया गया है ।
3. सं िवधान का अनु े द-34 सं सद को यह श दे ता है िक वह इस अपराध के िलये
िविध बन सकता है ।
उपयु कथनों म से कौन–सा/से कथन सही है /ह ?
A) केवल 1

B) केवल 2

C) केवल 1 और 3

D) केवल 2 और 3

Hide Answer -
उ रः (a)
ा ाः कथन 2 और 3 सही नहीं है , ोंिक न तो भारत के सं िवधान म और न
अ ृ ता के सं बंध म बनाए गए िकसी कानू न म इसे प रभािषत िकया गया है ।

सं िवधान का अनु े द-35 सं सद को यह श दे ता है िक वह अ ृ ता के अं त म


सं बंध म िविध बना सकती है ।

[11]भारतीय सं िवधान म मू ल अिधकार के िवषय म िन िल खत कथनों पर िवचार कीिजयेः


1. दे श के िकसी भाग म िनवास तथा बसने की तं ता।
2. भारत के सं पूण े म अबाध मण की तं ता।
3. अपराधों के िलये दोष िस के सं बंध म सं र ण।
4. वाक् एवं अिभ की तं ता।
5. ाण एवं दै िहक तं ता का सं र ण।
उपयु म से कौन-से मू ल अिधकार रा ीय आपातकाल म भी सीिमत या समा नहीं
िकये जा सकते ह?

A) केवल 1 और 2

B) केवल 3 और 4

C) केवल 3 और 5

D) केवल 4 और 5

Hide Answer -
उ रः (c)
ा ाः उपयु म से कथन 1, 2 एवं 4 गलत ह।

भारतीय सं िवधान के अनु े द-20 एवं 21 रा ीय आपातकाल म भी न तो सीिमत


िकया जा सकता है और न ही समा िकया जा सकता है ।
अनु े द-20: अपराधों के िलये दोषिस के सं बंध म को सं र ण दान
करता है । इसका सं बंध गत तं ता से है ।
अनु े द-21: को ाण एवं दै िहक तं ता का सं र ण दान करता है ।
इसका सं बंध जीवन के अिधकार से है । िसफ िविध ारा थािपत ि या के ारा ही
िकसी को जीवन से वं िचत िकया जा सकता है ।

[12]भारत म रहने वाला ि िटश नाग रक दावा नहीं कर सकता है :


A) ापार और वसाय की तं ता के अिधकार का।

B) िविध के सम समता के अिधकार का।

C) जीवन और गत तं ता की सु र ा के अिधकार का।

D) धम की तं ता के अिधकार का।

Hide Answer -
उ रः (a)
ा ाः भारतीय सं िवधान म पाँ च मू ल अिधकार गै र-नाग रकों को ा नहीं ह:
अनु े द-15, 16, 19, 29 और 30
ापार और वसाय की तं ता का अिधकार गै र-नाग रकों/िवदे िशयों को ा नहीं है ,
ोंिक यह अिधकार भारतीय सं िवधान के अनु े द 19(i)(छ) म िदया गया है ।

िविध के सम समता - अनु े द-14


जीवन का अिधकार - अनु े द-21
गत तं ता - अनु े द-20
धम की तं ता - अनु े द-25 से 28

[13]भारतीय सं िवधान के अं तगत िदये गए मू ल अिधकारों के सं दभ म िन िल खत पर िवचार


कीिजये :
1. िवचार एवं अिभ की तं ता।
2. दे श के िकसी भी भाग म घू मने और बसने का अिधकार।
3. सं पि अिजत करने का अिधकार।
4. भे दभाव के िव अिधकार।
उपयु म से कौन-से अिधकार गै र-नाग रकों को दान नहीं िकया गया है ?

A) केवल 3

B) केवल 3 और 4

C) केवल 2, 3 और 4

D) 1, 2, 3 और 4
Hide Answer -
उ रः (d)
ा ाः भारतीय सं िवधान म पाँ च मू ल अिधकार गै र-नाग रकों को ा नहीं ह:

अनु े द-15: िवभे द के िव अिधकार।


अनु े द-16: लोक िनयोजन के िवषय म अवसर की समता।
अनु े द-19(i)

(क): िवचार एवं अिभ की तं ता।


(ख): शां ितपू ण और िनरायु ध स े लन का अिधकार।
(ग): सं गम या सं घ या सहकारी सिमित बनाने का अिधकार।
(घ): भारत के रा े म सव अबाध घू मने का अिधकार।
(ङ): भारत के रा े के िकसी भी भाग म िनवास करने और बस जाने का अिधकार।
(च): 44व सं िवधान सं शोधन के ारा लोप कर िदया गया है ।
(छ): कोई वृ ि , उपजीिवका, ापार या कारोबार करने का अिधकार (सं पि अजन
करने का अिधकार)।

अनु े द-29: अ ं क वग के िहतों का सं र ण।


अनु े द-30: िश ण सं थाओं की थापना एवं शासन करने का अ सं क वग
का अिधकार।

[14]भारत का सं िवधान अपने सभी नाग रकों को धम की तं ता का मू ल अिधकार ितभू त


करता है । इस अिधकार के बारे म िन िल खत म से कौन-सा सही नहीं है ?

यह अं तःकरण की तं ता और िकसी भी धम को मानने , आचरण करने और


A)
चार करने की तं ता दे ता है ।

यह धम एवं पू त (Charitable) योजनों के िलये सं थाओं की थापना और पोषण


B)
की तं ता दे ता है ।

C) यह अिधकार लोक व था, सदाचार एवं ा के अधीन है ।

रा कोई भी ऐसा कानू न नहीं बना सकता, जो नाग रकों के इस अिधकार का


D)
िनवारण करे ।

Hide Answer -
उ रः (d)
ा ाः िवक (d) गलत है , ोंिक अनु े द-25(2) रा को इस सं बंध म कानू न
बनाने का अिधकार दे ता है ।

िवक (a) सं बंधी ावधान अनु े द-25(i) म िकया गया है ।


िवक (b) सं बंधी ावधान अनु े द-26(क) म िकया गया है ।
िवक (c) सं बंधी ावधान अनु े द 25(i) म िकया गया है ।

[15]भारत के सं िवधान के अनु े द-21 के अधीन ाण एवं दै िहक तं ता सं बंधी मू ल


अिधकार िदया गया है । इस सं बंध म िन िल खत म से कौन-से कथन सही ह?
1. यह अिधकार नाग रकों एवं अ गै र-नाग रकों को उपल है ।
2. इसम े ाचारी कायपािलका एवं िवधायी कायवाई से सं र ण का समावे श है ।
3. इसम मानवीय ग रमा के साथ जीने का अिधकार स िलत है ।
4. इसे िविध ारा थािपत ि या के ारा अनु सार छीना जा सकता है ।
उपयु कथनों म कौन–से कथन सही ह ?

A) केवल 1, 2 और 3

B) केवल 1 और 4

C) केवल 2, 3 और 4

D) 1, 2, 3 और 4

Hide Answer -
उ रः (d)
ा ाः उपयु सभी कथन सही ह।

अनु े द-21 सं बंधी अिधकार नाग रकों एवं गै र-नाग रकों को ा है । केवल पाँ च
ऐसे मू ल अिधकार ह, जो केवल नाग रकों को ा ह-अनु े द-15, 16, 19, 29
और 30।
रा ीय आपातकाल म इसे न तो सीिमत िकया जा सकता है और न ही समा । अतः
इस अिधकार को े ाचारी कायपािलका एवं िवधायी कायवायी से सं र ण ा है ।
अनु े द-20 को भी रा ीय आपातकाल म न तो सीिमत िकया जा सकता है और न
ही समा िकया जा सकता है ।
इसम मानवीय ग रमा के साथ जीने का अिधकार स िलत है । िविभ ाियक
िनणयों के ारा इसम ग रमापू ण जीवन जीने सं बंधी अिधकारों को शािमल िकया गया
है ।
जीवन के अिधकार से केवल िविध ारा थािपत ि या के आधार पर ही वं िचत
िकया जा सकता है । यह यु यु , ऋजु एवं ायपू ण होनी चािहये । इस अनु ेद
म िविध की स क् ि या को शािमल िकया गया है ।
अमे रकी सं िवधान म ‘िविध की स क् ि या’ का ावधान िकया गया है , जो
को कायपािलका एवं िवधानमं डल से सं र ण दान करता है ।
मे िनका गां धी बनाम भारत सं घ, 1978 मामले म ‘िविध की स क् ि या’ को
भारत म अपनाया गया।

[16]भारत के सं िवधान के भाग चार म रा के नीित िनदे शक िस ा ों का ावधान िकया


गया है । इस सं दभ म िन िल खत पर िवचार कीिजएः
1. भारतीय सं िवधान के अनु े द-35 से 51 तक इसका ावधान िकया गया है ।
2. इसे आयरलै के सं िवधान से िलया गया है ।
3. मू ल अिधकार वाद यो नहीं ह, जबिक नीित-िनदे शक त वाद यो ह, ोंिक
नीित िनदे शक िस ा ों को ायालय म चु नौती नहीं दी जा सकती है ।
उपयु म से कौन-सा/से कथन स है /ह?
A) केवल 2

B) केवल 1 और 2

C) केवल 1 और 3

D) केवल 2 और 3

Hide Answer -
उ रः (a)

ा ाः रा के नीित-िनदे शक िस ों के सं बंध म कथन-1 एवं 3 सही नहीं ह, ोंिक

इसकी चचा सं िवधान के अनु े द-36 से 51 तक की गई है न िक अनु े द 35-51 तक।

कथन-(3) इसिलये सही नहीं है , ोंिक मू ल अिधकार वाद यो ह, जबिक नीित-

िनदे शक त वाद यो नहीं ह। मू ल अिधकारों के वतन के िलये ायालय म चु नौती दी

जा सकती है , जबिक नीित िनदे शक त ों के उ ं घन पर ऐसा नहीं िकया जा सकता है ।

इसकी कृित गै र- ायोिचत है । रा इसे लागू करने के िलये बा नहीं है ।

कथन-(2) सही है , ोंिक सं िवधान िनमाता ने इसे आयरलै के सं िवधान से िलया है ।

[17] भारतीय सं िवधान म उ खत रा के नीित िनदे शक त ों के उ े ों पर िवचार


कीिजये :
1. सामािजक जातं की थापना करना।
2. राजनीितक जातं की थापना करना।
3. आिथक जातं की थापना करना।
उपयु म से कौन-सा/से उ े इसम शािमल है /ह?

A) केवल 1

B) केवल 1 और 2

C) केवल 1 और 3

D) 1, 2 और 3

Hide Answer -
उ रः (c)
ा ाः रा के नीित िनदे शक िस ों के उ े ों म राजनै ितक जातं की थापना
करना शािमल नहीं है ।

इसका उ े सामािजक एवं आिथक लोकतं की थापना करना है । इसके ारा


सामािजक एवं आिथक ाय को सु िनि त करना है ।

[18] भारतीय सं िवधान का कौन-सा भाग क ाणकारी रा का आदश घोिषत करता है ?


A) ावना

B) मू ल अिधकार

C) रा के नीित-िनदे शक िस ा

D) मू ल क

Hide Answer -
उ रः (c)
ा ाः भारत के सं िवधान के भाग-4 म रा को लोक क ाणकारी रा की थापना
के िलये िनदश िदये गये ह।
नोट: ावना म भी लोक क ाणकारी रा की थापना सं बंधी िवचार पाया जाता है ,
िक ु ावना या रा के नीित िनदे शक िस ा ों म से कोई एक िवक चु नना है तो
सही उ र रा के नीित-िनदे शक त होगा, िक ु िवक म दोनों साथ िदया है तो
दोनों साथ वाला िवक सही होगा।

[19]सू ची-I को सू ची-II से सु मेिलत कीिजये तथा नीचे िदए कूट की सहायता से सही उ र

चु िनये :

सू ची-II
सू ची-I (रा के नीित-िनदे शक िस ा )
(अनु े द)

A. रा ित ा, सु िवधाओं और अवसरों की असमानता


1. अनु े द-36
समा करने का यास करे गा।

B. रा का वही अथ है जो भाग-3 म है । 2. अनु े द-37

C. रा लोकक ाण की अिभवृ के िलये सामािजक


3. अनु े द-38(1)
व था बनाएगा।

D. यह िस ा दे श के शासन के िलये मू लभू त है । 4. अनु े द-38(2)

कूटः
A B C D

A) 4 1 3 2

B) 4 3 1 2

C) 2 3 1 4

D) 2 1 3 4
Hide Answer -
उ रः (a)
ा ाः उपयु म से िवक (a) सही है ।

भारत के सं िवधान के भाग-4 म अनु े द-36-51 तक रा के नीित-िनदे शक


िस ों का उ े ख िकया गया है ।
अनु े द-36 म कहा गया है रा का वही अथ है , जो सं िवधान के भाग-3 म है ।
अनु े द-37 म कहा गया है िक यह िस ा ायालय ारा वतनीय नहीं है , िफर
भी दे श के शासन म मू लभू त/आधारभू त है और िविध बनाने म इन त ों को लागू
करना रा का क होगा।
अनु े द-38(1) म ावधान िकया गया है िक रा लोकक ाण की अिभवृ के
िलये सामािजक, आिथक और राजनै ितक ाय ारा सामािजक व था सु िनि त
करे गी।
अनु े द-38(2) को 44व सं िवधान सं शोधन अिधिनयम, 1978 के ारा जोड़ा
गया, िजसम ावधान िकया गया है िक रा आय की असमानताओं को कम करने
का यास करे गा तथा ित ा, सु िवधाओं और अवसरों की असमानता समा करने
का यास करे गा।

[20]भारतीय सं िवधान म िति त रा के नीित-िनदे शक त ों के अ गत िन िल खत


ावधानों पर िवचार कीिजयेः
1. भारतीय नाग रकों के िलये समान िसिवल सं िहता सु रि त करना।
2. ाम पं चायतों को सं गिठत करना।
3. ामीण े ों म कुटीर उ ोगों को ो ािहत करना।
4. सभी कमकारों के िलये अवकाश तथा सां ृ ितक अवसर सु रि त करना।
उपय म से कौन-से गाँ धीवादी िस ा ह, जो रा के नीित-िनदे शक त ों म
ितिबं िबत होते ह?
कूटः

A) केवल 1, 2 और 4

B) केवल 2 और 3

C) केवल 1, 3 और 4

D) 1, 2, 3 और 4
Hide Answer -
उ रः (b)
ा ाः कथन 1 और 4 रा के नीित-िनदे शक िस ा ों के ावधान तो ह, िक ु इसे
गाँ धीवादी िस ा के प म शािमल नहीं िकया जाता है ।

रा के नीित-िनदे शक िस ा ों को मु तः तीन भागों म बाँ टा जा सकता है :

1. समाजवादी िस ा
2. गाँ धीवादी िस ा
3. उदार बौ क िस ा

गाँ धीवादी िस ा ः इसके अ गत नीित-िनदे शक िस ा ों के िन िल खत अनु े दों


को शािमल िकया जा सकता है ;

► अनु े द-40: ाम पं चायतों को सं गिठत करना।


► अनु े द-43: ामीण े ों म कुटीर उ ोगों को ो ाहन दे ना।
► अनु े द-43(ख): सहकारी सिमितयों के ै क गठन, ाय सं चालन,
लोकतां ि क िनयं ण आिद।
► अनु े द-46: अनु सूिचत जाितयों/जनजाितयों एवं अ दु बल वग के िश ा
और अथ सं बंधी िहतों की अिभवृ ।
► अनु े द-47: ा के िलये नु कसानदायक नशीली दवाओं, मिदरा एवं
उ े से िभ औषिधयों पर ितबं ध।
► अु न े द-48: गाय, बछड़ा तथा अ दु धा पशु ओं की बली पर रोक और
उसकी न ों म सु धार को ो ाहन।
[1] िन िल खत कथनों पर िवचार कीिजयेः

1. समान ाय एवं िनःशु िविधक सहायता।


2. उ ोगों के बं धन म कमकारों की भागीदारी।
3. सहकारी सिमितयों का उ यन।
4. पयावरण का सं र ण तथा सं वधन और वन तथा व जीवों की र ा ।
5. रा ीय मह के सं ारकों, थानों और व ु ओं का सं र ण।

उपयु म से कौन-से नीित-िनदे शक त ों को सं िवधान सं शोधन के ारा जोड़ा


गया है ?

A) केवल 1, 2 और 3

B) केवल 1, 3 और 4

C) केवल 1, 3 और 5

D) केवल 1, 2, 3 और 4

Show Answer +

[2] भारत के सं िवधान म काम पाने का अिधकार का ावधान िकया गया है । इस


सं दभ म िन िल खत म से कौन-सा कथन सही है ?

A) यह एक मू ल अिधकार है , िजसकी चचा सं िवधान के भाग-3 म की गई है ।

यह एक नीित-िनदे शक िस ा है , िजसकी चचा सं िवधान के अनु े द-46 म


B)
की गई है ।

महा ा गां धी रा ीय ामीण रोज़गार गार ी अिधिनयम, 2005 (मनरे गा) एक


C)
रोज़गार गार ी योजना है , जो इसके अ गत आती है ।

D) यह एक गाँ धीवादी िस ा है , जो सं िवधान के भाग-4 म ितिबं िबत होता है ।

Hide Answer -
उ रः (c)

ा ाः उपयु म से िवक (c) सही है , ोंिक मनरे गा एक रोजगार गार ी योजना है और

यह काम/रोज़गार पाने का अिधकार के अ गत आता है ।

यह मू ल अिधकार नहीं है , ब रा का नीित िनदे शक िस ा है और इसकी चचा

सं िवधान के भाग-4 म की गई है ।

सं िवधान के अनु े द-41 म इसकी चचा की गई है , न िक अनु े द-46 म।

यह गाँ धीवादी िस ा नहीं है ।

[3] भारतीय सं िवधान के अनु े द-39 के सं दभ म िन िल खत कथनों पर िवचार


कीिजयेः
1. ी और पु ष सभी नाग रकों को समान प से जीिवका के पया साधन
ा करने का अिधकार।
2. पु षों और यों दोनों को समान काय के िलये समान वे तन।
3. पु ष और ी कमकारों के ा और श का तथा बालकों की
सु कुमार अव था का दु पयोग से सं र ण।
4. बालकों को तं और ग रमामय वातावरण म ा िवकास के अवसर
और सु िवधाएँ ।

उपयु म से कौन-सा/से ावधान/ ावधानों को 42व सं िवधान सं शोधन


अिधिनयम, 1976 के ारा जोड़ा गया है ?

A) केवल 3

B) केवल 4

C) केवल 1 और 3

D) केवल 2 और 4

Hide Answer -
उ रः (b)

ा ाः उपयु म से कथन-4 सं बंधी ावधान 42व सं िवधान सं शोधन अिधिनयम, 1976 के ारा

जोड़ा गया है ।

भारतीय सं िवधान के अनु े द 39 म (क) से (च) तक कुल छः


ावधान िकये गए ह। उपयु कथन-4 सं बंधी ावधान
अनु े द-39(च) म िकया गया है ।
कथन-1 सं बंधी ावधान अनु े द 39(क) म िकया गया है ।
कथन-2 सं बंधी ावधान अनु े द 39(घ) म िकया गया है ।
कथन-3 सं बंधी ावधान अनु े द 39(ङ) म िकया गया है ।

[4] रा अपनी नीितयों का इस कार सं चालन करे गा िकः


1. समु दाय के भौितक सं साधनों का ािम और िनयं ण इस कार बँ टा हो, िजससे
सामू िहक िहत का सव म प से साधन हो।
2. आिथक व था इस कार चले , िजससे धन और उ ादन साधनों का सवसाधारण
के िलये अिहतकारी सं के ण न हो।
3. सभी कमकारों को िनवाह मज़दू री, िश जीवन र और अवकाश का स ू ण
उपभोग तथा सामािजक और सां ृ ितक अवसर उपल हो सकेगा।
4. उ ोगों के बं धन म भाग ले ने के िलये कमकारों को अवसर ा होंगे।|
भारतीय सं िवधान के अनु े द 31(ग) उपयु म से कौन-से नीित-िनदे शक त ों को
भावी बनाते ह?
A) केवल 1 और 2

B) केवल 1, 2 और 3

C) केवल 1, 2 और 4

D) 1, 2, 3 और 4

Hide Answer -
उ रः (a)
ा ाः उपयु कथन 1 और 2 को अनु े द 31(ग) भावी बनाते ह, जबिक कथन-3
और 4 का सं बंध इस अनु े द से नहीं है ।

कथन 1 और 2 का सं बंध सं िवधान के अनु े द 39(ख) और (ग) से है , जबिक


कथन 3 और 4 का सं बंध अनु े द 43 और 43(क) से है ।
25व सं िवधान सं शोधन अिधिनयम, 1971 के ारा सं िवधान का अनु े द-31(ग)
जोड़ा गया, जो कुछ मू ल अिधकारों का अपवाद है ।
अनु े द-31(ग) म कहा गया है िक अनु े द-39(ख) और (ग) सं बंधी कोई
कानू न लागू करते समय अनु े द-14 और 19 का उ ं घन होता है तो ऐसे कानू न
को अमा घोिषत नहीं िकया जा सकता अथात यह अनु े द कुछ नीित िनदे शक
त ों (अनु े द-39 (ख) और (ग)) को कुछ मू ल अिधकारों (अनु े द 14 और
19) पर वरीयता दे ता है ।

[5] िन िल खत यु ों म से कौन-सा/से यु सही सु मेिलत है /ह?

1. काम की ाय सं गत और मानवोिचत दशाओं तथा सू ित


अनु े द-41
सहायता उपल कराना।

2. ाय शासन की इकाइयों के प म काय करने वाली ाम


अनु े द-45
पं चायतों का सं गठन करना।

3. लोक ा का सु धार करना तथा मादक पे यों तथा ा


अनु े द-47
के िलये हािनकारक औषिधयों के उपभोग पर रोक।

4. ऐितहािसक अिभ िच तथा रा ीय मह के सभी ारकों


अनु े द-48
का सं र ण करना।

नीचे िदये गए कूट का योग कर सही उ र का चयन कीिजयेः

A) केवल 3

B) केवल 4

C) केवल 1, 2 और 3

D) केवल 1, 3 और 4
Hide Answer -
उ रः (a)
ा ाः उपयु म से केवल कथन-3 सही है ?

काम की ाय सं गत और मानवोिचत दशाओं तथा सू ित सहायता उपल करना -


अनु े द-42
कुछ दशाओं म काम पाने , िश ा पाने और बे कारी, बु ढ़ापा, बीमारी और िनःश ता
आिद की दशाओं म लोक सहायता पाने का अिधकार -अनु े द-41
लोक ा का सु धार करना तथा मादक पे यों तथा ा के िलये हािनकारक
औषिधयों के उपभोग का ितषे ध -अनु े द-47
ारं िभक शै शवाव था की दे खरे ख और सभी बालकों को छःवष की आयु तक िश ा
-अनु े द-45
एितहािसक अिभ िच तथा रा ीय मह के सभी सं ारकों, थानों और व ु ओं का
सं र ण- अनु े द-49

[6] िन िल खत म से कौन-सा कथन स है ?

नीित-िनदे शक िस ा ों को मू ल अिधकारों की अपे ा िकसी भी दशा म


A)
ाथिमकता नहीं दी जा सकती है ।

मू ल अिधकारों की अपे ा नीित-िनदे शक िस ा ों को सदै व ाथिमकता दी जाती


B)
है ।

नीित-िनदे शक िस ा ों की अपे ा मू ल अिधकारों को सदै व ाथिमकता दी जाती


C)
है ।

कुछ मामलों म नीित-िनदे शक िस ा ों को मू ल अिधकारों की अपे ा ाथिमकता


D)
दी जाती है ।

Hide Answer -
उ र (d)
ा ाः उपयु म से िवक (d) सही है , ोंिक वतमान थित के अनु सार कुछ
मामलों म रा के नीित-िनदे त ों को कुछ मू ल अिधकारों के अपे ा ाथिमकता दी
गई है ।

सं िवधान के अनु े द 31(ग) के ारा अनु े द 39(ख) और (ग) को कुछ मू ल


अिधकारों (अनु े द-14 एवं 19) पर ाथिमकता दी गई है ।
िमनवा िम बनाम भारत सं घ, 1980 मामले म ायालय ने मू ल अिधकार एवं
रा नीित-िनदे शक त ों को एक-दू सरे का पू रक माना है ।

[7] भारत के सं िवधान के भाग-4 म नाग रकों के िलये एक समान िसिवल सं िहता का
ावधान िकया गया है । इस सं दभ म िन िल खत म से कौन-सा कथन सही है ?
रा के िलये बा कारी थित है िक एक समान िसिवल सं िहता सब पर लागू की
A)
जानी चािहये ।

भारत के उ तम ायालय ने यह िनदश िदया है िक दस वष की अविध म समान


B)
िसिवल सं िहता सभी पर लागू कर दी जाए।

सं िवधान का अनु े द-40 भारत के सम रा े म नाग रकों के िलये एक


C)
समान िसिवल सं िहता का ावधान करता है ।

समान िसिवल सं िहता सं बंधी ावधान ज ू और क ीर रा पर लागू नहीं होता


D)
है ।

Hide Answer -
उ र (d)
ा ाः उपयु म से िवक (d) सही है , ोंिक ज ू -क ीर का अपना सं िवधान है ।
इसिलये एक समान िसिवल सं िहता सं बंधी ावधान इस रा पर लागू नहीं होता है ।

सं िवधान के अनु े द-44 भारत के सम रा - े म नाग रकों के िलये एक


समान िसिवल सं िहता ावधान करता है ।
एक समान िसिवल सं िहता सं बंधी ावधान लागू करने के िलये रा के िलए
बा कारी थित नहीं है ।
वतमान म भारत म केवल गोवा रा म सभी नाग रकों के िलये एक समान िसिवल
सं िहता/कोड लागू है ।

[8] भारत के सं िवधान के सं दभ म िन िल खत कथनों पर िवचार कीिजयेः


1. मू ल अिधकार
2. मू ल क
3. रा के नीित-िनदे शक त
उपयु म से कौन-सा/से उपबं ध रा ीय सामािजक सहायता काय म से सं बंिधत है /ह?

A) केवल 1

B) केवल 3

C) केवल 1 और 3

D) 1,2 और 3

Hide Answer -
उ र (b)
ा ाः उपयु म से केवल-3 उपबं ध के अ गत भारत सरकार ारा 15 अग
1995 म रा ीय सामािजक सहायता काय म ारं भ िकया गया।

सं िवधान के अनु े द-41 बे कारी, बु ढ़ापा, बीमारी और िनःश ता के थित म लोक


सहायता का ावधान करता है ।

[9]
रा के नीित-िनदे शक िस ां तों के सं दभ म िन िल खत कथनों पर िवचार कीिजयेः
1. इन िस ा ों म ौढ़ िश ा पर भारत का सं िवधान मौन है ।
2. इन िस ा ों की चचा भारत के सं िवधान म मू ल प से कुल 17 अनु े दों म की
गई है ।
उपयु म से कौन-सा/से कथन सही है /ह?

A) केवल 1

B) केवल 2

C) 1 और 2 दोनों

D) न तो 1 और न ही 2

Hide Answer -
उ र (a)
ा ाः उपयु कथनों म से केवल कथन-1 सही है , ोंिक सं िवधान के भाग-4 म
िश ा सं बंधी ावधान कई थानों पर िकया गया है , जै से अनु े द-41, 45 और 46,
िक ु ौढ़ िश ा के सं बंध म सं िवधान मौन है ।

नीित-िनदे शक िस ा ों की चचा मू ल प से सं िवधान के कुल 16 अनु े दों


(अनु े द 36-51) म की गई है न िक 17 अनु े दों म।

[10] िन िल खत म से कौन-सा/से सही सु मेिलत है /ह?

1. अ रा ीय शा और सु र ा अनु े द-50

2. कायपािलका से ायपािलका का पृ थ रण अनु े द-51

3. कृिष एवं पशु पालन को आधु िनक और वै ािनक प ितयों से


अनु े द-48
सं गिठत

4. दे श के पयावरण का सं र ण तथा सं व न अनु े द-49

5. पोषाहार और जीवन र को ऊँचा करना अनु े द 46

नीचे िदये गए कूट का योग कर सही उ र का चयन कीिजये :

A) केवल 3

B) केवल 1 और 2

C) केवल 2 और 5

D) केवल 4 और 5
Hide Answer -
उ र (a)
ा ाः उपयु म से केवल कथन-3 सही सु मेिलत है ।

अ रा ीय शा और सु र ा सं बंधी ावधानः अनु े द-51


कायपािलका से ायपािलका का पृ थकरण सं बंधी ावधान : अनु े द-50
दे श के पयावरण के सं र ण तथा सं व न सं बंधी ावधानः अनु े द-48(क)
पोषाहार और जीवन र को ऊँचा करने सं बंधी ावधान : अनु े द-47
अनु सूिचत जाितयों, अनु सूिचत जनजाितयों और अ दु बल वग के िलये िश ा और
अथ सं बंधी ावधान : अनु े द-46

[11]िन िल खत कथनों पर िवचार कीिजयेः

1. सं िवधान के अनु े द-49 म ावधान है िक कला क या ऐितहािसक अिभ िच


वाले सं ारक या थान या व ु का सं र ण करना रा की बा ता होगी।
2. सं िवधान के अनु े द-46 म ावधान है िक अनु सूिचत जाितयों, अनु सूिचत
जनजाितयों और अ दु बल वग के सामािजक अ ाय और सभी कार के शोषण
से रा उनकी र ा करे गा।
रा के नीित-िनदे शक िस ां तों के सं दभ म उपयु म से कौन-सा/से कथन
सही है /ह?

A) केवल 1

B) केवल 2

C) 1 और 2 दोनों

D) न तो 1 और न ही 2

Hide Answer -
उ र (c)
ा ाः उपयु दोनों कथन सही ह।
सं िवधान के अनु े द-49 म ावधान है िक सं सद ारा बनाई गई िविध या उसके अधीन
रा ीय मह वाले घोिषत िकये गए कला क या ऐितहािसक अिभ िच वाले ेक
सं ारक या थान या व ु का सं र ण करना रा की बा ता होगी।
अनु े द-46 म ावधान है िक अनू सूिचत जाितयों, अनु सूिचत जनजाितयों और अ दु बल
वग के िलये िश ा और अथ सं बंधी िहतों की अिभवृ तथा सामािजक अ ाय और सभी
कार के शोषण से रा सं र ा करे गा।

[12]मू ल अिधकार और रा के नीित िनदे शक त ों के सं दभ म िन िल खत मामलों/वादों


पर िवचार कीिजयेः
1. च ाकम दोराइराजन वाद
2. गोलकनाथ वाद
3. केशवान भारती वाद
4. िमनवा िम वाद
5. 24व सं िवधान सं शोधन
6. 42व सं िवधान सं शोधन
काल म के आधार पर सही म कौन-सा होगा?

A) 1-2-3-4-5-6

B) 1-2-5-3-6-4

C) 2-1-3-5-6-4

D) 2-1-5-3-4-6
Hide Answer -
उ र (b)

ा ाः उपयु वादों का सही काल म इस कार होगाः


म ास रा बनाम च ाकम दोराइराजन वाद, 1951: इस मामले म उ तम
ायालय ने िनणय िदया िक मू ल अिधकार एवं िनदे शक त ों के बीच िकसी तरह
की टकराव म मू ल अिधकार भावी होंगे, िक ु सं िवधान सं शोधन ि या के ारा
सं सद मू ल अिधकारों म सं शोधन कर सकती है । इसके प रणाम प सं सद ने
थम सं िवधान सं शोधन, 1951, चौथा सं िवधान सं शोधन अिधिनयम, 1955 और
17वाँ सं िवधान सं शोधन अिधिनयम, 1964 पा रत िकया।
गोलकनाथ बनाम पं जाब रा , 1967: इस वाद म उ तम ायालय ने िनणय
िदया िक नीित िनदे शक त ों को लागू करने के िलये मू ल अिधकारों म सं शोधन नहीं
िकया जा सकता है ।
24व सं िवधान सं शोधन अिधिनयम, 1971: गोलकनाथ वाद के िति या म सं सद
ारा यह सं शोधन पा रत िकया गया और व था दी गई िक सं सद को श है िक
सं िवधान सं शोधन अिधिनयम के ारा मू ल अिधकारों को समा या कम कर दे ।
25व सं िवधान सं शोधन अिधिनयम, 1971: इसके ारा सं सद ारा एक नया
अनु े द-31(ग) जोड़ा गया एवं दो व था दीः
- अनु े द-39(ख) और (ग) सं बंधी िविध को अनु े द 14 एवं 19 पर
वरीयता दी गई।
- इस सं बंधी कानू न को ायालय म इस आधार पर चु नौती नहीं दी जा
सकती िक यह अनु े द-39 (ख) और (ग) को भावी नहीं करता।
केशवान भारती बनाम केरल रा , 1973: इस वाद म ायालय के उपरो
दू सरे ावधान को असं वैधािनक एवं अवै ध घोिषत कर िदया एवं थम ावधान को
सं वैधािनक एवं वै ध माना।
- इस वाद म व था की गई िक सं सद मू ल अिधकारों म सं शोधन कर सकती है ,
पर ु मू ल ढाँ चे को नहीं बदल सकती।
42वाँ सं िवधान सं शोधन अिधिनयम, 1976: इसके ारा सभी िनदे शक त ों को
मू ल अिधकारों पर ाथिमकता दी गई।
िमनवा िम बनाम भारत सं घ, 1980: इस वाद म ायालय ने 42व सं िवधान
सं शोधन को असं वैधािनक एवं अवै ध घोिषत कर िदया एवं केशवान भारती वाद म
िदये गए िनणय को वै ध ठहराया गया।
- इस वाद म ायालय ने मू ल अिधकार और िनदे शक त ों को एक-दू सरे का
पू रक माना।

[13]िन िल खत म से कौन-से मामले /वाद ने भारतीय सं सद को 24व सं िवधान सं शोधन


अिधिनयिमत करने का अवसर िदया?
A) शं करी साद वाद

B) गोलकनाथ वाद

C) केशवान भारती वाद

D) च ाकम दोराइराजन वाद

Hide Answer -
उ र (b)
ा ाः गोलकनाथ बनाम पं जाब रा , 1967 वाद ने सं सद को 24व और 25व
सं िवधान सं शोधन अिधिनयम, 1971 अिधिनयिमत करने का अवसर िदया।

[14] े निवल ऑ न के अनु सार िन िल खत म से कौन-सा/से भारतीय सं िवधान का


अ ःकरण/मू ल आ ा है /ह?
1. मू ल अिधकार
2. रा के नीित-िनदे शक त
3. मू ल क
नीचे िदये गए कूट का योग कर सही उ र का चयन कीिजयेः

A) केवल 1

B) केवल 2

C) केवल 1 और 2

D) 1, 2 और 3

Hide Answer -
उ र (c)
ा ाः े निवल ऑ न ने ावना, मू ल अिधकारों और नीित-िनदे शक िस ा ों को
भारत के सं िवधान की मू ल आ ा/अ ःकरण कहा है ।

[15] ‘रा के नीित-िनदे शक िस ा एक ऐसा चे क है , जो बक म है , िक ु उसका


भु गतान बक के सु िवधानु सार िकया जाता है । इस िस ा के सं बंध म ऐसा िवचार
िन िल खत म से िकसने िदया था?

A) के.टी. शाह

B) के. एम. मुं शी

C) डॉ. राजे साद

D) डॉ. भीमराव अ े डकर


Hide Answer -
उ र (a)
ा ाः के.टी.शाह ने रा के नीित िनदे शक िस ां तों को ‘अितरे क कमका ’ कहा
और इसकी तु लना एक ऐसे चे क से की, िजसका भु गतान बकों के सु िवधानु सार िकया
जाता है ।
इसके सं बंध म अ के िवचारः
(ii) टी.टी. कृ मचारीः भावनाओं का एक थायी कूड़ाघर।

(ii) के.सी. े यरः ल और आकां ाओं का घोषणा-प एवं धािमक


उपदे श।
(iii) सर आइवर जोिनं ः कमका ी आकां ा।

[16] िन िल खत कथनों पर िवचार कीिजयेः

1. सं घ या िकसी रा के कायकलाप से सं बंिधत से वाओं और पदों के िलये िनयु याँ


करने म, अनु सूिचत जाितयों, अनु सूिचत जनजाितयों के सद ों के दावों का शासन की
द ता बनाए रखने की सं गित के अनु सार ान रखा जाएगा।
2. े क रा या रा के भीतर े क थानीय ािधकारी भाषाई अ सं क-वग
के बालकों को िश ा के ाथिमक र पर मातृ भाषा म िश ा की पया सु िवधाओं की
व था करने का यास करे गा।
3. सं घ का यह क होगा िक वह िह ी भाषा का सार बढ़ाए, उसका िवकास करे ,
िजससे वह भारत की सामािजक सं ृ ित के सभी त ों की अिभ का मा म बन
सके।
4. रा अनु सूिचत जाितयों, अनु सूिचत जनजाितयों और अ दु बल-वग के सामािजक
अ ाय और सभी कार के शोषण से उनकी सं र ा करे गा।
उपयु म से कौन-सा/से कथन रा के नीित-िनदे शक िस ा ों म शािमल है /ह?

A) केवल 1

B) केवल 4

C) केवल 1 और 3

D) केवल 2, 3 और 4

Hide Answer -
उ र (b)
ा ाः उपयु म से केवल कथन-4 रा के नीित-िनदे शक िस ा ों म शािमल है ।

कथन 1, 2 और 3 रा के नीित-िनदे शक िस ा ों म शािमल तो नहीं है , िक ु


भाग-4 के अित र ये सं िवधान के अ भागों म शािमल रा के िलये िनदश ह।
कथन-1 का उ े ख सं िवधान के भाग-16 के अनु े द-335 म िकया गया है ।
कथन-2 और 3 का उ े ख सं िवधान के भाग-17 के अनु े द 350 (क) एवं 351
म िकया गया है ।
[17]रा के नीित-िनदे शक िस ा के उ े ों के सं दभ म िन िल खत कथनों पर िवचार
कीिजयेः
1. एक लोक क ाणकारी रा की थापना करना।
2. एक धािमक रा की थापना करना।
3. सामािजक-आिथक ाय को सु िनि त करना।
4. एक पं थ-िनरपे रा की थापना करना।
नीचे िदये गए कूट का योग कर सही उ र का चयन कीिजयेः

A) केवल 1 और 3

B) केवल 1, 2 और 3

C) केवल 1, 3 और 4

D) 1, 2, 3 और 4

Hide Answer -
उ र (c)
ा ाः उपयु म से कथन-2 रा के नीित-िनदे शक िस ा का उ े नहीं है ।

नीित-िनदे शक िस ा ों का सार त सं िवधान के अनु े द-38 म िदया गया है ।


इसम सं िवधान की ावना की िन सु नाई दे ती है ।
अनु े द-38 म कहा गया है िक रा लोक क ाण की अिभवृ के िलये
सामािजक, आिथक और राजनै ितक ाय ारा सामािजक व था सु िनि त करे गा।
ये िस ा लोकतां ि क समाजवादी रा का खाका खींचते ह, िजसका ल
सामािजक और आिथक ाय दान करना है तथा ये लोक क ाणकारी रा की
थापना का माग श करते ह।

[18]भारतीय सं िवधान म विणत मू ल क ों के सं दभ म िन िल खत कथनों पर िवचार


कीिजये ;
1. ये केवल नाग रकों के िलये ह।
2. नीित-िनदे शक त ों के समान गै र- ायोिचत ह।
3. इसका उपबं ध मू ल सं िवधान म नहीं िकया गया है ।
उपयु म से कौन-सा/से कथन सही है /ह?

A) केवल 1

B) केवल 3

C) केवल 2 और 3

D) 1, 2 और 3
Hide Answer -
उ रः (d)
ा ाः मू ल क के सं बंध म उपयु तीनों कथन सही ह।

मू ल क गै र-नाग रकों/िवदे िशयों को ा नहीं ह।


ये ायालय ारा वतनीय नहीं ह। ायालय इसके िलये कोई रट नहीं िनकाल
सकता।
इ 42व सं िवधान सं शोधन अिधिनयम, 1976 के ारा जोड़ा गया।

[19] मू ल क ों के सं दभ म िन िल खत म से कौन-सा कथन सही नहीं है ?

A) ण िसं ह सिमित के िसफा रश पर इ सं िवधान म जोड़ा गया।

भारत के सं िवधान म इसे आयरलड के सं िवधान से भािवत होकर शािमल िकया


B)
गया।

C) मू ल प म दस मू ल क ों को भारत के सं िवधान म शािमल िकया गया है ।

86व सं िवधान सं शोधन अिधिनयम, 2002 के ारा ारहवाँ मू ल क सं िवधान


D)
म जोड़ा गया।

Show Answer +

[20] मू ल क ों के सं दभ म िन िल खत कथनों पर िवचार कीिजये :


1. इनका वणन सं िवधान के कुल ारह अनु े दों म िकया गया है ।
2. 42व सं िवधान सं शोधन अिधिनयम, 1976 के ारा इसे जनता दल की सरकार ने
सं िवधान ारा जोड़ा गया है ।
3. इसके ि या यन के िलये सं सद िविध बना सकती है ।
उपयु म से कौन-सा/से कथन सही है /ह?

A) केवल 2

B) केवल 3

C) केवल 2 और 3

D) 1, 2 और 3
Hide Answer -
उ रः (b)

ा ाः मू ल क ों के सं बंध म उपयु कथन 1 और 2 गलत ह, ोंिक इनका वणन


सं िवधान के केवल एक अनु े द-51(A) म िकया गया है , िजसे 42व सं िवधान सं शोधन
अिधिनयम ारा जोड़ा गया है ।
42व सं िवधान सं शोधन अिधिनयम, 1976 के ारा कां े स सरकार के ारा इसे
भारत के सं िवधान म जोड़ा गया।
सं सद इसके ि या यन के िलये िविध बना सकती है और इस सं बंध म सं सद ने
कई िविधयाँ बनाई ह।
[1] िन िल खत यु ों म से कौन-सा एक सही सु मेिलत नहीं है ?

भारतीय सं िवधान का िवषय भाग

A. नाग रकता भाग-II

B. मू ल अिधकार भाग-III

C. रा के नीित-िनदे शक त भाग-IV

D. मू ल क भाग-V

A) A

B) B

C) C

D) D

Hide Answer -
उ रः (d)

ा ाः िवक (d) सही सु मेिलत नहीं है , ोंिक 42व सं िवधान सं शोधन अिधिनयम, 1976

के ारा एक नया भाग-IV(क) जोड़ा गया और इसम मू ल क ों का उपबं ध िकया गया है ।

[2] मू ल क ों के सं दभ म िन िल खत म से कौन-सा कथन सही नहीं है ?

A) इ ले खों (Writs) के मा म से भावी बनाया जा सकता है ।

B) सं िवधान सं शोधन के ारा इनकी सं ा बढ़ाई जा सकती है ।

C) अ िविधयों की ा ा के िलये इनका उपयोग िकया जा सकता है ।

िकसी िविश क का पालन करना सं वैधािनक कानू न े म आता है ,


D)
िजसे ायालय सु िनि त करता है ।

Hide Answer -
उ रः (a)

ा ाः मू ल क ों के सं बंध म िवक (a) गलत है , ोंिक मू ल क ायालय ारा

वतनीय नहीं ह। ायालय इ भावी बनाने के िलये रट/ ले ख नहीं िनकाल सकता।

सं िवधान सं शोधन अिधिनयम के ारा इनकी सं ा बढ़ाई जा सकती है , जै सा िक 86व

सं िवधान सं शोधन अिधिनयम, 2002 के ारा बढ़ाई गई है ।

अ िविधयों की ा ा के िलये भी ायलय ारा इनका उपयोग िकया जा सकता है ।

[3] िन िल खत कथनों पर िवचार कीिजये :

1. मू ल क मौिलक अिधकारों का भाग है ।


2. मू ल क मौिलक अिधकारों का भाग नहीं है ।
3. मू ल क नीित-िनदे शक त ों का िह ा है ।
4. मू ल क नीित-िनदे शक त ों का िह ा नहीं है ।

उपयु म से कौन-सा/से कथन सही है /ह?

A) केवल 1

B) केवल 3

C) केवल 1 और 3

D) केवल 2 और 4

Hide Answer -
उ रः (d)

ा ाः उपयु कथन 1 और 3 सही नहीं ह, ोंिक 42व सं िवधान सं शोधन अिधिनयम,

1976 के ारा सं िवधान म एक नया भाग-IV(क) और एक नया अनु े द-51(A) जोड़ा गया

और इसम मू ल क ों को शािमल िकया गया है । अतः मू ल क न तो मू ल अिधकारों का

िह ा ह और न ही नीित-िनदे शक त ों का। ण िसं ह सिमित ने िसफा रश की थी िक सं िवधान

म मू ल क ों के िलये एक अलग पाठ होना चािहये ।

[4] पयावरण का सं र ण और सं व न’ के सं दभ म िन िल खत कथनों म से कौन-सा

सही है ?

A) इसकी चचा केवल रा के नीित-िनदे शक त ों के अं तगत की गई है ।

B) इसकी चचा केवल मू ल क ों के अं तगत की गई है ।

इसकी चचा रा के नीित-िनदे शक त ों और मू ल क ों दोनों म की गई


C)
है ।

इसकी चचा न तो रा के नीित िनदे शक त ों के अं तगत की गई है और न


D)
ही मू ल क ों के अं तगत।

Hide Answer -
उ रः (c)

ा ाः पयावरण का सं र ण और सं व न के सं बंध म उपबं ध रा के नीित-िनदे शक त ों

और मू ल क ों दोनों म की गई है । अं तर केवल यह है िक एक रा का क है तो दू सरा

नाग रक का।

नीित-िनदे शक त ों के अनु े द-48क म और मू ल क ों के अनु े द 51 क (छ) म

पयावरण का सं र ण और सं व न के सं बंध म ावधान िकया गया है ।

[5] िन िल खत पर िवचार कीिजये :

1. रा गान
2. रा ीय ज
3. अनु सूिचत जाित/अनु सूिचत जनजाित
4. व जीव/ ाणी

उपयु म से िकसका/िकनका सं र ण करना े क नाग रक का क होगा?

A) केवल 1

B) केवल 4

C) केवल 1, 2 और 4

D) 1, 2, 3 और 4

Hide Answer -
उ रः (b)

ा ाः उपयु म से केवल (4) सही है , ोंिक अनु े द 51(क) (छ) म कहा गया है िक

वन, झील, नदी और व जीव की र ा और सं व न करना े क नाग रक का क होगा।

रा गान और रा ीय ज के बारे म ावधान सं िवधान के अनु े द-51क (क) म िकया

गया है , िकंतु इसके सं र ण और सं व न की चचा नहीं, ब आदर करने की बात की

गई है ।

अनु सूिचत जाित और अनु सूिचत जनजाित की चचा मू ल कत ों के अं तगत नहीं की गई है ।

[6] िन िल खत कथनों पर िवचार कीिजये :

1. रा ीय मह के सं ारकों, थानों और व ु ओं की र ा करना।


2. सावजिनक सं पि को सु रि त रखना।
3. ाकृितक पयावरण की र ा और सं व न करना।
4. दे श की र ा करना।

उपयु म से कौन-सा/से मू ल क ों म शािमल नहीं है /ह?

A) केवल 1

B) केवल 2

C) केवल 1 और 4

D) केवल 2 और 3
Hide Answer -
उ रः (a)

ा ाः रा ीय मह के सं ारकों, थानों और व ु ओं का सं र ण सं बंधी ावधान सं िवधान के

अनु े द-49 म (रा के नीित िनदे शक त ) िकया गया है न िक मू ल क ों म।

सावजिनक सं पि को सु रि त रखने सं बंधी ावधान अनु े द-51क (झ) म िकया गया है ।

ाकृितक पयावरण का सं र ण एवं सं व न (िजसम वन, झील, नदी और व जीव आते

ह) करने सं बंधी ावधान अनु े द-51(A) छ म िकया गया है ।

दे श की र ा करने सं बंधी ावधान अनु े द 51क (घ) म िकया गया है ।

[7] ‘भारत की भु ता, एकता और अखं डता की र ा कर और उसे अ ु रख’। यह


उपबं ध िन िल खत म से िकसम िकया गया है ?

A) सं िवधान की ावना/उ े िशका

B) मू ल अिधकार

C) रा के नीित-िनदे शक िस ां त

D) मू ल क

Hide Answer -
उ रः (d)

ा ाः ‘दे श की भु ता, एकता और अखं डता की र ा कर और उसे अ ु रख’ का उपबं ध

सं िवधान के अनु े द-51क (ग) म िकया गया है ।

[8] सू ची-I को सू ची-II से सही सु मेिलत कीिजयेः

सू ची-I सू ची-II
(अनु े द 51क ) ( थान)

a. हमारी सामािसक सं ृ ित की गौरवशाली परं परा का 1.अनु े द 51क


प रर ण करना। (क)

2.अनु े द-51क
b. वै ािनक ि कोण का िवकास करना।
(ङ)

c. समरसता और समान ातृ की भावना का िनमाण 3.अनु े द-51क


करना। (च)

4.अनु े द-51क
d. सं िवधान का पालन करना।
(ज)

कूटः A B C D
A) 3 4 2 1

B) 3 2 4 1

C) 1 2 4 3

D) 1 4 2 3

Hide Answer -
उ रः (a)

ा ा: सू िचयों के सही सु मेलन िन िल खत ह-

• हमारी सामािसक सं ृ ित की गौरवशाली परं परा का प रर ण


अनु े द-51क (च)
करना।

• वै ािनक ि कोण का िवकास करना। अनु े द-51क (ज)

• समरसता और समान ातृ की भावना का िनमाण करना। अनु े द-51क (ङ)

• सं िवधान का पालन करना। अनु े द 51क (क)

[9] भारतीय सं िवधान ारा नाग रकों को द िन िल खत म से कौन-सा/से मू ल


क है /ह?

1. िमि त सं ृ ित की समृ िवरासत की र ा।


2. सामािजक अ ाय से कमजोर वग की र ा।
3. वै ािनक मनोदशा और खोज की भावना का िवकास।
4. वै य क और सामू िहक कायकलापों के सभी े ों म उ ृ ता के िलये
य ।

नीचे िदये गए कूटों का योग कर सही उ र चु िनये :

A) केवल 1 और 2

B) केवल 2

C) केवल 1, 3 और 4

D) 1, 2, 3 और 4
Hide Answer -
उ र (c)

ा ाः उपयु म से कथन 2 नाग रकों का मू ल क नहीं है , अिपतु रा के नीित-

िनदे शक त ों (अनु े द-46) म उपबं ध िकया गया है ।

िमि त सं ृ ित की समृ िवरासत की र ा का उपबं ध अनु े द-51क (च) म िकया गया

है ।

वै ािनक मनोदशा और खोज की भावना का िवकास का उपबं ध अनु े द-51क (ज) म

िकया गया है ।

वै य क और सामू िहक कायकलापों के सभी े ों म उ ृ ता के िलये य का उपबं ध

अनु े द-51क (ञ) म िकया गया है ।

[10]भारतीय सं िवधान के अनु े द-51क के अनु सार िन िल खत म से कौन-से


भारतीय नाग रकों के मू ल क ह?

1. समाजवाद, पं थ िनरपे तावाद और लोकतं म िव ास करना।


2. तं ता के िलये हमारे रा ीय आं दोलन को े रत करने वाले उ आदश
को दय म रखना।
3. भारत की सं भु ता, एकता और अखं डता की र ा करना।
4. समाज के गरीब और कमज़ोर वग की सहायता करना।
5. सं िवधान का पालन करना और रा ज तथा रा गान का आदर करना।

नीचे िदये गए कूट का योग कर सही उ र का चयन कीिजये -

A) केवल 1, 2 और 3

B) केवल 1, 4 और 5

C) केवल 2, 3 और 5

D) केवल 3, 4 और 5

Hide Answer -
उ रः (c)

ा ाः उपयु म से कथन 1 और 4 अनु े द-51क के अं तगत नाग रकों के मू ल क नहीं

ह।

कथन 2 का उपबं ध सं िवधान के अनु े द-51क (ख) म िकया गया है ।

कथन 3 का उपबं ध अनु े द-51क (ग) म िकया गया है ।

कथन 5 का उपबं ध अनु े द-51क (क) म िकया गया है ।

[11]यिद दे श की र ा के िलये मू ल क ों के अनु प सभी पु ष नाग रकों को से ना


म अिनवाय प से भत कर िलया जाए तो यह कानू न-
A) अिववे कपू ण होगा, चूँ िक इसम े णी का ावधान नहीं है ।

B) भे दभावपू ण होगा चूँ िक उ का आधार रखना होगा।

C) िलं ग के आधार पर भे दभावपू ण माना जाएगा।

D) सं िवधान के िकसी भी ावधान के ितकूल नहीं होगा।

Show Answer +

[12]भारत के सं िवधान म 86व सं िवधान सं शोधन अिधिनयम 2002 के ारा िश ा के


िवषय म उपबं ध मू ल अिधकारों, रा के नीित-िनदे शक िस ां तों और मू ल
क ों म िकया गया है । इस सं दभ म िन िल खत कथनों म से कौन-सा सही है ?

A) इसके सं बंध म उपबं ध सं िवधान के अनु े द-19(i) (क) म िकया गया है ।

B) इसके सं बंध म उपबं ध सं िवधान के अनु े द-51क (च) म िकया गया है ।

C) इसके सं बंध म उपबं ध सं िवधान के अनु े द-40 म िकया गया है ।

D) उपयु म से कोई नहीं।

Hide Answer -
उ रः (d)

ा ाः िश ा के सं बंध म उपबं ध मू ल अिधकारों के साथ-साथ नीित-िनदे शक िस ां तों और मू ल

क ों म िकया गया है , िकंतु इस सं बंध म उपयु तीनों िवक a, b, और c सही नहीं ह,

ोंिक-

मू ल अिधकारों म इसका उपबं ध अनु े द-21(A) म िकया गया है ।

नीित-िनदे शक िस ां तों म इसका उपबं ध अनु े द-45 म िकया गया है ।

मू ल क ों म इसका उपबं ध अनु े द-51A के (ट) म िकया गया है ।

उ े खनीय है िक सं सद ारा वष 2009 म अनु े द-21क के अनु सरण म एक

अिधिनयम बनाया गया और 1 अ ै ल, 2010 से यह अिधिनयम भावी हो गया।

[13]िन िल खत को सु मेिलत कीिजये :

सू ची-I सू ची-II
(अिधिनयम) (वष)

1. रा गौरव अपमान (िनवारण) अिधिनयम 1971

2. िविध िव ि याकलाप (िनवारण) अिधिनयम 1972

3. व जीव (सं र ण) अिधिनयम 1976

4. वन (सं र ण) अिधिनयम 1980

कूटः A B C D
A) 1 2 3 4

B) 1 3 2 4

C) 4 3 2 1

D) 4 2 3 1

Hide Answer -
उ र (b)

ा ाः वमा सिमित (1999) ने कुछ मू ल क ों की पहचान और उसके ि या यन के िलये

कानू नी ावधानों को लागू करने की व थाएँ की, जै से-

रा गौरव अपमान (िनवारण) अिधिनयम, 1971: यह अिधिनयम भारत के सं िवधान,

रा ीय ज और रा ीय गान के अनादर का िनवारण करता है ।

िविध िव ि याकलाप (िनवारण) अिधिनयम, 1976: िकसी सां दाियक सं गठन

को गै र-कानू नी घोिषत करने की व था करता है ।

व जीव (सं र ण) अिधिनयम, 1972: दु लभ और लु ाय जाितयों के ापार पर

ितबं ध लगाता है ।

वन (सं र ण) अिधिनयम, 1980: वनों की अिनयं ि त कटाई और वन भू िम के गै र-वन

उ े के िलये इ े माल पर रोक लगाता है ।

भारतीय दं ड सं िहता (IPC): रा ीय अखं डता के िलये पू वा ह से ता के िलये दं ड

का ावधान करता है ।

िसिवल अिधकार सं र ण अिधिनयम, 1955: जाित और धम से सं बंिधत अपराधों पर

दं ड की व था करता है ।

[14]सं िवधान का अनु े द-51क (ङ) भारत के सभी लोगों म समरसता और समान
ातृ की भावना के िनमाण का उपबं ध करता है । यह उपबं ध िन िल खत
भे दभाव से परे होने चािहये :

1. धम
2. भाषा
3. दे श
4. वग

उपयु म से कौन-से इसम शािमल ह?

A) केवल 1 और 2

B) केवल 1 और 4

C) केवल 1, 2 और 4

D) 1, 2, 3 और 4
Hide Answer -
उ रः (d)

ा ाः उपयु सभी का सं िवधान के अनु ै द-51क (ङ) म उपबं ध िकया गया है । इस

उपबं ध से परे े क नाग रक का क होगा िक वह भारत के सभी लोगों म समरसता और

समान भाईचारा की भावना का िनमाण करे तथा ऐसी थाओं का ाग करे जो यों के स ान

के िव है ।

[15]भारत के सं िवधान के भाग-IV (क) म विणत मू ल क ों के सं दभ म


िन िल खत कथनों पर िवचार कीिजये :
1. ये सदै व से सं िवधान का िह ा रहे ह।
2. सं िवधान सं शोधन के ारा इ सं िवधान म शािमल िकया गया है ।
3. ये सभी नाग रकों के िलये बा कारी ह।

उपयु कथनों म कौन-सा/से सही है /ह?

A) केवल 1

B) केवल 2

C) केवल 1 और 3

D) केवल 2 और 3

Hide Answer -
उ रः (b)

ा ाः मू ल क ों के सं बंध म कथन 1 और 3 सही नहीं है , ोंिक-

इसे 42व सं िवधान सं शोधन अिधिनयम, 1976 के ारा भारत के सं िवधान म शािमल िकया

गया है ।

ये नाग रकों के िलये बा कारी नहीं ह, अिपतु भारत के े क नाग रक का क है िक

इनका पालन कर।

[16]िन िल खत कथनों पर िवचार कीिजयेः

1. सं घ की कायपािलका श रा पित म िनिहत है ।


2. रा पित कायपािलका श का योग यं या अपने अधीन थ
अिधका रयों के ारा करे गा।
3. सं घ के र ा बलों का सव समादे श (Command) रा पित म िनिहत
है ।

उपयु म से कौन-सा/से कथन भारत के रा पित के िवषय म सही है /ह?


A) केवल 1

B) केवल 3

C) केवल 1 और 3

D) 1, 2 और 3

Hide Answer -
उ रः (d)

ा ाः उपयु तीनों कथन सही ह।

सं िवधान के अनु े द-53(1) म उपबं ध है िक सं घ की कायपािलका श रा पित म

िनिहत होगी और वह इस श का योग यं या अपने अधीन थ अिधका रयों के ारा

करे गा।

अनु े द-53(2) म उपबं ध है िक सं घ के र ा बलों का सव समादे श (Command)

रा पित म िनिहत होगा और उसका योग िविध ारा करे गा।

[17]रा पित के िनवाचक मं डल म शािमल होते ह:

1. लोकसभा के िनवािचत सद
2. रा सभा के िनवािचत सद
3. रा िवधानसभा के िनवािचत सद
4. रा िवधान प रषद् के िनवािचत सद
5. िद ी और पु डुचे री सं घ रा े की िवधानसभाओं के िनवािचत सद

िनचे िदये गये कूट का योग कर सही उ र का चयन कीिजये :

A) केवल 1, 2 और 3

B) केवल 1, 2, 3 और 4

C) केवल 1, 2, 3 और 5

D) उपरो सभी

Hide Answer -
उ रः (c)

ा ाः रा पित के िनवाचक मं डल/िनवाचकगण म रा िवधान प रषद के सद भाग नहीं

ले ते ह।

भारतीय सं िवधान के अनु े द-54 म रा पित के िनवाचक मं डल के सद ों के सं बंध म

उपबं ध िकया गया है ।

िद ी रा ीय राजधानी रा े और पु डुचे री सं घ रा े की िवधानसभाओं के िनवािचत

सद ों को 70व सं िवधान सं शोधन अिधिनयम, 1992 ारा िनवाचकगण का सद बनाया

गया है ।

[18]
भारत के रा पित के िनवाचन के सं दभ म िवचार कीिजयेः

1. भारत का ज जात नाग रक होना चािहये ।


2. ू नतम आयु 30 वष होनी चािहये ।
3. लोकसभा और रा सभा दोनों के सद िनवािचत होने के िलये यो होना
चािहये ।
4. पु निनवाचन के सं बंध म सं िवधान मौन है ।

उपयु म से कौन-सा/से सही है /ह?

A) केवल 1

B) केवल 3

C) केवल 1, 3 और 4

D) उपयु म से कोई नहीं

Hide Answer -
उ रः (d)

ा ाः उपयु म से कोई सही नहीं है ।

सं िवधान के अनु े द-58 म रा पित िनवािचत होने के िलये अहताएँ उ खत ह।

उसे िकसी भी प म भारत का नाग रक होना चािहये , न िक ज जात नाग रक।

िनवाचन के िलये ू नतम आयु 35 वष है न िक 30 वष।

उसे केवल लोकसभा सद िनवािचत होने के िलये यो होना चािहये ।

अनु े द-57 म रा पित के पु निनवािचत होने के िलये उपबं ध िकया गया है ।

[19]भारत के रा पित के स भ म िन िल खत कथनों पर िवचार कीिजयेः

1. चु नाव के नामां कन के िलये 100 ावक और 100 अनु समथक होने


चािहये ।
2. िनवाचन एकल सं मणीय मत ारा िकया जाता है ।
3. िनवाचन खु ले मतदान ारा होता है ।

उपयु म से कौन-सा/से सही है /ह?

A) केवल 1

B) केवल 2

C) केवल 1 और 2

D) 1, 2 और 3
Hide Answer -
उ रः (b)

ा ाः रा पित के सं बंध म उपयु कथन 1 और 3 सही नहीं ह।

रा पित के चु नाव के नामं कन के िलये कम से कम 50 ावक और 50 अनु समथक/

अनु मोदक होने चािहये ।

अनु े द-55 (3) म उपबं ध है िक रा पित का िनवाचन अनु पाितक ितिनिध प ित के

अनु सार एकल सं मणीय मत ारा होगा।

रा पित का िनवाचन गु मतदान के ारा होता है ।

[20]सं सद अथवा िवधानसभा का सद रा पित िनवािचत हो सकता है ,िकंतु वह सं सद


अथवा िवधानसभा का सद नहीं रहे गा?

A) पद हण करने के तु र पू व से

B) पद हण करने के तु र बाद से

C) पद हण करने की तारीख से

D) पद हण करने से छः माह के अं दर

Hide Answer -
उ रः (c)

ा ाः सं िवधान के अनु े द-59(1) म उपबं ध है िक कोई सं सद अथवा िवधान सभा सद

रा पित के पद हण करने की तारीख से सं सद या रा िवधानसभा का सद नहीं रहे गा।


[1] भारत के रा पित के िनवाचन की रीित के सं दभ म िन िल खत पर िवचार कीिजयेः

1. िवधानसभा के एक सद का मत मू = रा की जनसं ा / िवधान


सभा के िनवािचत सद ों की कुल सं ा x 100
2. सं सद के एक सद का मत मू = सभी रा ों के िवधानसभा सद ों
के मत मू / सं सद के िनवािचत सद ों की कुल सं ा

उपयु म से कौन-सा/से सही है /ह?

A) केवल 1

B) केवल 2

C) 1 और 2 दोनों

D) न तो 1 और न ही 2

Hide Answer -
उ रः (b)

ा ाः

सं िवधान के अनु े द-55 म रा पित के िनवाचन का तरीका िदया गया है । इस सं बंध म

उपयु कथन 1 सही नहीं है , ोंिक 1/100 के थान पर 1/1000 होगा।

उ े खनीय है िक िवधानसभा के िनवािचत सद ों के मतों की सं ा ात करने के िलये ,

उस रा की जनसं ा को, उस रा की िवधानसभा के िनवािचत सद ों की सं ा को

1000 से गु णा करने पर ा सं ा से भाग िदया जाता है । अतः सही तरीका िन िल खत

प म होगा-

िवधानसभा के एक सद का मत मू = रा की जनसं ा / िवधान सभा

के िनवािचत सद ों की कुल सं ा x 1000

[2] भारत के रा पित के िनवाचन म िवधानसभा के एक सद का मत मू उस


रा की जनसं ा को रा की िवधानसभा के िनवािचत सद ों की कुल सं ा
ारा िवभािजत कर ा भागफल के 1000 गु णकों के बराबर होता है । इस
सं दभ म यु ‘जनसं ा’ से ता य वतमान म िकस वष की जनगणना ारा
िनि त जनसं ा से है ?

A) जनगणना, 1971

B) जनगणना, 1981

C) जनगणना, 1991

D) जनगणना, 2001
Hide Answer -
उ रः (a)

ा ाः 42व सं िवधान सं शोधन अिधिनयम, 1976 के तहत रा पित के िनवाचन म जनसं ा से

आशय 1971 की जनगणना ारा िनि त जनसं ा से है । इसे वष 2000 तक के िलये िनधा रत

िकया गया था, िकंतु 84व सं िवधान सं शोधन अिधिनयम, 2002 के ारा इसे वष 2026 तक के

िलए बढ़ा िदया गया है ।

[3] रा पित के िनवाचन म रा का मु मं ी मतदान करने के िलये यो नहीं है ,


यिद वहः

A) यं ाशी है ।

B) रा की िवधानसभा म ब मत िस करना शे ष है ।

C) िवधानमं डल के िन सदन का सद है ।

D) िवधानमं डल के उ सदन का सद है ।

Hide Answer -
उ रः (d)

ा ाः सं िवधान के अनु े द-54 म रा पित के िनवाचन के िलये िनवाचक मं डल का उपबं ध

िकया गया है , िजसम रा के िवधानमं डल के उ सदन (िवधान प रषद) के सद िनवाचक

मं डल म भाग नहीं ले सकते ह। यिद मु मं ी उ सदन का सद है , तो वह रा पित के

िनवाचन म मतदान नहीं कर सकता है ।

[4] िन िल खत कथनों पर िवचार कीिजयेः

1. रा पित के िनवाचन से सं बंिधत सभी शं काओं और िववादों की जाँ च और


िनणय िनवाचन आयोग करता है ।
2. रा पित की िनयु अवै ध होने की दशा म उनके ारा पू व म िदये गए
काय अवै ध हो जाएँ गे ।

उपयु म से कौन-सा/से कथन सही है /ह?

A) केवल 1

B) केवल 2

C) 1 और 2 दोनों

D) न तो 1 और न ही 2
Hide Answer -
उ रः (d)

ा ाः उपयु दोनों कथन सही नहीं ह।

सं िवधान के अनु े द-71(1) म उपबं ध है िक रा पित चु नाव से सं बंिधत शं काओं और

िववादों की जाँ च सव ायालय ारा की जाएगी और उसका िनणय अं ितम होगा

सव ायालय, रा पित की िनयु को अवै ध घोिषत कर दे ता है तो पू व म रा पित

ारा िकये गए काय अवै ध नहीं होंगे। वे भावी बने रहगे [अनु े द-71(2)]

इस सं िवधान के अधीन रहते ए सं सद रा पित के िनवाचन से सं बंिधत िकसी िवषय का

िविनयमन िविध ारा कर सकेगी [अनु े द-71(3)] ।

रा पित के चु नाव को इस आधार पर चु नौती नहीं दी जा सकती िक िनवाचक मं डल अपू ण

है [अनु े द-71 (4)] ।

[5] िन िल खत कथनों पर िवचार कीिजयेः

रा पित चु नाव म कोई उ ीदवार कुल डाले गए मतों का 1/10 भाग ा


नहीं कर पाता है तो उसकी ज़मानत रािश ज़ हो जाएगी।
रा पित के ेक उ ीदवार को एक िनि त रािश भारतीय रज़व बक म
ज़मानत रािश के प म जमा करनी होगी।

उपयु म से कौन-सा/से कथन सही है /ह?

A) केवल 1

B) केवल 2

C) 1 और 2 दोनों

D) न तो 1 और न ही 2

Hide Answer -
उ रः (b)

ा ाः उपयु म से कथन 1 सही नहीं है ।

रा पित के चु नाव म ेक उ ीदवार को एक िनि त रािश (वतमान म 15000 पये )

भारतीय रज़व बक म ज़मानत रािश के प म जमा करनी पड़ती है ।

कोई उ ीदवार कुल डाले गए मतों का 1/6 भाग ा करने म असमथ रहता है तो

उसकी ज़मानत रािश ज़ हो जाएगी, िजसे ज़मानत ज़ होना भी कहते ह।

[6] िन िल खत पर िवचार कीिजयेः

1. सं िवधान की दू सरी अनु सूची म शपथ/ ित ान के सं बंध म उपबं ध है ।


2. सं िवधान की तीसरी अनु सूची म उपल यों, भ ों और िवशे षािधकारों का
उपबं ध है ।
3. भारत का रा पित शपथ/ ित ान उप-रा पित के सम ले ता है ।
4. उप-रा पित की अनु प थित म भारत के मु ायमू ित के सम रा पित
शपथ/ ित ान ले ता है ।

उपयु म से कौन-से कथन सही ह?

A) केवल 1 और 2

B) केवल 1, 2 और 3

C) उपयु सभी

D) उपयु म से कोई नहीं।

Hide Answer -
उ रः (d)

ा ाः उपयु म से कोई भी कथन सही नहीं है ।

सं िवधान के अनु े द-59(3) म रा पित की उपल यों, भ ों और िवशे षािधकारों का

उपबं ध है । वतमान म रा पित का वे तन ितमाह 1.50 पये लाख है । उ वे तन, भ े

एवं उप याँ सं िवधान की दू सरी अनु सूची के तहत िदए जाते ह।

सं िवधान की तीसरी अनु सूची म शपथ/ ित ा के सं बंध म उपबं ध है ।

सं िवधान के अनु े द-60 म उपबं ध है िक रा पित भारत के मु ायाधीश के सम

शपथ ले गा। मु ायाधीश की अनु प थित म उ तम ायालय के उपल वर

ायाधीश के सम शपथ ले गा।

[7] "सं िवधान और िविध का परी ण, सं र ण और ितर ण क ँ गा" के प म


िन िल खत म कौन शपथ हण करता है ?

A) भारत के रा पित

B) भारत के मु ायमू ित

C) भारत के उपरा पित

D) भारत के िनयं क-महाले खापरी क

Hide Answer -
उ रः (a)

ा ाः उपयु शपथ/ ित ा भारत के रा पित ारा ली जाती है ।

सं िवधान के अनु े द-60 म रा पित के शपथ के सं बंध म उपबं ध है ।

[8] िन िल खत कथनों पर िवचार कीिजयेः

1. भारत का रा पित शपथ हण करने की तारीख से पाँ च वष की अविध


तक पद धारण करे गा।
2. रा पित की पदाविध समा के प ात् वह अपने पद पर नहीं रहे गा, चाहे
उसके उ रािधकारी ने पद हण नहीं िकया हो।
3. रा पित अपना ाग प उप-रा पित को सौंपेगा।
4. रा पित ारा उप-रा पित को सं बोिधत ाग प की सू चना उसके ारा
लोकसभा के अ को तु रंत दी जाएगी।

उपयु म से कौन-से कथन भारत के रा पित के पदाविध के सं बंध म सही ह?

A) केवल 1 और 3

B) केवल 1, 3 और 4

C) केवल 3 और 4

D) 1, 2, 3 और 4

Hide Answer -
उ रः (c)

ा ाः रा पित की पदाविध के सं बंध म उपयु कथन 1 और 2 सही नहीं ह

सं िवधान के अनु े द-56(1) म उपबं ध है िक रा पित अपना पद हण करने की तारीख

से 5 वष की अविध तक पद धारण करे गा।

अनु े द-56(1)(क) म उपबं ध है िक रा पित अपना ाग प उप-रा पित को सौंपेगा।

अनु े द-56(1)(ग) म उपबं ध है िक रा पित अपने पद की अविध समा हो जाने पर

भी पद धारण करता रहे गा, जब तक िक उसका उ रािधकारी पद हण न कर ले ।

अनु े द-56(2) म उपबं ध है िक उप-रा पित ाग प की सू चना तु रंत लोकसभा अ

को दे गा।

[9] िन िल खत कथनों पर िवचार कीिजयेः

केवल सं िवधान के अित मण पर भारत के रा पित पर महािभयोग चलाया


जा सकता है ।
सं िवधान के अनु े द-56 1(ख) म उपबं ध है िक रा पित पर महािभयोग
अनु े द-61 के तहत चलाया जाएगा।

उपयु म से कौन-सा/से कथन सही है /ह?

A) केवल 1

B) केवल 2

C) 1 और 2 दोनों

D) न तो 1 और न ही 2

Hide Answer -
उ रः (c)

ा ाः उपयु दोनों कथन सही ह।

सं िवधान के अनु े द-561(ख) म उपबं ध है िक सं िवधान का अित मण करने पर रा पित

को अनु े द-61 म उपबं िधत रीित से महािभयोग ारा पद से हटाया जा सकेगा।


[10]भारतीय सं िवधान के अनु े द-61 के सं दभ म िवचार कीिजयेः

1. रा पित पर महािभयोग चलाने की ि या का उपबं ध है ।


2. केवल लोकसभा रा पित पर महािभयोग का आरोप लगा सकती है ।
3. अभी तक केवल एक बार महािभयोग का योग आ है ।

उपयु म से कौन-सा/से कथन सही है /ह?

A) केवल 1

B) केवल 1 और 2

C) केवल 1 और 3

D) 1, 2 और 3

Hide Answer -
उ रः (a)

ा ाः उपयु म से कथन 2 और 3 सही नहीं ह।

सं िवधान के अनु े द-61 म रा पित पर महािभयोग चलाने की ि या के सं बंध म उपबं ध

है ।

रा पित पर महािभयोग का आरोप सं सद के िकसी भी सदन ारा लगाया जा सकता है न

िक केवल लोकसभा ारा।

महािभयोग का योग अभी तक एक बार भी नहीं आ है ।

[11]भारत के रा पित पर महािभयोग के सं दभ म िन िल खत म से कौन-सा कथन

सही है ?

A) रा पित पर महािभयोग चलाने की ि या अ - ाियक है ।

सं सद के दोनों सदनों के नामां िकत सद , िज ोंने रा पित के चु नाव म भाग नहीं


B)
िलया था, रा पित के महािभयोग म भाग नहीं ले सकते ह।

रा िवधानसभाओं के िनवािचत सद और िद ी एवं पु दुचेरी के -शािसत रा


C) की िवधानसभाओं के िनवािचत सद , िज ोंने रा पित के चु नाव म भाग िलया था,
महािभयोग म भाग ले सकते ह।

रा पित पर िजस सदन ारा महािभयोग का आरोप लगाया गया है , उन


D) आरोपों पर सदन के दो ितहाई सद ों के ह ा र होने चािहये और रा पित
को 14 िदन का नोिटस िल खत प म दे ना चािहये ।
Hide Answer -
उ रः (a)

ा ाः उपयु म से कथन b, c और d सही नहीं ह।

कथन (b) के सं बंध म रा पित के महािभयोग म सं सद के दोनों सदनों के नामां िकत

सद भाग ले ते ह।

कथन (c) के सं बंध म रा की िवधानसभाओं के िनवािचत सद और िद ी व पु दुचेरी

िवधानसभाओं के िनवािचत सद महािभयोग म भाग नहीं ले ते ह।

कथन (d) के सं बंध म रा पित पर िजस सदन ारा आरोप लगाया गया है , उस आरोप पर

सदन के एक-चौथाई सद ों के ह ा र होने चािहये और रा पित को 14 िदन का नोिटस

दे ना चािहये ।

[12]िन िल खत कथनों पर िवचार कीिजयेः

भारत के रा पित को केवल महािभयोग के आधार पर पद से हटा सकते ह।


भारत के रा पित पर महािभयोग लगाने का अिधकार सं सद के दोनों सदनों
के पास है ।
भारत के रा पित पर महािभयोग का ाव उस सदन के उप थत और मत
दे ने वाले सद ों के दो-ितहाई ब मत ारा पा रत होना चािहये ।

उपयु म से कौन-सा/से कथन सही है /ह?

A) केवल 1

B) केवल 3

C) केवल 1 और 2

D) 1, 2 और 3
Hide Answer -
उ रः (c)

ा ाः उपयु म से कथन (3) सही नहीं है ।

भारत के रा पित को उसके पद से केवल महािभयोग (अु न े द-61) के आधार पर हटा

सकते ह। रा पित पर महािभयोग केवल सं िवधान के उ ं घन या अित मण के आधार पर

चलाया जा सकता है ।

सं सद का कोई भी सदन भारत के रा पित पर महािभयोग का आरोप लगा सकता है । एक

सदन महािभयोग का आरोप लगाता है तो दू सरा सदन उन आरोपों की जाँ च करता है ।

िजस सदन के ारा महािभयोग का आरोप लगाया जाता है , उस आरोप पर उस सदन के

एक-चौथाई सद ों के ह ा र होने चािहये और रा पित को 14 िदनों का नोिटस दे ना

चािहये ।

रा पित पर महािभयोग का आरोप सं बंधी ाव सदन की कुल सद सं ा के दो-

ितहाई ब मत ारा (न िक उप थत और मतदान करने वाले सद ों का दो-ितहाई ब मत

ारा) पा रत होना चािहये ।

दू सरा सदन इन आरोपों की जाँ च करे गा। यिद जाँ च सािबत हो जाती है और महािभयोग का

ाव सदन की कुल सद सं ा के दो-ितहाई ब मत ारा पा रत हो जाता है तो

ाव पा रत िकये जाने की तारीख से रा पित को पद से हटना होगा। िजस सदन म

महािभयोग के आरोप की जाँ च चल रही होती है , उस सदन म रा पित को उप थत होने

और अपना ितिनिध करने का अिधकार होगा।

[13]यिद भारत के रा पित ने अपनी पदाविध पाँ च वष पू री कर ली है , तब:

A) वह िकसी भी दशा म अपने पद पर नहीं रहे गा।

उ रािधकारी ारा पद हण करने तक उप-रा पित कायवाहक रा पित के प म


B)
पद हण करे गा।

C) पदाविध पू री करने के तु र बाद नया चु नाव करा ले ना चािहये ।

रा पित अपने पद पर बना रहे गा जब तक िक उसका उ रािधकारी पद हण न


D)
कर ले ।

Hide Answer -
उ रः (d)

ा ाः उपयु म से कथन (d) सही है ।

सं िवधान के अनु े द-56(1)(ग) म उपबं ध है िक वतमान रा पित अपने पद पर तब तक

बने रहगे , जब तक उनका उ रािधकारी पद हण न कर ले । सं िवधान के

अनु े द-62(1) म उपबं ध है िक रा पित की पदाविध पू ण होने से पहले नया चु नाव करा

ल।

[14]भारत के रा पित की मृ ु , पद ाग या पद से हटाए जाने या अ कारणों से ई


पद की र के सं दभ म िन िल खत कथनों पर िवचार कीिजयेः
1. र की तारीख के प ात् यथाशी या े क दशा म छःमाह से पहले
रा पित का िनवाचन कर िलया जाएगा।
2. भारत के उप-रा पित छः माह के िलये कायवाहक रा पित के प म
काय करगे ।
3. डॉ. जािकर सै न एकमा भारत के रा पित थे , िजनकी मृ ु उनके
कायकाल के दौरान ई थी।

उपयु म से कौन-सा/से कथन सही नहीं है /ह?

A) केवल 1

B) केवल 3

C) केवल 2 और 3

D) 1, 2 और 3

Hide Answer -
उ रः (b)

ा ाः उपयु म से कथन (3) गलत है ।

सं िवधान के अनु े द-62(2) म उपबं ध है िक रा पित के मृ ु , पद ाग या पद से हटाए

जाने या अ कारणों से ई र को र की तारीख के प ात् यथाशी अथवा ेक

दशा म छः माह से पू व रा पित का िनवाचन करा ले ना चािहये ।

भारत के उप-रा पित छः माह तक कायवाहक रा पित के प म काय करे ग। उप-

रा पित की अनु प थित म भारत के मु ायमू ित एवं मु ायमू ित के अनु प थित म

उ तम ायालय के व र तम ायिधक कायवाहक रा पित के प म काय करगे तथा

उसके क ों का िनवाह करगे । ऐसी थित म उसे रा पित की सभी श याँ व

उ ु याँ ा होंगी।

अब तक भारत म दो रा पित डॉ. जािकर सै न व फख ीन अली अहमद ऐसे ए ह,

िजनका िनधन उनके कायकाल के दौरान आ था।

डॉ. जािकर सै न की मृ ु के प ात् उप-रा पित वी.वी.िग र कायवाहक रा पित बने और

उनके ागप के प ात् भारत के मु ायमू ित एम. िहदायतु ाह कायवाहक रा पित

बने ।

अनु े द-62(2) के तहत िनवािचत रा पित अनु े द-56 के उपबं ध के अधीन रहते ए

अपने पद हण करने की तारीख से पाँ च वष तक पद धारण करे गा न िक शे ष अविध के

िलये ।

[15]भारत के रा पित के सं दभ म िन िल खत कथनों पर िवचार कीिजयेः

1. रा पित सं सद का एक सं घटक भाग है ।


2. भारत के सभी शासन सं बंधी काय उसके नाम पर िकये जाते ह।

उपयु म से कौन-सा/से सही है /ह?


A) केवल 1

B) केवल 2

C) 1 और 2 दोनों

D) न तो 1 और न ही 2

Hide Answer -
उ रः (c)

ा ाः उपयु दोनों कथन सही ह।

भारत का रा पित सं सद का एक भाग है ।

भारत के सभी शासन सं बंधी काय रा पित के नाम से िकये जाते ह। रा पित कायपािलका

का अ होता है और सम कायपािलका श उसम िनिहत होती है ।

[16]िन िल खत म से िकसकी िनयु भारत के रा पित ारा की जाती है ?

1. सं घ रा े के मु मं ी
2. अनु सूिचत जाित एवं अनु सूिचत जनजाित आयोग के अ
3. िव आयोग के अ
4. नीित आयोग के उपा

नीचे िदये गए कूट का योग कर सही उ र का चयन कीिजयेः

A) केवल 3

B) केवल 2 और 3

C) केवल 1, 2 और 3

D) 1, 2, 3 और 4

Hide Answer -
उ रः (c)

ा ाः नीित आयोग के उपा की िनयु भारत के धानमं ी ारा की जाती है ।

सं िवधान के अनु े द-239कक (5) म उपबं ध है िक सं घ रा े (िद ी) के मु मं ी

की िनयु रा पित करे गा और अ मं ि यों की िनयु मु मं ी की सलाह पर रा पित

करे गा।

सं िवधान के अनु े द-280(1) म उपबं ध है िक सं घीय िव आयोग के अ की िनयु

रा पित करे गा।

अनु े द-338 क(3) म उपबं ध है िक रा ीय अनु सूिचत जनजाित आयोग के अ ,

उपा और अ सद ों की िनयु रा पित करगे ।

[17]िन िल खत पर िवचार कीिजयेः

1. सं घ के मं ी
2. भारत का महा ायवादी
3. रा पाल

उपयु म से कौन-कौन भारत के रा पित के सादपय पद धारण करते ह?

A) केवल 1 और 3

B) केवल 1 और 2

C) केवल 2 और 3

D) 1, 2 और 3

Hide Answer -
उ रः (d)

ा ाः उपयु सभी कथन सही ह।

सं िवधान के अनु े द-75(2) म उपबं ध है िक मं ी रा पित के सादपय पद धारण

करगे ।

अनु े द-76(4) म उपबं ध है िक भारत का महा ायवादी रा पित के सादपय पद

धारण करे गा।

अनु े द-156(1) म उपबं ध है िक रा पाल रा पित के सादपय पद धारण करे गा।

[18]िन िल खत पर िवचार कीिजयेः

1. धानमं ी की िनयु
2. उप-रा पित की िनयु
3. िनयं क एवं महाले खा-परी क की िनयु
4. मु िनवाचन आयु की िनयु
5. महा ायवादी की िनयु

उपयु म से िकस-िकस की िनयु भारत के रा पित के ारा की जाती है ?

A) केवल 1, 2 और 3

B) केवल 2, 3 और 4

C) केवल 1, 3, 4 और 5

D) 1, 2, 3, 4 और 5
Hide Answer -
उ रः (c)

ा ाः रा पित, उप-रा पित की िनयु नहीं करता, ब उप-रा पित का चु नाव होता है ।

भारत के रा पित िन िल खत की िनयु करते ह

धानमं ी- अनु े द-75(1)

िनयं क एवं महाले खा-परी क- अनु े द-148(1)

मु िनवाचन आयु - अनु े द-324(2)

महा ायवादी- अनु े द-76(1)

रा पाल- अनु े द-155

सं घ लोक से वा आयोग के अ -अनु े द-316(1)

[19]भारतीय सं िवधान के अनु े द-74 के अं तगत िन िल खत कथनों पर िवचार


कीिजयेः

1. रा पित, मं ि प रषद की सलाह के अनु सार काय करे गा।


2. मं ि प रषद ारा दी गई सलाह को रा पित पु निवचार के िलये लौटा सकता
है ।
3. मं ि प रषद ारा दी गई सलाह की िकसी ायालय म जाँ च नहीं की जा
सकती।

उपयु म से कौन-से कथन सही ह?

A) केवल 1 और 2

B) केवल 1 और 3

C) केवल 2 और 3

D) 1, 2 और 3

Hide Answer -
उ रः (d)

ा ाः उपयु तीनों कथन सही ह।

सं िवधान के अनु े द-74(1) म उपबं ध है िक रा पित को सलाह व सहायता दे ने के िलये

एक मं ि प रषद होगी, िजसका धान, धानमं ी होगा और रा पित मं ि प रषद की सलाह

के अनु सार काय करे गा।

44व सं िवधान सं शोधन अिधिनयम, 1978 के ारा उपबं ध िकया गया िक रा पित

मं ि प रषद की सलाह को पु निवचार के िलये लौटा सकता है और रा पित पु निवचार के

प ात् दी गई सलाह के अनु सार काय करे गा।

अनु े द-74(2) म उपबं ध है िक मं ि प रषद ारा रा पित को दी गई सलाह की िकसी

ायालय म जाँ च नहीं की जा सकती है ।

[20]भारतीय सं िवधान के अनु सार िन िल खत कथनों पर िवचार कीिजयेः


1. धानमं ी की िनयु रा पित करता है जबिक धानमं ी रा पित की
सलाह पर अ मं ि यों की िनयु करता है ।
2. मं ी, धानमं ी के सादपय पद धारण करता है ।
3. सं िवधान की दू सरी अनु सूची के तहत रा पित मं ी को पद और गोपनीयता
की शपथ िदलाते ह।

उपयु म से कौन-सा/से कथन सही है /ह?

A) केवल 1

B) केवल 1 और 2

C) 1, 2 और 3

D) उपयु म से कोई नहीं

Hide Answer -
उ रः (d)

ा ाः उपयु तीनों कथन सही नहीं ह।

सं िवधान के अनु े द 75(1) म उपबं ध है िक धानमं ी की िनयु रा पित करे गा और

अ मं ि यों की िनयु रा पित, धानमं ी के सलाह पर करगे ।

अनु े द 75(2) म उपबं ध है िक मं ी, रा पित के सादपय पद धारण करगे , न िक

धानमं ी के।

अनु े द 75(4) म उपबं ध है िक रा पित सं िवधान की तीसरी अनु सूची के तहत मं ी को

पद और गोपनीयता की शपथ िदलाएं गे ।


[1] सं िवधान के अनु े द-78 म धानमं ी के क ों के सं बंध म उपबं ध है ।
िन िल खत म से कौन-सा एक धानमं ी का क नहीं है ?

सं घ के ि याकलाप के शासन एवं िवधायन सं बंधी मं ि प रषद के सभी


A)
िनणयों से रा पित को सू िचत कर।

सं घ के ि याकलाप के शासन एवं िवधायन सं बंधी जो जानकारी रा पित


B)
माँ गे, वह द।

िकसी िवषय िजस पर मं ी ने िनणय िदया है िकंतु मं ि प रषद ने िवचार नहीं


C) िकया है , रा पित ारा अपे ा िकये जाने पर प रषद के सम िवचार के िलये
रख।

िकसी िवशे ष िवधे यक को धन िवधे यक के प म मािणत िकये जाने के


D)
िनणय को रा पित को ु त करना।

Hide Answer -
उ रः (d)

ा ाः उपयु म से िवक (d) सही नहीं है ।

सं िवधान के अनु े द-78 म रा पित के ित धानमं ी के तीन क बतलाए गए ह-

उपयु , िवक (a) का उपबं ध अनु े द-78(क) म िकया गया है ।

िवक (b) का उपबं ध अनु े द-78(ख) म िकया गया है ।

िवक (c) का उपबं ध अनु े छ-78(ग) म िकया गया है ।

सं िवधान के अनु े द-110(4) म उपबं ध है िक अनु े द 111 के अधीन धन िवधे यक

रा पित के सम ु त िकया जाएगा, िजस पर लोकसभा के अ का ह ा र होगा

िक वह धन िवधे यक है ।

[2] िन िल खत म से कौन-सा एक कायपािलका का ाियक काय है ?

A) िवधे यकों को पास करना

B) रा पित पर महािभयोग

C) िवधायी िवशे षािधकारों का सं र ण

उन यों को द से िनल न िदलाना िज मृ ु द की सज़ा


D)
अिधिनण त की गई हो।
Hide Answer -
उ रः (d)

ा ाः सं िवधान का अनु े द-72 रा पित को मादान की श दान करता है । यह रा पित

की ाियक श है ।

िवधे यकों को पास करना रा पित की िवधायी श है ।

सं िवधान के अनु े द-61 म रा पित पर महािभयोग चलाया जाता है । यह सं सद की एक

अध- ाियक ि या है ।

[3] भारत के रा पित के ािधकार पर िवचार कीिजयेः

1. वा िवक और लोकि य
2. औपचा रक और िविधक
3. राजनीितक और नाममा
4. सं वैधािनक और नाममा

उपयु म से िकस कार के ािधकार भारत के रा पित को ा ह?

A) केवल 1 और 3

B) केवल 2 और 3

C) केवल 1 और 4

D) केवल 2 और 4

Hide Answer -
उ रः (d)

ा ाः रा पित के ािधकार के सं बंध म कथन 1 और 3 सही नहीं ह।

भारत का सं िवधान रा पित को सं वैधािनक और नाममा तथा औपचा रक और िविधक

अिधकार दान करता है ।

सं िवधान के अनु े द-53(1) म उपबं ध है िक सं घ की कायपािलका श रा पित म

िनिहत होगी और वह इसका योग यं या अपने अधीन थ अिधका रयों ारा करे गा।

सं िवधान के अनु े द-77(1) म उपबं ध है िक भारत सरकार कायपािलका के सम काय

रा पित के नाम से िकये जाएं गे , िकंतु कायपािलका श का वा िवक योग मं ि प रषद

ारा िकया जाता है । सं िवधान के अनु े द 74(1) म उपबं ध है िक रा पित, मं ि प रषद

की सलाह पर काय करे गा।

[4] िन कथनों पर िवचार कीिजयेः

1. भारत का रा पित सं सद का भाग है ।


2. सं सद ारा पा रत िवधे यक रा पित की ीकृित के िबना कानू न नहीं बन
सकता है ।

उपयु म से कौन-सा/से कथन सही है /ह?


A) केवल 1

B) केवल 2

C) 1 और 2 दोनों

D) न तो 1 और न ही 2

Hide Answer -
उ रः (c)

ा ाः उपयु दोनों कथन सही ह।

सं िवधान के अनु े द-79 म उपबं ध है िक भारत की सं सद, लोक सभा, रा सभा और

रा पित से िमलकर बने गी।

[5] भारत के रा पित िन िल खत िसफा रशों/ ितवे दनों को सं सद के पटल पर

रखवाएगाः

1. सं घ िव आयोग
2. लोक-ले खा सिमित
3. िनयं क एवं महाले खा-परी क
4. रा ीय अनु सूिचत जाित आयोग

उपयु म से कौन-सा/से इस सं दभ म सही है /ह?

A) केवल 1

B) केवल 1, 2 और 4

C) केवल 1, 3 और 4

D) 1, 2, 3 और 4

Hide Answer -
उ रः (c)

ा ाः उपयु म से कथन 2 सही नहीं है ।

लोक ले खा सिमित के ितवे दन को रा पित सदन के पटल पर नहीं रखवाते ह। लोक ले खा

सिमित अपनी रपोट लोकसभा अ को सौंपती है ।

सं िवधान के अनु े द-281 म उपबं ध है िक रा पित सं घ िव आयोग की िसफा रशों को

सं सद के े क सदन के सम रखवाएं गे ।

अनु े द-151(1) म उपबं ध है िक रा पित िनयं क एवं महाले खा-परी क के ितवे दनों

को सं सद के े क सदन के सम रखवाएं गे ।

अनु े द-338(6) म उपबं ध है िक रा पित रा ीय अनु सूिचत जाित आयोग के ितवे दनों

को सं सद के े क सदन के सम रखवाएं गे ।

अनु े द-111 म उपबं ध है िक सं सद के दोनों सदनों ारा पा रत िवधे यक रा पित की

ीकृित के प ात् कानू न का प ले गा


[6] भारत के रा पित िविभ े ों से रा सभा के िलये 12 सद ों को मनोनीत करते
ह। इस सं दभ म िन िल खत पर िवचार कीिजयेः

1. सािह के े से
2. िव ान के े से
3. कला के े से
4. सहकारी आं दोलन के े से
5. समाज से वा के े से

उपयु म से कौन-से कथन सही ह?

A) केवल 1, 2, 3 और 4

B) केवल 1, 2, 3 और 5

C) केवल 1, 3, 4 और 5

D) 1, 2, 3, 4 और 5

Hide Answer -
उ रः (b)

ा ाः सं िवधान के अनु े द-80(1) म उपबं ध है िक रा पित 12 सद ों को

अनु े द-80(3) के तहत रा सभा के िलये मनोनीत करगे । इसम शािमल े ह- सािह ,

िव ान, कला और समाज से वा।

सहकारी आं दोलन के े से रा पित नहीं, अिपतु रा के रा पाल िवधान प रषद के

िलये मनोनीत करगे ।

वतमान म रा सभा म रा पित ारा मनोनीत करने के सं बंध म बदलाव िकया गया है ।

इसी आधार पर ि केट खलाड़ी सिचन ते दुलकर को रा सभा के िलये मनोनीत िकया

गया।

[7] भारत के रा पित के सं बंध म िन िल खत कथनों पर िवचार कीिजयेः

1. वह लोक सभा या रा सभा या दोनों सदनों को िवघिटत कर सकता है ।


2. वह लोक सभा या रा सभा या दोनों सदनों का स ावसान कर सकता है ।
3. वह सं सद की बै ठक को कुछ घं टे, कुछ िदन, कुछ स ाह या अिनि त
काल के िलये थिगत कर सकता है ।

उपयु म से कौन-सा/से कथन सही है /ह?

A) केवल 1

B) केवल 2

C) केवल 1 और 2

D) केवल 2 और 3
Hide Answer -
उ रः (b)

ा ाः उपयु म से कथन 1 और 3 सही नहीं ह।

सं िवधान के अनु े द-85(2)(ख) म उपबं ध है िक रा पित केवल लोकसभा का िवघटन

कर सकता है , रा सभा का नहीं। ोंिक रा सभा एक थायी सदन है जबिक

लोकसभा की कायविध केवल पाँ च वष की होती है ।

अनु े द-85(2)(क) म उपबं ध है िक रा पित दोनों सदनों या िकसी एक सदन का

स ावसान कर सकेगा।n

सं सद को थिगत करने की श रा पित को नहीं है , अिपतु लोकसभा के मामले म लोक

सभा अ व रा सभा के मामले म सभापित (रा सभा के अ ) सदन की बै ठक

थिगत कर सकते ह।

[8] भारत के रा पित के सं दभ म िन िल खत पर िवचार कीिजयेः

1. रा सभा और लोकसभा की बै ठक बु लाना।


2. रा सभा और लोकसभा की सं यु बै ठक बु लाना।
3. रा सभा और लोकसभा के सं यु बै ठक की अ ता करना।

उपयु म से कौन-सा/से कथन स है /ह?

A) केवल 1

B) केवल 2

C) केवल 1 और 2

D) 1, 2 और 3

Hide Answer -
उ रः (c)

ा ाः उपयु म से कथन (3) सही नहीं है ।

सं िवधान के अनु े द 108 म उपबं ध है िक रा पित कुछ दशाओं म दोनों सदनों का

सं यु अिधवे शन बु ला सकता है , िकंतु इसकी अ ता लोकसभा अ ारा की जाती

है , न िक रा पित ारा।

अनु े द-85(1) म उपबं ध है िक रा पित समय-समय पर सं सद के े क सदन की

बै ठक बु ला सकता है िकंतु सं सद के े क सदन की एक स की अं ितम बै ठक और

आगामी स की थम बै ठक के बीच छः माह का अं तर नहीं होना चािहये ।

[9] िन िल खत म से कौन-सा/से सही सु मेिलत है /ह?

(सं िवधान का ावधान) (अनु े द)

सं घ की कायपािलका श का िव ार अनु े द-73

सं सद म रा पित का अिभभाषण अनु े द-86


सं सद म रा पित का िवशे ष अिभभाषण अनु े द-87

कूटः

A) केवल 1

B) केवल 1 और 2

C) 1, 2 और 3

D) केवल 2 और 3

Hide Answer -
उ रः (d)

ा ाः उपयु तीनों सही सु मेिलत ह।

सं िवधान के अनु े द-73 म सं घ की कायपािलका श के िव ार के सं बंध म उपबं ध है ।

अनु े द-86(1) म उपबं ध है िक रा पित सं सद के िकसी एक सदन म या एक साथ दोनों

सदनों म अिभभाषण कर सकेगा और सद ों की उप थित की अपे ा कर सकेगा।

अनु े द 86(2) म उपबं ध है िक रा पित सं सद के िकसी सदन को िवधे यक के सं बंध म

सं देश भे ज सकेगा।

अनु े द-87(1) म उपबं ध है िक रा पित एक साथ सं सद के दोनों सदनों म अिभभाषण

करे गा और सं सद को उसके आ ान का कारण बताएगा। लोक सभा के सं दभ म ेक

साधारण िनवाचन के प ात् थम स के ारं भ म और े क वष थम स के आरं भ म

अिभभाषण करे गा।

[10]िन िल खत सू ची-I को सू ची-II से सु मेिलत कीिजयेः

सू ची-I
सू ची-II
(अनु े द-72 म
(अथ)
उपबं ध)

1.दोषी, सभी द ादे शों,द ों और िनरहताओं से


A. मा
मु ।

B.लघु करण 2.द के प म बदलाव कर कम करना।

C. ितहार 3.द की कृित बदले िबना अविध कम करना

4.मू ल प से दी गई सज़ा को िकसी िवशे ष प र थित


D.िवराम
म कम करना।

कूटः
A) 1 2 3 4

B) 1 3 2 4

C) 4 3 2 1

D) 4 2 3 1

Hide Answer -
उ रः (a)

ा ाः भारत के सं िवधान के अनु े द-72 म रा पित को मादान की श का ावधान है ।

यह रा पित की ाियक श है ।

माः दोषी को सभी द ों, द ादे शों और िनरहताओं से पू णतः मु िमल जाती है ।

लघु करणः द के प को बदलकर कम कर िदया जाता है । जै सेः मृ ु द का

लघु करण कर कठोर कारावास म प रवितत करना।

प रहारः द की कृित म कमी िकये िबना उसकी अविध कम करना, जै से- दो वष के

कठोर कारावास को एक वष के कठोर कारावास म बदल िदया जाता है ।

िवरामः दोषी को मू ल प म दी गई सज़ा को िकसी िवशे ष प र थित म कम करना, जै से-

मिहला अपराधी का गभवती होना।

ितलं बनः िकसी द (िवशे षकर मृ ु द ) पर अ थाई रोक लगाना।

[11] मादान के सं बंध म िन िल खत म से कौन-सा कथन सही नहीं है ?

सं िवधान के अनु े द-72 के तहत रा पित और रा पाल को मादान की


A)
श दी गई है ।

केवल रा पित को मृ ु द को मा करने की श ा है , रा पाल को


B)
यह श नहीं दी गई है ।

केवल रा पित सै ायालय ारा दी गई सज़ा को मा कर सकता है ,


C)
रा पाल को यह श नहीं ा है ।

रा पाल को मृ ु द का िनलं बन, प रहार और लघु करण करने की श


D)
रा पित के समान है ।
Hide Answer -
उ रः (a)

ा ाः उपयु म से िवक (a) सही नहीं है ।

सं िवधान के अनु े द-72 म रा पित को मादान की श दी गई है जबिक रा पाल

को अनु े द-161 म मादान की श दी गई है ।

मृ ु द को मा करने की श केवल रा पित को है , रा पाल को यह श नहीं

ा है । मृ ु द के िनलं बन, प रहार या लघु करण करने की श रा पाल और

रा पित को समान है ।

रा पित के मादान की श की ाियक समी ा नहीं की जा सकती, बशत रा पित का

िनणय े ाचारी, िववे करिहत, दु भावना अथवा भे दभावपू ण न हो।

रा पित मादान श का योग मं ि मं डल के परामश से करे गा।

यिद मादान की पू व यािचका रा पित ारा र कर दी गई है तो दू सरी यािचका दायर

नहीं की जा सकती है ।

[12]भारत म ‘अ ादे श’ के योग के सं दभ म िन िल खत म से कौन-सा कथन सही


है ?

सं िवधान के अनु े द-123 के अं तगत रा पित और रा पाल इसका उपयोग


A)
करते ह।

B) अ ादे श का योग रा पित िववे कानु सार करता है ।

रा पित ारा अ ादे श तब जारी िकया जाता है , जब लोक सभा या रा


C)
सभा स म नहीं रहते ह।

भारत के साथ-साथ अमे रका, और ि टे न म भी अ ादे श के योग के सं बंध


D)
म उपबं ध है ।

Hide Answer -
उ रः (c)

ा ाः उपयु म से कथन (a), (b) और (d) सही नहीं ह।

सं िवधान के अनु े द-123 के तहत भारत के रा पित अ ादे श का योग करते ह,

जबिक रा ों के रा पाल सं िवधान के अनु े द-213 के अं तगत इसका योग करते ह।

अ ादे श का योग रा पित िववे कानु सार नहीं करते अिपतु मं ि मं डल की सलाह पर

इसे जारी करते ह और वापस ले ते ह।

अनु े द-123(1) म उपबं ध है िक रा पित, उस समय को छोड़कर जब सं सद के दोनों

सदन स म ह, अ ादे श जारी कर सकता है ।

भारत के रा पित की अ ादे श जारी करने की श अनोखी है जबिक अमे रका व ि टे न

म इसका योग नहीं िकया जाता है ।

[13]सं िवधान के अनु े द-123 के अं तगत रा पित अ ादे श का योग करता है । इस


सं दभ म िन िल खत म से कौन-से कथन सही ह?
1. रा पित अ ादे श को िकसी भी समय वापस ले सकता है ।
2. अ ादे श का वही बल होता है जो सं सद ारा पा रत अिधिनयम का होता
है ।
3. अ ादे श सं सद के पु नः समवे त होने की तारीख से छः माह की समा
पर वृ नहीं रहे गा।
4. सं िवधान म सं शोधन हे तु अ ादे श जारी िकया जा सकता है ।

नीचे िदये गए कूटों का योग कर सही उ र का चयन कीिजयेः

A) केवल 1 और 2

B) केवल 1, 2 और 3

C) केवल 1, 2 और 4

D) 1, 2, 3 और 4

Hide Answer -
उ रः (a)

ा ाः उपयु म से कथन (3) और (4) सही नहीं ह।

रा पित ारा जारी अ ादे श का जीवनकाल सीिमत है । सं िवधान के अनु े द-123(2)

(क) म उपबं ध है िक अ ादे श सं सद के दोनों सदनों के सम रखा जाएगा और सं सद के

पु नः समवे त होने की तारीख से छः स ाह (न िक छः माह) की समा पर वृ नहीं

रहे गा।

सं िवधान म सं शोधन हे तु अ ादे श जारी नहीं िकया जा सकता। सं िवधान के अनु े द-13

म ‘िविध’ की जो प रभाषा दी गई है उसम इसे शािमल िकया गया है । अतः अ ादे श मू ल

अिधकारों को न तो छीन सकता है , न ू न कर सकता है और न उ ं घन कर सकता है ।

अ ादे श भू तल ी हो सकता है , अथात िपछले तारीख से इसे भावी िकया जा सकता है ।

रा पित को समाधान हो जाए िक ऐसी प र थितयाँ िव मान ह िजसके कारण तु रंत

कायवाही करना आव क है तो रा पित अ ादे श जारी कर सकता है , परं तु सं सद का

कोई एक सदन या दोनों सदन स म नहीं होने चािहये ।

रा पित की अ ादे श जारी करने की श और रा ीय आपात (अनु े द-352) से कोई

सं बंध नहीं है । अ ादे श यु , बा आ मण या सश िव ोह न हो तो भी जारी िकया जा

सकता है ।

अ ादे श के योग को ायालय म चु नौती दी जा सकती है ।

भारत शासन अिधिनयम, 1935 म गवनर जनरल को अ ादे श िनकालने की श दी

गई थी।

[14]भारत के रा पित के ‘पॉकेट वीटो’ पर िवचार कीिजयेः

1. िवधे यक को अिनि त काल के िलये लं िबत रख सकता है ।


2. िवधे यक को एक िनि त समय सीमा तक लं िबत रख सकता है ।
3. भारतीय डाक (सं शोधन) अिधिनयम म इसका योग डॉ. राजे साद ने
िकया था।
4. सं िवधान सं शोधन अिधिनयम म इसका योग नहीं कर सकते ह।

उपयु म से कौन-से कथन सही ह?

A) केवल 1 और 3

B) केवल 1 और 4

C) केवल 2 और 3

D) केवल 2 और 4

Hide Answer -
उ रः (b)

ा ाः उपयु म से कथन (2) और (3) सही नहीं ह।

रा पित के सम आए िकसी िवधे यक पर िनणय दे ने के िलये सं िवधान म कोई समय सीमा

िनधा रत नहीं है , जबिक अमे रका म 10 िदनों के अ र िवधे यक को पु निवचार के िलये

लौटाना पड़ता है ।

भारतीय डाक (सं शोधन) अिधिनयम, 1986 म रा पित जै ल िसं ह (न िक डॉ. राजे

साद) ारा इस वीटो का योग िकया गया था।

इसे पॉकेट वीटो इसिलये कहते ह िक रा पित िकसी िवधे यक को अिनि त काल के िलये

टे बल पर पड़ा रहने दे ता है , न तो ीकृित दे ता है और न ही लौटाता है ।

सं िवधान सं शोधन से सं बंिधत अिधिनयमों म रा पित के पास कोई वीटो नहीं है । 24व

सं िवधान सं शोधन अिधिनयम, 1971 के ारा रा पित को ीकृित दे ने के िलये बा कारी

बना िदया गया।

[15]रा पित के िनलं बनकारी वीटो के सं दभ म िन िल खत म से कौन-सा/से कथन

सही है /ह?

1. इसका योग धन िवधे यक के सं बंध म नहीं कर सकते ह।


2. िकसी िवधे यक के पु निवचार के िलये इस वीटो का योग करते ह।

नीचे िदये गए कूट का योग कर सही उ र का चयन कीिजयेः

A) केवल 1

B) केवल 2

C) 1 और 2 दोनों

D) न तो 1 और न ही 2
Hide Answer -
उ रः (c)

ा ाः उपयु दोनों कथन सही ह।

रा पित ारा इस वीटो का योग तब िकया जाता है , जब िकसी िवधे यक (धन िवधे यक को

छोड़कर) को पु निवचार के िलये भे जा जाता है , िकंतु सं सद उस िवधे यक को सं शोधन या

िबना सं शोधन के साथ साधारण ब मत से पा रत कर रा पित के पास भे जता है तो रा पित

ीकृित दे ने के िलये बा है ।

धन िवधे यक के सं बंध म इस वीटो का योग नहीं िकया जा सकता है ।

[16]पे ु (PEPSU) िविनयोग िवधे यक म रा पित डॉ. राजे साद ने िन िल खत म


से कौन-से वीटो का योग िकया था?

A) अ ां ितक वीटो

B) िवशे िषत वीटो

C) िनलं बनकारी वीटो

D) पॉकेट वीटो

Hide Answer -
उ रः (a)

ा ाः वष, 1954 म रा पित डॉ. राजे साद ने पे ु िविनयोग िवधे यक पर इस वीटो का

योग िकया था।

वष, 1991 म रा पित डॉ. आर. वकटरमण ारा सं सद सद ों के वे तन, भ ों और पशन

(सं शोधन) िवधे यक पर इस वीटो का योग िकया था।

इस वीटो के ारा रा पित सं सद ारा पा रत िवधे यक को अनु मित दे ने से इनकार कर

सकता है ।

धन िवधे यक के सं बंध म इस वीटो का योग नहीं कर सकते ।

िवशे िषत वीटोः भारत के रा पित के सं दभ म यह वीटो मह हीन है । अमे रकी रा पित

ारा इस वीटो का योग िकया जाता है ।

[17]िन िल खत म से कौन-सा/से कथन सही है /ह?

1. सं सद ारा पा रत िवधे यक रा पित के पास भे जा जाता है , तब रा पित


उस िवधे यक को (धन िवधे यक से िभ ) पु निवचार के िलये सं सद को लौटा
सकता है , िकंतु पु नः िबना सं शोधन या सं शोधन के साथ रा पित के पास
भे जा जाता है तो रा पित ीकृित दे ने के िलये बा है ।
2. रा िवधानसभा ारा पा रत िवधे यक को रा पाल रा पित के िवचाराधीन
आरि त कर सकता है । रा पित रा पाल को िनदश दे सकता है िक उस
िवधे यक को (धन िवधे यक से िभ ) रा िवधानसभा के पास पु निवचार के
िलये लौटा दे , िकंतु यिद रा िवधानसभा ारा पु नः िबना सं शोधन या
सं शोधन के साथ रा पित के पास भे जा जाता है तो रा पित ीकृित दे ने के
िलये बा नहीं है ।

नीचे िदये गए कूट का योग कर सही उ र का चयन कीिजये :

A) केवल 1

B) केवल 2

C) 1 और 2 दोनों

D) न तो 1 और न ही 2

Hide Answer -
उ रः (c)

ा ाः उपयु दोनों कथन सही ह।

सं िवधान के अनु े द-111 म उपबं ध है िक सं सद ारा पा रत िवधे यक के सं बंध म

रा पित के पास तीन िवक होते ह-

िवधे यक पर अपनी ीकृित दे सकता है ; अथवा

िवधे यक पर अपनी ीकृित सु रि त रख सकता है ; अथवा

िवधे यक को पु निवचार के िलये (धन िवधे यक से िभ ) लौटा सकता है , िकंतु यिद सं सद

ारा पु नः िबना सं शोधन या सं शोधन के साथ रा पित के पास भे जा जाता है , तो रा पित

ीकृित दे ने के िलये बा है ।

सं िवधान के अनु े द-200 म उपबं ध है िक रा पाल रा पित के िवचार के िलये िवधे यक

को आरि त रख सकता है ।

सं िवधान के अनु े द-201 के तहत रा पित के पास तीन िवक रहते ह-

िवधे यक पर ीकृित दे सकता है ।

िवधे यक पर ीकृित सु रि त रख सकता है ।

रा पाल को िनदश दे सकता है िक िवधे यक को (धन िवधे यक से िभ ) पु निवचार के िलये

लौटा दे , िकंतु रा िवधानसभा ारा पु नः िबना सं शोधन या सं शोधन के साथ रा पित के

पास भे जा जाता है , तो रा पित उस पर ीकृित दे ने के िलये बा नहीं है ।

[18]सं िवधान के अनु े द-143 के सं दभ म िन िल खत पर िवचार कीिजये :

1. इस अनु े द के तहत रा पित उ तम या उ ायालय से परामश माँ ग


सकता है ।
2. रा पित ारा माँ गा गया परामश दे ने के िलये उ तम ायालय बा है ।
3. उ तम ायालय का परामश रा पित के िलये बा कारी है

उपयु म से कौन-सा/से कथन सही है /ह?


A) केवल 1

B) केवल 2

C) 1, 2 और 3

D) उपयु म से कोई नहीं

Hide Answer -
उ रः (d)

ा ाः उपयु तीनों कथन सही नहीं ह।

सं िवधान के अनु े द-143 म उपबं ध है िक रा पित को तीत हो िक िविध या त का

कोई ऐसा उ आ है या उ होने की सं भावना है िजस पर उ तम ायालय की

राय आव क है , तो रा पित उ तम ायालय से परामश ले सकता है । रा ों के उ

ायालय इसके अं तगत नहीं आते ह।

रा पित उ तम ायालय का परामश मानने के िलये बा नहीं है ।

उ तम ायालय रा पित के ारा मां गा गया परामश दे ने के िलये बा नहीं है ।

[19]भारत के रा पित के िववे क की श के सं दभ म िन िल खत पर िवचार


कीिजयेः

1. मादान की श
2. अ ादे श जारी करने की श
3. िवधे यक को आपि यों सिहत वापस भे जना
4. िवधे यक को रोक कर रखना
5. सं सद को सं देश भे जना

उपयु म से कौन-से कथन सही ह?

A) केवल 1 और 2

B) केवल 3, 4 और 5

C) केवल 1, 2, 3 और 4

D) 1, 2, 3, 4 और 5
Hide Answer -
उ रः (b)

ा ाः उपयु म से 1 और 2 सही नहीं ह। रा पित मादान और अ ादे श की श का

योग मं ि मं डल की सलाह पर करता है

रा पित को िन मामलों म िववे क की श ा है ः

िवधे यक को आपि यों सिहत वापस भे जना।

िवधे यक को रोक कर रखना।

सं सद को सं देश भे जना।

लोकसभा म िकसी दल को ब मत न िमलने पर धानमं ी की िनयु करना।

लोकसभा को िवघिटत करना यिद मं ि मडल ने अपना ब मत खो िदया है ।

मं ि मं डल को िवघिटत कर सकता है यिद वह सदन म िव ासमत िस नहीं कर पाया है ।

[20]िन िल खत पर िवचार कीिजयेः

1. नए रा ों के िनमाण या वतमान रा की सीमाओं म प रवतन।


2. धन िवधे यक
3. ऐसा िवधे यक जो सही अथ म धन िवधे यक नहीं है , िकंतु भारत की सं िचत
िनिध से य करना पड़ता है ।

उपयु म से कौन-सा/से िवधे यक है /ह, िजसम रा पित की पू व मं ज़ूरी या


िसफा रश आव क है ?

A) केवल 1

B) केवल 1 और 2

C) केवल 2 और 3

D) 1, 2 और 3

Hide Answer -
उ रः (d)

ा ाः उपयु तीनों िवधे यकों के सं बंध म भारत के रा पित की पू व मं ज़ूरी या िसफा रश

आव क है ।

कथन-1 का उपबं ध सं िवधान के अनु े द-3 म है ।

कथन-2 का उपबं ध अनु े द-117(1) म है ।

कथन-3 का उपबं ध अनु े द-117(3) म है ।

अनु े द-31(क)(1) म िनिद िवषयों से सं बंिधत िवधे यक।

अनु े द-304 म उपबं िधत रा के ऐसे िवधे यक जो ापार और वािण की तं ता

को भािवत करते ह।
[1] िन िल खत म से िकसकी िनयु भारत के रा पित ारा की जाती है ?

1. िनयं क महाले खा परी क


2. सं घ लोक से वा आयोग का अ
3. िव आयोग का अ
4. रा ों के रा पाल
5. महा ायवादी

नीचे िदये गए कूट का योग कर सही उ र का चयन कीिजयेः

A) केवल 1 और 2

B) केवल 1, 2 और 3

C) केवल 2, 3, 4 और 5

D) 1, 2, 3, 4 और 5

Hide Answer -
उ रः (d)

ा ाः भारत के रा पित ारा उपयु िनयु यों के अलावा अ िनयु याँ भी की जाती ह, जो

िन िल खत ह-

अनु सूिचत े ों के शासन पर ितवे दन दे ने के िलये आयोग।

अनु सूिचत जाितयों और जनजाितयों के िलये िवशे ष अिधकारी।

मु िनवाचन आयु और िनवाचन आयोग के अ सद आिद।

[2] िन िल खत सू ची-I को सू ची-II से सु मेिलत कीिजये तथा नीचे िदए गये कूट की

सहायता से सही उ र चु न:

सू ची-I
(भारतीय सं िवधान के ावधान)
सू ची-II
(अनु े द)

A. कुछ सं घ रा े ों के िलये िविनयम बनाने की रा पित


1.अनु े द-240
की श

B. रा पित सामािजक व शै िणक ि से िपछड़े वग की


दशाओं के अ े षण के िलये एक आयोग की िनयु कर 2. अनु े द-339
सकेगा

C. रा पित रा ों के अनु सूिचत े ों के शासन और


अनु सूिचत जनजाितयों के क ाण पर ितवे दन दे ने के िलये 3. अनु े द-340
एक आयोग की िनयु कर सकेगा

D. े क रा व के शािसत दे शोंम अनु सूिचत


जनजाितयोंकी सू ची बनाने वअिधसू िचत करने की रा पित की 4. अनु े द-342

कूटः
A B C D
A) 1 2 3 4

B) 1 3 2 4

C) 4 3 2 1

D) 4 2 3 1

Hide Answer -
उ रः (b)

ा ाः

सं िवधान के अनु े द-240 सं घ रा े का शासन रा पित के नाम से चलाया जाता है ।

रा पित िन िल खत सं घ रा े ों के िलये िविनयम बना सकते ह:

अ मान और िनकोबार ीप समू ह

ल ीप

दादर व नागर हवे ली

दमन व दीव

इसके सं बंध म रा पित के पास अं ितम श है और सं सद ारा बनाई गई िकसी िविध को

समा या सं शोधन कर सकता है ।

अनु े द-339: रा पित सं िवधान के ारं भ से 10 वष की समा पर अनु सूिचत े ों के

शासन एवं अनु सूिचत जनजाितयों के क ाण पर ितवे दन के िलये एक आयोग की िनयु

कर सकेगा।

अनु े द-342: े क रा व के शािसत दे शों म अनु सूिचत जनजाितयों के सू ची बनाने व

अिधसू िचत करने की रा पित की श के सं बंध म उपबं ध है ।

[3] सं िवधान की पाँ चवीं अनु सूची के पै रा-6 म अनु सूिचत े के सं बंध म उपबं ध िकया
गया है िक िकसी भौगोिलक े को अनु सूिचत े घोिषत िकया जा सकता है । यह
सं वैधािनक अिधकार िकसको ा है ?

A) रा पित

B) रा पाल

C) मु मं ी

D) धानमं ी

Hide Answer -
उ रः (a)

ा ाः रा पित आदे श ारा िकसी भौगोिलक े को अनु सूिचत े घोिषत कर सकता है या

समा कर सकता है या सीमाओं म प रवतन कर सकता है ।

रा के रा पाल के परामश से अनु सूिचत े के े फल को बढ़ा सकता है ।

पाँ चवीं अनु सूची के पै रा-3 म उपबं ध है िक रा का रा पाल ितवष या जब भी रा पित

अपे ा करे रा के अनु सूिचत े के शासन के सं बंध म ितवे द

[4] जनजाित सलाहकार प रषद’ के सं बंध म िन िल खत कथनों पर िवचार कीिजये :

1. इसका ावधान सं िवधान की पाँ चवीं अनु सूची म िकया गया है ।


2. इसकी थापना रा पित के आदे श पर की जाती है ।
3. ऐसे रा ों म की जाती है , जहाँ अनु सूिचत जनजाितयाँ ह, िकंतु अनु सूिचत े
नहीं।

उपयु कथनों म कौन-सा /से कथन सही है /ह?


A) केवल 1

B) केवल 1 और 2

C) केवल 1 और 3

D) 1, 2 और 3

Hide Answer -
उ रः (d)

ा ाः उपयु तीनों कथन सही ह।

सं िवधान की पाँ चवीं अनु सूची म उपबं ध है िक ऐसे रा म िजसम अनु सूिचत जनजाितयाँ ह,

िकंतु अनु सूिचत े नहीं है , रा पित ऐसा आदे श दे तो जनजाित सलाहकार प रषद की

थापना की जाएगी।

[5] िन िल खत कथनों पर िवचार कीिजयेः

1. रा पित एन. सं जीव रे ी भारत के िनिवरोध रा पित िनवािचत ए थे ।


2. रा पित वी.वी. िग र का सं बंध टे ड यू िनयन आं दोलन से रहा है ।
3. रा पित डॉ. राधाकृ न को दाशिनक शासक के प म जाना जाता है ।
4. भारत म िकसी भी मु ायमू ित ने कभी रा पित के प म काय नहीं
िकया है ।

उपयु म से कौन-से कथन सही ह?

A) केवल 1 और 3

B) केवल 1, 2 और 3

C) केवल 1, 3 और 4

D) 1, 2, 3 और 4

Hide Answer -
उ रः (b)

ा ाः उपयु म से कथन (4) सही नहीं है ।

भारत के मु ायमू ित एम.िहदायतु ा 20 जु लाई से 24 अग , 1969 के दौरान भारत

के कायवाहक रा पित रहे ।

वष 1977 म एन.सं जीव रे ी को सवस ित (िनिवरोध) से भारत का रा पित चु ना गया।

रा पित वी.वी. िग र दो बार अ खल भारतीय टे ड यू िनयन के अ रहे ।

रा पित डॉ. सवप ी राधाकृ न को दाशिनक राजा/दाशिनक शासक के प म जाना जाता

है ।

रा पित डॉ. ए.पी.जे . अ ु ल कलाम को िमसाइल मै न के नाम से जाना जाता है । वे भारत के

11व रा पित बने ।

[6] रा पित डॉ. राजे साद के सं बंध म कौन-सा कथन सही नहीं है ?

A) वे भारत के थम रा पित थे ।

B) दू सरी बार िनवािचत होने वाले भारत के एकमा रा पित थे ।

C) वह थम बार 1950 म भारत के रा पित बने ।

D) उपयु म से कोई नहीं।


Hide Answer -
उ रः (d)

ा ाः उपयु सभी कथन सही ह। डॉ. राजे साद सं िवधान सभा के अ होने के नाते

थम बार 26 जनवरी, 1950 को रा पित बने , िकंतु रा पित के िलये थम िनवाचन 1952 म

आ और दू सरी बार 1957 म आ।

डॉ. राजे साद दो बार रा पित िनवािचत ये थे ।

[7] िन िल खत कथनों पर िवचार कीिजयेः

1. ीमती ितभा पािटल भारत की दू सरी मिहला रा पित बनी।


2. गत मम णब मु खज दे श के 14व रा पित बन।

उपयु म से कौन-सा/से कथन सही है /ह?

A) केवल 1

B) केवल 2

C) 1 और 2 दोनों

D) न तो 1 और न ही 2

Hide Answer -
उ रः (d)

ा ाः उपयु दोनों कथन सही नहीं ह।

ीमित ितभा पािटल भारत की पहली मिहला रा पित बनी। वह गत म के अनु सार

दे श की 12वीं रा पित बनी।

गत म के अनु सार णब मु खज दे श के 13व रा पित बन; जबिक कायकाल के

अनु सार 14व। वष 2012 म रा पित का चु नाव स ए।

[8] सं सद के िलये रा पित का अिभभाषण कौन तै यार करता है ।

A) यं रा पित

B) लोकसभा अ

C) रा सभा के सभापित

D) के ीय मं ि मं डल

Hide Answer -
उ रः (d)

ा ाः सं सद के िलये रा पित का अिभभाषण के ीय मं ि मं डल तै यार करता है , ोंिक

कायपािलका की वा िवक श के ीय मं ि मं डल म िनिहत होती है ।

[9] िन िल खत कथनों पर िवचार कीिजयेः

1. सं िवधान के अनु े द-63 म भारत के िलये एक उप-रा पित के सं बंध म


उपबं ध है ।
2. सं िवधान के अनु े द-89 म रा सभा के िलये सभापित के सं बंध म उपबं ध
है ।
3. उप-रा पित भारत का ि तीय उ तम िति त पदािधकारी है ।
4. उप-रा पित पद से सं बंिधत कोई औपचा रक काय/दािय नहीं ह।

उपयु म से कौन-से कथन सही ह?


A) केवल 1, 2, और 3

B) केवल 1, 3 और 4

C) केवल 2, 3 और 4

D) 1, 2, 3 और 4

Hide Answer -
उ रः (d)

ा ाः उपयु सभी कथन सही ह।

सं िवधान के अनु े द-63 म उपबं ध है िक भारत का एक उप-रा पित होगा।

सं िवधान के अनु े द-89 म उपबं ध है िक भारत का उप-रा पित रा सभा का पदे न

सभापित होगा।

उप-रा पित का पद दे श का दू सरा सव पद होता है । आिधका रक म म उसका पद

रा पित के बाद आता है ।

उप-रा पित पद से सं बंिधत उप-रा पित का कोई औपचा रक काय/दािय नहीं होता, अिपतु

वह रा सभा का पदे न सभापित होता है और उसी प म काय करता है ।

अनु े द-64 म उपबं ध है िक उप-रा पित रा सभा का पदे न सभापित होगा, और कोई

लाभ का पद धारण नहीं करे गा।

[10]भारत का उप-रा पित जब रा पित के प म काय करता है , तो इसके सं दभ म


कौन-सा कथन सही नहीं है ?

A) वह रा सभा के सभापित के पद के क ों का पालन नहीं करे गा।

B) वह रा सभा के सभापित को दे य वे तन व भ ों का हकदार नहीं होगा।

उसे रा पित की सभी श याँ और उ ु याँ ा होंगी तथा रा पित की


C)
उपल यों, भ ों व िवशे षािधकारों का हकदार होगा।

उसे रा सभा के सभापित को दे य वे तन व भ े ा होंगे और रा पित की


D)
उपल यों, भ ों व िवशे षािधकारों का हकदार नहीं होगा।

Hide Answer -
उ रः (d)

ा ाः उपयु म से िवक (d) सही नहीं है ।

सं िवधान के अनु े द-65 म उपबं ध है िक रा पित की मृ ु , पद ाग या पद से हटाए जाने या

अ कारण से र ए पद पर उस तारीख से उप-रा पित, रा पित के सभी काय /

उ रदािय ों का िनवहन करे गा और उस अविध के दौरान वह रा सभा के सभापित पद के

क ों का पालन नहीं करे गा।

उस अविध के दौरान उप-रा पित को रा पित की सभी श याँ व उ ु याँ ा होंगी और

सं िवधान की दू सरी अनु सूची म िनिद रा पित की उपल यों, भ ों व िवशे षािधकारों का

हकदार होगा।

[11] िन िल खत कथनों पर िवचार कीिजयेः

1. उप-रा पित के िनवाचन के िलये ू नतम आयु 30 वष है ।


2. उप-रा पित के िनवाचन के िलये उ ीदवार को लोकसभा या रा सभा के
िनवाचन के िलये पा होना चािहये ।

उपयु म से कौन-सा/से कथन सहीं है /ह?


A) केवल 1

B) केवल 2

C) 1 और 2 दोनों

D) न तो 1 और न ही 2

Hide Answer -
उ रः (d)

ा ाः उपयु दोनों कथन सही नहीं ह।

सं िवधान के अनु े द-66 म उप-रा पित के िनवाचन के सं बंध म उपबं ध है ।

अनु े द-66(3) म उपबं ध है िक वह भारत का नाग रक हो, ू नतम आयु 35 वष हो और

रा सभा का सद िनवािचत होने के िलये पा हो।

अनु े द-66(2) म उपबं ध है िक सं सद या िवधानमं डल का कोई सद उप-रा पित के प

म िनवािचत हो जाता है तो पद हण करने की तारीख से सं सद या िवधानमं डल का सद

नहीं रहे गा।

अनु े द-60(1) म उपबं ध है िक उप-रा पित का चु नाव आनु पाितक ितिनिध प ित के

अनु सार एकल सं मणीय मत ारा होगा और िनवाचन गु मतदान के ारा होगा।

[12]भारत के उप-रा पित के िनवाचक मं डल म िन िल खत म से कौन शािमल होते है /

ह?

1. केवल लोकसभा के िनवािचत सद


2. केवल रा सभा के िनवािचत सद
3. िवधानसभा के िनवािचत सद

नीचे िदये गए कूटों का योग कर सही उ र का चयन कीिजयेः

A) केवल 1

B) केवल 1 और 2

C) 1, 2 और 3

D) उपयु म से कोई नहीं

Hide Answer -
उ रः (d)

ा ाः उपयु म से कोई सही नहीं है ।

सं िवधान के अनु े द-66(1) म उप-रा पित के िनवाचन के सं बंध म उपबं ध है । इसके

िनवाचक मं डल म लोकसभा और रा सभा के सभी िनवािचत सद ों के अलावा लोकसभा व

रा सभा के मनोनीत सद भी शािमल होते ह।

उप-रा पित के िनवाचक मं डल म िवधानसभा व िवधान प रषद के सद शािमल नहीं होते ।

[13]भारत के उप-रा पित के िनवाचन के सं बंध म िन िल खत म से कौन-सा कथन सही

नहीं है ?

A) िकसी रा के िवधानमं डल का सद उप-रा पित का उ ीदवार हो सकता है ।

B) सं सद का कोई सद उप-रा पित का उ ीदवार हो सकता है ।

उप-रा पित के चु नाव के नामां कन के िलये 50 ावक और 50 अनु मोदक


C)
होने चािहये ।

उप-रा पित के उ ीदवार को भारतीय रज़व बक म एक िनि त रािश जमा


D)
करना आव क है ।
Hide Answer -
उ रः (c)

ा ाः उपयु म से िवक C सही नहीं है ।

उप-रा पित चु नाव के नामां कन के िलये 20 ावक व 20 अनु मोदक होने चािहये ।

सं िवधान के अनु े द-66(2) म उपबं ध है िक यिद सं सद या रा िवधानमं डलों का कोई

सद उप-रा पित िनवािचत हो जाता है तो पद- हण करने की तारीख से वह सं सद या

िवधानमं डल का सद नहीं रहे गा।

उप-रा पित के उ ीदवार को भारतीय रज़व बक म 15000 ज़मानत रािश के प म जमा

करना आव क है ।

[14]भारत के उप-रा पित के सं बंध म िन िल खत म से कौन-सा कथन सही है ?

उप-रा पित के चु नाव से सं बंिधत सभी शं काओं व िववादों की जाँ च व िनणय


A)
भारत के िनवाचन आयोग ारा की जाती है ।

B) भारत का मु ायमू ित उप-रा पित को पद की शपथ या ित ा िदलाता है ।

उप-रा पित को पद ु त करने सं बंधी ाव लोकसभा या रा सभा म ुत


C)
िकया जा सकता है ।

रा सभा की अ ता उप-रा पित ारा की जाती है िक ु वह रा सभा का


D)
सद नहीं होता है ।

Hide Answer -
उ रः (d)

ा ाः उपयु म से िवक a, b और c सही नहीं ह।

सं िवधान के अनु े द-71(1) म उपबं ध है िक रा पित या उप-रा पित के चु नाव से सं बंिधत

सभी िववाद या शं काओं की जाँ च व िनणय सव ायालय ारा िकया जाएगा और उसका

िनणय अ म होगा।

अनु े द 71(2) म उपबं ध है िक यिद उ तम ायालय रा पित या उप-रा पित के िनवाचन

को अवै ध घोिषत कर दे ता है तो उसके ारा पू व म िकये गए काय अवै ध नहीं होंगे।

अनु े द-71(3) म उपबं ध है िक रा पित या उप-रा पित के िनवाचन से सं बंिधत िकसी

िवषय का िविनयमन सं सद िविध ारा कर सकेगी।

अनु े द-71(4) म उपबं ध है िक रा पित या उप-रा पित के िनवाचन को िनवाचक मं डल म

से िकसी सद की र के आधार पर चु नौती नहीं दी जा सकती।

सं िवधान के अनु े द-69 म उपबं ध है िक उप-रा पित, रा पित या उसके ारा िनयु िकसी

के सम शपथ/ ित ा हण करगे ।

सं िवधान के अनु े द-67(ख) म उपबं ध है िक उप-रा पित को पद ु त करने सं बंधी ाव

केवल रा सभा म लाया जा सकता है ।

रा सभा की अ ता सभापित (उप-रा पित) ारा की जाती है िकंतु वह रा सभा का

सद नहीं होता है ।

[15]उप-रा पित की पदाविध के सं बंध म िन िल खत म से कौन-सा कथन सही नहीं है ?

A) उप-रा पित अपना ाग-प रा पित को सौंपता है ।

इसे रा सभा और लोकसभा के त ालीन सम सद ों के ब मत ारा पा रत


B)
सं क से पद से हटाया जा सकता है ।

C) सं क ािवत करने से पू व 14 िदन की सू चना दे ना आव क है ।

उप-रा पित पदाविध समा के प ात् भी पद पर बना रहे गा, जब तक उसका


D)
उ रािधकारी पद हण न कर ले ।
Hide Answer -
उ रः (b)

ा ाः उपयु म से िवक (b) सही नहीं है ।

सं िवधान के अनु े द-67(क) म उपबं ध है िक उप-रा पित अपना ाग-प रा पित को

सौंपेगा।

अनु े द-67(ख) म उपबं ध है िक रा सभा के त ालीन सम सद ों के ब मत ारा

पा रत सं क िजससे लोकसभा सहमत हो उप-रा पित को उसके पद से हटा सकते ह।

सं क ािवत करने से पू व 14 िदन की सू चना दे ना आव क है ।

अनु े द-67(ग) म उपबं ध है िक उप-रा पित पद की अविध समा होने के प ात् भी पद

पर बना रहे गा जब तक उसका उ रािधकारी पद हण न कर ले ।

अनु े द-60 म उपबं ध है िक उप-रा पित की पदाविध समा से पहले िनवाचन पू ण कर

िलया जाएगा।

उप-रा पित की मृ ु , पद ाग या पद से हटाये जाने या अ कारण से ई र के प ात्

िनवाचन यथाशी करा िलया जाएगा और िनवािचत अनु े द-67 के उपबं धों के अधीन

अपने पद हण करने की तारीख से पाँ च वष की पू री अविध तक पद धारण करे गा।

उप-रा पित पु निनवािचत हो सकता है ।

[16]िन िल खत कथनों पर िवचार कीिजयेः

1. उप-रा पित कायवाहक रा पित के प म अिधकतम एक वष तक काय


कर सकता है ।
2. सं िवधान उप-रा पित पद के िलये दू सरी अनु सूची के तहत वे तन व भ े आिद
का उपबं ध करता है ।

उपयु म से कौन-सा/से कथन सही है /ह?

A) केवल 1

B) केवल 2

C) 1 और 2 दोनों

D) न तो 1 और न ही 2

Hide Answer -
उ रः (d)

ा ाः उपयु दोनों कथन सही नहीं ह।

उप-रा पित कायवाहक रा पित के प म अिधकतम छः माह तक काय कर सकता है ।

सं िवधान उप-रा पित पद के िलये िकसी कार के वे तन व भ े का उपबं ध नहीं करता ब

उसे रा सभा के पदे न सभापित के तहत एवं दू सरी अनु सूची म उपबं िधत वे तन व भ े िमलते

ह। वतमान म रा सभा के सभापित का वे तन ितमाह ` 1.25 लाख है ।

[17]यिद भारत के उप-रा पित को पद से हटाने का कोई सं क रा सभा म


िवचाराधीन है , इस थित के सं दभ म िन िल खत म से कौन –सा कथन सही होगा?
नीचे िदए गये कूट की सहायता से सही उ र चु न:

1. रा सभा म नहीं बोल सकता।


2. सभा की कायवाही म भाग नहीं ले सकता।
3. रा सभा के िकसी िवषय पर मतदान नहीं कर सकता।

कूट:
A) केवल 3

B) केवल 1 और 3

C) केवल 2 और 3

D) 1, 2 और 3

Hide Answer -
उ रः (a)

ा ाः उपयु म से कथन-1 और 2 सही नहीं ह।

सं िवधान के अनु े द-92(2) म उपबं ध है िक उप-रा पित को पद से हटाने सं बंधी कोई

सं क रा सभा म िवचारधीन है , तब सभापित (उप-रा पित) को रा सभा म बोलने और

कायवाही म भाग ले ने का अिधकार होगा, िक ु वह िकसी िवषय पर मतदान नहीं कर

सकता।

अनु े द-92(1) म उपबं ध है िक जब उप-रा पित को पद से हटाने सं बंधी कोई सं क

िवचाराधीन है , तब रा सभा की िकसी बै ठक म उप थत होने पर भी पीठासीन नहीं होगा।

[18]रा सभा की बै ठक की अ ता सभापित और उप-सभािपत के अनु प थित म


िकसके ारा की जाती है ?

A) रा सभा के व र सद

B) रा पित ारा िनयु रा सभा का कोई सद

C) रा सभा की ि या के िनयमों ारा अवधा रत

D) लोकसभा अ ारा िनयु रा सभा का कोई सद

Hide Answer -
उ रः (c)

ा ाः उपयु म से िवक c सही है ।

सं िवधान के अनु े द-91(2) म उपबं ध है िक रा सभा की बै ठक की अ ता सभापित

और उप-सभापित की अनु प थित म रा सभा की ि या के िनयमों ारा अवधा रत

करे गा और ऐसा भी अनु प थत है तो रा सभा ारा अवधा रत अ करे गा।

सं िवधान के अनु े द-91(1) म उपबं ध है िक सभापित और उप-सभापित के अनु प थित म

रा पित ारा िनयु रा सभा का सद उसके क ों का पालन करे गा।

[19]रा सभा के उप-सभापित के सं बंध म िन िल खत म से कौन-सा कथन सही है ?

उप-सभापित का पद र होने पर रा सभा के सद ों म से ही िकसी सद


A)
को उप-सभापित के प म चु नेगी।

रा सभा के िलये पा उप-सभापित बन सकता है , इसके िलये रा सभा


B)
का सद होना आव क नहीं है ।

उप-सभापित को पद से रा सभा म उप थत और मत दे ने वाले सद ों के दो-


C)
ितहाई ब मत ारा हटा सकते ह।

उप-सभापित को पद से हटाने के िलये 14 िदन पू व सू चना दे ना आव क नहीं


D)
है ।
Hide Answer -
उ रः (a)

ा ाः उपयु म से िवक -a सही है और िवक b, c और d सही नहीं ह।

सं िवधान के अनु े द-89(2) म उपबं ध है िक रा सभा अपने सद ों म से उप-सभापित का

चु नाव करे गी।

केवल रा सभा का सद ही उप-सभापित बन सकता है और रा सभा का सद नहीं

रहने पर पद र कर दे गा- अनु े द-90(क)।

उप-सभापित को उसके पद से रा सभा के त ालीन सम सद ों के ब मत से पा रत

सं क ारा हटाया जा सकता है - अनु े द-90(ग)।

उप-सभापित को पद से हटाने के िलये सं क ािवत होने के 14 िदन पहले सू चना दे ना

आव क है ।

अनु े द-90(ख) म उपबं ध है िक उप-सभापित अपना ाग-प सभापित को दे गा।

[20]भारत के उप-रा पित के सं बंध म िन िल खत कथनों पर िवचार कीिजयेः

1. उप-रा पित बनने के िलये के पास जनता के सदन का सद


िनवािचत होने की यो ता होनी चािहये ।
2. िकसी रा की िवधाियका का सद इस पद के िलये उ ीदवार हो सकता
है ।
3. उप-रा पित पद का कायकाल उतना ही होता है िजतना िक रा पित का।
4. उसे एक औपचा रक महािभयोग ारा पद से हटाया जा सकता है ।

उपयु म से कौन-से कथन सही नहीं ह?

A) 1 और 2

B) 2 और 3

C) 1 और 4

D) 3 और 4

Hide Answer -
उ रः (c)

ा ाः उपयु म से कथन-1 और 4 सही नहीं ह।

भारत का उप-रा पित बनने के िलये के पास रा सभा का सद िनवािचत होने की

यो ता होनी आव क है न िक जनता के सदन (लोकसभा) का सद िनवािचत होने की

यो ता।

सं सद या िकसी रा के िवधानमं डल का सद उप-रा पित पद का उ ीदवार हो सकता

है , िक ु उप-रा पित पद हण करने की तारीख से वह सं सद या रा िवधानमं डल का

सद नहीं रहे गा।

रा पित व उप-रा पित का कायकाल समान (5 वष) होता है ।

उप-रा पित को पद से हटाने के िलये रा सभा के त ालीन सम सद ों के ब मत ारा

पा रत सं क तथा लोकसभा की सहमित आव क है । इसे एक औपचा रक महािभयोग ारा

नहीं हटाया जा सकता।


[1] भारत के उप-रा पित के सं बंध म िवचार कीिजयेः

1. डॉ. एस. राधाकृ न


2. वी.वी. िग र
3. के. आर. नारायणन
4. डॉ. जािकर सै न

उपयु म से कौन उप-रा पित का पद सं भालने से पू व राजदू त अथवा उ ायु


पद पर रहे ह?

A) केवल 1 और 2

B) केवल 2 और 3

C) केवल 1, 2 और 3

D) 1, 2, 3 और 4

Hide Answer -
उ रः (c)

ा ाः उपयु म से 1, 2 और 3 सही ह िक ु 4 गलत है ।

उप-रा पित बनने से पहले िन िल खत भारत के राजदू त/उ ायु पद पर

कायरत थे -

एस. राधाकृ न- 1949-1952- पू व सोिवयत सं घ ( स)

वी.वी. िग र -1947-1951- िसलोन ( ीलं का)

के. आर. नारायणन - 1976-1978 - चीन

[2] िन िल खत पर िवचार कीिजयेः-

1. मोह द िहदायतु ाह
2. फख ीन अली अहमद
3. नीलम सं जीव रे ी
4. शं कर दयाल शमा

उपयु रा पित पद पर रहे यों म से कौन भारत के उप-रा पित पद पर


भी रहे ?

A) केवल 1 और 4

B) केवल 2 और 3

C) केवल 3 और 4

D) 1, 2, 3 और 4
Hide Answer -
उ रः (a)

ा ाः उपयु म से मोह द िहदायतु ाह (1979-84) और डॉ. शं कर दयाल शमा

(1987-92) भारत के उप-रा पित भी रहे ।

नीलम सं जीव रे ी और फख ीन अली अहमद भारत के उप-रा पित के पद पर काय

नहीं िकये ब सीधे रा पित चु ने गए। नीलम सं जीव रे ी िनिवरोध/सवस ित से रा पित

चु ने गए थे ।

[3] िन िल खत पर िवचार कीिजयेः-

1. जी. एस. पाठक


2. बी. डी. ज ी
3. कृ का
4. बी.एस. शे खावत

उपयु म से कौन भारत के उप-रा पित तो रहे ह िक ु रा पित के पद पर


काय नहीं िकये ?

A) केवल 1 और 4

B) केवल 1, 3 और 4

C) केवल 2, 3 और 4

D) 1, 2, 3 और 4

Hide Answer -
उ रः (d)

ा ाः उपयु सभी उप-रा पित तो रहे परं तु भारत के रा पित के प म काय नहीं

िकये ।

[4] िन िल खत यु ों म से कौन-सा/से सही सु मेिलत है /ह?

1. ानी जै ल िसं ह - रा पित और उप-रा पित


2. डॉ. ए.पी.जे . अ ु ल कलाम - रा पित और उप-रा पित
3. डॉ. जािकर सै न - रा पित और उप-रा पित

नीचे िदये गए कूट का योग कर सही उ र का चयन कीिजयेः

A) केवल 1

B) केवल 3

C) केवल 1 और 3

D) 1, 2 और 3
Hide Answer -
उ रः (b)

ा ाः उपयु म से 1 और 2 सही सु मेिलत नहीं ह।

ानी जै ल िसं ह- भारत के उप-रा पित पद पर काय नहीं िकया।

डॉ. ए.पी.जे . अ ु ल कलाम- भारत के उप-रा पित पद पर नहीं रहे ।

डॉ. जािकर सै न- भारत के रा पित और उप-रा पित दोनों पदों पर रहे ।

रा पित डॉ. राजे साद, ीमती ितभा पािटल और णब मु खज कभी भी उप-रा पित

पद पर नहीं रहे ।

[5] िन िल खत पर िवचार कीिजयेः

1. डॉ. राधाकृ न
2. आर. वकटरमण
3. के.आर. नारायणन
4. मो. हािमद अं सारी

उपयु म से कौन उप-रा पित के िलये दो बार िनवािचत ए?

A) केवल 1

B) केवल 1 और 4

C) केवल 1, 2 और 4

D) 1, 3 और 4

Hide Answer -
उ रः (b)

ा ाः डॉ. राधाकृ न और मो. हािमद अं सारी दो बार भारत के उप-रा पित के िलये

िनवािचत ए।

डॉ राधाकृ न - 1952 और 1957

मो. हािमद अं सारी - 2007 और 2012

आर. वकेटरमन और के. आर. नारायणन एक-एक बार उप-रा पित के िलये िनवािचत

ए।

[6] भारत म रा पित/उप-रा पित के सं दभ म िवचार कीिजयेः

1. नीलम सं जीव रे ी
2. डॉ. ए.पी.जे . अ ु ल कलाम
3. डॉ. एस. राधाकृ न
4. एम. िहदायतु ाह
5. डॉ. शं कर दयाल शमा

उपयु म से कौन-से रा पित/उप-रा पित िनिवरोध/सवस ित से चु ने गये ?


A) केवल 1 और 2

B) केवल 1, 2 और 3

C) केवल 1, 3, 4 और 5

D) 1, 2, 3, 4 और 5

Hide Answer -
उ रः (c)

ा ाः उपयु म से 2 सही नहीं है , ोंिक रा पित चु नाव म डॉ. ए.पी.जे . अ ु ल कलाम

की मु ित ी ल ी सहगल थी।

वष 1977 म नीलम सं जीव रे ी को सवस ित से िनिवरोध रा पित चु ना गया। जबिक वष

1969 म वी.वी िग र के खलाफ रा पित का चु नाव हार गए थे ।

डॉ. एस. राधाकृ न, एम. िहदायतु ाह और डॉ. शं कर दयाल शमा उप-रा पित के पद

पर िनिवरोध चु ने गये थे ।

[7] िन िल खत कथनों पर िवचार कीिजयेः

1. सं िवधान म उपबं ध है िक मं ि प रषद का धान, धानमं ी होगा।


2. धानमं ी के िनवाचन के सं बंध म सं िवधान म उपबं ध िकया गया है ।

उपयु म से कौन-सा/से कथन सही है /ह?

A) केवल 1

B) केवल 2

C) 1 और 2 दोनों

D) न तो 1 और न ही 2

Hide Answer -
उ रः (a)

ा ाः उपयु म से कथन-2 सही नहीं है ।

सं िवधान के अनु े द-75(1) म उपबं ध है िक धानमं ी की िनयु रा पित करे गा। अतः

धानमं ी की िनयु होती है , न िक िनवाचन। धानमं ी का न तो चयन होता है और न

ही मनोनयन होता है ।

सं िवधान के अनु े द 74(1) म उपबं ध है िक मं ि प रषद का धान, धानमं ी होगा।

[8] िन िल खत कथनों पर िवचार कीिजयेः

1. धानमं ी रा पित के सादपयत पद धारण करता है ।


2. गै र-सं सद सद धानमं ी िनयु नहीं हो सकता।
3. धानमं ी पद और गोपनीयता की शपथ रा पित के सम ले ता है ।
4. धानमं ी के वे तन और भ े के सं बंध म उपबं ध सं िवधान के दू सरी
अनु सूची म िकया गया है ।

उपयु म से कौन-सा/से कथन सही नहीं है /ह?

A) केवल 1

B) केवल 2

C) केवल 1 और 2

D) केवल 2, 3 और 4

Hide Answer -
उ रः (b)

ा ाः उपयु म से कथन (2) सही नहीं है ।

वष 1997 म उ तम ायालय ने िनणय िदया िक एक जो सं सद के िकसी भी

सदन का सद नहीं है , छः माह के िलये धानमं ी िनयु हो सकता है । इस अविध म

उसे सं सद के िकसी भी सदन का सद बनना पड़े गा, अ था छः माह प ात वह

धानमं ी नहीं रहे गा।

धानमं ी का कायकाल िनि त नहीं है और वह रा पित के सादपयत पद पर बना रहता

है िकंतु धानमं ी को जब तक लोकसभा म ब मत हािसल है रा पित उसे बखा नहीं

कर सकता है ।

रा पित धानमं ी को पद और गोपनीयता की शपथ िदलवाता है जो िक सं िवधान के

तीसरी अनु सूची म उपबं ध है । धानमं ी वै से ही शपथ ले ते ह, जै से कोई अ मं ी।

धानमं ी के वे तन व भ े सं सद ारा समय-समय पर िनधा रत होते रहते ह, िजसका

सं िवधान की दू सरी अनु सूची म उपबं ध है ।

[9] भारत के धानमं ी के सं बंध म िवचार कीिजयेः

1. इ रा गाँ धी
2. एच.डी. दे वे गौड़ा
3. अटल िबहारी बाजपे यी
4. डॉ. मनमोहन िसं ह

उपयु म से कौन-सा/से धानमं ी रा सभा के सद नहीं था/थे ?

A) केवल 2

B) केवल 3

C) केवल 1, 2 और 4

D) केवल 2 और 3
Hide Answer -
उ रः (b)

ा ाः उपयु म से (3) सही नहीं है , ोंिक अटल िबहारी बाजपे यी अपने धानमं ि काल

म लोकसभा के सद थे , न िक रा सभा के।

िन िल खत धानमं ी रा सभा के सद थेः-

इ रा गाँ धी - 1966

एच.डी. दे वे गौड़ा - 1996

डॉ. मनमोहन िसं ह - 2004 एवं 2009

आमतौर पर धानमं ी लोकसभा का सद होता है िकंतु रा सभा के सद को

धानमं ी बनाने पर कोई रोक नहीं है । जब इ रा गां धी थम बार धानमं ी बनी थी तो

वह रा सभा की सद थी।

ि टे न म धानमं ी केवल िन सदन (हाउस ऑफ कॉमन) का सद ही बने गा।

[10]िन िल खत कथनों पर िवचार कीिजयेः

1. रा पित, भारत सरकार का काय अिधक सु िवधापू वक िकये जाने के िलये


और मं ि यों म उ काय के आवं टन के िलये िनयम बनाएगा।
2. भारत सरकार की कायपािलका सं बंधी सम कायवािहयाँ धानमं ी के
नाम से की ई कही जाएं गी।

उपयु म से कौन-सा/से कथन सही है /ह?

A) केवल 1

B) केवल 2

C) 1 और 2 दोनों

D) न तो 1 और न ही 2

Hide Answer -
उ रः (a)

ा ाः उपयु म से कथन (2) सही नहीं है ।

सं िवधान के अनु े द 77(1) म उपबं ध है िक भारत सरकार की कायपािलका सं बंधी

सम कायवािहयाँ रा पित के नाम से की ई कही जाएं गी, न िक धानमं ी के नाम से ।

कथन (1) का उपबं ध सं िवधान के अनु े द 77(3) म िकया गया है ।

[11]िन िल खत कथनों पर िवचार कीिजयेः

1. भारत का धानमं ी बनने के िलये ू नतम आयु 30 वष होनी चािहये ।


2. भारत म उप- धानमं ी पद का सृ जन सं िवधान के ावधान से हटकर आ
है ।

उपयु म से कौन-सा/से सही है /ह?


A) केवल 1

B) केवल 2

C) 1 और 2 दोनों

D) न तो 1 और न ही 2

Hide Answer -
उ रः (b)

ा ाः उपयु म से कथन (1) सही नहीं है ।

भारत का धानमं ी बनने के िलये ू नतम आयु 25 वष होनी चािहये ।

सं िवधान म उप- धानमं ी के सं बंध म कोई उपबं ध नहीं है । भारत म इस पद का सृ जन

राजनीितक बा ताओं के कारण सं िवधान के ावधानों से हटकर आ है । यह गै र-

सं वैधािनक पद है ।

[12]भारत का धानमं ी यिद उ सदन (रा सभा) का सद हो तो

A) अिव ास ाव की थित म अपने प म वोट नहीं दे सकेगा।

धानमं ी पद की शपथ हण करने के बाद छः माह के अ र िन सदन का


B)
सद बनना पड़े गा।

C) िन सदन म बजट पर नहीं बोल सकेगा।

D) केवल उ सदन म ही व दे सकता है ।

Hide Answer -
उ रः (a)

ा ाः उपयु म से िवक (b), (c) और (d) सही नहीं ह।

यिद धानमं ी सं सद के िकसी सदन का सद नहीं है तो उसे छः माह के अ र िन

सदन या उ सदन का सद बनना पड़े गा, अथात रा सभा का सद भी धानमं ी

बन सकता है ।

यिद धानमं ी उ सदन का सद है , तो िन सदन (लोक सभा) म बोल सकता है ,

िकंतु मतदान नहीं कर सकता। वह िजस सदन का सद है , केवल उसी सदन म मतदान

कर सकता है ।

[13]भारत के धानमं ी के िनयु के समयः-


ज़ री नहीं है िक वह सं सद के दोनों सदनों म से िकसी एक का आव क
A) प से सद हो, परं तु उसे छः माह के अं दर आव क प से दोनों म से
एक सदन का सद को जाना चािहये ।

ज़ री नहीं है िक वह सं सद के दोनों सदनों म से एक का आव क प से


B) सद हो, परं तु उसे छः माह के अं दर लोकसभा का सद हो जाना
चािहये ।

C) सं सद के दोनों सदनों म से एक का आव क प से सद होना चािहये ।

D) आव क प से लोकसभा का सद होना चािहये ।

Hide Answer -
उ रः (a)

ा ाः उपयु म से िवक (b), (c) और (d) सही नहीं ह।

धानमं ी की िनयु के समय आव क नहीं है िक वह लोकसभा या रा सभा का

सद हो, िकंतु छः माह के अं दर उसे लोकसभा या रा सभा का सद हो जाना चािहये

अ था छः माह की समा के प ात वह धानमं ी के पद पर नहीं रहे गा।

[14]भारत के धानमं ी के सं बंध म िन िल खत म से कौन-सा कथन सही है ?

वह सं सद के दोनों सदनों के सद ों म से अपने मं ि यों का चयन करने के


A)
िलये तं है ।

वह रा पित के साथ उिचत परामश करके अपने मं ि मं डल के सहयोिगयों का


B)
चयन कर सकता है ।

अपने मं ि मं डल के मं ि यों के प म काय करने के िलये यों के चयन


C)
म पू णतः िववे क का योग करता है ।

अपने मं ि मं डल के सहयोिगयों का चयन करने म सीिमत श याँ रखता है ,


D)
ोंिक िववे क योग की श याँ भारत के रा पित म िनिहत है ।

Hide Answer -
उ रः (c)

ा ाः उपयु म से िवक (a), (b) और (d) सही नहीं ह।

धानमं ी मं ि प रषद का अ होता है । उसे अपने सहयोगी के प म ऐसे यों को

चु नने का पू ण अिधकार है जो उसके साथ िमलकर काय कर सक। वह मं ि यों के चयन म

िववे क का योग करता है । वह सं सद के बाहर के यों को भी मं ी बना सकता है

िकंतु छः माह के अं दर उसे लोकसभा या रा सभा का सद बनना अिनवाय है , अ था

छः माह प ात वह मं ी नहीं रहे गा।

[15]िन िल खत कथनों पर िवचार कीिजयेः

1. के ीय मं ि प रषद म धानमं ी सिहत मं ि यों की कुल सं ा सं सद


सद ों की कुल सं ा के 15 ितशत से अिधक नहीं होनी चािहये ।
2. के शािसत दे श िद ी म मु मं ी सिहत मं ि यों की कुल सं ा
िवधानसभा की कुल सद सं ा के 10 ितशत से अिधक नहीं होनी
चािहये ।
3. िकसी रा की मं ि प रषद म मु मं ी सिहत मं ि यों की कुल सं ा
िवधान मं डल के सद ों की कुल सं ा के 15 ितशत से अिधक नहीं
होनी चािहये ।

उपयु म से कौन-सा/से कथन सही है /ह?

A) केवल 1

B) केवल 2

C) केवल 1 और 2

D) 1, 2 और 3

Hide Answer -
उ रः (b)

ा ाः उपयु म से कथन (1) और (3) सही नहीं ह।

91व सं िवधान सं शोधन अिधिनयम, 2003 के ारा अनु े द-75(1)(क) जोड़ा गया और

उपबं ध िकया गया िक मं ि प रषद म धानमं ी सिहत मं ि यों की कुल सं ा लोकसभा (न

िक सं सद) के सद ों की कुल सं ा के 15 ितशत से अिधक नहीं होनी चािहये ।

91व सं िवधान सं शोधन अिधिनयम, 2003 के ारा सं िवधान म अनु े द 164(1)(क)

जोड़ा गया और उपबं ध िकया गया िक िकसी रा की मं ि प रषद म मु मं ी सिहत

मं ि यों की कुल सं ा उस रा के िवधानसभा (न िक िवधान मं डल) के सद ों की कुल

सं ा के 15 ितशत से अिधक नहीं होनी चािहये और मु मं ी सिहत मं ि यों की कुल

सं ा 12 से कम नहीं होनी चािहये ।

69व सं िवधान सं शोधन अिधिनयम, 1991 के ारा अनु े द-239(क)(क) म िद ी के

सं बंध म िवशे ष उपबं ध है । 239(क)(क) के (4) म उपबं ध है िक मं ि प रषद म

मु मं ी सिहत मं ि यों की कुल सं ा िवधान सभा के सद ों की कुल सं ा के 10

ितशत से अिधक नहीं होनी चािहये ।

[16]िन िल खत म से िकसका भारत के सं िवधान म तो उ े ख नहीं है िकंतु


प रपाटी के प म पालन िकया जाता है ?

A) िव मं ी को िन सदन का सद होना चािहये ।

B) धानमं ी यिद िन सदन म ब मत खो दे तो उसे ाग प दे दे ना चािहये ।

C) मं ि प रषद म भारत के सभी लोगों का ितिनिध हो।

अपनी पदाविध की समा से पू व रा पित व उप-रा पित दोनों के एक साथ


D)
पद ाग करने पर िन सदन का अ रा पित का काय वहन करे गा।
Hide Answer -
उ रः (b)

ा ाः उपयु म से कथन (a), (c) और (d) सही नहीं ह।

सं िवधान म यह उपबं ध कही नहीं है िक िन सदन (लोक सभा) म ब मत खो दे ने पर

धानमं ी ागप दे दे ता है िकंतु सं िवधान के अनु े द-75(3) म उपबं ध है िक

मं ि प रषद सामू िहक प से लोकसभा के ित उ रदायी होगी। अतः इसी आधार पर यह

पर रा/प रपाटी िवकिसत ई है ।

मं ी, सं सद के िकसी सदन का सद हो सकता है और यिद िकसी सदन का सद नहीं

है तो छः माह के अ र िकसी सदन का सद बनना अिनवाय है ।

मं ि प रषद म मं ि यों की िनयु धानमं ी के िववे क पर िनभर करता है िक वह िकसे

मं ी बनाएं गे और िकसे नहीं।

रा पित व उप-रा पित दोनों पदाविध की समा के पू व ाग प दे द, तो भारत का

मु ायमू ित रा पित का कायभार सं भाले गा।

[17]सं िवधान के अनु े द-75(3) म मं ि प रषद के सामू िहक उ रदािय का उपबं ध


िकया गया है । इस सं दभ म िन िल खत कथनों पर िवचार कीिजयेः

1. िकसी मं ी की मृ ु या ागप दे ने पर मं ि प रषद िवघिटत हो जाएगी।


2. धानमं ी की मृ ु या ाग प दे ने पर मं ि प रषद िवघिटत हो जाएगी।

उपयु म से कौन-सा/से कथन सही है /ह?

A) केवल 1

B) केवल 2

C) 1 और 2 दोनों

D) न तो 1 और न ही 2

Hide Answer -
उ रः (b)

ा ाः उपयु म से कथन (1) सही नहीं है ।

िकसी मं ी की मृ ु या ागप दे ने पर मं ि प रषद पर कोई भाव नहीं पड़ता है अथात

मं ि प रषद िवघिटत नहीं होगी। केवल र ता उ होगी िजसे भरने के िलये धानमं ी

तं ह।

धानमं ी की मृ ु या ागप दे ने पर मं ि प रषद यं िवघिटत हो जाएगी, ोंिक

धानमं ी मं ि प रषद का धान होता है ।

[18]िन िल खत पर िवचार कीिजयेः

1. अं तरा ीय प रषद
2. े ीय प रषद
3. वै ािनक एवं औ ोिगक अनु संधान प रषद
4. रा ीय सु र ा सिमित
5. रा ीय एकता प रषद

उपयु म से िकसका अ भारत का धानमं ी होता है ?

A) केवल 1 और 2

B) केवल 1, 2 और 4

C) केवल 1, 3, 4 और 5

D) 1, 2, 3, 4 और 5

Hide Answer -
उ रः (c)

ा ाः उपयु म से केवल (2) सही नहीं है , ोंिक े ीय प रषद का अ भारत का

गृ हमं ी होता है ।

भारत के धानमं ी िन िल खत के अ होते ह;

1. नीित आयोग

2. रा ीय िवकास प रषद (NDC)

3. रा ीय एकता प रषद

4. अं तरा ीय प रषद

5. रा ीय जल सं साधन प रषद

6. वै ािनक एवं औ ोिगक अनु संधान प रषद

7. रा ीय सु र ा सिमित

[19]िन िल खत म से कौन-सा कथन सही नहीं ह?

धानमं ी चौधरी चरण िसं ह अपने कायकाल के समय कभी भी सं सद म


A)
उप थत नहीं ए।

B) धानमं ी लाल बहादु र शा ी की मृ ु दे श से बाहर ई।

C) धानमं ी मनमोहन िसं ह ारा नई आिथक नीित की घोषणा की गई।

D) सं िवधान म मं ि मं डल श का योग केवल एक बार आ है ।


Hide Answer -
उ रः (c)

ा ाः उपयु म से िवक (c) सही नहीं है ।

नई आिथक नीित की घोषणा सन् 1991 म त ालीन धानमं ी नरिस ा राव ारा की

गई, िजससे भारत म उदारीकरण, िनजीकरण और वै ीकरण का सार ते ज़ी से आ।

चौधरी चरण िसं ह 28 जु लाई, 1779 से 14 जनवरी 1980 तक भारत के धानमं ी रहे ,

परं तु इस दौरान लोकसभा की कोई बै ठक नहीं ई, ोंिक लोकसभा की िनधा रत

बै ठक से एक िदन पू व कां े स ारा समथन वापस ले िलया गया और सरकार िगर गई।

धानमं ी लाल बहादु र शा ी की मृ ु 11 जनवरी 1966 को ताशकंद (उ े िक ान) म

ई। वे भारत-पािक ान समझौते के तहत वहाँ गए ए थे ।

सं िवधान के अनु े द-352(3) म केवल एक बार ‘मं ि मं डल’ श का योग िकया गया

है ।

[20]िन िल खत पर िवचार कीिजयेः

1. मोरारजी दे साई
2. चौधरी चरण िसं ह
3. वी.पी. िसं ह
4. पी.वी. नरिस ा राव
5. एच.डी. दे वगौड़ा

उपयु म से कौन भारत के धानमं ी बनने से पू व रा ों के मु मं ी भी रहे ?

A) केवल 1, 2 और 3

B) केवल 1, 2, 3 और 4

C) केवल 3, 4 और 5

D) 1, 2, 3, 4 और 5

Hide Answer -
उ रः (d)

ा ाः उपयु सभी भारत के धानमं ी बनने से पू व रा ों के मु मं ी भी रहे । िववरण

िन ानु सार है ः-

1. मोरारजी दे साई - त ालीन ब ई रा

2. चौधरी चरण िसं ह - उ र दे श

3. वी.पी.िसं ह- उ र दे श

4. पी.वी. नरिस ा राव - आं दे श

5. एच.डी. दे वगौड़ा- कनाटक

6. नरे मोदी - गु जरात

मोरारजी दे साई दे श के पहले गै र-कां े सी धानमं ी(1977) थे ।

नरे मोदी धानमं ी (वतमान म) बनने से पू व गु जरात के चार बार मु मं ी रह चु के ह।


[1] िन िल खत कथनों पर िवचार कीिजयेः

1. भारतीय सं घ की कायपािलका श धानमं ी म िनिहत है ।


2. धानमं ी िसिवल से वा बोड का पदे न अ होता है ।

उपयु म से कौन-सा/से कथन सही है /ह?

A) केवल 1

B) केवल 2

C) 1 और 2 दोनों

D) न तो 1 और न ही 2

Hide Answer -
उ रः (d)

ा ाः उपयु दोनों कथन सही नहीं ह।

सं िवधान के अनु े द-53(1) म उपबं ध है िक सं घ की कायपािलका श रा पित म

िनिहत होगी और इसका योग वह यं या अपने अधीन थ अिधका रयों के ारा करे गा।

िसिवल से वा बोड का पदे न अ कैिबने ट सिचव होता है , न िक धानमं ी।

[2] के ीय सरकार के सं दभ म िन िल खत कथनों पर िवचार कीिजयेः

1. सं िवधान म उपबं ध है िक सम कैिबने ट मं ी अिनवाय प से केवल


लोकसभा के ही आसीन सद होंगे।
2. के ीय कैिबने ट सिचवालय सं सदीय काय मं ालय के िनदशाधीन काय
करता है ।

उपयु म से कौन-सा/से कथन सही है /ह?

A) केवल 1

B) केवल 2

C) 1 और 2 दोनों

न तो 1 और न ही 2
D)
Hide Answer -
उ रः (d)

ा ाः उपयु दोनों कथन सही नहीं ह।

सं िवधान धानमं ी या मं ी बनने के िलये रा सभा के सद ों पर िकसी कार की रोक

नहीं लगाता है । अतः सं सद के िकसी भी सदन का सद मं ी बन सकता है । सं िवधान म

उपबं ध है िक यिद काई मं ी सं सद के िकसी भी सदन का सद नहीं है , तो छः माह के

अं दर उसे िकसी सदन का सद बनना पड़ता है , अ था छः माह बाद वह मं ी नहीं

रहे गा।

के ीय कैिबने ट सिचवालय सीधे धानमं ी के अधीन होता है । इसका शासिनक मु ख

कैिबने ट सिचव होता है और कैिबने ट सिचव िसिवल से वा बोड का पदे न अ होता है ।

[3] मं ि मं डल सिचवालय के सं बंध म िन िल खत पर िवचार कीिजयेः

1. मं ि मं डल की बै ठकों के िलये काय सू ची तै यार करना।


2. मं ि मं डलीय सिमितयों के िलये सिचवालयी सहायता दान करना
3. मं ालयों को िव ीय सं साधनों का आवं टन करना।

उपयु म से कौन-सा/से मं ि मं डल सिचवालय का/के काय है /ह?

A) केवल 1

B) केवल 1 और 2

C) केवल 2 और 3

D) 1, 2 और 3

Hide Answer -
उ रः (b)

ा ाः उपयु म से कथन 3 सही नहीं है ।

मं ि मं डल सिचवालय के िन िल खत काय है ः

धानमं ी के आदे श पर मं ि मं डल की बै ठक आयोिजत करना।

मं ि मं डलीय बै ठकों के िलये कायसू ची तै यार करना और िवत रत करना।

कायसू ची से सं बंिधत द ावे ज़ों का िवतरण करना।

िकये गए िवचार-िवमश का रकाड तै यार करना।

मं ि मं डल ारा िलये गए िनणयों के ि या यन की िनगरानी करना।

[4] िन िल खत कथनों पर िवचार कीिजयेः

1. िकसी को उप- धानमं ी कहना केवल राजनीितक िनणय है ।


2. सं िवधान उप- धानमं ी के अिधकारों का ावधान नहीं करता है ।

उपयु म से कौन-सा/से कथन सही है /ह?


A) केवल 1

B) केवल 2

C) 1 और 2 दोनों

D) न तो 1 और न ही 2

Hide Answer -
उ रः (c)

ा ाः उपयु दोनों कथन सही ह।

सं िवधान म उप- धानमं ी के सं बंध म कोई उपबं ध नहीं है । राजनीितक बा ताओं के

कारण इसका ावधान सं िवधान से हटकर िकया गया है । धानमं ी के परामश पर रा पित

मं ि मं डल के िकसी सद को उप- धानमं ी चु न सकते ह।

सं िवधान उप- धानमं ी के िलये कोई उ रदािय या अिधकार का ावधान नहीं करता है ।

उप- धानमं ी के उ रदािय ों का िनधारण धानमं ी का िवशे षािधकार है ।

[5] के सरकार के सं दभ म िन िल खत कथनों पर िवचार कीिजयेः

1. मं ी, सं सद की िकसी सिमित का सद हो सकता है ।


2. भारत का महा ायवादी सं सद की िकसी सिमित का सद हो सकता है ।

उपयु म से कौन-सा/से कथन सही है /ह?

A) केवल 1

B) केवल 2

C) 1 और 2 दोनों

D) न तो 1 और न ही 2

Hide Answer -
उ रः (c)

ा ाः उपयु दोनों कथन सही ह।

सं िवधान के अनु े द-88 म उपबं ध है िक े क मं ी और भारत के महा ायवादी सं सद

की िकसी सिमित का सद बन सकते ह और िजस सिमित के वे सद होते ह उ

उसम बोलने और कायवाही म भाग ले ने का अिधकार होता है , िकंतु मत दे ने का अिधकार

नहीं होता।

े क मं ी और भारत के महा ायवादी िकसी भी सदन म या सदनों की सं यु बै ठक म

बोल सकते ह और िकसी कायवाही म भाग ले सकते ह िकंतु मत नहीं दे सकते ह।

[6] अिव ास ाव के सं दभ म िन िल खत म से कौन-सा कथन सही नहीं है ?


A) भारत के सं िवधान म अिव ास ाव का उ े ख नहीं है ।

B) अिव ास ाव केवल लोकसभा म लाया जा सकता है ।

यह ाव लाने के िलये लोकसभा के कम से कम 100 सद ों का समथन


C)
आव क है ।

लोकसभा के सद ों ारा ब मत से अिव ास ाव पास होने पर


D)
मं ि प रषद ागप दे दे गी।

Hide Answer -
उ रः (c)

ा ाः उपयु म से िवक (c) सही नहीं है ।

‘अिव ास ाव’ लाने के िलये लोकसभा के कम से कम 50 सद ों का समथन

आव क है ।

भारत के सं िवधान म िकसी अिव ास ाव का उ े ख नहीं है । अनु े द-118 म उपबं ध

है िक सं सद का े क सदन अपनी ि या और काय सं चालन के िविनयमन के िलये

िनयम बना सकेगा। अतः लोकसभा ने िनयम 198 म मं ि प रषद के िव अिव ास

ाव की ि या के िलये िनयम बनाया है ।

सं िवधान के अनु े द-75(3) म उपबं ध है िक मं ि प रषद लोकसभा के ित सामू िहक

प से उ रदायी होगी। अतः अिव ास ाव केवल लोकसभा म ही लाया जा सकता है ।

लोकसभा के सद ों के ब मत ारा अिव ास ाव पा रत होने पर मं ि प रषद ागप

दे दे गी।

िव ास ाव लोकसभा म सरकार ारा रखा जाता है । यह ाव धानमं ी या उसकी

मं ि प रषद या उनम से कोई एक सद रख सकता है ।

रा पित लोकसभा म िव ासमत खोए ए मं ि प रषद की सलाह मानने के िलये बा नहीं

है ।

[7] मं ि प रषद सामू िहक प से उ रदायी होती है ः

1. लोकसभा के ित
2. एक सं वैधािनक बा ता के अं तगत
3. अनु े द 75(3) के अनु सार
4. अनु े द (76) (3) के अनु सार

उपयु म से कौन-से सही ह?

A) केवल 1 और 2

B) केवल 1, 2 और 3

C) केवल 1, 2 और 4

D) 1, 2, 3 और 4
Hide Answer -
उ रः (b)

ा ाः उपयु म से कथन (4) सही नहीं है , ोंिक अनु े द 76(3) म भारत के

महा ायवादी के कत ों का उ े ख िकया गया है ।

मं ि प रषद का सामू िहक उ रदािय एक सं वैधािनक ावधान है , िजसका

अनु े द-75(3) म उपबं ध िकया गया है । मं ि प रषद सामू िहक प से लोकसभा के ित

उ रदायी होती है ।

[8] के ीय मं ि प रषद के ागप दे ने के उपरां तः

1. रा पित, वै क क व था बनने तक उसे बने रहने के िलये कहगे ।


2. अपद थ मं ि प रषद नई सरकार बनने तक पदभार का िनवहन करे गी।

उपयु म से कौन-सा/से कथन सही है /ह?

A) केवल 1

B) केवल 2

C) 1 और 2 दोनों

D) न तो 1 और न ही 2

Hide Answer -
उ रः (c)

ा ाः उपयु दोनों कथन सही ह।

मं ि प रषद के ागप दे ने के प ात रा पित वै क क व था बनने तक उसे पद पर

बने रहने के िलये कहगे । वै क क व था से अिभ ाय है ः यथा सं भव शी नई सरकार

के गठन हे तु आम चु नाव कराए जाएँ ।

वै क क व था (नई सरकार के गठन) होने तक अपद थ मं ि प रषद काय करती

रहे गी।

[9] मं ि प रषद म शािमल होता/होते है /ह?

1. कैिबने ट र के मं ी
2. रा र के मं ी
3. उपमं ी

नीचे िदये गए कूट का योग कर सही उ र का चयन कीिजयेः

A) केवल 1

B) केवल 1 और 2

C) केवल 1 और 3

D) 1, 2 और 3
Hide Answer -
उ रः (d)

ा ाः उपयु सभी सही ह।

मं ि प रषद म मं ि यों की तीन े िणयाँ शािमल होती ह और धानमं ी मं ि प रषद का

अ होता है :

1. कैिबने ट मं ीः ये मं ी कैिबने ट के सद होते ह और इसकी बै ठकों म भाग ले ते ह।

इनके उ रदािय ों की प रिध सं पूण के सरकार पर होती है ।

2. रा मं ीः रा मं ि यों को मं ालय/िवभागों का तं भार िदया जा सकता है अथवा

कैिबने ट मं ी के साथ सहयोगी बनाया जा सकता है । वे कैिबने ट के सद नहीं होते ह

और न ही कैिबने ट की बै ठक म भाग ले ते ह, िकंतु मं ालय से सं बंिधत िकसी काय हे तु

िवशे ष आमं ण पर भाग ले सकते ह।

उपमं ीः कैिबने ट अथवा रा मं ि यों के शासिनक, राजनै ितक और सं सदीय काय म

सहायता के िलये इ िनयु िकया जाता है । ये कैिबने ट सद नहीं होते और न ही

कैिबने ट की बै ठक म भाग ले ते ह।

1. सं सदीय सिचवः ये मं ि प रषद की अ म े णी म आते ह िजसे मं ालय भी कहा जाता

है । इनके पास कोई िवभाग नहीं होता। ये व र मं ि यों के साथ उनके सं सदीय काय म

सहायता के िलये िनयु होते ह। 1967 से राजीव गाँ धी के थम िव ार को छोड़कर

कोई भी सं सदीय सिचव िनयु नहीं िकया गया है ।

2. मं ि प रषद म उप- धानमं ी को भी शािमल िकया जा सकता है । उप- धानमं ी मु तः

राजनीितक कारणों से िनयु िकया जाता है ।

[10]िन िल खत कथनों पर िवचार कीिजयेः

1. मं ि प रषद श की चचा सं िवधान के मू ल प म की गई है ।


2. मं ि मं डल श का उ े ख सं िवधान के मू ल प म नहीं था।
3. मं ि मं डल श की चचा सं िवधान म केवल एक बार की गई है ।

उपयु म से कौन-सा/से कथन सही है /ह?

A) केवल 1

B) केवल 1 और 2

C) केवल 1 और 3

D) 1, 2 और 3
Hide Answer -
उ रः (d)

ा ाः उपयु सभी कथन सही ह।

सं िवधान के अनु े द-74 और 75 म मं ि प रषद की िव ृ त चचा है । अतः इस श का

उपबं ध मू ल सं िवधान म िकया गया है ।

मं ि मं डल श को 44व सं िवधान सं शोधन अिधिनयम, 1978 के ारा शािमल िकया

गया। सं िवधान के मू ल प म इसकी चचा नहीं थी।

मं ि मं डल श का उ े ख सं िवधान म केवल एक बार अनु े द-352(3) म िकया गया

है ।

[11]िन िल खत म से कौन-सा कथन सही है ?

सं िवधान के भाग-V और अनु े द-148 म अटॉन जनरल (Attorney


A)
General) का उपबं ध िकया गया है ।

भारत के रा पित ारा ऐसे को अटॉन जनरल (Attorney


B) General) िनयु िकया जाता है जो उ तम ायालय के िकसी ायाधीश
की िनयु के िलये यो ता रखता हो।

अटॉन जनरल (Attorney General) की पदाविध 6 वष या आयु 65 वष, जो


C)
भी पहले हो, होती है ।

सं िवधान के दू सरी अनु सूची म अटॉन जनरल (Attorney General) के


D)
िलये पा र िमक का उपबं ध िकया गया है ।

Hide Answer -
उ रः (b)

ा ाः उपयु म से िवक (b) सही है और िवक (a), (c) और (d) गलत ह।

सं िवधान के भाग-V के अ ाय-1 म और अनु े द-76 म भारत के महा ायवादी

(Attorney General) का उपबं ध िकया गया है ।

सं िवधान के अनु े द-76(1) म उपबं ध है िक रा पित, उ तम ायालय का ायाधीश

िनयु होने के यो िकसी को भारत का महा ायवादी िनयु करे गा।

सं िवधान म महा ायवादी के कायकाल का िनधारण नहीं िकया गया है । इसे हटाने को

ले कर कोई व था नहीं है । यह रा पित के सादपयतं पद धारण करता है अथात रा पित

िकसी भी समय उसे हटा सकता है ।

अनु े द-76(3) म उपबं ध है िक रा पित ारा तय पा र िमक उसे ा होगा। सं िवधान

म इसका पा र िमक तय नहीं िकया गया है ।

[12]भारत के महा ायवादी के सं दभ म िन िल खत कथनों पर िवचार कीिजयेः

1. महा ायवादी रा पित का थम िविध अिधकारी या सलाहकार होता है ।


2. मं ि प रषद (सरकार) के बदल जाने या ागप दे ने के प ात वह पद पर
बना रहे गा।
3. उसे ऐसी िविध से हटाया जाएगा जै से िक उ तम ायालय के िकसी
ायाधीश को हटाते ह।

उपयु म से कौन-सा/से कथन सही है /ह?

A) केवल 1

B) केवल 1 और 2

C) 1, 2 और 3

D) उपयु म से कोई नहीं।

Hide Answer -
उ रः (d)

ा ाः उपयु सभी कथन गलत ह।

भारत का महा ायवादी भारत सरकार का थम िविध अिधकारी/मु िविध सलाहकार

होता है , न िक रा पित या धानमं ी का।

महा ायवादी अपना ागप रा पित को सौंपता है िकंतु जब सरकार (मं ि प रषद) बदल

जाए या ागप दे दे तो उसे ागप दे ना होता है , ोंिक उसकी िनयु सरकार की

िसफा रश से की जाती है ।

महा ायवादी को रा पित िकसी भी समय उसके पद से हटा सकता है । उसे हटाने के

िलये िकसी कार के महािभयोग की आव कता नहीं होती।

[13]िन िल खत कथनों पर िवचार कीिजयेः

1. भारत के महा ायवादी को अपने क ों के पालन म भारत के रा े म


सभी ायालयों म सु नवाई का अिधकार है ।
2. भारत का महा ायवादी सं सद के िकसी भी सदन का सद नहीं होता, िकंतु
उसे सं सद के दोनों सदनों म बोलने , कायवाही म भाग ले ने या सं यु बै ठक म
भाग ले ने तथा मत दे ने का अिधकार है ।

उपयु म से कौन-सा/से कथन सही है /ह?

A) केवल 1

B) केवल 2

C) 1 और 2 दोनों

D) न तो 1 और न ही 2
Hide Answer -
उ रः (a)

ा ाः उपयु म से कथन (2) सही नहीं है ।

सं िवधान के अनु े द-76(3) म उपबं ध है िक महा ायवादी को अपने क ों के पालन

म भारत के रा े म सभी ायालयों म सु नवाई का अिधकार होगा।

भारत का महा ायवादी सं सद के िकसी भी सदन का सद नहीं होता, िकंतु उसे सं सद

के दोनों सदन म बोलने , कायवाही म भाग ले ने या सं यु बै ठक म भाग ले ने का अिधकार

है , परं तु मत दे ने का अिधकार नहीं है ।

[14]भारत के महा ायवादी के सं दभ िन िल खत कथनों पर िवचार कीिजयेः

1. लोकसभा की िकसी सिमित का सद हो सकता है ।


2. भारत सरकार का पू णकािलक वकील होता है ।
3. िनजी िविधक कायवाही नहीं कर सकता है ।

उपयु म से कौन-सा/से कथन सही है /ह?

A) केवल 1

B) केवल 1 और 2

C) केवल 2 और 3

D) 1, 2 और 3

Hide Answer -
उ रः (a)

ा ाः उपयु म से कथन (2) और (3) सही नहीं ह।

भारत का महा ायवादी भारत सरकार का पू णकािलक वकील नहीं होता। वह सरकारी

कम की े णी म नहीं आता, इसिलये उसे िनजी िविधक कायवाही (िनजी ै स) से नहीं

रोका जा सकता है ।

भारत का महा ायवादी िकसी भी सं सदीय सिमित का सद हो सकता है , बोल सकता

है , काय म भाग ले सकता है , िकंतु मतदान नहीं कर सकता।

[15]सॉिलिसटर जनरल िन िल खत म से ा होता है ?

A) भारत सरकार का िविधक सलाहकार

B) रा सरकार का िविधक सलाहकार

C) भारत सरकार का शासिनक अिधकारी

D) रा सरकार का शासिनक अिधकारी


Hide Answer -
उ रः (a)

ा ाः उपयु म से िवक (a) सही है ।

अटॉन जनरल (महा ायवादी) के अित र भारत सरकार के अ कानू नी सलाहकार

सॉिलिसटर जनरल (महािधव ा) और अपर महािधव ा ह, िकंतु इसकी चचा सं िवधान के

अनु े द-76 म नहीं की गई है । ये अटॉन जनरल को उसके दािय ों के िनवहन म

सहायता करते ह।

[16]िन िल खत म से कौन-सा कथन सही है ?

एडवोकेट जनरल (Advocate General) रा का सव िविधक


A)
अिधकारी होता है ।

उ ायालय के मु ायाधीश की यो ता रखने वाले को ही


B)
रा पाल एडवोकेट जनरल (Advocate General) िनयु करता है ।

एडवोकेट जनरल (Advocate General) को भारत के िकसी ायालय के


C)
सम सु नवाई का अिधकार है ।

सं िवधान के दू सरी अनु सूची म एडवोकेट जनरल (Advocae General) के


D)
वे तन व भ ों के सं बंध म उपबं ध है ।

Hide Answer -
उ रः (a)

ा ाः उपयु म से कथन (a) सही है और कथन (b), (c) और (d) गलत ह।

सं िवधान के अनु े द-165 म रा के िलये महािधव ा (Advocate General) का

उपबं ध िकया गया है । महािधव ा रा का सव िविधक/कानू नी अिधकारी/सलाहकार

होता है ।

रा के रा पाल ऐसे को रा का महािधव ा िनयु करता है जो उ

ायालय के ायधीश बनने की यो ता रखता हो न िक केवल मु ायाधीश की।

रा के महािधव ा को रा के िकसी ायालय के सम सु नवाई का अिधकार है , न

िक भारत के िकसी भी ायालय म।

रा के महािधव ा के वे तन व भ े रा पाल ारा िनधा रत िकये जाते ह न िक सं िवधान

की दू सरी अनु सूची म उपबं ध िकया गया है ।

महािधव ा रा पाल के सादपयत पद धारण करता है । रा पाल िकसी भी समय उसे

हटा सकता है । सं िवधान म इसके पदाविध का िनधारण नहीं िकया गया है ।

रा सरकार (रा मं ि प रषद) के बदलने या ागप दे ने पर महािधव ा को ागप

दे ना पड़ता है , ोंिक इसकी िनयु रा सरकार की सलाह पर होती है ।

रा का महािधव ा िवधानमं डल के िकसी भी सदन का सद नहीं होता िकंतु उसे दोनों

सदनों म बोलने , कायवाही म भाग ले ने या सं यु बै ठक म भाग ले ने का अिधकार है , परं तु

मतदान दे ने का अिधकार नहीं। है ।


[17]िनयं क एवं महाले खापरी क के सं दभ म िन िल खत म से कौन-सा कथन सही

है ?

इस पद का सृ जन सं िवधान के ारा िकया गया है और इसकी िनयु का


A)
उपबं ध अनु े द-248 म िकया गया है ।

इसकी िनयु रा पित ारा की जाती है और तथा रा पित के सादपयत ही


B)
वह पद धारण करता है ।

सं िवधान की तीसरी अनु सूची के तहत यह भारत के मु ायमू ित के सम


C)
शपथ ले गा।

इसे पद से रा पित के ारा उसी रीित से पद ु त िकया जा सकता है िजस


D)
रीित से उ तम ायालय के ायाधीशों को पद ु त िकया जाता है ।

Hide Answer -
उ रः (d)

ा ाः उपयु म से िवक (d) सही है और िवक (a), (b) और (c) गलत ह।

सं िवधान के अनु े द 148 म उपबं ध है िक िनयं क एवं महाले खा परी क को उसके पद

से उसी रीित से रा पित के ारा हटाया जा सकता है िजस रीित से उ तम ायालय के

ायाधीशों को हटाते ह। अथात सं सद म िवशे ष ब मत के साथ उसके दु वहार या

अयो ता पर ाव पा रत कर पद से हटा सकते ह। सं सद म िवशे ष ब मत का अथ है ः

े क सदन की कुल सद सं ा के ब मत तथा सदन म उप थत और मतदान करने

वाले सद ों के कम से कम 2/3 ब मत ारा पा रत होना चािहये । ऐसा सं सद के एक ही

स म ािवत और ीकृत होना चािहये ।

सं िवधान के अनु े द-148 म िनयं क एवं महाले खा परी क के सं बंध उपबं ध म है ।

िनयं क एवं महाले खा परी क की िनयु भारत के रा पित ारा की जाती है और वह

अपने पद पर 6 वष या 65 वष की आयु (जो भी पहले हो) तक बना रहता है , न िक

रा पित के सादपयत पद धारण करता है । उसे केवल रा पित ारा सं िवधान म उ े खत

ि या के तहत हटाया जा सकता है ।

सं िवधान के अनु े द-148(2) म उपबं ध है िक िनयं क एवं महाले खापरी क सं िवधान की

तीसरी अनु सूची के तहत भारत के रा पित के सम शपथ ले गा या ित ा करे गा, न िक

भारत के मु ायमू ित के सम ।

[18]िन िल खत कथनों पर िवचार कीिजयेः

1. िनयं क एवं महाले खा परी क के वे तन व से वा की अ शत के सं बंध म


उपबं ध सं िवधान की दू सरी अनु सूची म िकया गया है ।
2. िनयं क एवं महाले खा परी क के कायालय के शासिनक य भारत
सरकार की सं िचत िनिध पर भा रत होत ह तथा इस पर सं सद म मतदान
होता है ।

उपयु म से कौन-सा/से कथन सही है /ह?


A) केवल 1

B) केवल 2

C) 1 और 2 दोनों

D) न तो 1 और न ही 2

Hide Answer -
उ रः (a)

ा ाः उपयु म से कथन (2) सही नहीं है ।

भारत के िनयं क एवं महाले खा परी क के कायालय के शासिनक य िजसके अं तगत

उस कायालय म से वा करने वाले यों को या उसके सं बंध म सं देय सभी वे तन, भ े

और पशन है , भारत सरकार के सं िचत िनिध पर भा रत होंग, अतः इन पर सं सद म

मतदान नहीं होगा [अनु े द-148(6)]

सं िवधान के अनु े द-148(2) म उपबं ध है िक िनयं क एवं महाले खा परी क के वे तन व

से वा की अ शत सं िवधान की दू सरी अनु सूची के तहत होंगी।

िनयं क एवं महाले खा परी क का वे तन उ तम ायालय के ायाधीश के बराबर होता

है । वष 2009 म उ तम ायालय के ायाधीश का वे तन ` 90,000 ितमाह कर

िदया गया।

इसका वे तन एवं अ से वा श सं सद ारा िनधा रत होती ह।

[19]भारत के िनयं क एवं महाले खा परी क के सं दभ म िवचार कीिजयेः

1. वह के और रा ों का ले खा परी ण करता है ।
2. वह के और रा सरकारों के ले खों से सं बंिधत रपोट रा पित को
सौंपता है ।
3. वह अपने पद पर नहीं रहने के प ात भारत सरकार या रा सरकार के
अधीन िकसी और पद का पा नहीं होगा।

उपयु म से कौन-सा/से कथन सही है /ह?

A) केवल 3

B) केवल 1 और 2

C) केवल 1 और 3

D) 1, 2 और 3
Hide Answer -
उ रः (c)

ा ाः उपयु म से कथन (2) सही नहीं है । n

सं िवधान के अनु े द-151(1) म उपबं ध है िक िनयं क एवं महाले खा परी क के

सरकार के ले खों से सं बंिधत अपनी रपोट रा पित को सौंपता है ।

अनु े द-151(2) म उपबं ध है िक रा सरकार के ले खों से सं बंिधत अपनी रपोट

रा पाल को सौंपेगा।

अनु े द-149 म उपबं ध है िक सं घ और रा ों के ले खाओं का परी ण िनयं क एवं

महाले खा परी क ारा िकया जाएगा।

सं िवधान के अनु े द-148(4) म उपबं ध है िक अपने पद पर नहीं रहने के प ात वह

भारत सरकार या रा सरकार के अधीन िकसी और पद का पा नहीं होगा।

अनु े द-150 म उपबं ध है िक सं घ और रा ों के ले खाओं को ऐसे ा प म रखा जाएगा

जो रा पित, िनयं क एवं महाले खा परी क की सलाह पर तय कर।

[20]िन िल खत क ों म से कौन-सा एक भारत के िनयं क एवं ले खा परी क ारा


पालन नहीं िकया जाता है ?

भारत की सं िचत िनिध से होने वाले सभी यों का ले खा परी ण करना और


A)
उसके सं बंध म ितवे दन ु त करना।

आक क िनिध और लोक ले खाओं से होने वाले सभी यों का ले खा परी ण


B)
करना और उसके सं बंध म ितवे दन ु त करना।

सभी ापार, िनमाण, लाभ और हािन के ले खाओं के ले खा परी ण करना


C)
और उसके सं बंध म ितवे दन ु त करना।

सावजिनक धन की ा और िनगम पर िनयं ण करना और यह सु िनि त


D)
करना िक सावजिनक राज राजकोष म जमा हो।

Hide Answer -
उ रः (d)

ा ाः उपयु म से िवक (d) सही नहीं है ।

भारत के सं िवधान म िनयं क एवं महाले खा परी क (कैग) की प रक ना िनयं क के

साथ-साथ महाले खा परी क के प म की गई है , िकंतु वहार म भारत म कैग केवल

महाले खा परी क की भू िमका का िनवाह करता है । कैग सावजिनक धन की ा और

िनगम का िनयं ण नहीं कर सकता उसका भारत की सं िचत िनिध से धन िनकासी पर कोई

िनयं ण नहीं है । उसकी भू िमका य होने के प ात केवल ले खा परी ा की है । इसके

िवप रत ि टे न म कैग को िनयं क के साथ-साथ महापरी क की श भी ा है ।


[1] भारत के िनयं क एवं महाले खा परी क के सं दभ म िन िल खत म से कौन-सा

कथन सही नहीं है ?

A) वह लोक ले खा सिमित का िम एवं मागदशक होता है ।

B) उसे लोक िनिध का अिभभावक कहा जाता है ।

C) उसके रपोट पर अं ितम प से कायवाही सं सद ारा की जाती है ।

D) वह लोक-ले खा सिमित की बै ठकों म भाग नहीं ले सकता है ।

Hide Answer -
उ रः (d)

ा ाः उपयु म से कथन (d) सही नहीं है ।

भारत के िनयं क एवं महाले खा परी क सं सद की लोक ले खा सिमित की बै ठकों म भाग

ले सकता है ।

लोक ले खा सिमित की कायवाही की म थता िनयं क एवं महाले खा परी क करता है ,

इसिलये इसे लोक ले खा सिमित का िम और मागदशक कहा जाता है ।

िनयं क एवं महाले खा परी क सं घ और रा ों की लोक िनिधयों के सभी यों की ले खा-

परी ा करता है । इसिलये इसे लोक िव /लोक िनिध का सं र क/अिभभावक कहा जाता

है ।

सं िवधान के अनु े द-151(1) म उपबं ध है िक कैग अपनी रपोट रा पित को सौंपेगा

और रा पित उसे सं सद के दोनों सदनों के सम रखवाएगा। इस आधार पर कैग की

रपोट पर उिचत कायवाही करने की अं ितम िज़ े दारी सं सद की है ।

[2] लोक िव को सु रि त करने के साथ-साथ भारत म िनयं क एवं महाले खा


परी क (कैग) कायालय का ा मह है ?

1. जब भारत का रा पित रा ीय आपात/िव ीय आपात की घोषणा करता है


तब कैग सं सद की ओर से राजकोष पर िनयं ण रखता है ।
2. कैग ारा काया त प रयोजनाओं या काय मों पर ज़ारी िकये गए
ितवे दनों पर लोक ले खा सिमित िवचार-िवमश करती है ।
3. कैग के ितवे दनों से िमली जानका रयों के आधार पर जाँ चक ा एजिसयाँ
उन लोगों के िव आरोप दा खल कर सकती है िज ोंने लोक िव
बं धन म कानू न का उ ं घन िकया हो।
4. कैग को ऐसी िनि त ाियक श याँ ा ह िक सरकारी कंपिनयों के
ले खा-परी ा और ले खा जाँ चते समय वह कानू न का उ ं घन करने वालों
पर अिभयोग चला सके।

उपयु म से कौन-सा/से कथन सही है /ह?


A) केवल 1

B) केवल 2 और 3

C) केवल 1, 3 और 4

D) 1, 2, 3 और 4

Hide Answer -
उ रः (b)

ा ाः उपयु म से कथन (1) और (4) सही नहीं ह।

सं िवधान के अनु े द-149 म कैग की श यों और क ों के सं बंध म उपबं ध िकया

गया है ।

वहार म कैग केवल महाले खा परी क की भू िमका का िनवाह करता है , िनयं क की

भू िमका वह नहीं िनभाता जै सा िक ि टे न म िनभाता है । कैग का भारत की सं िचत िनिध से

धन िनकासी पर कोई िनयं ण नहीं है , कैग की भू िमका य होने के प ात केवल ले खा

परी ण का होता है ।

कैग सरकारी य की तकसं गतता, िन ा और िमत यता की जाँ च करता है िकंतु उसे

ऐसी ाियक श ा नहीं है िक वह सरकारी कंपिनयों के िव अिभयोग चला सके।

कैग अपनी रपोट रा पित को सौंपता है और रा पित इस रपोट को सं सद के दोनों सदनों

के सम रखवाता है और सं सद की लोक ले खा सिमित इसके रपोट पर िवचार-िवमश

करती है । यह सिमित अपनी रपोट लोकसभा अ को सौंपती है ।

कैग के ितवे दनों के आधार पर जाँ चक ा एजिसयाँ उन लोगों के िव आरोप दा खल

कर सकती है िज ोंने लोक िनिध के बं धन म कानू न का उ ं घन िकया है ।

कैग सं सद का एजट होता है और उसी के मा म से खच का ले खा परी ण करता है ,

इसिलये वह केवल सं सद के ित िज़ े दार होता है ।

[3] ले खा परी ा बोड के सं दभ म िन िल खत म से कौन-सा कथन सही नहीं ह?

A) इसकी थापना शासिनक सु धार आयोग की अनु शंसाओं पर की गई।

B) यह िनयं क एवं महाले खा परी क कायालय का एक अं ग है ।

C) इसके अ एवं दो सद होते ह।

D) इसके अ एवं सद ों की िनयु रा पित ारा की जाती है ।


Hide Answer -
उ रः (d)

ा ाः उपयु म से िवक (d) सही नहीं है ।

ले खा परी ा बोड के अ एवं सद ों की िनयु िनयं क एवं महाले खा परी क (कैग)

ारा की जाती है , न िक रा पित के ारा।

इस बोड की थापना भारतीय शासिनक सु धार आयोग की अनु शंसाओं पर वष 1968 म

की गई।

इसकी थापना िनयं क एवं महाले खा परी क कायालय के अं ग के प म की गई।

इसका एक अ और दो सद होते ह।

[4] िन िल खत म से कौन िनयं क एवं महाले खा परी क (कैग) की रपोट की

समी ा करता/करती है ?

A) लोक ले खा सिमित

B) य िवभाग, के ीय िव मं ालय

C) ले खा महािनयं क

D) मं ि मं डलीय सिचवालय

Hide Answer -
उ रः (a)

ा ाः िनयं क एवं महाले खा परी क (कैग) अपनी रपोट रा पित को सौंपता है और रा पित

उस रपोट को सं सद के दोनों सदनों के सम रखवाता है । सं सद उस रपोट को लोक ले खा

सिमित को सौंपती है और सिमित उस रपोट की समी ा करती है । कैग सिमित की सहायता

करता है । सिमित अपना ितवे दन लोकसभा अ को सौंपती है ।

[5] िन िल खत कथनों पर िवचार कीिजयेः

1. भारत का रा पित सं सद के िकसी भी सदन का सद नहीं होता है ।


2. सं सद रा पित और दोनों सदनों से िमलकर बनती है ।
3. सं सद के दोनों सदनों ारा पा रत कोई िवधे यक रा पित की ीकृित के
िबना कानू न नहीं बन सकता।

उपयु म से कौन-सा/से कथन सही है /ह?

A) केवल 1

B) केवल 1 और 2

C) केवल 2 और 3

D) 1, 2 और 3
Hide Answer -
उ रः (d)

ा ाः उपयु तीनों कथन सही ह।

सं िवधान के अनु े द-59(1) म उपबं ध है िक रा पित सं सद के िकसी भी सदन या िकसी

रा के िवधानमं डल के िकसी भी सदन का सद नहीं होगा।

सं िवधान के अनु े द-79 म उपबं ध है िक सं घ के िलये एक सं सद होगी जो रा पित और

दोनों सदनों से िमलकर बने गी। अतः रा पित सं सद का अिभ अं ग है ।

सं सद के अिभ अं ग होने के तहत दोनों सदनों ारा पा रत कोई िवधे यक रा पित की

ीकृित के प ात् ही कानू न बन पाता है ।

[6] सं सद की सं रचना के सं दभ म िन िल खत म से कौन-सा कथन सही नहीं है ?

रा सभा 250 से अनिधक सद ों से िमलकर बने गी जबिक लोकसभा 552


A)
से अनिधक सद ों से िमलकर बने गी।

रा सभा म 238 से अनिधक सद रा ों और सं घ रा े ों से होंगे। जबिक


B)
लोकसभा म 530 से अनिधक सद रा ों और सं घ रा े ों से होंगे।

रा सभा के िलये रा पित ारा 12 से अनिधक सद ों के नाम िनिद िकये


C) जाएं गे जबिक लोकसभा के िलये 2 से अनिधक सद ों के नाम िनिद िकये
जाएं गे ।

सं िवधान की चौथी अनु सूची म रा सभा म थानों के आवं टन का उपबं ध है


D)
जबिक लोकसभा के िलये सं िवधान म ऐसी कोई अनु सूची नहीं है ।

Hide Answer -
उ रः (b)

ा ाः िवक (b) सही नहीं है ।

सं िवधान के अनु े द-80(1)(ख) म उपबं ध है िक रा सभा म रा ों एवं सं घ रा े ों

से 238 से अिधक ितिनिध नहीं होंगे। सं िवधान के अनु े द-81(1)(क) म उपबं ध है िक

लोकसभा म रा ों से 530 और सं घ रा े ों से 20 से अिधक ितिनिध नहीं होंगे।

वतमान म रा सभा म 245 और लोकसभा म 545 सद ह।रा सभा म रा ों के

229 ितिनिध और सं घ रा े ों के 4 ितिनिध ह तथा 12 सद रा पित ारा

मनोनीत िकये गए ह।लोकसभा म 530 सद रा ों से , 13 सद सं घ रा े ों से

और 2 सद रा पित ारा मनोनीत िकये गए ह।

लोकसभा म अिधकतम सद सं ा 552 और रा सभा म अिधकतम सद सं ा

250 हो सकती है ।

सं िवधान की चौथी अनु सूची म उपबं ध है िक रा सभा म रा ों और सं घ रा े ों से

िकतने ितिनिध होंगे जबिक लोकसभा के िलये सं िवधान म ऐसी कोई अनु सूची नहीं है ।

[7] िन िल खत कथनों पर िवचार कीिजयेः

1. रा सभा म भारत के सभी सं घ रा े ों के ितिनिध शािमल ह।


2. रा सभा म सं घ रा े ों म िद ी के ितिनिधयों की सं ा सबसे
अिधक है ।

उपयु म से कौन-सा/से कथन सही है /ह?

A) केवल 1

B) केवल 2

C) 1 और 2 दोनों

D) न तो 1 और न ही 2

Hide Answer -
उ रः (b)

ा ाः उपयु म से कथन (1) सही नहीं है ।

रा सभा म सभी सं घ रा े ों का ितिनिध नहीं है । केवल िद ी और पु डुचे री के

ितिनिध शािमल ह।

सं घ रा े ों म िद ी के ितिनिध सबसे अिधक तीन (3) ह जबिक पु डुचे री का एक

ितिनिध है ।

[8] िन िल खत म से कौन लोकसभा और रा सभा दोनों के िनवाचन म मतदान कर


सकता है ?

A) सं सद के िन सदन के िनवािचत सद

B) सं सद के उ सदन के िनवािचत सद

C) रा िवधानमं डल के उ सदन के िनवािचत सद

D) रा िवधानमं डल के िन सदन के िनवािचत सद

Hide Answer -
उ रः (d)

ा ाः उपयु म से िवक (d) सही है ।

रा िवधानमं डल के िन सदन के िनवािचत सद ों को और रा सभा दोनों के िनवाचन

म मतदान के अिधकार ा ह। भारत के नाग रक होने के नाते वे लोकसभा के िनवाचन

म मतदान कर सकता है और रा सभा के िनवाचन म सं िवधान के अनु े द-80(4) के

तहत मतदान कर सकते ह।

[9] िन िल खत कथनों पर िवचार कीिजयेः

1. भारत के सं िवधान म उपबं ध है िक लोकसभा या रा सभा के मनोनीत


सद मं ी नहीं बन सकते ।
2. रा सभा और लोकसभा के मनोनीत सद रा पित और उप-रा पित के
िनवाचन म मतदान नहीं कर सकते ।
उपयु म से कौन-सा/से कथन सही है /ह?

A) केवल 1

B) केवल 2

C) 1 और 2 दोनों

D) न तो 1 और न ही 2

Hide Answer -
उ रः (d)

ा ाः उपयु दोनों कथन सही नहीं ह।

रा सभा के िलये 12 और लोकसभा के िलये 2 सद रा पित मनोनीत कर सकते ह

और सं िवधान इ मं ी बनाने से मना नहीं करता। अथात् वे मं ी बन सकते ह।

रा सभा और लोकसभा के मनोनीत सद उप-रा पित के िनवाचन म तो मतदान कर

सकते ह िकंतु रा पित के िनवाचन म मतदान नहीं कर सकते ।

[10]िन िल खत कथनों पर िवचार कीिजयेः

1. रा सभा के सभी सद रा ों की िवधानसभाओं के सद ों ारा


िनवािचत होते ह।
2. रा सभा म रा ों का ितिनिध उनकी जनसं ा और े फल के
अनु पात म िकया गया है , िजसकी चचा सं िवधान की चौथी अनु सूची म की
गई है ।

उपयु म से कौन-सा/से कथन सही है /ह?

A) केवल 1

B) केवल 2

C) 1 और 2 दोनों

D) न तो 1 और न ही 2

Hide Answer -
उ रः (d)

ा ाः उपयु दोनों कथन सही नहीं ह।

रा सभा के कुल 250 सद ों म से 238 सद रा ों की िवधानसभाओं के सद ों

ारा िनवािचत होते ह जबिक 12 सद रा पित ारा मनोनीत िकये जाते ह।

रा सभा म रा ों का ितिनिध उनकी जनसं ा के अनु पात ( े फल के अनु पात म

नहीं होगा) म िकया गया है और सं िवधान की चौथी अनु सूची म इसका िनधारण कर िदया

गया है िक िकस रा से रा सभा की िकतनी सीट होंगी।

[11]िन िल खत कथनों पर िवचार कीिजयेः


1. रा सभा एक थायी सदन है और इसे कभी भं ग नहीं िकया जा सकता।
2. रा सभा के 2/3 सद े क दो वष की समा पर से वािनवृ हो
जाते ह।
3. रा सभा के सद ों का कायकाल छः वष का होता है ।
4. रा सभा के सद चु ने जाने के िलये ू नतम आयु 25 वष होनी चािहये ।

उपयु म से कौन-सा/से कथन सही है /ह?

A) केवल 1 और 3

B) केवल 1, 2 और 3

C) केवल 1, 3 और 4

D) 1, 2, 3 और 4

Hide Answer -
उ रः (a)

ा ाः उपयु म से कथन (2) और (4) सही नहीं ह।

सं िवधान के अनु े द 83(1) म उपबं ध है िक रा सभा का िवघटन नहीं होगा।

रा सभा के एक-ितहाई (1/3) सद े क दो वष की समा पर से वािनवृ होंगे।

ात हो िक रा सभा के थम बै च के 2 वष की समा के बाद 1/3 सद ों की

से वािनवृ ि लॉटरी के मा म से की गई थी।

रा सभा के सद ों का कायकाल छः वष का होता है । इसका िनधारण सं िवधान म नहीं

िकया गया है ब जन ितिनिध अिधिनयम, 1951 के आधार पर तय आ।

सं िवधान के अनु े द-84(ख) म उपबं ध है िक रा सभा की सद ता के िलये ू नतम

आयु 30 वष होनी चािहये ।

[12]मू ल सं िवधान म लोकसभा सद ों की सं ा 500 िनधा रत की गई थी िकंतु दो


सं िवधान सं शोधनों के ारा इनकी सं ा म वृ की गई। वे सं िवधान सं शोधन ह -

A) 7वाँ और 31वाँ

B) 6ठवाँ और 22वाँ

C) 12वाँ और 38वाँ

D) 11वाँ और 42वाँ
Hide Answer -
उ रः (a)

ा ाः उपयु म से िवक (a) सही है ।

7व सं िवधान सं शोधन अिधिनयम, 1956 के ारा रा ों का पु नगठन िकया गया और

लोकसभा की सद सं ा 500 से बढ़ाकर 520 िनि त की गई।

31व सं िवधान सं शोधन अिधिनयम, 1973 के ारा लोकसभा की सद सं ा 520 से

बढ़ाकर 545 िनि त की गई,िजसम 530 सद रा ों से , 13 सद सं घ रा े ों से

तथा रा पित ारा मनोनीत 2 एं ो-इं िडयन सद , यिद रा पित को ऐसा लगे िक

लोकसभा म एं ो-इं िडयन का ितिनिध पया नहीं ह।

लोकसभा की अिधकतम सं ा 552 हो सकती है ,िजसम 530 सद रा ों से 20

सद सं घ रा े ों से और 2 सद रा पित ारा मनोनीत एं ो-इं िडयन।

एं ो-इं िडयन को लोकसभा म मनोनीत करने का उपबं ध 95व सं िवधान सं शोधन

अिधिनयम, 2009 के ारा वष 2020 तक के िलये बढ़ा िदया गया है ।

[13]िन िल खत कथनों पर िवचार कीिजयेः

1. लोकसभा का सद बनने के िलये ू नतम आयु 21 वष है ।


2. लोकसभा म मतदान करने के िलये ू नतम आयु 18 वष होनी चािहये ।
3. मतदान का अिधकार वै धािनक अिधकार है न िक सं वैधािनक।

उपयु म से कौन-सा/से कथन सही है /ह?

A) केवल 2

B) केवल 1 और 2

C) केवल 2 और 3

D) 1, 2 और 3

Hide Answer -
उ रः (a)

ा ाः उपयु म से कथन (1) और (3) सही नहीं है ।

सं िवधान के अनु े द 84(ख) म उपबं ध है िक लोकसभा का सद बनने के िलये ू नतम

आयु 25 वष होनी चािहये ।

लोकसभा म मतदान करने के िलये ू नतम आयु 18 वष है । 61व सं िवधान सं शोधन

अिधिनयम, 1988 के ारा मतदान करने की ू नतम आयु 21 वष से घटाकर 18 वष

कर दी गई है ।

सं िवधान के अनु े द-326 म लोकसभा और िवधानसभा म मत दे ने के अिधकार के सं बंध

म उपबं ध है । अतः मत दे ने का अिधकार सं वैधािनक और वै धािनक दोनों है ।

[14]िन िल खत कथनों पर िवचार कीिजयेः


लोकसभा म सीटों का आवं टन े क रा को उसकी जनसं ा के आधार
पर िनधा रत है ।
लोकसभा म सीटों का आवं टन उन रा ों पर लागू नहीं होता िजनकी
जनसं ा 60 लाख से कम है ।
वतमान लोकसभा म े क रा के िलये सीटों का आवं टन 2001 की
जनगणना पर आधा रत है ।

उपयु म से कौन-सा/से कथन सही है /ह?

A) केवल 1

B) केवल 1 और 2

C) केवल 1 और 3

D) 1, 2 और 3

Hide Answer -
उ रः (b)

ा ाः उपयु म से कथन (3) सही नहीं है ।

सं िवधान के अनु े द 81(2)(क) म उपबं ध है िक लोकसभा म सीटों का आवं टन उस

रा की जनसं ा के अनु पात म होगा, िकंतु यह उपबं ध उन रा ों पर लागू नहीं होता

िजनकी जनसं ा 60 लाख से कम है ।

सं िवधान के अनु े द-81(2)(ख) म रा ों को ादे िशक िनवाचन े ों म िवभािजत करने

सं बंधी उपबं ध है ।

वतमान लोकसभा म े क रा के िलये थानों/सीटों का आवं टन 1971 की जनगणना

के आधार पर िकया गया है । इसका उपबं ध सं िवधान के अनु े द-82 म िकया गया है ।

यह व था वष 2000 तक के िलये थी िकंतु 84व सं िवधान सं शोधन अिधिनयम, 2001

के ारा इसे 2026 तक के िलये बढ़ा िदया गया है । वष 2026 तक लोकसभा की सीटों/

थानों म प रवतन नहीं होगा।

[15]87व सं िवधान सं शोधन अिधिनयम, 2003 का सं बंध िनवाचन े ों के प रसीमन


से है । इस सं शोधन के ारा िनवाचन े ों के प रसीमन म िकस जनगणना को
अपनाने के िलये कहा गया है ?

A) जनगणना, 1971

B) जनगणना, 1991

C) जनगणना, 2001

D) जनगणना, 2011
Hide Answer -
उ रः (c)

ा ाः उपयु म से िवक (c) सही है ।

84व सं िवधान सं शोधन अिधिनयम, 2001 के ारा रा ों के ादे िशक िनवाचन े ों का

पु नः समायोजन के िलये 1991 की जनगणना को आधार बनाने के िलये कहा गया। बाद म

87व सं िवधान सं शोधन, अिधिनयम, 2003 के ारा िनवाचन े ों के प रसीमन के िलये

2001 की जनगणना को आधार बनाने के िलये कहा गया।

सं सद ने प रसीमन आयोग अिधिनयम, 1952, 1962, 1972 एवं 2002 पा रत िकया

है ।

[16]लोकसभा म अनु सूिचत जाितयों एवं जनजाितयों के िलये आर ण का उपबं ध िकया

गया है । इस सं दभ म िन िल खत म से कौन-सा कथन सही है ?

A) सं िवधान के अनु े द-331 म इनके िलये आर ण का उपबं ध िकया गया है ।

B) ये सामा िनवाचन े से चु नाव नहीं लड़ सकते ह।

वतमान म इनके आरि त सीटों के आवं टन म 1971 की जनगणना को


C)
आधार बनाया गया है ।

D) इनकी आर ण की समय सीमा बढ़ाकर वष 2020 कर दी गई है ।

Hide Answer -
उ रः (d)

ा ाः उपयु म से िवक (d) सही है ।

95व सं िवधान सं शोधन अिधिनयम, 2009 के ारा इनकी आर ण की समय सीमा

बढ़ाकर वष 2020 कर दी गई है ।

सं िवधान के अनु े द-330 म लोकसभा म इनके िलये आर ण के सं बंध म उपबं ध िकया

गया है ।

वतमान म लोकसभा म इनके िलये आरि त सीटों के आवं टन के िलये 2001 की

जनगणना को आधार बनाया गया है । यह ावधान 87व सं िवधान सं शोधन अिधिनयम,

2003 के ारा िकया गया।

अनु सूिचत जाित एवं जनजाित के िलये आरि त े ों से केवल वही चु नाव लड़ सकते ह,

िकंतु ये सामा िनवाचन े से भी चु नाव लड़ सकते ह। इ इनकी जनसं ा के आधार

पर सीटों का आर ण िदया जाता है । अनु सूिचत जनजाित की सवािधक आरि त सीट म

दे श(6) म ह।

[17]लोकसभा के कायकाल के सं दभ म िन िल खत म कौन–सा कथन सही है ?


A) िकसी भी प र थित म नहीं बढ़ाया जा सकता।

B) एक बार म छः माह तक के िलये बढ़ाया जा सकता है ।

आपातकाल की घोषणा के समय एक बार म एक वष तक के िलये बढ़ाया जा


C)
सकता है ।

आपातकाल की घोषणा के समय एक बार म दो वष के िलये बढ़ाया जा


D)
सकता है ।

Hide Answer -
उ रः (c)

ा ाः उपयु म से िवक (c) सही है ।

लोकसभा का कायकाल आम चु नाव के बाद पहली बै ठक से पाँ च वष का होता है ।

अनु े द-83(2)

लोकसभा को पाँ च वष पहले भी रा पित मं ि प रषद की सलाह पर िवघिटत कर सकता है

और इसके खलाफ ायालय म चु नौती नहीं दी जा सकती।

आपातकाल की घोषणा के समय लोकसभा का कायकाल एक बार म एक वष के िलये

बढ़ाया जा सकता है , िकंतु आपातकाल समा होने के प ात इसका िव ार छः माह से

अिधक नहीं हो सकता है ।

[18]भारत के िनयं क एवं महाले खा परी क के सं दभ म िन िल खत म से कौन-सा

कथन सही नहीं है ?

A) वह लोक ले खा सिमित का िम एवं मागदशक होता है ।

B) उसे लोक िनिध का अिभभावक कहा जाता है ।

C) उसके रपोट पर अं ितम प से कायवाही सं सद ारा की जाती है ।

D) वह लोक-ले खा सिमित की बै ठकों म भाग नहीं ले सकता है ।

Hide Answer -
उ रः (d)

ा ाः उपयु म से कथन (d) सही नहीं है ।

भारत के िनयं क एवं महाले खा परी क सं सद की लोक ले खा सिमित की बै ठकों म भाग

ले सकता है ।

लोक ले खा सिमित की कायवाही की म थता िनयं क एवं महाले खा परी क करता है ,

इसिलये इसे लोक ले खा सिमित का िम और मागदशक कहा जाता है ।,

िनयं क एवं महाले खा परी क सं घ और रा ों की लोक िनिधयों के सभी यों की ले खा-

परी ा करता है । इसिलये इसे लोक िव /लोक िनिध का सं र क/अिभभावक कहा जाता

है ।

सं िवधान के अनु े द-151(1) म उपबं ध है िक कैग अपनी रपोट रा पित को सौंपेगा

और रा पित उसे सं सद के दोनों सदनों के सम रखवाएगा। इस आधार पर कैग की

रपोट पर उिचत कायवाही करने की अं ितम िज़ े दारी सं सद की है ।


[19]िन िल खत म से कौन-से सं सद सद बनने के िलये पा नहीं ह?

1. ायालय ने घोषणा कर दी है िक िवकृत िच है ।


2. िकसी अपराध म को दो वष या उससे अिधक की सज़ा ई हो।
3. िनधा रत समय के अं दर चु नावी खच का ौरा दे ने म असफल रहा है ।
4. छु आछूत, दहे ज़ या सती था जै से अपराधों म सं िल पाया गया है ।

नीचे िदये गए कूट का योग कर सही उ र का चयन कीिजयेः

A) केवल 1 और 2

B) केवल 1, 2 और 3

C) केवल 1, 2 और 4

D) 1, 2, 3 और 4

Hide Answer -
उ रः (d)

ा ाः उपयु सभी कथन सही ह।

सं िवधान और जन ितिनिध अिधिनयम, 1951 म िकसी के सं सद सद बनने

सं बंधी पा ता का िनधारण िकया गया है ।

सं िवधान म उपबं ध है िक कोई सं सद सद नहीं बन सकता यिद ायालय ने

िकसी को िवकृत िच घोिषत कर िदया है ।

जन ितिनिध अिधिनयम, 1951 म अपा ता के िलये िन िल खत उपबं ध िकये गए ह;

1. िकसी अपराध म को दो वष या उससे अिधक की सज़ा ई हो।

2. िनधा रत समय के अ र चु नावी खच का ौरा नहीं िदया हो।

3. उसे छु आछूत, दहे ज़ या सती था जै से अपराधों म सं िल पाया गया हो।

सं िवधान के अनु े द-103 म उपबं ध है िक सं सद सद अनु े द-102(1) के तहत

यो है या अयो है का िनणय रा पित करे गा और रा पित का िनणय अ म होगा, िकंतु

रा पित ऐसे िकसी का िनणय ले ने से पहले िनवाचन आयोग की राय ले गा और उसी

राय के अनु सार िनणय करे गा।

[20]िन िल खत म से कौन-सा कथन सही है ?


सं िवधान के अनु े द-102(1) के तहत सं सद सद ों के िलये यो ता या
A)
अयो ता सं बंधी िनणय भारत के िनवाचन आयोग ारा िकया जाता है ।

सं िवधान की दसवीं अनु सूची दल-बदल अिधिनयम से सं बंिधत है िजसे 85व


B)
सं िवधान सं शोधन अिधिनयम ारा शािमल िकया गया है ।

91व सं िवधान सं शोधन अिधिनयम के ारा दल-बदल अिधिनयम को और


C)
अिधक सश बनाया गया।

नािमत या नाम िनदिशत कोई सं सद सद छः माह बाद िकसी राजनीितक


D)
दल म शािमल होता है तो सद ता नहीं ागनी पड़ती है ।

Hide Answer -
उ रः (c)

ा ाः उपयु म से कथन (c) सही है ।

91व सं िवधान सं शोधन अिधिनयम, 2003 के ारा दल-बदल कानू न म सं शोधन िकया

गया। इसके ारा दल-बदल कानू न को और अिधक सश बनाया गया।

सं िवधान के अनु े द 103 म उपबं ध है िक सं सद का कोई सद अनु े द 102(1) के

तहत यो है या नहीं है , इसका िनणय रा पित ारा िकया जाता है और उसका िनणय

अं ितम होता है , िकंतु इससे पू व रा पित िनवाचन आयोग से राय ले गा और उसी के अनु सार

काय करे गा।

सं िवधान की दसवीं अनु सूची का सं बंध दल-बदल अिधिनयम से है िजसे 52व सं िवधान

सं शोधन अिधिनयम, 1985 के ारा सं िवधान म जोड़ा गया।

यिद कोई सं सद सद े ा से अपनी राजनीितक दल की सद ता ागता है या अपने

राजनीितक दल के िनदशों के िव सं सद म मतदान करता है या नहीं करता है या कोई

िनदलीय सद िकसी राजनीितक दल म शािमल हो जाता है या कोई नािमत या नाम

िनदिशत सद छः माह बाद िकसी राजनीितक दल म शािमल हो जाता है तो सद ता

ागनी पड़ती है ।

10वीं अनु सूची के तहत अयो ता के सवालों का िनपटारा रा सभा म सभापित और

लोकसभा म लोकसभा अ करता है । वष 1992 म उ तम ायालय ने िनणय िदया

िक सभापित/अ ारा िदये गए िनणयों की ाियक समी ा हो सकती है ।


[1] सं सद सद ों की पद र के सं दभ म िन िल खत म से कौन-सा कथन सही
नहीं है ?

सं सद के िकसी सदन का सद 60 िदनों की अविध तक सदन की आ ा के


A) िबना उसके सभी अिधवे शनों से अनु प थत रहता है तो सदन उसके थान को
र घोिषत कर सकता है ।

कोई सं सद के दोनों सदनों का सद चु न िलया जाता है और 10 िदनों


B) के अं दर सू चना नहीं दे ता है िक उसे िकस सदन म रहना है तो रा सभा म
उसकी सद ता र मानी जाएगी।

कोई सं सद और रा िवधानमं डल के िकसी सदन का सद एक ही


समय म िनवािचत हो जाता है और 14 िदनों के अं दर रा के िवधानमं डल
C)
की सीट खाली नहीं करता है तो िवधानमं डल की उसकी सद ता तः ही
समा हो जाएगी।

सं सद के िकसी सदन का सद , दू सरे सदन का भी सद चु न िलया जाता है


D)
तो पहले वाले सदन म उसका पद र हो जाएगा।

Hide Answer -
उ रः (c)

ा ाः उपयु म से िवक (c) सही नहीं है ।

सं िवधान के अनु े द 101(2) म उपबं ध है िक कोई भी सं सद और िकसी रा के

िवधानमं डल के िकसी सदन का सद एक साथ नहीं हो सकता। यिद एक साथ िनवािचत

हो जाता है और 14 िदनों के अं दर रा िवधानमं डल की सद ता का ाग नहीं करता है

तो इस अविध की समा के प ात् वह सं सद का सद नहीं रहे गा।

सं िवधान के अनु े द-101(1) म उपबं ध है िक कोई एक सं सद के दोनों सदनों का

सद नहीं हो सकता। यिद कोई दोनों सदनों का सद िनवािचत हो जाता है और

10 िदनों के अं दर इसकी सू चना नहीं दे ता है िक उसे िकस सदन का सद रहना है तो

उस अविध की समा के प ात् उसकी सद ता रा सभा से र मानी जाएगी।

अनु े द 101(4) म उपबं ध िकया गया है िक सं सद के िकसी भी सदन का कोई सद

60 िदनों की अविध तक सदन की आ ा के िबना उसके सभी अिधवे शनों म अनु प थत

रहता है तो सदन उसके थान को र घोिषत कर सकेगा, िकंतु इस दौरान सदन का

स ावसान या सदन लगातार चार से अिधक िदनों के िलये थिगत रहता है तो इस अविध

को इसम शािमल नहीं िकया जाएगा।

अनु े द-101(3) म उपबं ध है िक रा सभा का सद सभापित को और लोकसभा का

सद अ को अपना ागप सौंप सकता है । सभापित/अ ारा ागप ीकार

कर िलया जाता है तो उसका थान र हो जाएगा।

कोई अयो सं सद सद िनवािचत हो जाता है तो लोक- ितिनिध अिधिनयम,

1951 के तहत उ ायालय चु नाव को अमा या शू घोिषत कर सकता है । असं तु

उ ायालय के िनणय के िव उ तम ायालय म अपील कर सकता है ।


[2] िन िल खत कथनों पर िवचार कीिजयेः

1. सं सद के े क सदन का े क सद रा पित या उसके ारा िनयु


के सम सं िवधान की दू सरी अनु सूची के तहत शपथ या ित ा
ले गा।
2. शपथ या ित ान के पू व यिद कोई सदन के सद के प म
सदन म बै ठता है तो उसे ितिदन ₹500 जु माना भरना होगा।

उपयु म से कौन-सा/से कथन सही है /ह?

A) केवल 1

B) केवल 2

C) 1 और 2 दोनों

D) न तो 1 और न ही 2

Hide Answer -
उ रः (b)

ा ाः उपयु म से कथन (1) सही नहीं है ।

सं सद सद रा पित या उसके ारा िनयु के सम सं िवधान की तीसरी अनु सूची

(न िक दू सरी अनु सूची) के तहत शपथ या ित ान ले गा (अनु े द-99)

सं सद का कोई सद िबना शपथ या ित ान िलये सदन की बै ठक म भाग ले ता है तो

₹500 ितिदन जु माना भरना पड़े गा (अनु े द-104)

जब तक सं सद सद शपथ या ित ान नहीं ले ता सदन की िकसी बै ठक म भाग नहीं ले

सकता और न ही मत दे सकता है ।

[3] िन िल खत कथनों पर िवचार कीिजयेः

1. कोरम/गणपू ित के िलये लोकसभा म कम से कम 100 सद और


रा सभा म कम से कम 50 सद अव होने चािहये ।
2. लोकसभा या रा सभा के पीठासीन अिधकारी केवल बराबर की थित म
मतदान कर सकता है ।

उपयु म से कौन-सा/से कथन सही है /ह?

A) केवल 1

B) केवल 2

C) 1 और 2 दोनों

D) न तो 1 और न ही 2
Hide Answer -
उ रः (b)

ा ाः उपयु म से कथन (2) सही है ।

सं िवधान के अनु े द-100(3) म उपबं ध है िक सं सद के े क सदन का अिधवे शन

गिठत करने के िलये गणपू ित/कोरम सदन के सद ों की कुल सं ा का 1/10 भाग होना

चािहये । अतः लोकसभा के िलये कम-से -कम 55 सद और रा सभा के िलये कम-से -

कम 25 सद उप थत होने चािहये ।

अनु े द-100(1) म उपबं ध है िक े क सदन की बै ठक म या सदनों के सं यु बै ठक

म िनणय, उप थत व मत दे ने वाले सद ों के ब मत से िकया जाएगा। िजसम अ या

सभापित को शािमल नहीं िकया जाता है । अ /सभापित केवल मत बराबर की थित म

िनणायक मत दे ता है ।

[4] िन िल खत म से कौन-सा कथन सही नहीं है ?

सं सद की बै ठक को थिगत करने की श लोकसभा अ /रा सभा के


A)
सभापित के पास है ।

B) सं सद के स ावसान की श रा पित के पास है ।

C) सं सद को िवघिटत करने की श रा पित के पास है ।

D) सं सद के दो स ों के बीच अिधकतम अं तराल छः माह का हो सकता है ।

Hide Answer -
उ रः (c)

ा ाः उपयु म से िवक (c) सही नहीं है ।

रा पित रा सभा का िवघटन नहीं कर सकता, ोंिक रा सभा एक थायी सदन है ।

रा पित केवल लोकसभा को िवघिटत कर सकता है । अनु े द-85(2)(ख)

लोकसभा के अ /रा सभा के सभापित सं सद की बै ठक को कुछ घं टों के िलये , कुछ

िदन के िलये , कुछ स ाह के िलये या अिनि त काल के िलये थिगत कर सकता है ।

सं िवधान के अनु े द-85(2)(क) म उपबं ध है िक रा पित सं सद के दोनों सदनों या िकसी

एक सदन का स ावसान कर सकता है ।

अनु े द-85(1) म उपबं ध है िक सं सद के दो स ों के बीच अिधकतम अं तराल छः माह

से अिधक नहीं हो सकता है ।

[5] रा सभा के सं दभ म िन िल खत म से कौन-से कथन सही ह? नीचे िदए गये कूट


की सहायता से सही उ र चु न :

1. इसको भं ग नहीं िकया जा सकता।


2. इसका कायकाल छः वष का होता है ।
3. इसके एक-ितहाई सद हर दो वष बाद िनवृ हो जाते ह।
4. इसके सद 30 वष से कम उ के नहीं होंगे।
कूट:

A) केवल 1 और 3

B) केवल 1, 3 और 4

C) केवल 2, 3 और 4

D) 1, 2, 3 और 4

Hide Answer -
उ रः (b)

ा ाः उपयु म से कथन (2) सही नहीं है ।

रा सभा एक थायी सदन है । इसे कभी भं ग नहीं िकया जा सकता है । इसके सद ों का

कायकाल छः वष का होता है ।

इसके एक-ितहाई सद े क दो वष पर िनवृ होते ह।

रा सभा सद बनने के िलये ू नतम आयु 30 वष होनी चािहये ।

[6] िकसी सं सद सद की दल प रवतन से उ अयो ता के सं दभ म िन िल खत


कथनों पर िवचार कीिजयेः

1. यिद उस मू ल राजनीितक दल म िजससे वह चु ना गया था एक िवभाजन हो


जाए।
2. यिद उसने े ा से अपने राजनीितक दल की सद ता छोड़ दी हो।
3. यिद उसने अपने राजनीितक दल के िनदशों के िव सदन के मतदान म
भाग नहीं िलया हो।
4. यिद सदन का कोई मनोनीत सद छः माह बाद िकसी राजनीितक दल
का सद बन जाए।

उपयु म से कौन-से कथन सही ह?

A) केवल 2 और 3

B) केवल 2 और 4

C) केवल 2, 3 और 4

D) 1, 2, 3 और 4
Hide Answer -
उ रः (d)

ा ाः उपयु सभी कथन सही ह।

52व सं िवधान सं शोधन अिधिनयम, 1985 के ारा सं िवधान म 10वीं अनु सूची जोड़ी गई,

िजसे दल-बदल कानू न के नाम से जाना जाता है । इस कानू न म दल-बदल से उ सं सद

सद की अयो ता के सं बंध म ावधान है ।

यिद उसने े ा से अपने राजनीितक दल की सद ता छोड़ दी है ।

यिद वह अपने राजनीितक दल के िनदशों के िव सदन के मतदान म भाग ले ता है या

अनु प थत रहता है ।

कोई िनदलीय सद चु नाव के बाद िकसी राजनीितक दल की सद ता हण कर ले ता है ।

कोई नाम िनदिशत सद अपनी सद ता हण करने के छः माह बाद िकसी राजनीितक

दल की सद ता हण कर ले ता है ।

91व सं िवधान सं शोधन अिधिनयम, 2003 के ारा दल-बदल कानू न म प रवतन िकया

गया और इसे और अिधक सश बनाया गया। 52व सं िवधान सं शोधन के तहत सं सद के

िकसी सद को दल म टू ट (Split) के कारण बाहर होने पर अयो नहीं माना जाता था।

अथात यिद िकसी राजनीितक दल के 1/3 सद सदन म एक नए दल का गठन कर

ले ते ह तो उसे अयो नहीं माना जाएगा, िकंतु 91व सं िवधान सं शोधन के ारा इस

ावधान को समा कर िदया गया।

[7] सं सद के िकसी भी सदन के सद की अयो ता सं बंधी ों का िनणय करता है -

A) िनवाचन आयोग

B) सं सद का सं बंिधत सदन

C) भारत का मु ायमू ित

D) िनवाचन आयोग की सलाह पर रा पित

Hide Answer -
उ रः (d)

ा ाः उपयु म से िवक (d) सही है ।

सं िवधान के अनु े द-103 म उपबं ध है िक यिद यह उठता है िक सं सद के िकसी

सदन का कोई सद अनु े द-102(1) के तहत अयो ता से हो गया है या नहीं,

इसका िनणय रा पित करे गा, और उसका िनणय अं ितम होगा। िकंतु ऐसा करने से पहले

रा पित िनवाचन आयोग की राय ले गा और उसकी राय के अनु सार काय करे गा।

यिद दल प रवतन के आधार पर सं सद सद ों की अयो ता का उठता है तो इसका

िनणय सदन का अ करता है िकंतु अ का िनणय अं ितम नहीं होगा और उसकी

ाियक समी ा की जा सकती है (िकहोतो होलोहन वाद, 1993)

[8] सं िवधान के अनु सार लोकसभा के अिधवे शनः-


वष म तीन बार होने चािहये और दो अिधवे शनों के म दो माह से अिधक का
A)
अं तराल नहीं होना चािहये ।

वष म दो बार होने चािहये और दो अिधवे शनों के म तीन माह से अिधक का


B)
अं तराल नहीं होना चािहये ।

वष म दो बार होने चािहये और दो अिधवे शनों के म चार माह से अिधक का


C)
अं तराल नहीं होना चािहये ।

वष म दो बार होने चािहये और दोनों अिधवे शनों के म छः माह से अिधक


D)
का अं तराल नहीं होना चािहये ।

Hide Answer -
उ रः (d)

ा ाः उपयु म से िवक (d) सही है ।

सं िवधान के अनु े द 85(1) म उपबं ध है िक रा पित समय-समय पर सं सद के ेक

सदन को ऐसे समय और थान पर, जो ठीक समझे , अिधवे शन के िलये आ त करे गा िकंतु

उसके एक स की अं ितम बै ठक और आगामी स की थम बै ठक के िलये िनयत तारीख

के बीच छः माह से अिधक का अं तराल नहीं होना चािहये ।

[9] िन िल खत कथनों पर िवचार कीिजयेः

1. रा सभा म रा ों के ितिनिध रा की िवधानसभाओं के िनवािचत


सद ों ारा आनु पाितक ितिनिध णाली के अनु सार एकल सं मणीय
मत ारा चु ने जाते ह।
2. रा सभा म सं घ रा े ों के ितिनिध एक िनवाचक मं डल के सद ों
ारा अ प से आनु पाितक ितिनिध णाली के अनु सार एकल
सं मणीय मत ारा चु ने जाते ह।
3. रा सभा म रा ों और सं घ रा े ों के ितिनिधयों ारा भरे जाने वाले
थानों का आवं टन सं िवधान की चौथी अनु सूची म िनधा रत उपबं धों के
अनु सार होता है ।उपयु से कौन-सा/से कथन सही है /ह?

A) केवल 1

B) केवल 1 और 3

C) केवल 2 और 3

D) 1, 2 और 3
Hide Answer -
उ रः (d)

ा ाः उपयु सभी कथन सही ह।

सं िवधान के अनु े द-80(4) म उपबं ध है िक रा सभा म े क रा के ितिनिधयों का

िनवाचन उस रा के िवधानसभा के िनवािचत सद ों ारा आनु पाितक ितिनिध प ित

के अनु सार एकल सं मणीय मत ारा िकया जाएगा। मतदान की इस णाली को ‘हे यर

योजना’ कहते ह। भारत म रा पित और रा सभा के सद ों के िनवाचन म इसे अपनाया

जाता है ।

सं िवधान के अनु े द-80(5) म उपबं ध है िक रा सभा म सं घ रा े ों के ितिनिध

ऐसी रीित से चु ने जाएं गे जो सं सद िविध ारा व था करे । सं सद ारा लोक ितिनिध

अिधिनयम, 1950 म िनवाचन के रीित के सं बंध म उपबं ध िकया गया है िक िनवाचक-

मं डल के सद ों ारा अ प से आनु पाितक ितिनिध णाली के अनु सार एकल

सं मणीय मत के ारा चु ने जाएं गे ।

अनु े द-80(2) म उपबं ध है िक रा सभा म रा ों और सं घ रा े ों के ितिनिधयों

ारा भरे जाने वाले थानों का आवं टन सं िवधान की चौथी अनु सूची म िनधा रत उपबं धों के

अनु सार होगा।

[10]लोकसभा अ के सं दभ म िन िल खत म से कौन-सा कथन सही है ?

A) इसका अ मनोनीत िकया जाता है ।

B) इसका अ िनवािचत िकया जाता है ।

C) इसका अ िनयु िकया जाता है ।

D) इसका अ चयिनत िकया जाता है ।

Hide Answer -
उ रः (d)

ा ाः उपयु म से िवक (d) सही है ।

सं िवधान के अनु े द-93 म उपबं ध है िक लोकसभा अपने दो सद ों को अपना अ

और उपा चु नेगी। जब-जब अ या उपा का पद र होगा तब लोकसभा

िकसी अ सद को अ या उपा चु नेगी।

[11]लोकसभा अ के सं दभ म िन िल खत कथनों पर िवचार कीिजयेः

1. लोकसभा के अ के िलये चु नाव की तारीख रा पित िनधा रत करता है ।


2. लोकसभा िवघटन के प ात् होने वाले थम अिधवे शन के ठीक पहले तक
अ अपने पद पर बना रहता है ।

उपयु म से कौन-सा/से कथन सही है /ह?


A) केवल 1

B) केवल 2

C) 1 और 2 दोनों

D) न तो 1 और न ही 2

Hide Answer -
उ रः (c)

ा ाः उपयु दोनों कथन सही ह।

भारत का रा पित लोकसभा अ के चु नाव की तारीख िनधा रत करता है । लोकसभा की

पहली बै ठक के प ात् उप थत सद ों के बीच से अ का चु नाव िकया जाता है ।

अनु े द-94(ग) म उपबं ध है िक अ लोकसभा िवघटन के प ात् नई लोकसभा के

थम अिधवे शन के ठीक पहले तक पद पर बना रहे गा।

[12]लोकसभा अ के पद के िवषय म िन िल खत कथनों पर िवचार कीिजयेः

1. लोकसभा अ रा पित के सादपयत पद धारण करता है ।


2. अ बनने के िलये आव क नहीं िक वह लोकसभा का सद हो,
िकंतु अ चु ने जाने के प ात छः माह के अं दर उसे लोकसभा की
सद ता हण करनी होगी।
3. यिद सामा अविध से पू व लोकसभा को भं ग कर िदया जाता है तो उसे
अपना पद छोड़ना होगा।
4. अ अपना ागप दे ना चाहे तो उसे अपना ागप रा पित को
सं बोिधत करना होगा।

उपयु म से कौन-सा/से कथन सही है /ह?

A) केवल 1

B) केवल 3 और 4

C) केवल 1, 2 और 3

D) उपयु म से कोई नहीं


Hide Answer -
उ रः (d)

ा ाः उपयु सभी कथन गलत ह।

लोकसभा का अ आमतौर पर लोकसभा के जीवन काल तक पद धारण करता है । वह

रा पित के सादपयत पद धारण नहीं करता है ।

लोकसभा अ बनने के िलये यह आव क है िक वह लोकसभा का सद हो। यिद वह

लोकसभा का सद नहीं है तो अ नहीं रहे गा। [अनु े द 94(क)]

लोकसभा के िवघटन के प ात् भी अ नई लोकसभा के थम अिधवे शन के ठीक पहले

तक पद धारण करता रहे गा। [अनु े द 94(ग)]

अ अपना ाग प दे ना चाहे तो उपा को सौंपेगा, न िक रा पित को। [अनु ेद

94(ख)]

[13]िन िल खत कथनों पर िवचार कीिजयेः

1. लोकसभा के अ को पद से लोकसभा के त ालीन सम सद ों के


दो-ितहाई ब मत से पा रत सं क के ारा हटा सकते ह।
2. लोकसभा अ को पद से हटाने सं बंधी सं क ु त िकये जाने के
आशय की सू चना कम से कम 14 िदन पहले दे ना आव क है ।

उपयु म से कौन-सा/से कथन सही है /ह?

A) केवल 1

B) केवल 2

C) 1 और 2 दोनों

D) न तो 1 और न ही 2

Hide Answer -
उ रः (b)

ा ाः उपयु म से कथन (1) सही नहीं है ।

सं िवधान के अनु े द 94(ग) म उपबं ध है िक लोकसभा के अ को पद से लोकसभा के

त ालीन सम सद ों के ब मत से (न िक दो-ितहाई ब मत) पा रत सं क ारा हटा

सकते ह और यह सं क ु त करने के आशय की सू चना कम से कम 14 िदन पहले

दे ना आव क है ।

[14]यिद लोकसभा अ को पद से हटाने सं बंधी सं क लोकसभा की िकसी बै ठक


म िवचाराधीन है , तो उसके िवषय म ा सही नहीं है ?
A) बै ठक म उप थत रहने पर भी पीठासीन नहीं होगा।

B) लोकसभा म बोलने और कायवाही म भाग ले ने का अिधकार होगा।

C) कायवाही के दौरान िकसी िवषय पर थमतः मत नहीं दे सकता।

D) मत बराबर होने की थित म िनणायक मत नहीं दे सकता।

Hide Answer -
उ रः (c)

ा ाः उपयु म से िवक (c) सही नहीं है ।

सं िवधान के अनु े द-96 म उपबं ध है िक जब लोकसभा अ को उसके पद से हटाने

सं बंधी सं क िवचाराधीन हो तब अ पीठासीन नहीं होगा, िकंतु उसे लोकसभा म

बोलने और कायवाही म भाग ले ने का अिधकार होगा। उसे िकसी सं क या कायवािहयों के

दौरान िकसी िवषय पर थमतः मतदान करने का अिधकार होगा, परं तु मत बराबर होने

की थित म वह िनणायक मत नहीं दे सकता है ।

[15]िन िल खत कथनों पर िवचार कीिजयेः

1. लोकसभा का अिभर क भारत का रा पित होता है ।


2. लोकसभा के अ एवं उपा का पद र होने पर रा पित ारा
िनयु अ के क ों का िनवहन करे गा।
3. लोकसभा के अ और उपा की अनु प थित म अ ारा
सभापित तािलका म नामां िकत सद अ के क ों का िनवहन
करे गा।
4. लोकसभा के उपा के चु नाव की तारीख रा पित ारा िनधा रत की
जाती है ।

उपयु म से कौन-से कथन सही ह?

A) केवल 2 और 3

B) केवल 1, 2 और 3

C) केवल 1, 3 और 4

D) 1, 2, 3 और 4
Hide Answer -
उ रः (a)

ा ाः उपयु म से कथन (1) और (4) गलत ह।

लोकसभा का सं र क/अिभर क/अिभभावक लोकसभा अ होता है न िक भारत का

रा पित। वह लोकसभा और उसके ितिनिधयों का मु खया होता है । वह सद ों की

श यों और िवशे षािधकारों का अिभभावक होता है । वह सदन का मु व ा होता है

और सभी सं सदीय मामलों म उसका िनणय अं ितम होता है ।

लोकसभा के उपा के चु नाव की तारीख लोकसभा अ िनधा रत करता है , न िक

भारत का रा पित।

लोकसभा के अ और उपा का पद र होने पर भारत के रा पित ारा िनयु

अ के क ों का िनवहन करे गा।

लोकसभा अ सद ों म से 10 को सभापित तािलका के िलये नामां िकत करता है ।

इनम से कोई भी अ या उपा की अनु प थित म अ के क ों का िनवहन कर

सकेगा।

[16]िन िल खत म से कौन-सा कथन सही नहीं है ?

सं सद के दोनों सदनों की सं यु बै ठक की अ ता लोकसभा अ ारा


A)
की जाती है ।

B) सं सद के दोनों सदनों की सं यु बै ठक रा पित ारा आ त की जाती है ।

कोई िवधे यक धन िवधे यक है या नहीं, इसके अं ितम िनणय का अिधकार


C)
लोकसभा अ का होता है ।

D) लोकसभा अ िकसी सं सदीय सिमित का अ नहीं हो सकता।

Hide Answer -
उ रः (d)

ा ाः उपयु म से िवक (d) सही नहीं है ।

लोकसभा अ लोकसभा की सभी सं सदीय सिमितयों के सभापित िनयु करता है और

उनके काय का पयवे ण करता है । वह सं सदीय सिमित का अ हो सकता है , जै से-

काय मं णा-सिमित, िनयम सिमित और सामा योजन सिमित का अ लोकसभा का

अ होता है ।

सं िवधान के अनु े द 108 के तहत सं सद के दोनों सदनों की सं यु बै ठक रा पित ारा

आ त की जाती है िकंतु सं यु बै ठक की अ ता लोकसभा अ करता है ।

सं िवधान के अनु े द 110(3) म उपबं ध है िक कोई िवधे यक धन िवधे यक है या नहीं,

लोकसभा अ का िनणय अं ितम होता है ।

[17]िन िल खत कथनों पर िवचार कीिजयेः


1. यिद लोकसभा उपा को िकसी सं सदीय सिमित का सद बनाया जाता
है तो वह उस सिमित का सभापित/अ बन जाता है ।
2. लोकसभा अ स ाधारी दल से और उपा िवप ी दल से होता है ।

उपयु म से कौन-सा/से कथन सही है /ह?

A) केवल 1

B) केवल 2

C) 1 और 2 दोनों

D) न तो 1 और न ही 2

Hide Answer -
उ रः (c)

ा ाः उपयु दोनों कथन सही ह।

जब कभी लोकसभा उपा को िकसी सं सदीय सिमित का सद बनाया जाता है तो

ाभािवक प से उसका सभापित बन जाता है ।

11वीं लोकसभा से एक सहमित बनी िक लोकसभा अ स ाधारी दल से जबिक

उपा िवप ी दल से होगा। इससे पू व लोकसभा अ और उपा दोनों स ाधारी

दल से ही होते थे ।

[18]दसवीं अनु सूची के तहत दल-बदल उपबं ध के आधार पर सं सद सद की


अयो ता के सं दभ म िन िल खत कथनों पर िवचार कीिजयेः

1. इसका िनणय लोकसभा अ के ारा िकया जाता है ।


2. लोकसभा अ ारा िदये गए िनणय को ायालय म चु नौती नहीं दी जा
सकती।

उपयु म से कौन-सा/से कथन सही है /ह?

A) केवल 1

B) केवल 2

C) 1 और 2 दोनों

D) न तो 1 और न ही 2
Hide Answer -
उ रः (a)

ा ाः दल-बदल उपबं ध के आधार पर सं सद सद ों के अयो ता के सं बंध म लोकसभा

अ के िनणय की ाियक समी ा हो सकती है । यह िनणय िकहोतो होलोहन वाद, 1992 म

ायालय ारा िदया गया।

सं िवधान के अनु े द 102(2) म दसवीं अनु सूची के तहत दल-बदल उपबं ध के आधार

पर सं सद सद को अयो घोिषत करने के सं बंध म उपबं ध है । इस सं बंध म िनणय

लोकसभा अ ारा िकया जाता है ।

[19] ो-टे म (Pro-Tem) ीकर के सं दभ म िन िल खत म से कौन-सा/से कथन


सही है /ह?

1. इसका पद अ कािलक होता है ।


2. यह नई लोकसभा की पहली बै ठक म पीठासीन अिधकारी होता है ।
3. इसका मु क नए सद ों को शपथ िदलाना है ।

नीचे िदये गए कूट का योग कर सही उ र का चयन कीिजयेः

A) केवल 3

B) केवल 1 और 3

C) केवल 2 और 3

D) 1, 2 और 3

Hide Answer -
उ रः (d)

ा ाः उपयु सभी कथन सही ह।

सं िवधान म यह व था है िक पहली लोकसभा का अ नई लोकसभा की पहली बै ठक

के ठीक पहले तक अपने पद पर रहता है । इसिलये रा पित सामा तः लोकसभा के व र

सद को ो-टे म ीकर (सामाियक अ ) चु नता है और उसे शपथ िदलाता है ।

इसका पद अ कािलक होता है । नए अ चु ने जाने के बाद यह पद तः समा हो

जाता है ।

वह नई लोकसभा की पहली बै ठक म पीठासीन अिधकारी होता है ।

इसका मु क नई लोकसभा के सद ों को शपथ िदलवाना है ।

[20]लोकसभा अ और उपा के सं दभ म िवचार कीिजयेः

1. डॉ. स दान िस ा के ीय िवधान प रषद के थम भारतीय उपा


िनयु ए थे ।
2. िव ल भाई जे . पटे ल क ीय िवधान प रषद के थम भारतीय अ
िनवािचत ए थे ।
3. लोकसभा के थम अ और उपा मशः अयं गर और मावलं कर
थे ।

उपयु म से कौन-सा/से कथन सही है /ह?

A) केवल 1

B) केवल 1 और 2

C) केवल 2 और 3

D) 1, 2 और 3

Hide Answer -
उ रः (b)

ा ाः उपयु म से कथन (3) सही नहीं है ।

लोकसभा के थम अ जी.वी. मावलं कर एवं उपा अयं गर थे ।

अ एवं उपा पद का उ व भारत शासन अिधिनयम 1919 के उपबं ध के तहत

सन् 1921 म आ था। उस समय अ को े सीडे और उपा को िड ी े सीडे

कहते थे ।

1921 म गवनर जनरल, जो यं पीठासीन अिधकारी था, ने े ड रक ाइट को क ीय

िवधान प रषद का अ एवं डॉ. स दान िसं हा को उपा िनयु िकया। डॉ.

स दा द िस ा इसके उपा िनयु होने वाले थम भारतीय थे ।

सन 1925 म िव लभाई जे .पटे ल क ीय िवधान प रषद के अ िनवािचत होने वाले

थम भारतीय थे ।

लोकसभा की थम मिहला अ मीरा कुमार बनी।

लोकसभा के थम जनजातीय अ पी.ए. सं गमा थे ।

थम लोकसभा अ जी.वी. मावलं कर के िव लोकसभा म अिव ास ाव लाया गया था

िकंतु लोकसभा ारा इसे अ ीकृत कर िदया गया।


[1] रा सभा के सभापित/उपसभापित के सं दभ म िन िल खत म से कौन-सा/से
कथन सही है /ह?

1. सभापित रा सभा का सद नहीं होता।


2. उपसभापित रा सभा का सद होता है ।
3. सभापित को हटाने सं बंधी कोई सं क रा सभा म िवचाराधीन की थित
म वह सदन म बोल सकता है , कायवाही म भाग ले सकता है , िकंतु मत
नहीं दे सकता।

नीचे िदये गए कूट का योग कर सही उ र का चयन कीिजयेः


कूटः

A) केवल 2

B) केवल 1 और 2

C) केवल 2 और 3

D) 1, 2 और 3

Hide Answer -
उ रः (d)

ा ाः उपयु सभी कथन सही ह।

रा सभा का सभापित उसका सद नहीं होता है , ोंिक भारत का उप-रा पित ही

रा सभा का सभापित होता है और उप-रा पित का िनवाचन होता है ।

रा सभा अपने सद ों के बीच से उप-सभापित को चु नती है । इसिलये उप-सभापित

रा सभा का सद होता है । रा सभा का सद नहीं रहने पर वह उप-सभापित नहीं

रहे गा।

रा सभा के सभापित/उप-सभापित को पद से हटाने सं बंधी कोई सं क रा सभा म

िवचाराधीन है तो वह सदन की बै ठक म उप थत रहने पर भी पीठासीन नहीं रहे गा।

जब उपरा पित (सभापित) को उसके पद से हटाने सं बंधी सं क रा सभा म िवचाराधीन

है तब वह रा सभा म बोल सकता है , कायवाही म भाग ले सकता है , िकंतु मत नहीं दे

सकता है । लोकसभा अ को थमतः मत दे ने का अिधकार है , िक ु िनणायक मत नही

दे सकता है ।

[2] िन िल खत म से कौन-सा कथन सही नहीं है ?

A) रा सभा का उपसभापित अपना ाग-प सभापित को सौंपेगा।

रा सभा के त ालीन सम सद ों के ब मत से पा रत सं क ारा


B)
उपसभापित को हटाया जा सकता है ।

रा सभा के सभापित/उपसभापित का वे तन व भ े भारत के सं िचत िनिध पर


C)
भा रत होते ह और सदन म इस पर मतदान होता है ।

D) सं िवधान म उपरा पित के िलये वे तन व भ े के सं बंध म उपबं ध नहीं है ।


Hide Answer -
उ रः (c)

ा ाः उपयु म से कथन (c) सही नहीं है ।

रा सभा के सभापित/उपसभापित का वे तन व भ े भारत के सं िचत िनिध पर भा रत होते

ह और सदन म उस पर मतदान नहीं होता है । इसका वे तन सं सद ारा िनधा रत होता है

और सं िवधान की दू सरी अनु सूची म इसके िलये उपबं ध है ।

रा सभा का उपसभापित अपना ाग-प सभापित को सौंपता है ।

रा सभा के त ालीन सम सद ों के ब मत से पा रत सं क के ारा उपसभापित

को पद से हटा सकते ह। इस सं क के आशय की सू चना 14 िदन पहले दे ना आव क

है ।

सं िवधान म उपरा पित के वे तन व भ े के सं बंध म उपबं ध नहीं है । उसे रा सभा के पदे न

सभापित के प म वे तन व भ े िमलते ह।

[3] सं सद के अिधका रयों म स िलत ह।

1. लोकसभा का अ
2. लोकसभा का उपा
3. रा सभा का सभापित
4. लोकसभा का महासिचव

नीचे िदये गए कूट का योग कर सही उ र का चयन कीिजयेः

A) केवल 1 और 2

B) केवल 1 और 3

C) केवल 1, 2 और 3

D) 1, 2, 3 और 4

Hide Answer -
उ रः (d)

ा ाः उपयु सभी सं सद के अिधका रयों म शािमल ह।

सं सद के अिधका रयों म िन को शािमल िकया जाता है -

1. लोकसभा अ

2. लोकसभा उपा

3. रा सभा का सभापित

4. रा सभा का उपसभापित

5. लोकसभा का महासिचव

6. रा सभा का महासिचव

[4] भारतीय सं सद का सिचवालयः-


A) सं सदीय काय मं ी के अधीन है ।

B) रा पित के अधीन है ।

C) सरकार से तं है ।

D) सव ायालय के अधीन है ।

Hide Answer -
उ रः (c)

ा ाः उपयु म से िवक (c) सही है ।

सं िवधान के अनु े द 98 म उपबं ध है िक सं सद के िलये पृ थक सिचवालय होगा।

जनवरी 1926 म सरकार से तं और असं ब सिचवालय अ म आया और

तं ता प ात् इसे अपना िलया गया और अनु े द 98 म इसके िलये उपबं ध िकया गया।

सं सद का सिचवालय सरकार से तं है ।

सं िवधान के अनु े द 98(1) म उपबं ध है िक सं सद के े क सदन के पृ थक सिचवीय

कमचारीवृ होंग।

अनु े द 98(2) म उपबं ध है िक सं सद िविध ारा सं सद के े क सदन के सिचवीय

कमचारीवृं द म भत का और िनयु यों की से वा की शत का िविनयमन कर

सकेगी।

अनु े द 98(3) म उपबं ध है िक जब तक सं सद ऐसा उपबं ध नहीं करती तब तक

रा पित लोकसभा के अ और रा सभा के सभापित के परामश के प ात् ऐसा कर

सकेगा।

लोकसभा सिचवालय लोकसभा अ और रा सभा सिचवालय रा सभा के सभापित के

पयवे ण म रहते ह।

दोनों सदनों के सिचवालयों का मु खया महासिचव होता है जो एक थायी अिधकारी होता है

और उसकी िनयु सदन का अिधकारी करता है ।

[5] अ , सदन के िकसी भी सद को बोलने से रोक सकता है और अ िकसी


सद को बोलने दे सकता है । यह घटना कहलाती है -

A) मयादा (Decorum)

B) प ाग (Crossing the Floor)

C) अं त दन (Inter-Pellation)

D) बै ठ जाना (Yielding the Floor)


Hide Answer -
उ रः (d)

ा ाः उयु म से िवक (d) सही है ।

बै ठ जाना (Yielding the floor): अ िकसी भी सद को बोलने से रोक कर

अ को बोलने के िलये कह सकता है ।

मयादा (Decorum): िवन एवं ीकाय वहार बनाए रखना।

प ाग (Crossing the Floor): सदन के िकसी सद ारा वह िजस दल से

चु नकर आया है , उसका ाग कर दू सरे दल म शािमल होना।

अं त दन (Inter Pellation): िकसी सरकारी अिधकारी से िकसी नीित या काय के

िववरण की मां ग हे तु सं सदीय ि या।

[6] िन िल खत कथनों पर िवचार कीिजयेः-

1. रा सभा का सभापित तथा उप-सभापित उस सदन के सद नहीं होते ।


2. रा पित के िनवाचन म सं सद के दोनों सदनों के मनोनीत सद ों को
मतदान का कोई अिधकार नही होता, जबिक उनको उप-रा पित के
िनवाचन म मतदान का अिधकार होता है ।

उपयु म से कौन-सा/से कथन सही है /ह?

A) केवल 1

B) केवल 2

C) 1 और 2 दोनों

D) न तो 1 और न ही 2

Hide Answer -
उ रः (b)

ा ाः उपयु म से कथन (1) सही नहीं है ।

रा सभा का सभापित उस सदन का सद नहीं होता, जबिक उप-सभापित उस सदन

का सद होता है । रा सभा का सभापित उप-रा पित होता है और उप-रा पित का

िनवाचन होता है ।

रा पित के िनवाचन म सं सद के मनोनीत सद भाग नहीं ले ते, जबिक उप-रा पित के

िनवाचन म सं सद के सभी सद भाग ले ते ह।

[7] िन िल खत म से कौन-सा कथन सही है ?


A) सं सद म लं िबत िवधे यक सदनों के थगन के प ात् पगत हो जाते ह।

B) सं सद म लं िबत िवधे यक सदनों के स ावसान के प ात् पगत हो जाते ह।

सं सद म िवचाराधीन ाव, सं क नोिटस या यािचका सदनों के स ावसान


C)
के प ात् पगत हो जाते ह।

सं सद म िवचाराधीन ाव, सं क नोिटस या यािचका सदनों के थगन के


D)
प ात् पगत हो जाते ह।

Hide Answer -
उ रः (c)

ा ाः उपयु म से िवक (c) सही है ।

सं सद म िवचाराधीन ाव, सं क नोिटस या यािचका सदनों के स ावसान के प ात

पगत हो जाते ह, िकंतु सं सद म लं िबत िवधे यक पर स ावसान के प ात् कोई भाव नहीं

पड़ता है ।

सं सद म लं िबत िवधे यक, िवचाराधीन ाव, सं क नोिटस या यािचका पर सदनों के

थगन का कोई भाव नहीं पड़ता है ।

ि टे न म सदनों के स ावसान के कारण िवधे यक या अ लं िबत काय पगत/समा हो

जाते ह।

[8] िन िल खत म से कौन-सा कथन सही नहीं है ?

ऐसा िवधे यक जो सं सद के दोनों सदनों म असहमित के कारण पा रत न आ


A) हो और लोकसभा के िवघटन के पू व रा पित ने दोनों सदनों की सं यु बै ठक
बु लाने की नोिटस दे दी हो, लोकसभा िवघटन के प ात् पगत नहीं होगा।

ऐसा िवधे यक जो सं सद के दोनों सदनों ारा पा रत हो गया हो और रा पित


B) की ीकृित के िलये िवचाराधीन हो, लोकसभा िवघटन के प ात् पगत नहीं
होगा।

ऐसा िवधे यक जो दोनों सदनों ारा पा रत हो परं तु रा पित ारा पु निवचार के


C)
िलये लौटा िदया गया हो, लोकसभा िवघटन के प ात् पगत नहीं होगा।

लोकसभा म पा रत िकंतु रा सभा म िवचाराधीन िवधे यक लोकसभा के


D)
िवघटन के प ात् पगत नहीं होगा।
Hide Answer -
उ रः (d)

ा ाः उपयु म से िवक (d) सही नहीं है ।

लोकसभा म लं िबत सभी िवधे यक, ाव, सं क , यािचका आिद लोकसभा के िवघटन के

प ात् समा हो जाते ह। िकंतु लं िबत िवधे यकों और सभी लं िबत आ ासनों िजनकी जाँ च

सरकारी आ ासन सं बंधी सिमित ारा की जानी होती है लोकसभा के िवघटन पर समा

नहीं होते ।

लोकसभा के िवघटन के प ात् िवधे यकों पर िन िल खत भाव पड़ता है ः

1. लोकसभा म पा रत िकंतु रा सभा म िवचाराधीन िवधे यक लोकसभा िवघटन के प ात्

समा हो जाता है ।

2. लोकसभा म िवचाराधीन िवधे यक िवघटन के प ात् समा हो जाता है ।

3. ऐसा िवधे यक जो रा सभा म िवचाराधीन हो परं तु लोकसभा ारा पा रत न हो, िवघटन

के प ात् समा नहीं होगा

[9] सं सद म भाषा के सं दभ म िन िल खत म से कौन-सा/से कथन सही है /ह?

1. भारत के सं िवधान म िहं दी और अं े ज़ी भाषा सदन की कायवाही की भाषा


घोिषत है ।
2. राजभाषा अिधिनयम, 1963 िहं दी के साथ अं े ज़ी की िनरं तरता की
अनु मित दे ता है ।
3. सं सद के पीठासीन अिधकारी िकसी सद को अपनी मातृ भाषा म बोलने
का अिधकार दे सकता है ।

नीचे िदये गए कूट का योग कर सही उ र का चयन कीिजयेः

A) केवल 1

B) केवल 1 और 2

C) केवल 1 और 3

D) 1, 2 और 3

Hide Answer -
उ रः (d)

ा ाः उपयु सभी कथन सही ह।

सं िवधान म उपबं ध है िक सं घ की राजभाषा िहं दी और िलिप दे वनागरी होगी। राजभाषा

अिधिनयम,1963 उपबं ध करता है की सदन की कायवाही अं े जी के साथ-साथ िहं दी म

होगी।

राजभाषा अिधिनयम, 1963 िहं दी के साथ अं े ज़ी की िनरं तरता की अनु मित दे ता है ।

सं सद म पीठासीन अिधकारी िकसी सद को अपनी मातृ भाषा म बोलने का अिधकार दे

सकता है ।
[10]सं सदीय कायवाही ‘ काल’ के सं दभ म िन िल खत म से कौन-सा कथन सही
नहीं है ?

A) सं सद का पहला घं टा काल के िलये होता है ।

B) तारां िकत के बाद पू रक पू छ सकते ह।

C) अतारां िकत के बाद पू रक नहीं पू छ सकते ह।

अ -सू चना के पू छने के िलये कम से कम 30 िदन का नोिटस दे ना


D)
आव क है ।

Hide Answer -
उ रः (d)

ा ाः उपयु म से िवक (d) सही नहीं है ।

अ -सू चना के पू छने के िलये कम से कम 10 िदन का नोिटस दे ना आव क है न

िक 30 िदन।

सं सद का पहला घं टा ‘ काल’ के िलये होता है । इस दौरान सं सद सद पू छ सकते

ह और इसका उ र सामा तः मं ी दे ते ह, िकंतु कोई ऐसा िजसकी िवषय व ु िकसी

ाइवे ट सद (जो मं ी नहीं है ) से सं बंिधत है तो उससे भी पू छ सकते ह।

तीन कार के होते ह;

तारां िकत ः- इसका उ र मौ खक िदया जाता है और इसके बाद पू रक पू छ सकते

ह।

अतारां िकत ः- इसका उ र िल खत होता है और इसके बाद पू रक नहीं पू छ सकते

ह।

अ -सू चना के ः- कम से कम 10 िदन का नोिटस दे कर ऐसे पू छ सकते ह।

[11]सू ची I को सू ची II से सु मेिलत कीिजयेः

सू ची-I ( ) सू ची-I I(सं बंिधत रं ग)

A. तारां िकत 1. पीले

B. अतारां िकत 2. ह े गु लाबी

C. अ -सू चना के 3. सफेद

D. ाइवे ट सद ों से पू छा गया 4. हरे

नीचे िदये गए कूट का योग कर सही उ र का चयन कीिजयेः


A B C D
A) 1 2 3 4

B) 1 3 2 4

C) 4 3 2 1

D) 4 2 3 1

Hide Answer -
उ रः (c)

ा ाः उपयु म से िवक (c) सही है ।

तारां िकत, अतारां िकत, अ -सू चना के और ाइवे ट सद ों से पू छे जाने वाले ों की

सू ची अलग-अलग रं गों की छपी होती है , तािक वे एक-दू सरे से अलग िदख।

तारां िकत ों की सू ची- हरा रं ग

अतारां िकत ों की सू ची- सफेद रं ग

अ -सू चना के ों की सू ची- ह े गु लाबी रं ग

ाइवे ट सद ों से पू छे जाने वाले ों की सू ची- पीला रं ग

[12]सं सदीय कायवाही ‘शू काल’ (Zero Hour) के सं दभ म िन िल खत कथनों पर

िवचार कीिजयेः

1. शू काल ि िटश सं सदीय व था की दे न है ।


2. इसकी अविध दोपहर 12 बजे से 1. 00 बजे तक होती है ।
3. सं सदीय ि या म इसका उ े ख नहीं िकया गया है ।

उपयु म से कौन-सा/से कथन सही है /ह?

A) केवल 2

B) केवल 1 और 2

C) 2 और 3 दोनों

D) 1, 2 और 3

Show Answer +

[13]सं सद के ‘ थगन ाव’ का ल है ः

A) मं ि यों से सू चना एक करना।

B) बजट ावों म कटौती करना।

C) सरकार की िकसी िविश नीित की आलोचना करना।

D) सरकार को िगराना।
Hide Answer -
उ रः (c)

ा ाः उपयु म से िवक (c) सही है ।

यह एक िन ा ाव है । इसके ारा सरकार की िकसी िविश नीित की आलोचना की

जाती है । यह ाव मं ि मं डल पर िनयं ण का एक मह पू ण साधन है ।

इस ाव को लाने के िलये 50 सद ों का समथन आव क है । इसे लोकसभा एवं

रा सभा दोनों म पे श िकया जा सकता है । सदन का कोई भी सद इस ाव को पे श

कर सकता है ।

[14]भारतीय सं सद म ‘ थगन ाव’ लाने का ाउ े है ?

A) सावजिनक मह के िनि त आव क मु े पर बहस करने हे तु।

B) िवप ी सद ों का मं ि यों से सू चना ा करने हे तु।

C) िकसी अनु दान की मां ग म एक िनि त मा ा म कटौती करने हे तु।

कुछ सद ों के िहं सक अथवा अनु िचत वहार पर रोक लगाने के िलये


D)
कायवाही थिगत करने हे तु।

Hide Answer -
उ रः (a)

ा ाः उपयु म से िवक (a) सही है ।

थगन ाव िकसी अिवलं बनीय लोक मह के मामले पर सदन म चचा करने के िलये

सदन को थिगत करने का ाव है ।

इसके ारा िन िल खत लोक मह के मामलों को उठाये जा सकते ह-

जो िनि त, त ा क, अ ं त ज़ री एवं लोक मह का हो।

इसम एक से अिधक मु ों को शािमल नहीं िकया जा सकता।

वतमान घटनाओं के िकसी मह पू ण िवषय को उठा सकते ह।

िवशे षािधकार के को नहीं उठा सकते ।

ायालय म िवचाराधीन मामलों पर चचा नहीं की जा सकती।

उस िवषय पर चचा नहीं की जा सकती िजस पर उसी स म चचा हो चु की है ।

[15]सू ची-I को सू ची-II से सु मेिलत कीिजयेः-

सू ची-
सू ची-II(सं बंिधत त )
I( ाव)

A.कटौती 1. अ ाव क सावजिनक मह के मामले पर मं ि यों का


ाव ान आकिषत करना।

B. 2. मं ि यों ारा िदये गए गलत या अपू ण उ र की ओर


ानाकषण लोकसभा अ का ान आकिषत करना।
ाव

C.
3. बजट ावों पर य कम करने का ाव ािवत
िवशे षािधकार
करना।
ाव

D. थगन
4. समय से पू व सदन को थिगत करना।
ाव

नीचे िदये गए कूट का योग कर सही उ र का चयन कीिजयेः


A B C D

A) 3 1 2 4

B) 3 2 1 4

C) 1 3 4 2

D) 1 4 3 2

Hide Answer -
उ रः (a)

ा ाः

कटौती ावः- बजट ावों पर य कम करने का ाव ािवत करना।

ानाकषण ावः- सं सद का कोई सद सदन के पीठासीन अिधकारी की अि म

अनु मित से िकसी मं ी का ान अिवलं बनीय लोक मह के मामले पर आकिषत कर

सकता है । यह भारत की दे न है और यह 1954 म अ म आया। सं सदीय ि या के

िनयमों म इसका उ े ख है ।

िवशे षािधकार ावः- इसे सं सद का कोई भी सद पे श कर सकता है । इसम मं ि यों

ारा िदये गए गलत या अपू ण उ र की ओर लोकसभा अ का ान आकिषत करना

होता है । इसका उ े सं बंिधत मं ि यों की िनं दा करना है ।

थगन ावः- िकसी अिवलं बनीय लोक मह के मामले पर सदन म चचा करने के िलये ,

सदन की कायवाही को थिगत करने का ाव है । इसके ारा समय से पू व सदन का

थगन हो जाता है ।

[16] ानाकषण ाव के सं दभ म िन िल खत म से कौन-सा/से कथन सही है /ह?

1. इसने थगन ाव के े को सीिमत िकया है ।


2. इस ाव म िन ा का प शािमल नहीं होता है ।

नीचे िदये गए कूट का योग कर सही उ र का चयन कीिजयेः


कूट:
A) केवल 1

B) केवल 2

C) 1 और 2 दोनों

D) न तो 1 और न ही 2

Hide Answer -
उ रः (c)

ा ाः उपयु दोनों कथन सही ह।

शू काल म समान ानाकषण ाव भारतीय सं सदीय ि या की दे न है और यह

1954 से अ म आया।

इसने थगन ाव के े को सीिमत िकया है ।

यह थगन ाव के स श है िकंतु इसम िनं दा का प शािमल नहीं रहता। इसिलये इसे

मं ि मं डल को उ रदायी बनाने का उपाय नहीं माना जाता है ।

[17]िनं दा ाव के सं दभ म िन िला खत कथनों पर िवचार कीिजये -

1. इसे केवल सं पूण मं ि प रषद के िव ही लाया जा सकता है ।


2. इसे पा रत होने पर मं ि प रषद को ाग-प दे ना अिनवाय है ।

उपयु म से कौन-सा/से कथन सही है /ह?

A) केवल 1

B) केवल 2

C) 1 और 2 दोनों

D) न तो 1 और न ही 2

Hide Answer -
उ रः (d)

ा ाः उपयु दोनों कथन गलत ह।

िनं दा ाव सं सदीय कायवाही का एक साधन है ।इसे िकसी एक मं ी या मं ि यों के समू ह

या सं पूण मं ि प रषद के िव लाया जा सकता है ।

इसे पा रत होने पर मं ि प रषद को ागप दे ना आव क नहीं है । इससे केवल

मं ि प रषद की कुछ नीितयों एवं काय की आलोचना होती है ।

इसे लोकसभा म ीकार करने का कारण बताना अिनवाय होता है ।

[18]‘अिव ास ाव’ के सं दभ म िन िल खत म से कौन-सा कथन सही है ?


इसे िकसी एक मं ी, मं ि यों के समू ह या सं पूण मं ि प रषद के िव लाया जा
A)
सकता है ।

इसे लोकसभा म पा रत होने पर मं ि प रषद को ाग-प दे ना आव क नहीं


B) है , अिपतु इससे िसफ सरकार की कुछ नीितयों एवं काय की आलोचना होती
है ।

C) इसे लोकसभा म ीकार करने का कारण बताना आव क नहीं है ।

D) इस ाव के समथन म कम से कम 100 सद ों की सहमित अिनवाय है ।

Hide Answer -
उ रः (c)

ा ाः उपयु म से िवक (c) सही है ।

सं िवधान के अनु े द- 75 (3) म उपबं ध है िक मं ि प रषद सामू िहक प से लोकसभा के

ित उ रदायी होगी। मं ि प रषद तभी तक बनी रहती है जब तक लोकसभा म उसे ब मत

ा है । अतः इसे िसफ सं पूण मं ि प रषद के िव लोकसभा के िव ास म िनधारण हे तु

लाया जाता है । अिव ास ाव को लोकसभा म ीकार करने का कारण बताना

आव क नहीं होता।

लोकसभा म पा रत होने का अथ है िक मं ि प रषद का लोकसभा म ब मत नहीं है । उसने

लोकसभा म िव ास को खो िदया है । ऐसी थित म मं ि िपरषद को ाग-प दे ना पड़ता

है ।

अिव ास ाव के समथन म 50 सद ों की सहमित अिनवाय है , न िक 100 सद ों

की।

[19]‘ध वाद ाव’ के सं दभ म िन िल खत म से कौन-सा कथन सही नहीं है ?

इसके ारा भारत का रा पित े क आम चु नाव के पहले स और िव ीय


A)
वष के पहले स म सदन को सं बोिधत करता है ।

यह ाव सदन के सद ों को सरकार एवं शासन की आलोचना के अवसर


B)
उपल कराता है ।

C) रा पित के इस सं बोधन को ि टे न के राजा के अिभभाषण से िलया गया है ।

इस ाव पर चचा केवल लोकसभा म होती है ।


D)
Hide Answer -
उ रः (d)

ा ाः उपयु म से िवक (d) गलत है ।

ध वाद ाव के ारा भारत का रा पित े क आम चु नाव के पहले स और िव ीय

वष के पहले स म सदन को सं बोिधत करता है । अपने सं बोधन म रा पित पू ववत वष

और आने वाले वष म सरकार की नीितयों एवं योजनाओं का खाका खींचता है ।

यह ाव सदन म सद ों (लोकसभा या रा सभा) को चचा एवं वाद-िववाद के मु े

उठाने और ु िटयों एवं किमयों हे तु सरकार एवं शासन की आलोचना का अवसर उपल

कराता है ।

रा पित के इस सं बोधन को ि टे न के राजा के अिभभाषण से िलया गया है ।

सं सद के दोनों सदनों म इस ाव पर चचा होती है । बहस के बाद ाव को मत-

िवभाजन के िलये रखा जाता है । इस ाव का सदन म पा रत होना आव क है , नहीं तो

इसका अथ सरकार की पराजय मानी जाएगी।

[20]सं सदीय ि यािविध के अनु सार ‘िगलोिटन’ का अथ है िक लोकसभा अ -

जारी काय को रोक कर सदन के सम िकसी िवषय को मतदान के िलये


A)
रखता है ।

B) समय समा होने से पू व सदन को थिगत कर दे ता है ।

C) िकसी ाव पर मतदान की अनु मित नहीं दे ता है ।

D) िकसी सद को दु वहार के िलये फटकारता है ।


Hide Answer -
उ रः (a)

ा ाः उपयु म से िवक (a) सही है ।

सं सद म बजट पा रत होने की ि या के दौरान काय मं णा सिमित िकसी िवशे ष


मां ग को और बजट सिहत अनु दानों की सब मां गों को ीकृत करने के िलये समय
सीमा िनधा रत करती है , जै से ही समय सीमा समा हो जाती है , उस पर चचा के
समापन की ि या लागू की जाती है और मां ग मतदान के िलये रख दी जाती है ,
चाहे उस पर चचा ई हो या न ई हो। इसी ि या को ‘िगलोिटन’ कहते ह

सं सदीय कायवाही के कुछ अ साधन िन िल खत ह:

आधे धं टे की बहसः अ ऐसी बहस के िलये स ाह म तीन िदन िनधा रत कर


सकता है । इसके िलये सदन म कोई औपचा रक ाव या मतदान नहीं होता। यह
चचा पया लोक मह के मामलों आिद के िलये है ।
अ कािलक चचाः इसे 2 घं टे की चचा भी कहते ह। अ एक स ाह म तीन
िदन उपल करा सकता है । इसे सं सद सद ज़ री सावजिनक मह के मामलों
पर बहस के िलये रख सकते ह।
औिच ः यह सदन का ान आकिषत करने की एक यु है । जब सदन के
सं चालन म सामा िनयमों का पालन नहीं होता तो कोई भी सद इसके मा म से
सदन का ान आकिषत कर सकता है । यह सदन की कायवाही को समा करती
है । इसम िकसी तरह के बहस की अनु मित नहीं है ।
[1] ‘यु वा सं सद योजना’ के सं दभ म िन िल खत कथनों पर िवचार कीिजयेः

1. यह चौथे अ खल भारतीय प (whip) स े लन की अनु शंसा पर ारं भ


की गई।
2. इसे बढ़ावा एवं ो ाहन दे ने के िलये सं सदीय काय मं ालय रा ों को
ज़ री िश ण दे ता है ।

उपयु म से कौन-सा/से कथन सही है /ह?

A) केवल 1

B) केवल 2

C) 1 और 2 दोनों

D) न तो 1 और न ही 2

Hide Answer -
उ रः (c)

ा ाः उपयु दोनों कथन सही ह।

यु वा सं सद योजना चौथे अ खल भारतीय प स े लन की अनु शंसा पर ारं भ की गई।

इस योजना को बढ़ावा एवं ो ाहन सं सदीय काय मं ालय के ारा िदया जा रहा है ।

[2] िन िल खत म से कौन-सा एक सही अनु म है , िजससे होकर सं सद म कोई

साधारण िवधे यक पा रत होता है ।

A) पु रः थापना, सै ा क चचा, ख शः िवचार, माणीकरण, सहमित

B) पु रः थापना, माणीकरण, सै ा क चचा, ख शः िवचार, सहमित

C) माणीकरण, पु रः थापना, सै ा क चचा, ख शः िवचार, सहमित

D) माणीकरण, सै ा क चचा, ख शः िवचार, पु रः थापना, सहमित


Hide Answer -
उ रः (a)

ा ाः उपयु म से िवक (a) सही है ।

सं सद ारा पा रत िकसी साधारण िवधे यक का अनु म इस कार है ;

पु रः थापना ( ु तीकरण/पे श करना): यह िवधे यक का थम चरण है । साधारण िवधे यक

सं सद के िकसी भी सदन म पे श िकया जा सकता है । थम चरण म िवधे यक पर िकसी कार की

चचा नहीं होती है ।

पु रः थापनो र ावः िवधे यक के ु तीकरण के प ात् िवधे यक का भारी सद िवधे यक

के सं बंध म ाव पे श करता है ।

सै ा क चचाः पु रः थापनो र ाव के प ात् िवधे यक के िस ा ों एवं उपबं धों पर चचा

होती है , पर ु िवधे यक पर िव ार से िवचार-िवमश नहीं होता। इस म म िवधे यक म कोई

सं शोधन ु त नहीं िकया जाता। इं ै के हाउस ऑफ कॉम म इसे थम वाचन (First

Reading) कहते ह।

सिमित की अव थाः यिद कोई सद यह ाव पे श करता है िक िवधे यक को वर सिमित या

सं यु सिमित को िनिद िकया जाए और यह ाव पा रत हो जाता है तो सिमित िवधे यक पर

ख शः िवचार करती है , िक ु इसके मू ल िवषय म प रवतन नहीं कर सकती है ।

ख शः िवचारः सिमित से िवधे यक ा करने के प ात् सदन िवधे यक के सम उपबं धों की

समी ा करता है । िवधे यक के े क उपबं ध पर ख वार चचा एवं मतदान होता है । इस अव था

म सद सं शोधन ु त करते ह और अगर सं शोधन ीकार हो जाते ह तो वे िवधे यक का

िह ा बन जाते ह। इं ै के हाउस ऑफ कॉम म इसे ि तीय वाचन (Second Reading)

कहते ह।

ि तीय वाचन के बाद भारी सद यह ाव पे श करता है िक िवधे यक पा रत िकया

जाए या सं शोिधत प म पा रत िकया जाए। इसे इं ै के हाउस ऑफ कॉम म तृ तीय

वाचन (Third Reading) कहते ह। जब िवधे यक एक सदन ारा पा रत कर िदया जाता

है तो िवधे यक पर िवचार एवं ीकृित के िलये दू सरे सदन म भे ज िदया जाता है ।

माणीकरणः जब िवधे यक दोनों सदनों ारा पा रत कर िदया जाता है तो पीठासीन अिधकारी

(अ या सभापित) िवधे यक पर ह ा र करता है । इसे ही माणीकरण कहते ह।

सहमित/अनु मितः अ या सभािपत ारा मािणत िवधे यक को रा पित की सहमित के िलये

भे जा जाता है और रा पित की सहमित के प ात् िवधे यक अिधिनयम का प ले ले ता है ।

[3] सं सद म पु रः थािपत साधारण िवधे यक के सं दभ म िन िल खत म से कौन-सा/से

कथन सही है /ह?

1. सरकारी िवधे यक वह िवधे यक होता है , िजसे कोई मं ी या सं सद सद


ु त करता है ।
2. िकसी िवधे यक पर ख शः िवचार तृ तीय वाचन म होता है ।

नीचे िदये गए कूट का योग कर सही उ र का चयन कीिजये ।


A) केवल 1

B) केवल 2

C) 1 और 2 दोनों

D) न तो 1 और न ही 2

Hide Answer -
उ रः (d)

ा ाः उपयु दोनों कथन गलत ह।

साधारण िवधे यक सं सद के िकसी भी सदन म ु त िकया जा सकता है । िजस िवधे यक को

िकसी मं ी ारा ु त िकया जाता है उसे सरकारी िवधे यक कहते ह और मं ी से िभ

िकसी अ सद ारा ु त िकया गया िवधे यक, गै र-सरकारी िवधे यक कहलाता है ।

मं ी से िभ कोई सद (गै र-सरकारी सद ) िवधे यक ु त करना चाहता है तो सदन

को इसकी अि म सू चना दे नी पड़ती है । सू चना की िविहत अविध एक मास है ।

यिद िवधे यक ु तीकरण से पू व राजप म कािशत हो गया है तो ु तीकरण के िलये

अनु मित के ाव की आव कता नहीं पड़ती और जो िवधे यक ु तीकरण के पू व

राजप म कािशत नहीं आ है उसे ु तीकरण के प ात् कािशत िकया जाता है ।

िवधे यक पर ख शः िवचार अथवा आम बहस ि तीय वाचन म होती है । िवधे यक पर

ख वार चचा एवं मतदान होता है । सद सं शोधन ु त कर सकते ह और सं शोधन

ीकार हो जाते ह तो िवधे यक का िह ा बन जाते ह।

[4] सं सद के साधारण िवधे यक के सं दभ म िन िल खत कथनों पर िवचार कीिजये :

1. एक सदन ारा पा रत िवधे यक को दू सरा सदन अिधकतम एक वष तक


रोककर रख सकता है ।
2. सं सद के दोनों सदनों म गितरोध होने की थित म भारत का रा पित
सं िवधान के अनु े द-110 के तहत सं यु बै ठक बु लाता है ।
3. सं सद के दोनों सदनों की सं यु बै ठक की अ ता भारत के रा पित
ारा की जाती है ।

उपयु म से कौन-सा/ से कथन सही है /ह?

A) केवल 1

B) केवल 1 और 2

C) 1, 2 और 3

D) उपयु म से कोई नहीं।


Hide Answer -
उ रः (d)

ा ाः उपयु सभी कथन गलत ह।

एक सदन ारा पा रत कोई साधारण िवधे यक जब दू सरे सदन म जाता है , तो दू सरा सदन

िन िल खत म से कोई भी कदम उठा सकता है ः-

⇒ िवधे यक को िबना सं शोधन के पा रत कर दे ।

⇒ िवधे यक को सं शोधन के साथ पा रत करे ।

⇒ िवधे यक को अ ीकार कर दे ।

⇒ िवधे यक को पड़ा रहने दे और कोई कायवाही न करे ।

ऐसी थित म एक सदन ारा पा रत िवधे यक को दू सरा सदन अिधकतम छः माह तक (न

िक एक वष तक) रोककर रख सकता है ।

छः माह बीत जाने के प ात् रा पित सं िवधान के अनु े द-108 (न िक अनु े द-110)

के तहत सं सद के दोनों सदनों का सं यु अिधवे शन बु लाता है ।

सं सद के सं यु अिधवे शन की अ ता लोकसभा अ (न िक रा पित ारा या

रा सभा के सभापित ारा) ारा की जाती है ।

[5] सं सद के दोनों सदनों ारा पा रत कोई साधारण िवधे यक भारत के रा पित के


पास अनु मित के िलये भे जा जाता है तो सं िवधान के अनु े द-111 के तहत
रा पित िवधे यक को पु निवचार के िलये वापस लौटा सकता है , िक ु दोनों सदनों
ारा िवधे यक म सं शोधन या िबना सं शोधन के रा पित के पास वापस अनु मित के
िलये भे जा जाता है तो रा पित िन िल खत म से कौन-सा कदम उठा सकता है ?

A) िवधे यक को पु निवचार के िलये िफर से वापस लौटा सकता है ।

B) िवधे यक पर अनु मित दे ने के िलये रा पित बा है ।

िवधे यक पर अपनी अनु मित रोक सकता है और ऐसी थित म िवधे यक


C)
समा हो जाता है ।

D) उपयु म से कोई नहीं।


Hide Answer -
उ रः (b)

ा ाः उपयु म से िवक (b) सही है ।

कोई साधारण िवधे यक दोनों सदनों ारा पा रत होने के प ात् रा पित के सम ु त िकया

जाता है तो रा पित िन िल खत कदम उठा सकता है ः-

िवधे यक को अनु मित दे दे । ऐसी थित म िवधे यक अिधिनयम बन जाता है ।

िवधे यक को अपनी अनु मित न दे । ऐसी थित म िवधे यक समा हो जाता है और वह

अिधिनयम नहीं बन पाता।

िवधे यक को पु निवचार के िलये लौटा सकता है , ले िकन दोनों सदनों ारा सं शोधन या िबना

सं शोधन के िवधे यक को रा पित के पास वापस भे जा जाता है तो रा पित अनु मित दे ने के

िलये बा है ।

[6] िन िल खत म से कौन-सा कथन धन-िवधे यक के सं दभ म सही नहीं है ?

लोकसभा अ यह िनणय करने के िलये अ म ािधकारी है िक कोई


A)
िवधे यक धन िवधे यक है या नहीं।

लोकसभा ारा पा रत िकसी धन िवधे यक को रा सभा ारा 14 िदनों के


B)
अ र लौटाया जाना और िवचाराथ भे जा जाना आव क है ।

रा पित िकसी धन-िवधे यक को लोकसभा म पु निवचार के िलये नहीं लौटा


C)
सकता।

धन िवधे यक सं सद के दोनों सदनों म से िकसी भी सदन म पु रः थािपत िकया


D)
जा सकता है ।
Hide Answer -
उ रः (d)

ा ाः उपयु म से िवक (d) सही नहीं है ।

सं िवधान के अनु े द-109 म धन-िवधे यक के सं बंध म िवशे ष ि या का उपबं ध िकया

गया है ।

अनु े द-109(1) म उपबं ध िकया गया है िक धन िवधे यक रा सभा म पु रः थािपत

(Introduced) नहीं िकया जाएगा। अथात धन िवधे यक को केवल लोकसभा म ही ुत

कर सकते ह।

सं िवधान के अनु े द-110(3) म उपबं ध िकया गया है िक यिद उठता है िक कोई

िवधे यक धन िवधे यक है या नहीं, तो उस पर लोकसभा अ का िनणय अ म होगा।

अनु े द-109(2) म उपबं ध है िक लोकसभा ारा पा रत िकसी धन िवधे यक को

रा सभा ारा 14 िदनों के अ र लौटाया जाना आव क है । अनु े द-109(5) म

उपबं ध है िक उ 14 िदन की अविध के भीतर लोकसभा को नहीं लौटाया जाता है तो

उ अविध की समा पर वह दोनों सदनों ारा उस प म पा रत िकया गया समझा

जाएगा, िजस प म लोकसभा ारा पा रत िकया गया था।

धन िवधे यक को जब रा पित के सम ु त िकया जाता है , तो रा पित या तो ीकृित दे

दे ता है या रोककर रख सकता है , िक ु पु निवचार के िलये लोकसभा को नहीं लौटा सकता

है । धन िवधे यक को लोकसभा म ु त करने से पहले रा पित की पू व अनु मित ली जाती

है ।

[7] सं िवधान के अनु े द-110 म धन िवधे यक की प रभाषा दी गई है । इस सं दभ म


िन िल खत म से कौन-सा िवषय इस प रभाषा के अ गत शािमल नहीं है ?

A) िकसी कर का अिधरोपण, उ ादन, प रहार, प रवतन या िविनयमन।

B) भारत सरकार ारा धन उधार ले ने का या कोई ाभू ित दे ने का िविनयमन।

C) भारत की सं िचत िनिध या आक क िनिध की अिभर ा।

D) जु माने या अथद से सं बंिधत उपबं ध।


Hide Answer -
उ रः (d)

ा ाः उपयु म से िवक (d) सही नहीं है ।

सं िवधान के अनु े द-110 के तहत धन िवधे यक के अ गत िन िल खत िवषयों को

शािमल िकया गया है -

िकसी कर का अिधरोपण, उ ादन, प रहार, प रवतन या िविनयमन।

भारत सरकार ारा धन उधार ले ने या कोई ितभू ित दे ने का िविनयमन।

भारत की सं िचत िनिध या आक क िनिध की अिभर ा तथा ऐसी िकसी िनिध म धन जमा

करना या उसम से धन िनकालना।

भारत की सं िचत िनिध से धन का िविनयोग।

िकसी य को भारत की सं िचत िनिध पर भा रत य घोिषत करना या ऐसे य की रकम

को बढ़ाना।

भारत की सं िचत िनिध या लोक ले खों म िकसी कार की धन की ा या अिभर ा या

इनम य या सं घ या रा के ले खाओं का परी ण आिद।

जु माने या अथद सं बंधी िवषय इसकी प रभाषा म शािमल नहीं ह।

[8] िन िल खत कथनों पर िवचार कीिजएः

1. जब कोई धन िवधे यक या साधारण िवधे यक रा सभा या रा पित को


पारे िषत िकया जाता है तो उस पर लोकसभा अ या रा सभा के
सभापित का ह ा र सिहत मािणत होता है ।
2. िकसी मं ी ारा ु त साधारण िवधे यक या धन िवधे यक को लोकसभा
ारा अ ीकृत कर दे ने पर सरकार को ागप दे ना पड़ता है ।

उपयु म से कौन-सा/से कथन सही है /ह?

A) केवल 1

B) केवल 2

C) 1 और 2 दोनों

D) न तो 1 और न ही 2
Hide Answer -
उ रः (b)

ा ाः उपयु म से कथन (a) सही नहीं है ।

सं िवधान के अनु े द-109 के तहत रा सभा को या अनु े द-111 के तहत रा पित को

जब िकसी धन िवधे यक को पारे िषत िकया जाता है तो उस पर लोकसभा अ के

ह ा र सिहत मािणत रहता है , जबिक साधारण िवधे यक म अ /सभापित के माण

की आव कता नहीं पड़ती। जब साधारण िवधे यक को िकसी मं ी ारा ु त िकया जाता

है और लोकसभा म अ ीकृत हो जाता है तो सरकार को ाग-प दे ना पड़ सकता है ।

इसी तरह धन-िवधे यक को लोकसभा ारा अ ीकृत कर िदया जाता है तो सरकार को

ाग-प दे ना पड़ता है । धन िवधे यक को केवल मं ी ारा ही ु त िकया जा सकता है ।

इस कार सभी धन िवधे यक सरकारी िवधे यक होते ह।

[9] िन िल खत कथनों पर िवचार कीिजएः

1. कोई िवधे यक यिद लोकसभा के अ ारा धन-िवधे यक मािणत िकया


जाता है , तो उसके कार पर िकसी ायालय म नहीं िकया जा
सकता।
2. भारत के रा पित या सं सद म यह श िनिहत है िक वे उस िवधे यक के
कार को, िजसे धन िवधे यक माना गया है , गत कर सकता है , भले ही
लोकसभा के अ ारा धन-िवधे यक के पम मािणत िकया गया हो।

उपयु म से कौन-सा/से कथन सही नहीं है /ह?

A) केवल 1

B) केवल 2

C) 1 और 2 दोनों

D) न तो 1 और न ही 2

Hide Answer -
उ रः (b)

ा ाः उपयु म से कथन 2 सही नहीं है ।

सं िवधान के अनु े द-110(3) म उपबं ध है िक धन िवधे यक के सं बंध म लोकसभा अ

का िनणय अ म होगा और उसके िनणय को िकसी ायालय, सं सद या रा पित ारा

चु नौती नहीं दी जा सकती है ।

धन िवधे यक सं िवधान के अनु े द-109 के तहत रा सभा को और अनु े द-111 के

तहत रा पित के सम पारे िषत या ु त िकया जाता है । े क धन िवधे यक पर

लोकसभा अ के ह ा र सिहत यह माण पृ ां िकत रहता है िक यह धन िवधे यक है ।

[10]िन िल खत कथनों पर िवचार कीिजये ।


1. सभी धन िवधे यक िव िवधे यक होते ह।
2. सं िवधान के अनु े द-109 म धन िवधे यक (Money Bill) को प रभािषत
िकया गया है ।
3. िव िवधे यक (Financial Bill) के सं बंध म िवशे ष उपबं ध सं िवधान के
अनु े द-117 म िकया गया है ।

उपयु म से कौन-सा/से कथन सही है /ह?

A) केवल 1

B) केवल 1 और 3

C) केवल 2 और 3

D) 1, 2 और 3

Hide Answer -
उ रः (b)

ा ाः उपयु म से कथन-2 सही नहीं है ।

सं िवधान के अनु े द-110 म धन िवधे यक की प रभाषा दी गई है , जबिक अनु े द-109

म धन िवधे यक के सं बंध म िवशे ष ि या का उपबं ध है ।

सभी धन िवधे यक, िव िवधे यक होते ह, िक ु सभी िव िवधे यक धन िवधे यक नहीं होते ।

सं िवधान के अनु े द-117 म िव िवधे यक के सं बंध म िवशे ष उपबं ध है ।

िव िवधे यक तीन कार के होते ह-

धन िवधे यक- अनु े द-110

िव िवधे यक-I अनु े द-117(1)

िव िवधे यक II- अनु े द-117(3)

[11]िव िवधे यक (Financial Bill) के सं दभ म िन िल खत म से कौन-सा कथन


सही है ?

A) सं सद के िकसी भी सदन म पु रः थािपत िकया जा सकता है ।

सं सद के िकसी भी सदन म रा पित की िसफा रश से ही पु रः थािपत िकया जा


B)
सकता है ।

C) रा सभा िव िवधे यक म सारभू त सं शोधन कर सकती है ।

D) रा पित सं यु बै ठक नहीं बु ला सकता है ।


Hide Answer -
उ रः (c)

ा ाः उपयु म से िवक (c) सही है ।

रा सभा िव िवधे यक को ीकार या अ ीकार कर सकती है या इसम सारभू त सं शोधन

कर सकती है , िक ु लोकसभा, रा सभा की अनु शंसाओं को ीकार करने या अ ीकार

करने के िलये तं है , और िवधे यक को आगे भे ज सकती है । य िप िकसी भी सदन ारा

िवधे यक म ािवत कर (Tax) आिद को तब तक कम या समा नहीं िकया जा सकता,

जब तक की रा पित इसकी सहमित न दे द।

िव िवधे यक दो मामलों म धन िवधे यक के समान है ।

इसे केवल लोकसभा म ही पु रः थािपत िकया जा सकता है ।

लोकसभा म पु रः थािपत करने से पहले रा पित की िसफा रश आव क है । अथात्

रा पित की िसफा रश से ही इसे पु रः थािपत या ािवत िकया जा सकता है ।

दोनों सदनों के बीच गितरोध होने पर रा पित अनु े द-108 के तहत सं यु बै ठक बु ला

सकता है , िजसकी अ ता लोकसभा के अ ारा की जाती है ।

उपयु दो मामलों के अित र िव िवधे यक के साथ वै सा ही वहार िकया जाता है ,

जै सा िक िकसी साधारण िवधये क के सं बंध म िकया जाता है ।

सं िवधान के अनु े द-117(3) म उपबं ध है िक िजस िवधे यक को अिधिनयिमत और

वितत िकये जाने पर भारत की सं िचत िनिध म से य करना पड़े गा, वह िवधे यक सं सद

के िकसी भी सदन ारा तब तक पा रत नहीं िकया जाएगा, जब तक ऐसे िवधये क पर

िवचार करने के िलये उस सदन से रा पित ने िसफा रश न की हो।

[12]िव िवधे यक (Financial Bill) को सं सद के दोनों सदनों ारा पा रत कर


रा पित के सम ु त िकया जाता है तो इस सं दभ म िन िल खत म से कौन-सा
कथन सही नहीं है ?

A) िवधे यक पर अपनी ीकृित दे सकता है ।

B) िवधे यक पर अपनी ीकृित रोक सकता है ।

C) िवधे यक को पु निवचार के िलये सदन को वापस नहीं भे ज सकता।

D) िवधे यक को पु निवचार के िलये सदन को वापस भे ज सकता है ।


Hide Answer -
उ रः (c)

ा ाः उपयु म से िवक (c) सही नहीं है ।

िव िवधे यक केवल दो मामलों म धन िवधे यक के समान है ः- (i) िव िवधे यक केवल

लोकसभा म ही पु रः थािपत िकया जा सकता है । और (ii) रा पित की िसफा रश से ही

िवधे यक को पु रः थािपत या ािवत िकया जा सकता है ।

उपयु दो मामलों को छोड़कर िव िवधे यक के साथ साधारण िवधये क जै सा वहार

िकया जाता है । अतः सं सद के दोनों सदनों से पा रत होने के प ात् अनु े द-111 के तहत

िवधे यक पर अपनी ीकृित दे सकता है या अपनी ीकृित रोक सकता है या पु निवचार के

िलये िवधे यक को सदन के पास वापस भे ज सकता है ।

[13]सं िवधान सं शोधन के सं दभ म िन िल खत कथनों पर िवचार कीिजयेः

1. सं िवधान के भाग-22 के अनु े द-368 म सं िवधान सं शोधन के सं बंध म


उपबं ध िकया गया है ।
2. सं िवधान सं शोधन िवधे यक रा पित की पू व ीकृित से ही पु रः थािपत िकया
जा सकता है ।
3. ससं द और रा के िवधानमं डल को सं िवधान म सं शोधन करने की श
है ।
4. सं िवधान सं शोधन िवधे यक सरकारी या गै र-सरकारी सद ारा
पु रः थािपत िकया जा सकता है ।

उपयु म से कौन-सा/से कथन सही है /ह?

A) केवल 1 और 2

B) केवल 1, 2 और 4

C) केवल 4

D) 1, 2, 3 और 4
Hide Answer -
उ रः (c)

ा ाः उपयु म से कथन1, 2 और 3 गलत ह।

सं िवधान के भाग 20 के अनु े द-368 म सं िवधान सं शोधन के सं बंध म उपबं ध है ।

सं िवधान सं शोधन िवधे यक पु रः थािपत करने के िलये रा पित की पू व ीकृित आव क

नहीं है ।

सं िवधान सं शोधन की ि या केवल सं सद म ही ारं भ हो सकती है , रा के िवधानमं डल

को यह श ा नहीं है ।

सं िवधान सं शोधन िवधे यक मं ी (सरकारी सद ) या गै र-मं ी (गै र-सरकारी सद ) पु रः

थािपत कर सकता है ।

भारत म सं िवधान सं शोधन की ि या न तो ि टे न के समान लचीली और न अमे रका के

समान कठोर है । यह दोनों का िम ण है ।

[14]सं िवधान सं शोधन िवधे यक के सं दभ िन िल खत म से कौन-से कथन सही ह?

1. सं सद के िकसी भी सदन म पु रः थािपत िकया जा सकता है ।


2. दोनों सदनों के बीच गितरोध होने पर सं यु बै ठक का उपबं ध नहीं है ।
3. िवधे यक को सं सद के े क सदन म अलग-अलग पा रत कराना अिनवाय
है ।
4. िवधे यक को रा पित न तो अपने पास रख सकता है और न पु निवचार के
िलये लौटा सकता है ।

नीचे िदए गए कूट का योग कर सही उ र का चयन कीिजये ।

A) केवल 1 और 2

B) केवल 1, 2 और 3

C) केवल 1, 2 और 4

D) 1, 2, 3 और 4
Hide Answer -
उ रः (d)

ा ाः उपयु सभी कथन सही ह।

सं िवधान सं शोधन िवधये क को सं सद के सरकारी या गै र-सरकारी सद ारा िकसी भी

सदन (लोकसभा या रा सभा) म पु रः थािपत िकया जा सकता है ।

दोनों सदनों के बीच गितरोध होने पर सं िवधान के अनु े द-108 के तहत सं यु बै ठक

बु लाने का उपबं ध नहीं है ।

िवधे यक को सं सद के े क सदन म अलग-अलग पा रत कराना अिनवाय है , अ था

िवधे यक समा हो जाएगा।

दोनों सदनों ारा पा रत िवधे यक जब रा पित के पास अनु मित के िलये जाता है तो रा पित

अनु े द-111 के तहत िवधे यक को अपने पास नहीं रख सकता और न िवधे यक को

पु निवचार के िलये वापस लौटा सकता है । अथात् िवधे यक पर रा पित को अनु मित दे ना

अिनवाय हो जाता है ।

24व सं िवधान सं शोधन 1971 से पू व रा पित िवधे यक पर अनु मित दे ने के बदले यह कह

सकते थे िक ‘अनु मित िव ा रत’ करता है । ऐसा होने पर वह अिधिनयम नहीं बन पाता था,

िक ु इस सं शोधन के ारा िवधे यक पर अनु मित दे ने के िलये अिनवाय कर िदया गया।

[15]यिद भारत के सं िवधान के िकसी सं शोधन ारा कितपय िवषयों म प रवतन लाना
हो, तो ऐसे सं शोधन के िवधायी उपबं ध को रा पित की सहमित के िलये भे जे जाने
से पू व सं शोधन का उतने रा ों के िवधानमं डलों ारा जो आधे रा ों से कम नहीं
हों, अनु समथन िकया जाना आव क है । िन िल खत म स वे कौन-से िवषय ह?

1. रा ों का सं सद म ितिनिध
2. सातवीं अनु सूची की कोई सू ची
3. भारत के रा पित के िनवाचन की रीित
4. सं घ एवं रा ों की कायपािलका श का िव ार

नीचे िदये गए कूट का योग कर सही उ र चु िनयेः

A) केवल 2 और 3

B) केवल 1, 2 और 3

C) केवल 2, 3 और 4

D) 1, 2, 3 और 4
Hide Answer -
उ रः (d)

ा ाः उपयु सभी कथन सही ह।

सं िवधान म सं शोधन तीन कार से िकया जाता है ः

1. सं सद म साधारण ब मत ारा

2. सं सद म िवशे ष ब मत ारा

3. सं सद म िवशे ष ब मत और कम से कम आधे रा ों के िवधानमं डलों के अनु समथन से ।

सं सद म िवशे ष ब मत और कम से कम आधे रा ों के िवधानमं डलों के अनु समथन ारा

िन िवषयों म सं शोधन िकया जा सकता है ः

1. रा पित के िनवाचन की िविध (अनु े द-54, 55)

2. रा ों का सं सद म ितिनिध (अनु े द-80, 81 चौथी अनु सूची)

3. सातवीं अनु सूची की कोई सू ची।

4. सं घ एवं रा ों की कायपािलका श का िव ार (अनु े द- 73, 162)।

5. सं घ एवं रा ों के बीच िवधायी श का िवतरण (भाग-11 का अ ाय-1)।

6. उ तम ायालय और उ ायालय (अनु े द-241, भाग-5 का अ ाय-4, भाग-6

का अ ाय-5)

7. सं िवधान का सं शोधन करने की सं सद की श (अनु े द-368)

[16]िन िल खत कथनों पर िवचार कीिजयेः

1. सं िवधान के अनु े द-368 के उपबं धों म सं शोधन करने के िलये िवधे यक


सं सद के े क सदन म उस सदन की कुल सद सं ा के ब मत ारा
और उस सदन म उप थत और मत दे ने वाले सद ों के कम से कम दो-
ितहाई ब मत ारा पा रत िकया जाना अिनवाय है ।
2. उपयु सं शोधन को रा पित के सम अनु मित के िलये ु त िकये जाने
से पहले कम-से -कम तीन-चौथाई रा ों के िवधानमं डलों का अनु समथन
अिनवाय है ।

उपयु म से कौन-सा/से कथन सही है /ह?

A) केवल 1

B) केवल 2

C) 1 और 2 दोनों

D) न तो 1 और न ही 2
Hide Answer -
उ रः (a)

ा ाः उपयु म से कथन-2 सही नहीं है ।

सं िवधान के अनु े द-368(2) म उपबं ध है िक सं िवधान के अनु े द-368 के िकसी

उपबं ध म सं शोधन करने के िलये िवधे यक को सं सद के े क सदन के ारा अलग-

अलग, उस सदन की कुल सद सं ा के ब मत (50% से अिधक) ारा तथा उस

सदन म उप थत मत दे ने वाले सद ों के कम से कम दो -ितहाई (2/3) ब मत ारा

पा रत होना अिनवाय है । इसे ही िवशे ष ब मत से पा रत होना कहते ह।

इस िवधे यक को रा पित से सम ु त करने से पहले कम से कम आधे रा ों के

िवधानमं डलों का अनु समथन अिनवाय है , जबिक अमे रका म तीन चौथाई (3/4) रा ों के

अनु समथन की आव कता होती है ।

अनु े द-368(3) म उपबं ध है िक अनु े द-13 की कोई बात इस अनु े द के अधीन

िकये गए िकसी सं शोधन पर लागू नहीं होगी। इसे 24व सं िवधान सं शोधन, 1971 ारा

जोड़ा गया।

[17]िन िल खत म से वे कौन-सा/से िवषय है /ह, िजससे सं बंिधत सं वैधािनक ावधानों


म भारत की सं सद साधारण ब मत से प रवतन कर सकती है ?

1. रा ों के नाम, उनकी सीमाएं और उनके े बदलना।


2. रा िवधान प रषद का िनमाण या उसकी समा ।
3. अनु सूिचत े ों एवं अनु सूिचत जनजाितयों का शासन।

नीचे िदये गए कूट का योग कर सही उ र का चयन कीिजयेः

A) केवल 1

B) केवल 1 और 2

C) केवल 1 और 3

D) 1, 2 और 3
Hide Answer -
उ रः (d)

ा ाः उपयु म से सभी कथन सही ह।

सं िवधान के कुछ उपबं धों म प रवतन सं िवधान का सं शोधन नहीं समझा जाता है । सं सद

सं िवधान के ऐसे उपबं धों म सादे ब मत/साधारण ब मत (साधारण िवधायी ि या) से

प रवतन कर सकती है । साधारण ब मत े क सदन म उप थत और मतदान करने वाले

का ब मत होता है । इसके ारा सं सद सं िवधान के िन िल खत उपबं धों को बदल सकती

है -

1. नए रा ों का वे श या गठन।

2. नए रा ों का िनमाण, उसके े ों, सीमाओं या सं बंिधत रा ों के नामों म प रवतन।

3. िकसी रा म िवधान प रषद का िनमाण या उसकी समा ।

4. अनु सूिचत े ों एवं अनु सूिचत जनजाितयों का शासन (पां चवीं अनु सूची)।

5. जनजातीय े ों का शासन (छठी अनु सूची)।

6. नाग रकता की ा एवं समा ।

7. िनवाचन े ों का पु निनधारण।

8. सं सद एवं रा िवधानमं डल के िलये िनवाचन।

9. रा पित, रा पाल, लोकसभा अ , ायाधीश आिद के िलये प रला यां , भ े ,

िवशे षािधकार आिद (दू सरी अनु सूची)।

10. सं सद, उसके सद ों और उसकी सिमितयों के िवशे षािधकार।

11. उ तम ायालयों म अवर ायाधीशों की सं ा।

12. सं सद सद ों के वे तन एवं भ े ।

[18]िन िल खत कथनों पर िवचार कीिजयेः

1. सं सद सं िवधान के अनु े द-368 म उपबं िधत ि या का योग कर


सं िवधान के िकसी भाग म सं शोधन कर सकती है ।
2. सं सद सं िवधान सं शोधन के ारा सं िवधान के आधा रक ल ण म प रवतन
नहीं कर सकती है ।
3. सं िवधान सं शोधन सं िवधान के अनु े द-13 के तहत िविध नहीं है ।

उपयु म से कौन-सा/से कथन सही है /ह?

A) केवल 2

B) केवल 1 और 2

C) केवल 2 और 3

D) 1, 2 और 3
Hide Answer -
उ रः (d)

ा ाः उपयु तीनों कथन सही ह।

केशवान भारती बनाम केरल रा , 1973 और उसके बाद ायालय ारा िलये गए

िविभ िनणयों से यह िन ष िनकला िक-

सं सद सं िवधान के अनु े द-368 म िविहत ि या का अनु सरण कर सं िवधान के िकसी

भाग को सं शोिधत कर सकती है , िक ु सं िवधान के आधारभू त ढाँ चे/आधा रक ल ण/मू ल

ढाँ चे म प रवतन/सं शोधन नहीं कर सकती।

24व सं िवधान सं शोधन 1971 के ारा सं िवधान म अनु े द -368(3) और अनु ेद

13(4) जोड़े गए, िजसम उपबं ध िकया गया िक सं िवधान सं शोधन सं िवधान के

अनु े द-13 के तहत िविध नहीं है । अथात अनु े द-13 की कोई बात अनु े द-368 के

अधीन िकये गए सं िवधान के िकसी सं शोधन पर लागू नहीं होगी।

42व सं िवधान सं शोधन, 1976 के ारा अनु े द-368 (4) एवं (5) जोड़े गए। अनु ेद

368(4) म उपबं ध िकया गया है िक सं िवधान सं शोधन अिधिनयम की िविधमा ता को

िकसी ायालय म िकसी आधार पर गत नहीं िकया जा सकेगा और अनु े द-368(5)

म उपबं ध िकया गया िक अनु े द-368 के अधीन सं िवधान के उपबं धों का प रवधन,

प रवतन या िनरसन के प म सं शोधन करने के िलये सं सद की सं िवधायी श पर िकसी

कार का िनबधन नहीं होगा। हालाँ िक िमनवा िम वाद, 1980 म उपयु दोनों का

ख न िकया गया और ाियक पु निवलोकन को सं िवधान का आधा रक ल ण माना गया।

[19]भारत के सं िवधान के अनु सार िन म से िकस/िकन िवधे यक/िवधे यकों को लागू


करने के िलये रा पित की पू व ीकृित या िसफा रश ज़ री है ?

1. नए रा ों का गठन या रा की सीमाओं के पु नगठन के िलये ।


2. धन िवधे यक।
3. ापार की तं ता पर िनबधन अिधरोिपत करने वाले रा िवधे यक।
4. कोई िवधे यक जो सही अथ म धन िवधे यक नहीं है , िक ु भारत की सं िचत
िनिध म से य करना पड़े गा।

नीचे िदये गए कूट का योग कर सही उ र का चयन कीिजयेः

A) केवल 1

B) केवल 1 और 2

C) केवल 1, 2 और 4

D) 1, 2, 3 और 4
Hide Answer -
उ रः (d)

ा ाः उपयु सभी कथन सही ह।

भारत के सं िवधान म उपबं ध है िक कुछ िवषयों पर िवधान पु रः थािपत करने के िलये

रा पित की पू व मं ज़ूरी या िसफा रश आव क है । इसम शािमल िवषय ह-

नए रा ों का िनमाण या वतमान रा ों की सीमाओं म प रवतन। (अनु े द 3)

धन िवधे यक। [अनु े द-117 (1)]

ापार की तं ता पर रोक लगाने वाले रा िवधे यक। (अनु े द-304)

कोई िवधे यक जो सही अथ म धन िवधे यक नहीं है ,िक ु िजसम भारत की सं िचत िनिध म

से य करना पड़े गा। [अनु े द-117 (3)]

ऐसा कर िवधे यक िजसम रा ों का िहत िनिहत हो। [अनु े द-274 (1)]

[20]िकसी एक िदन रा सभा की बै ठक म 120 सद उप थत थे , िज लोकसभा


ारा पहले ही पा रत एक साधारण िवधये क पर मतदान करना था। 75 सद ों ने
िवधे यक के प म मतदान िकया। इस सदं भ म िन प रणामों म से कौन-सा सही
है ?

िवधये क पा रत नहीं समझा जाएगा, ोंिक रा सभा की कुल सद सं ा


A) के आधे से अिधक सद अनु प थत थे और वे मतदान म भागीदार भी नहीं
थे ।

िवधे यक पा रत नहीं माना जाएगा, ोंिक मतदान करने वालों म से 2/3


B)
सद ों ने इसके प म मतदान नहीं िकया।

िवधे यक पा रत नहीं माना जाएगा और पु निवचार के िलये इसे लोकसभा को


C)
लौटाना होगा।

D) िवधे यक पा रत घोिषत कर रा पित की अनु मित के िलये भे ज िदया जाएगा।

Hide Answer -
उ रः (d)

ा ाः उपयु म से िवक (d) सही है ।

साधारण िवधे यक दोनों सदनों म अलग-अलग उप थत और मत दे ने वाले सद ों के

ब मत से पा रत िकया जाता है । इसके बाद ीकृित के िलये रा पित के पास भे ज िदया

जाता है । चूँ िक रा सभा म उप थत 120 सद ों म से 75 सद ों ने िवधे यक के प म

मतदान िकया है , जबिक ब मत के िलये मा 61 सद ों की सहमित आव क थी। अतः

िवधये क पा रत घोिषत िकया जाएगा और इसे ीकृित के िलये रा पित के पास भे ज िदया

जाएगा।
[1] लोकसभा और रा सभा के बीच गितरोध की िकस थित/िकन थितयों म सं सद
की सं यु बै ठक बु लाई जाती है ।

1. सं िवधान सं शोधन िवधे यक को पा रत करने की थित म।


2. धन िवधये क (Money bill) को पा रत करने की थित म।
3. िव िवधे यक (Financial bill) को पा रत करने की थित म।
4. साधारण िवधे यक को पा रत करने की थित म।

नीचे िदये गए कूट का योग कर सही उ र का चयन कीिजये ।

A) केवल 4

B) केवल 3 और 4

C) केवल 1, 3 और 4

D) 1, 2 3 और 4

Hide Answer -
उ रः (b)

ा ाः उपयु म से कथन-1 और 2 सही नहीं ह।

रा पित सं िवधान के अनु े द-108 के तहत सं सद के दोनों सदनों के बीच गितरोध की

थित म सं यु बै ठक बु लाता है । रा पित िन थितयों म सं यु बै ठक बु ला सकता है :

1. साधारण िवधे यक

2. िव िवधे यक (Financial Bill)

धन िवधे यक एवं सं िवधान सं शोधन की थित म सं यु बै ठक का ावधान नहीं है ।

[2] िन िल खत कथनों पर िवचार कीिजयेः

1. सं सद के िकसी एक सदन ारा पा रत िकसी साधारण िवधे यक को दू सरा


सदन एक वष से अिधक के िलये रोक ले ता है तो ऐसी थित म रा पित
सं सद की सं यु बै ठक बु लाता है ।
2. सं सद की सं यु बै ठक म कोई साधारण िवधे यक दोनों सदनों के उप थत
और मत दे ने वाले सद ों के दो-ितहाई ब मत से पा रत होना चािहये ।
3. सं सद की सं यु बै ठक के िलये कोरम दोनों सदनों की कुल सद सं ा
का 1/6 भाग होना चािहये ।
4. लोकसभा एवं रा सभा की थम सं यु बै ठक दहे ज़ ितषे ध िवधे यक को
पा रत करने के िलये ई थी।

उपयु म से कौन-सा/से कथन सही है /ह?


A) केवल 4

B) केवल 1 और 3

C) केवल 1, 2 और 3

D) 1, 2, 3 और 4

Hide Answer -
उ रः (a)

ा ाः उपयु म से कथन 1, 2 और 3 गलत ह।

सं सद का सं यु अिधवे शन िन िल खत तीन प र थितयों म बु लाया जा सकता है -

1. जब एक सदन ारा पा रत िवधे यक दू सरे सदन ारा अ ीकृत कर िदया जाता है ।

2. यिद एक सदन दू सरे सदन ारा िवधे यक म िकये गए सं शोधनों को मानने से इनकार कर

दे ।

3. एक सदन ारा पा रत िवधे यक को दू सरे सदन ारा िबना पा रत िकये छः माह (न िक

एक वष) से ादा समय हो जाए।

सं यु बै ठक म िवधे यक को दोनों सदनों म उप थत और मत दे ने वाले सद ों की कुल

सं ा के ब मत ारा (न िक 2/3 ब मत ारा पा रत) पा रत कर िदया जाता है , तो माना

जाता है िक िवधये क को दोनों सदनों ने पा रत कर िदया है ।

सं सद की सं यु बै ठक का कोरम/गणपू ित दोनों सदनों की कुल सद सं ा का 1/10

भाग (न िक 1/6 भाग) होता है ।

सं सद के दोनों सदनों की सं यु बै ठक अभी तक तीन बार बु लाई जा चु की है ः

1. दहे ज़ ितषे ध िवधे यक

2. बक से वा आयोग िनरसन िवधे यक

3. आतं कवाद िनवारण िवधे यक (POTA)

सं यु बै ठक की कायवाही लोकसभा के ि या िनयमों के अनु सार सं चािलत होती है न

िक रा सभा के िनयमों के अनु सार।

सं िवधान के अनु े द-108 (5) म उपबं ध है िक यिद रा पित ने लोकसभा के िवघटन से

पू व सं यु बै ठक बु लाने की नोिटस जारी कर दी है तो लोकसभा िवघटन के प ात् भी इस

अनु े द के अधीन सं यु बै ठक होगी, और िवधे यक पा रत हो सकेगा।

[3] सं सद के दोनों सदनों म गितरोध की थित म यिद लोकसभा का अ और


उपा दोनों अनु प थत हों तो सं यु बै ठक की अ ता िकसके ारा की
जाएगी?
A) रा सभा के सभापित के ारा।

B) रा सभा के उपसभापित के ारा।

C) भारत के रा पित के ारा।

D) सं यु बै ठक म उप थित व र सद के ारा।

Hide Answer -
उ रः (b)

ा ाः उपयु म से िवक (b) सही है ।

सं िवधान के अनु े द-108 के तहत, यिद सं सद के दोनों सदनों के बीच गितरोध उ

हो जाता है तो रा पित दोनों सदनों की सं यु बै ठक बु लाता है और उसकी अ ता

लोकसभा अ करता है । यिद लोकसभा अ अनु प थत है तो लोकसभा के उपा

के ारा और उपा की अनु प थित म रा सभा के उपसभापित के ारा और

उपसभापित की अनु प थित म सं यु बै ठक म उप थत सद ों ारा इस बात का िनणय

िकया जाता है िक इस सं यु बै ठक की अ ता कौन करे गा।

रा सभा के सभापित (उप-रा पित) सं यु बै ठक की अ ता नहीं करता, ोंिक वह

सं सद के िकसी भी सदन का सद नहीं होता है ।

[4] िन िल खत कथनों पर िवचार कीिजयेः-

1. सं िवधान के अनु े द-112 म ‘बजट’ श का उ े ख िकया गया है ।


2. एकवथ सिमित की िसफा रश पर रे ल बजट को आम बजट से अलग िकया
गया।

उपयु म से कौन-सा/से कथन सही है /ह?

A) केवल 1

B) केवल 2

C) 1 और 2 दोनों

D) न तो 1 और न ही 2
Hide Answer -
उ रः (b)

ा ाः उपयु म से कथन-1 गलत है ।

सं िवधान के अनु े द-112 म ‘वािषक िव ीय िववरण’ श का योग है न िक ‘बजट’

श का। वािषक िव ीय िववरण को ही साधारण बोलचाल की भाषा म ‘बजट’ कहते ह।

वष 1921 म एकवथ सिमित की िसफा रश पर रे ल बजट को आम बजट से अलग िकया

गया।

अग 2016 म पु नः रे ल बजट को आम बजट म समािहत करने का िनणय िलया गया

और इसके िलये िव मं ालय ने पाँ च सद ीय सिमित का गठन िकया, िजसकी

अनु शंसाओं पर वष 2017 म इसे आम बजट म समािहत कर िदया गया।

वािषक िव ीय िववरण (बजट) म िव ीय वष के दौरान भारत सरकार की अनु मािनत

ा यों और खच का िववरण होता है , िजसकी समय अविध 1 अ ै ल से 31 माच तक की

होती है ।

[5] िन िल खत कथनों पर िवचार कीिजयेः

1. वािषक िव ीय िववरण (बजट) को भारत का रा पित े क वष सं सद के


दोनों सदनों म रखवाता है ।
2. राज िवभाग िव मं ालय के अ गत आता है और सं घीय बजट की
तै यारी और सदन म पे श करने के िलये उ रदायी है ।

उपयु म से कौन-सा/से कथन सही है /ह?

A) केवल 1

B) केवल 2

C) 1 और 2 दोनों

D) न तो 1 और न ही 2

Hide Answer -
उ रः (a)

ा ाः उपयु म से कथन-2 गलत है ।

राज िवभाग िव मं ालय के अ गत आता है , िक ु सं घीय बजट की तै यारी और उसे

सं सद म पे श करने के िलये िव मं ालय का ‘आिथक काय िवभाग’ (Department of

Economic Affairs) उ रदायी है न की राज िवभाग।

सं िवधान के अनु े द-112 म उपबं ध है िक रा पित े क िव ीय वष के सं बंध म सं सद

के दोनों सदनों के सम भारत सरकार की उस वष के िलये अनु मािनत ा यों और य

का िववरण रखवाएगा, िजसे इस भाग म वािषक िव ीय िववरण कहा गया है ।

[6] ‘आिथक सव ण’ सं सद म ितवष ु त िकया जाता है -


A) आगामी वष के बजट के ु तीकरण के पू व।

B) आगामी वष के बजट के ु तीकरण के प ात् ।

C) िव िवधे यक के ु तीकरण के प ात् ।

D) िविनयोग िवधये क के ु तीकरण के प ात् ।

Hide Answer -
उ रः (a)

ा ाः उपयु म से िवक - (a) सही है ।

आिथक सव ण सं सद म ितवष आगामी वष के बजट के ु तीकरण से पू व ु त िकया

जाता है ।

[7] ‘अनु दान की मां ग’ के सं दभ म िन िल खत कथनों पर िवचार कीिजयेः

1. लोकसभा म अनु दान की मां ग पर चचा एवं मतदान होता है ।


2. रा सभा म अनु दान की मां ग पर न तो चचा होती है और न ही मतदान।
3. रा पित की िसफा रश के िबना िकसी अनु दान की मां ग नहीं की जा सकती
है ।

उपयु म से कौन-सा/से कथन सही है /ह?

A) केवल 1

B) केवल 1 और 2

C) केवल 1 और 3

D) 1, 2 और 3

Hide Answer -
उ रः (c)

ा ाः उपयु म से कथन 2 गलत है ।

सं िवधान के अनु े द-113 म ‘अनु दान की मां ग’ के सं बंध म चचा की गई है ।

अनु दान की मां ग पर चचा रा सभा म की जाती है , िक ु अनु दान की मां ग पर मतदान

करने की श रा सभा को नहीं है ।

लोकसभा म अनु दान की मां ग पर चचा होती है और उस पर मतदान भी होता है ।

लोकसभा को यह श है िक िकसी मां ग को अनु मित दे या अनु मित दे ने से इनकार कर

दे ।

सं िवधान के अनु े द-113(3) म उपबं ध है िक िकसी अनु दान की मां ग रा पित की

िसफा रश पर ही की जाएगी, अ था नहीं।

वािषक िव ीय िववरण (बजट) म िदये गए य के अनु मानों म भारत की सं िचत िनिध पर

भा रत य एवं भारत की सं िचत िनिध से िकये जाने वाले अ यों की रािश अलग-अलग

िदखाई जाती है । भारत की सं िचत िनिध पर अनु मािनत अ यों को लोकसभा के सम

अनु दानों की मां ग के प म रखा जाता है ।


[8] िन िल खत कथनों पर िवचार कीिजयेः

1. भारत की सं िचत िनिध पर भा रत य पर लोकसभा म मतदान होता है ,


िक ु रा सभा म नहीं।
2. सं सद को भारत की सं िचत िनिध पर भा रत य को बढ़ाने की श है ।
3. सं सद को रा पित की िसफा रश के िबना कोई कर आरोिपत करने की
श नहीं है ।
4. सं सद को रा पित की िसफा रश के िबना िकसी कर म वृ करने की
श नहीं है ।

उपयु म से कौन-सा/से कथन सही है /ह?

A) केवल 3 और 4

B) केवल 1, 2 और 3

C) केवल 1, 3 और 4

D) 1, 2, 3 और 4

Hide Answer -
उ रः (a)

ा ाः उपयु म से कथन-1 और 2 गलत ह।

वािषक िव ीय िववरण (बजट) ु त िकये जाने के प ात् सं सद के दोनों सदनों म

साधारण चचा होती है । इसके प ात् भारत की सं िचत िनिध पर भा रत य को छोड़कर

य के अ अनु मान लोकसभा के सम अनु दान की मां ग के प म रखे जाते ह।

भारत की सं िचत िनिध पर भा रत य को सं सद म मतदान के िलये नहीं रखा जाता है ।

केवल सं सद म इस पर चचा होती है । इसिलये सं सद को भा रत यों (Charged

Expenditures) म कोई प रवतन करने की श नहीं है ।

सं सद रा पित की िसफा रश के िबना न कोई कर आरोिपत कर सकती है और न िकसी

कर म वृ कर सकती है ।

[9] यिद वािषक सं घीय बजट लोकसभा ारा पा रत नहीं होता है , तो इस दशा म -

A) बजट म सं शोधन कर यह दु बारा पे श िकया जाता है ।

B) सु झाव हे तु बजट रा सभा को भे ज िदया जाता है ।

C) सं घीय िव मं ी से ाग-प दे ने के िलये कहा जा सकता है ।

D) धानमं ी अपनी मं ि प रषद का ाग-प पे श कर दे ता है ।


Hide Answer -
उ रः (d)

ा ाः उपयु म से िवक - (d) सही है ।

वािषक सं घीय बजट का लोकसभा ारा पा रत नहीं होने का अथ है िक लोकसभा म

सरकार ने ब मत खो िदया है । ऐसी थित म धानमं ी अपनी मं ि प रषद का ाग-प

ु त करे गा।

[10]िव िवधे यक के िवषय म ि या का सही अनु म िन िल खत म से कौन-सा है ?


नीचे िदए गये कूट की सहायता से सही उ र चु न:

1. बजट का ु तीकरण
2. अनु दान की मां ग पर मतदान
3. कटौती ाव
4. िविनयोग िवधे यक
5. िव िवधे यक

कूट :

A) 1, 2, 3, 4, 5

B) 1, 2, 3, 5, 4

C) 1, 3, 2, 4, 5

D) 1, 3, 2, 5, 4
Hide Answer -
उ रः (a)

ा ाः उपयु म से िवक (a) सही है ।

सं सद म बजट िन िल खत चरणों से गु जरता है ः

1. बजट का ु तीकरणः इसे दो पों म ु त िकया जाता है ः रे ल बजट और आम

बजट। रे ल बजट, आम बजट से पहले ु त िकया जाता है । िव मं ी सं सद म आम

बजट पे श करता है । वष 2017 से रे ल बजट को आम बजट म समायोिजत कर िदया

गया है । अतः अब बजट को एक की प म ु त िकया जाता है ।

2. आम बहसः दोनों सदनों म आम बहस होती है ।

3. िवभागीय सिमित ारा जाँ चः बजट पर आम बहस के प ात् सं सद की थायी सिमित

अनु दान की िव ार से जाँ च करती है और रपोट तै यार कर दोनों सदनों के सम रखी

जाती है । थायी सिमित की यह व था वष 1993 से शु की गई। यह व था

िविभ मं ालयों पर सं सदीय िव ीय िनयं ण के उ े से ारं भ की गई थी। वष 2004

म इसका िव ार िकया गया।

4. अनु दान की मां गों पर मतदानः अनु दान की मां गों पर मतदान केवल लोकसभा म ही होता

है , रा सभा को इसकी श नहीं है । रा सभा म केवल चचा होती है ।

5. कटौती ावः े क मां ग पर लोकसभा म अलग से मतदान होता है । इस दौरान सं सद

सद बहस करते ह। सद अनु दान मां गों पर कटौती के िलये ाव ला सकते ह। ये

कटौती ाव तीन कार के होते ह-

नीितगत कटौती ाव

सां केितक कटौती ाव

आिथक कटौती ाव

6. िविनयोग िवधे यकः सं िवधान के अनु े द-114 म उपबं ध है िक भारत की सं िचत िनिध से

िबना िविनयोग िवधे यक के धन की िनकासी नहीं होगी।

7. िव िवधे यक/धन िवधे यकः भारत सरकार के उस वष के िलये िव ीय ावों को

भावी करने के िलये पु रः थािपत/पे श िकया जाता है ।

ले खानु दान (vote on account): इसे अि म अनु दान भी कहते ह। सं िवधान के

अनु े द-116 म इसके िलये ावधान है । िविनयोग िवधे यक के लागू होने तक भारत की सं िचत

िनिध से धन नहीं िनकाल जा सकता है । इसम काफी समय लगता है और अ ै ल तक चला जाता

है , िक ु 31 माच के बाद सरकार को िविभ काय के िलये धन की आव कता पड़ती है । अतः

लोकसभा िवशे ष यासों के मा म से धन की व था करती है । िजसे ले खानु दान कहते ह। इसे

बजट पर आम बहस के उपरा या अनु दान की मां गों पर मतदान के उपरा पा रत िकया जाता

है । यह कुल बजट का 1/6 भाग के बराबर होता है और इसे ू नतम दो माह और अिधकतम छः

माह के य हे तु ीकृित दी जाती है ।

[11]िन िल खत म से कौन-सा एक खच भारत की सं िचत िनिध पर भा रत नहीं िकया

जाएगा?
A) रा पित की उपल याँ और भ े तथा उसके पद से सं बंिधत अ य।

B) ऐसे ऋण भार, िजनका दािय भारत सरकार पर है ।

िकसी ायालय या मा थम् अिधकरण के िनणय, िड ी या पं चाट की तु ि


C)
के िलये अपे ि त रािशयाँ ।

वह रािश जो ाकृितक आपदाओं म ए खच को चु काने के िलये आव क


D)
हो।

Hide Answer -
उ रः (d)

ा ाः उपयु म से िवक (d) सही नहीं है ।

सं िवधान के अनु े द-112(3) म भारत की सं िचत िनिध पर भा रत य के सं बंध म

उपबं ध है , जो िन िल खत है ः-

रा पित की उपल याँ एवं भ े तथा उसके पद से सं बंिधत अ य।

ऐसे ऋण-भार िजसका दािय भारत सरकार पर है , िजसके अ गत ाज, िन े प-िनिध

भार (Sinking fund charges) और मोचन भार (Redemption charges) तथा

उधार ले ने और ऋण से वा एवं ऋण मोचन से सं बंिधत अ य।

िकसी ायालय या मा थम् अिधकरण के िनणय, िड ी या पं चाट की तु ि के िलये

अपे ि त रािशयाँ ।

भारत, िनयं क एवं महाले खा परी क को सं देय वे तन, भ े और पशन।

रा सभा के सभापित (उपरा पित रा सभा का सभापित होता है और उपरा पित के

प म उसे वे तन व भ े नहीं िमलते ह।) और उप-सभापित तथा लोकसभा के अ और

उपा के वे तन एवं भ े ।

उ तम ायालय के ायाधीशों को दे य वे तन, भ े एवं पशन।

फेडरल ायालय के ायाधीशों को दे य पशन।

उ ायालय के ायाधीशों को दे य पशन।

कोई अ य जो इस सं िवधान ारा या सं सद ारा इस कार भा रत घोिषत िकया जाता

है ।

[12]लोक िव पर सं सद के िनयं ण के सं दभ म िन िल खत कथनों पर िवचार


कीिजयेः

1. वािषक िव ीय िववरण को सं सद के सम ु त करना।


2. िविनयोग िवधे यक सं सद से पा रत होने के प ात् ही भारत की सं िचत िनिध
से धन िनकाला जा सकता है ।
3. सं सद म िव िवधे यक का ु त िकया जाना।
4. ले खानु दान तथा अनु पूरक अनु दान का ावधान।

नीचे िदये गए कूट का योग कर सही उ र का चयन कीिजयेः


A) केवल 1 और 2

B) केवल 1, 2 और 3

C) केवल 1, 2 और 4

D) 1, 2, 3 और 4

Hide Answer -
उ रः (d)

ा ाः उपयु सभी कथन सही ह।

सं िवधान के अनु े द-112 के तहत रा पित े क िव ीय वष के सं बंध म सं सद के दोनों

सदनों के सम भारत सरकार की उस वष के िलये अनु मािनत ा यों और यों का

िववरण रखवाएगा। यह िववरण सं सद के िविभ ावधानों से होकर गु जरता है ।

सिवधान के अनु े द-114 म िविनयोग िवधे यक के सं बंध म ावधान है । िबना िविनयोग

िवधे यक पा रत िकये भारत की सं िचत िनिध से धन नहीं िनकाला जा सकता है ।

सं सद ारा िव िवधे यक/धन िवधे यक को पा रत करने के प ात् ही भारत सरकार के उस

वष के िलये िव ीय ाव को भावी बनाया जा सकता है ।

ले खानु दान को अि म अनु दान भी कहते ह। इसका उपबं ध सं िवधान के अनु े द-116 म

िकया गया है । सं सद भारत सरकार के िलये अि म अनु दान का ावधान करती है । इसी

तरह सं िवधान के अनु े द-115 म अनु पूरक अनु दान का उपबं ध िकया गया है ।

[13]िन िल खत कथनों पर िवचार कीिजयेः

1. सं सद के ािधकार के िबना कोई धन भारत की सं िचत िनिध से नहीं


िनकाला जा सकता है ।
2. लोक ले खा िनिध से िकये जाने वाले सभी िवतरणों के िलये भी भारत की
सं सद के ािधकार की आव कता होती है ।

उपयु म से कौन-सा/से कथन सही है /ह?

A) केवल 1

B) केवल 2

C) 1 और 2 दोनों

D) न तो 1 और न ही 2
Hide Answer -
उ रः (a)

ा ाः उपयु म से कथन 2 सही नहीं है ।

सं िवधान के भाग-12 के अ ाय-1 म तीन कार की िनिधयों की चचा है ः

1. भारत की सं िचत िनिध : अनु े द 266

2. भारत की लोक ले खा िनिध : अनु े द 266

3. भारत की आक क िनिध : अनु े द 267

भारत की लोक ले खा िनिध को कायकारी ि या ारा िनयं ि त िकया जाता है । अथात् इस

खाते से भु गतान सं सदीय ािधकार के िबना भी कर सकते ह।

भारत की सं िचत िनिध से कोई धन सं सद के ािधकार के िबना नहीं िनकाल सकते । इसके

िलये सं सद ारा िविनयोग िवधे यक पा रत करना पड़ता है ।

भारत की लोक ले खा िनिध के भाँ ित भारत की आक क िनिध को भी कायकारी ि या

से सं चािलत िकया जाता है । अथात् इस िनिध से भु गतान भी सं सद के ािधकार के िबना

कर सकते ह। भारत की आक क िनिध की शु आत 1960 म ई। यह रा पित के

अिधकार म रहती है ।

[14]िन िल खत म से कौन-सा/से भारत की सं िचत िनिध का/के भाग है /ह?

1. ाियक जमा
2. बचत
3. बक जमा
4. राज

नीचे िदये गए कूट का योग कर सही उ र का चयन कीिजयेः

A) केवल 4

B) केवल 2 और 4

C) केवल 2, 3 और 4

D) 1, 2, 3 और 4
Hide Answer -
उ रः (a)

ा ाः उपयु म से 1, 2 और 3 गलत ह।

भारत की लोक ले खा िनिधः इसके अ गत भिव िनिध जमा, ाियक जमा, बचत बक

जमा, िवभागीय जमा आिद को शािमल िकया जाता है ।

भारत की सं िचत िनिधः इसके अ गत िन िल खत को शािमल िकया जाता है ः-

1. भारत सरकार ारा ा सभी राज /कर।

2. राजकोषीय िवधे यकों, ऋणों या अथ पाय अि मों को जारी के सरकार ारा िलये गए

सभी ऋण।

3. ऋणों की पु नादायगी म सरकार ारा ा धनरािश आिद।

भारत की आक क िनिधः सं िवधान सं सद को भारत की आक क िनिध के गठन की

अनु मित दे ता है । इसकी शु आत भारत की आक क िनिध अिधिनयम, 1950 से ई।

िनिध को रा पित की ओर से िव सिचव ारा रखा जाता है । यह िनिध रा पित के

अिधकार म रहती है । वह िकसी अ ािशत य के िलये अि म दे सकता है , िजसे बाद

म सं सद ारा ािधकृत करवाया जा सकता है ।

[15]िन िल खत कथनों पर िवचार कीिजयेः

1. िविनयोग िवधे यक की रकम म प रवतन करने या अनु दान के ल को


बदलने अथवा भारत की सं िचत िनिध पर भा रत य की रकम म प रवतन
का भाव रखने वाले िकसी सं शोधन को सं सद के िकसी भी सदन म
पु रः थािपत नहीं िकया जा सकता है ।
2. िव िवधे यक म कर को बढ़ाने या घटाने सं बंधी सं शोधन सं सद के िकसी
सदन म पु रः थािपत नहीं िकया जा सकता है ।

उपयु म से कौन-सा/से कथन सही है /ह?

A) केवल 1

B) केवल 2

C) 1 और 2 दोनों

D) न तो 1 और न ही 2
Hide Answer -
उ रः (a)

ा ाः उपयु म से कथन 2 सही नहीं है ।

िव िवधे यक भारत सरकार के उस वष के िलये िव ीय ावों को भावी करने के िलये

पु रः थािपत िकया जाता है । इस पर धन िवधे यक की शत लागू होती ह। अथात इसे

रा सभा म पु रः थािपत नहीं िकया जा सकता है । िव िवधे यक म करों को बढ़ाने या

घटाने सं बंधी सं शोधन पे श िकया जा सकता है ।

िविनयोग िवधे यक की रकम म प रवतन करने या अनु दान के ल को बदलने या भारत

की सं िचत िनिध पर भा रत य की रकम म प रवतन करने वाला कोई सं शोधन सं सद के

िकसी भी सदन म पु रः थािपत नहीं िकया जा सकता। इस मामले म रा पित की सहमित के

बाद ही कोई अिधिनयम बनाया जा सकता है ।

[16]िन िल खत म से कौन-सा/से सही सु मेिलत है /ह?

1. सां केितक कटौती : ₹ 1 की कमी


2. नीित कटौती : ₹ 100 की कमी
3. आिथक कटौती : िकसी िनि त रािश का उ े ख नहीं रहता
नीचे िदये गए कूट का योग सही उ र का चयन कीिजयेः

A) केवल 3

B) केवल 1 और 2

C) 1, 2 और 3

D) उपयु म से कोई नहीं


Hide Answer -
उ रः (a)

ा ाः उपयु म से 1 और 2 गलत ह।

अनु दान की मां गों म कटौती ाव केवल लोकसभा म लाया जा सकता है । कटौती ाव

तीन कार के होते ह;

1. नीित कटौती ाव (Policy Cut Motion): इसम कहा जाता है िक मां ग की रािश म

₹ 1 की कटौती कर दी जाए। सद कोई वै क क नीित भी पे श कर सकते ह।

2. आिथक कटौती/िमत ियता कटौती ाव (Economy Cut Motion): इसम िकसी

िनि त रािश का उ े ख नहीं होता। इसम कहा जाता है िक मां ग की रािश को एक

िनि त सीमा तक कम िकया जाए।

3. सां केितक कटौती ाव (Token cut Motion): इसम कहा जाता है िक मां ग म ₹

100 की कमी की जाए।

कटौती ाव म भारत की सं िचत िनिध पर भा रत य से सं बंिधत कोई िवषय नहीं होना

चािहये ।

इसम िकसी ायालयीन करण का उ े ख नहीं होना चािहये ।

इसम िवशे षािधकार का कोई नहीं उठाया जा सकता।

[17]िन िल खत पर िवचार कीिजयेः

1. कटौती ाव
2. िगलोिटन
3. थगन ाव
4. ानाकषण ाव

उपयु ि याओं म से कौन-सी भारत के सं घीय बजट से सं बंिधत है /ह?

A) केवल 1 और 2

B) केवल 1 और 3

C) केवल 1, 2 और 3

D) 1, 2, 3 और 4
Hide Answer -
उ रः (a)

ा ाः उपयु म से (1) और (2) का सं दिभत सं बंध भारत के सं घीय बजट से है ।

कटौती ावः अनु दानों की मां गों पर बहस के समय लोकसभा म सद ों ारा लाया जाता

है ।

िगलोिटनः अनु दान की मां गों पर मतदान के िलये कुल 26 िदन िनधा रत िकये गए ह।

अ म िदन लोकसभा अ शे ष सभी मां गों को मतदान के िलये पे श करता है , चाहे

सद ारा उस पर चचा ई हो या न ई हो। इसे ही िगलोिटन कहते ह।

[18]ले खानु दान और अं त रम बजट के बीच ा अ र है ?

1. थायी सरकार केवल ले खानु दान के ावधान का योग करती है , जबिक


कायवाहक सरकार अं त रम बजट के ावधान का योग करती है ।
2. ले खानु दान सरकार के बजट के यप मा से सं बं होता है ।

नीचे िदये गए कूट का योग कर सही उ र का चयन कीिजयेः

A) केवल 1

B) केवल 2

C) 1 और 2 दोनों

D) न तो 1 और न ही 2

Hide Answer -
उ रः (b)

ा ाः उपयु म से कथन 1 गलत है ।

ले खानु दान के ावधानों का योग थायी एवं कायवाहक दोनों सरकार कर सकती ह। उसी

तरह अ रम बजट के ावधान का योग थायी एवं कायवाहक दोनों सरकार कर सकती

ह।

ले खानु दान सरकार के बजट य प से सं बंिधत मा है । इसे एक िनि त अविध ( ू नतम

2 माह और अिधकतम छः माह) के िलये कुल बजट का 1/6 भाग सरकार को िदया जाता

है , जबिक अ रम बजट म य एवं आय दोनों प स िलत होते ह।

ले खानु दान (Vote on account) की ीकृित सं सद ारा तब दान की जाती है , जब

सामा बजट की समय-सीमा के अ र (बजट की समय-सीमा 1 अ ै ल से 31 माच तक

होती है ) पा रत होने की आशा नहीं होती है ।

[19]सू ची-I को सू ची-II से सु मेिलत कीिजयेः

सू ची-I सू ची-II

A. ले खानु दान 1. रकम जो अनु दान से अिधक य की गई हो।


B. यानु दान 2.अित र य
जो अनु मोिदत बजट म स िलत नहीं है

C. अनु पूरक 3. िव ृत अनु मान


अनु दान के िबना ही ीकृत एक मु धनरािश।

D. अिधक अनु दान 4. बजट अनु मोदन के लं िबत रहने तक पे शगी अनु दान।

नीचे िदये गये कूट का योग कर सही उ र का चयन कीिजयेः


A B C D

A) 4 3 2 1

B) 4 2 3 1

C) 1 2 3 4

D) 1 3 2 4

Hide Answer -
उ रः (a)

ा ाः

ले खानु दान (Vote on account): इसे अि म अनु दान भी कहा जाता है । बजट 31 माच

तक पा रत न होने की थित म सं सद एक िनि त अविध के िलये ( ू नतम 2 माह और

अिधकतम छः माह) बजट का 1/6 भाग अि म अनु दान के प म ीकृत करती है

यानु दान (Vote of credit): इसका उपबं ध सं िवधान के अनु े द-116(ख) म िकया

गया है िक जब िकसी से वा की मह ा या उसके अिनि त प के कारण इसे वािषक

िव ीय िववरण के साथ विणत नहीं िकया जा सकता। अथात िव ृ त अनु मान के िबना ही

ीकृत एक मु रािश।

अपवादानु दान (Exceptional grants): इसका वणन सं िवधान के अनु े द-116(ग) म

िकया गया है । इसे िवशे ष उ े के िलये मं ज़ूर िकया जाता है और यह वतमान िव ीय वष

या से वा से सं बंिधत नहीं होता है ।

अनु पूरक अनु दान (Supplementary grants): इसका उपबं ध सं िवधान के

अनु े द-115(क) म िकया गया है । िकसी िविश से वा पर चालू िव वष के िलये य

िकये जाने के िलये ािधकृत कोई रकम उस वष के उ े के िलये अपया पाई जाती है

या उस वष के वािषक िव ीय िववरण म शािमल न की गई हो।

अिधक अनु दान (Excess grants): िकसी िव ीय वष के दौरान िकसी से वा पर, उस

वष और उस से वा के िलये अनु दान की गई रकम से अिधक कोई धन य हो गया हो।

सं िवधान के अनु े द-115(ख) म इसका उपबं ध िकया गया है ।

[20]िन िल खत म से कौन-सा/से ावधान भारत के सं िवधान के अनु े द-249 ारा


द है /ह?

1. िवधान की अविश श याँ ।


2. रा सू ची के िवषय के सं बंध म रा ीय िहत म िविध बनाने की सं सद की
श ।
3. अं तरा ीय करारों को भावी करने के िलये िवधान।

नीचे िदये गए कूट का योग कर सही उ र का चयन कीिजयेः

A) केवल 2

B) केवल 3

C) केवल 2 और 3

D) 1, 2 और 3

Hide Answer -
उ रः (a)

ा ाः उपयु म से कथन (1) और (3) सही नहीं ह।

िवधान की अविश श यों के सं बंध म उपबं ध सं िवधान के अनु े द-248 म िकया गया

है । यिद िकसी िवषय का उ े ख सं घ सू ची, रा सू ची एवं समवत सू ची म नहीं है तो ऐसे

िवषय को अविश सू ची म डालते ह। इसम िविध बनाने का अिधकार अ न प से सं सद

को है ।

अं तरा ीय करारों/समझौतों को भावी करने के िलये िवधान बनाने के सं बंध म उपबं ध

सं िवधान के अनु े द-253 म िकया गया है । सं सद को िकसी अ दे श या दे शों के साथ

की गई िकसी सं िध, करार या अिभसमय अथवा िकसी अं तरा ीय स े लन, सं गम या अ

िनकाय म िकये गए िकसी िनणय के काया यन के िलये भारत के सं पूण रा े या

उसके िकसी भाग के िलये िविध बनाने की श है ।

रा सू ची के िवषय के सं बंध म रा ीय िहत म िविध बनाने सं बंधी उपबं ध सं िवधान के

अनु े द-249 म िकया गया है । रा सभा उप थत और मत दे ने वाले सद ों म से कम-

से -कम 2/3 सद ों ारा समिथत सं क ारा घोिषत करती है िक रा ीय िहत म यह

आव क या समीचीन है , तो सं सद, रा सू ची म विणत िवषय पर िविध बनाए, सं सद

भारत के सं पूण रा े या उसके िकसी भाग के िलये िविध बना सकती है । यह िविध एक

वष के िलये होती है , िकंतु सं सद सं क पा रत कर एक-एक वष के िलये बढ़ा सकती है ।


होम › सामा अ यन टे Hits: 599

[1] सं सद रा सू ची के िवषय म िविध बना सकती है ः

A) यिद सं सद रा ीय िहत म इस आशय का सं क पा रत करती है ।

B) यिद उ तम ायालय सं सद को आव कस ा दान करता है ।

यिद रा सभा दो-ितहाई ब मत से पा रत सं क ारा घोिषत करती है िक


C)
रा का िवचाराधीन िवषय रा ीय मह का है ।

यिद रा पित अ ादे श जारी कर िकसी िवषय को रा सू ची से सं घ सू ची या


D)
समवत सू ची म ह ां त रत करता है ।

Hide Answer -
उ रः (c)

ा ाः उपयु म से िवक (c) सही है ।

सं िवधान के अनु े द-249 के तहत रा सभा यिद उप थत और मत दे ने वाले सद ों म से

कम से कम 2/3 सद ों ारा समिथत सं क ारा घोिषत करती है िक रा ीय िहत म यह

आव क या समीचीन है तो सं सद भारत के सं पूण भाग या उसके िकसी भाग के िलये िविध बना

सकती है । इस िविध की अविध एक वष की होती है ।

[2] रा सभा की अन अिधका रता के सं दभ म िन िल खत कथनों पर िवचार


कीिजयेः

1. नई अ खल भारतीय से वा को सृ िजत करने के सं दभ म।


2. रा सू ची म विणत िकसी िवषय पर िविध बनाने के िलये सं सद को
ािधकृत करने के सं दभ म।
3. रा ों के सृ जन तथा उ ू लन के सं दभ म।

उपयु म से कौन-सा/से कथन सही है /ह?

A) केवल 1

B) केवल 2

C) केवल 1 और 2

D) 1, 2 और 3
Hide Answer -
उ रः (c)

ा ाः उपयु म से कथन (3) सही नहीं है ।

रा ों के सृ जन तथा उ ू लन के सं बंध म रा सभा को कोई िवशे षािधकार नहीं है ।

रा सभा को दो िवषयों म अन अिधका रता ा है अथवा रा सभा का िवशे षािधकार

है ः-

अ खल भारतीय से वाः सं िवधान के अनु े द-312 म इस सं बंध म उपबं ध िकया गया है

िक रा सभा उप थत और मत दे ने वाले सद ों म से कम से कम 2/3 सद ों ारा

समिपत सं क ारा यह घोिषत करती है िक रा ीय िहत म ऐसा करना आव क या

समीचीन है तो सं सद, सं घ और रा ों के िलये स िलत एक या अिधक अ खल भारतीय

से वाओं का सृ जन कर सकती है ।

रा सू ची म विणत िकसी िवषय पर िविध बनाने के िलये सं सद को अिधकृत कर सकती

है । सं िवधान के अनु े द-249 म इसका उपबं ध है ।

[3] भारतीय सं सद रा सू ची के िवषय म िविध बना सकती है ः

1. अं तरा ीय करारों को भावी बनाने हे तु।


2. सं बंिधत रा ों की सहमित से ।
3. रा ों म रा पित शासन लागू रहने की थित म।
4. रा ीय आपात लागू होने की थित म।
5. रा ीय िहत म जब रा सभा सद दो-ितहाई ब मत से ाव पा रत कर।

उपयु म से कौन-से कथन सही ह?

A) केवल 1और 2

B) केवल 2 और 3

C) केवल 3, 4 और 5

D) 1, 2, 3, 4 और 5
Hide Answer -
उ रः (d)

ा ाः उपयु सभी कथन सही ह।

अं तरा ीय करारों/समझौतों को भावी बनाने के िलये सं सद सं िवधान के अनु े द-253 के

तहत कोई भी िविध बना सकती है ।

दो या दो से अिधक रा ों के िलये उनकी सहमित से सं सद रा सू ची के िवषय म

सं िवधान के अनु े द-252 के तहत िविध बना सकती है ।

रा ों म रा पित शासन लागू रहने की थित म सं सद सं िवधान के अनु े द-357 के तहत

रा सू ची के िवषय म िविध बना सकती है ।

यिद रा सभा रा ीय िहत म 2/3 ब मत से ाव पा रत कर दे ती है तो सं सद सं िवधान

के अनु े द-249 के तहत रा सू ची के िवषय म िविध बना सकती है ।

यिद रा ीय आपात की घोषणा वतन हो तो सं सद सं िवधान के अनु े द 250 के तहत

िविध बना सकती है ।

[4] अं तरा ीय सं िधयों को भारत के िकसी भाग अथवा सं पूण भारत म लागू करने के

िलये सं सद कोई भी कानू न बना सकती है ः-

A) सभी रा ों की सहमित से

B) सं बंिधत रा ों की सहमित से

C) िबना िकसी रा की सहमित से

D) कम से कम आधे रा ों की सहमित से

Hide Answer -
उ रः (c)

ा ाः उपयु म से िवक (c) सही है ।

सं िवधान के अनु े द 253 म यह ावधान है िक सं सद अं तरा ीय करारों/सं िधयों/समझौतों को

भावी बनाने के िलये भारत के सं पूण रा े या उसके िकसी भाग के िलये कोई भी िविध बना

सकती है ।

[5] िन िल खत कथनों पर िवचार कीिजयेः

1. भारत के िकसी वतमान रा की सीमाओं म प रवतन करने के िलये सं सद


सश है ।
2. भारत के िकसी वतमान रा की सीमाओं म प रवतन करने से सं बंिधत
कोई िवधे यक केवल रा सभा म तथा केवल भारत के रा पित की
िसफा रश पर पु रः थािपत िकया जा सकता है ।

उपयु म से कौन-सा/से सही है /ह?


A) केवल 1

B) केवल 2

C) 1 और 2 दोनों

D) न तो 1 और न ही 2

Hide Answer -
उ रः (a)

ा ाः उपयु म से कथन (2) सही नहीं है ।

सं िवधान के अनु े द (3) म उपबं ध िकया गया है िक भारत के िकसी वतमान रा की

सीमाओं म प रवतन करने से सं बंिधत कोई िवधे यक सं सद के िकसी भी सदन म रा पित

की िसफा रश पर ही पु रः थािपत िकया जाएगा।

सं िवधान के अनु े द (3) के तहत सं सद नए रा ों के िनमाण, वतमान रा ों के े ,

सीमाओं या नामों म प रवतन करने के िलये सश है ।

[6] लोकसभा और रा सभा की समान श याँ िकस े म है ?

A) नई अ खल भारतीय से वाएँ गिठत करने के सं बंध म।

B) रा ीय िहत म रा सू ची के िवषय म िविध बनाने के सं बंध म।

C) कटौती ाव ु त करने के िवषय म।

D) सं िवधान के सं शोधन के िवषय म।

Hide Answer -
उ रः (d)

ा ाः उपयु म से िवक (d) सही है ।

सं िवधान म सं शोधन करने के िलये सं सद के दोनों सदनों की श समान है । दोनों सदनों

ारा पा रत होने पर ही सं िवधान सं शोधन माना जाएगा, अ था नहीं। इसिलये इसम सं यु

बै ठक का ावधान नहीं है । इसे भारत का रा पित पु निवचार के िलये नहीं लौटा सकता है ।

अ खल भारतीय से वा (अनु े द-312) और रा ीय िहत म रा सू ची पर िविध बनाने का

रा सभा का िवशे षािधकार है (अनु े द-249)

अनु दान की मां ग पर कटौती ाव केवल लोकसभा म ु त िकया जाता है ।

[7] िन िल खत कथनों पर िवचार कीिजयेः

1. सं िवधान के अनु े द-105 के तहत सं सद और उसके सद ों तथा रा ों


के िवधानमं डलों और उसके सद ों को िवशे षािधकार ा है ।
2. सं सद की कायवाही के दौरान कायवाही चलने से 40 िदन पू व तथा ब
होने के 40 िदन प ात् सं सद के िकसी सद को िसिवल एवं आपरािधक
या िनवारक िनरोध के मामले म िगर ार नहीं कर सकते ।
3. सं सद स के दौरान सं सद के िकसी सद को िकसी ायालय म सा ी
के प म उप थत होने से मु ा है ।

उपयु म से कौन-सा/से कथन सही है /ह?

A) केवल 3

B) केवल 2

C) केवल 2 और 3

D) 1, 2 और 3

Hide Answer -
उ रः (a)

ा ाः उपयु म से कथन (1) और (2) गलत ह।

सं िवधान के अनु े द 105 के तहत सं सद, उसके सद ों एवं उसकी सिमितयों को

िवशे षािधकार ा है , जबिक रा ों के िवधानमं डलों, उसके सद ों और उसकी सिमितयों

को सं िवधान के अनु े द-194 के तहत िवशे षिधकार ा है ।

सं सद की कायवाही के दौरान कायवाही चलने से 40 िदन पू व और ब होने के 40 िदन

बाद तक सं सद के िकसी सद को केवल िसिवल मामलों म िगर ार नहीं कर सकते ह।

आपरािधक या िनवारक िनरोध के तहत यह िवशे षािधकार ा नहीं है ।

सं सद सद को सं सद या उसकी सिमित म कही गई िकसी बात के िलये ायालय म

दोषी नहीं ठहराया जा सकता है । सद ारा कही गई िकसी बात के िलये कोई कायवाही

नहीं की जा सकती। दू सरी ओर सं िवधान के अनु े द-121 म उपबं ध िकया गया है िक

सं सद का कोई सद सं सद की अ र सव ायालय या उ ायालय के िकसी

ायाधीश के अपने क ों के िनवहन के िलये आचरण के िवषय म चचा नहीं करे गा।

[8] सं सदीय िवशे षािधकार के सं दभ म िन िल खत म से कौन-सा कथन सही नहीं है ?

सं सद का े क सदन चचा एवं कायवाही को कािशत कर सकता है ,और


A)
अ यों को कािशत करने से रोक सकता है ।

सं सद के े क सदन को सद ों एवं बाहरी यों को सदन के


B)
िवशे षािधकारों के उ ं घन करने पर द त करने की श है ।

सं सद के े क सदन को बाहरी यों को अपविजत करने का अिधकार


C)
है ।

सं सद के े क सद को सदन के अं दर बहस और कायवाहीयों की


D)
तं ता है , िकंतु इसकी जाँ च ायालय ारा की जा सकती है ।
Hide Answer -
उ रः (d)

ा ाः उपयु म से कथन (d) गलत है ।

सं सद के े क सद को सदन के अं दर बहस और कायवाही की तं ता ा है और

सं सद के े क सदन म जो कुछ कहा या िकया जाता है , ायालय उसकी जाँ च नहीं कर

सकता है । सं सद का े क सद सदन के भीतर उ होने वाले मामलों पर ायालय

के ह े प के िबना िनणय ले सकती है ।

सं सद के े क सदन को बहस एवं कायवाही को कािशत करने का अिधकार तथा अ

यों को काशन से रोकने का अिधकार है ।

सं सद का े क सदन बाहरी यों को अपविजत कर सकता है । सदन को यह

अिधकार है िक वह अपनी बै ठक को गु बना ले ।

सं सद के े क सदन को सद ों एवं बाहरी यों को िवशे षािधकारों के उ ं घन पर

द त करने की श है , जै सेः-

एच.सी. मु ल को वष 1951 म लोकसभा से िन ािसत िकया गया।

सु म ामी को 1976 म रा सभा से िन ािसत िकया गया। वे रा सभा की

सद ता से िन ािसत होने वाले पहले सद थे ।

ीमित इ रा गां धी को वष 1970 म लोकसभा से िन ािसत िकया गया।

सं सद के े क सदन को यह श है िक वह अपनी कायवािहयों को िनयं ि त एवं िविनयिमत

करे ।

[9] िन िल खत कथनों पर िवचार कीिजयेः

1. सदन की अनु मित के िबना उसकी साठ लगातार बै ठकों म अनु प थत


रहने वाला थम सां सद लोकसभा से था।
2. े फल की ि से लोकसभा का सवािधक बड़ा िनवाचन े उ ाव है ।

उपयु म से कौन-सा/से कथन सही है /ह?

A) केवल 1

B) केवल 2

C) 1 और 2 दोनों

D) न तो 1 और न ही 2
Hide Answer -
उ रः (d)

ा ाः उपयु दोनों कथन गलत ह।

सदन की अनु मित के िबना उसकी साठ लगातार बै ठकों से अनु प थत रहने वाला थम

सां सद रा सभा से था। नव र 2000 म पं जाब के िनदलीय रा सभा सां सद बरिजं दर

िसं ह हमदद को सदन की अनु मित के िबना 60 लगातार बै ठकों म अनु प थित के कारण

सद ता से अयो घोिषत कर िदया गया।

े फल की ि से लोकसभा का सवािधक बड़ा िनवाचन े ल ाख है , तथा सबसे छोटा

िनवाचन े चाँ दनी चौक है ।

[10]लोक ले खा सिमित के सं दभ म िन िल खत म से कौन-सा कथन सही है ?

लोक ले खा सिमित भारत शासन अिधिनयम, 1935 के अं तगत पहली बार


A)
1937 म अ म आई।

लोक ले खा सिमितयों की कुल सद सं ा 25 है , िजसम लोकसभा और


B)
रा सभा के सद होते ह।

C) कोई भी मं ी इस सिमित का सद नहीं हो सकता है ।

D) इस सिमित का अ हमे शा स ाधारी दल से होता है ।

Hide Answer -
उ रः (c)

ा ाः उपयु म से िवक (c) सही है ।

लोक ले खा सिमित म िकसी भी मं ी का िनवाचन नहीं हो सकता है । ितवष सं सद ारा

अपने सद ों म से समानु पाितक ितिनिध णाली के आधार पर एकल सं मणीय मत

के मा म से इसके सद ों का चु नाव िकया जाता है ।

लोक ले खा सिमित भारत शासन अिधिनयम, 1919 के अं तगत वष 1921 म पहली बार

अ म आई।

वतमान म लोक ले खा सिमित की कुल सद सं ा 22 है , िजसम लोकसभा के 15 और

रा सभा के 7 सद ह।

इस सिमित का अ िवप ी दल से चु ना जाता है । यह पर रा 1967 म िवकिसत ई।

लोकसभा अ , लोकसभा सद ों म से इसके अ की िनयु करता है । रा सभा

का सद इस सिमित का अ नहीं हो सकता।

इस सिमित का कायकाल एक वष का होता है ।

यह सं सद की थायी िव ीय सिमित है ।

[11] ा लन सिमित के सं दभ म िन िल खत कथनों पर िवचार कीिजयेः

1. वतमान म इस सिमित की कुल सद सं ा 30 है , िजसम केवल


लोकसभा के सद शािमल होते ह।
2. इस सिमित का अ हमे शा स ाधारी दल से होता है ।
उपयु म से कौन-सा/से कथन सही है /ह?

A) केवल 1

B) केवल 2

C) 1 और 2 दोनों

D) न तो 1 और न ही 2

Hide Answer -
उ रः (c)

ा ाः उपयु दोनों कथन सही ह।

वतमान म इस सिमित की कुल सद सं ा 30 है । मू लतः इसम 25 सद थे , िकंतु

1956 म इसकी सद सं ा बढ़ाकर 30 कर दी गई, िजसम केवल लोकसभा के सद

शािमल होते ह। रा सभा के सद इस सिमित म शािमल नहीं िकये जाते ह।

इस सिमित का अ हमे शा स ाधारी दल से चु ना जाता है । लोकसभा अ , लोकसभा

सद ों म से अ की िनयु करता है ।

कोई मं ी इस सिमित का सद नहीं हो सकता है ।

यह सं सद की एक थायी िव ीय सिमित है , िजसका गठन पहली बार वष 1950 म

त ालीन िव मं ी जॉन मथाई की िसफा रश पर िकया गया।

इस सिमित का कायकाल एक वष का होता है ।

इसके सद ों का चु नाव ितवष लोकसभा ारा अपने सद ों म से समानु पाितक

ितिनिध णाली के आधार पर एकल सं मणीय मत ारा िकया जाता है ।

[12]सावजिनक उप म सिमित के सं दभ म िन िल खत कथनों पर िवचार कीिजयेः

1. इस सिमित की थापना तं ता ा के प ात् की गई।


2. यह एक थायी िव ीय सं सदीय सिमित है , िजसकी कुल सद सं ा 25
है । इसम लोकसभा एवं रा सभा के सद शािमल होते ह।
3. कोई मं ी इस सिमित का सद नहीं हो सकता, िकंतु इस सिमित का
अ रा सभा या लोकसभा का सद हो सकता है ।

उपयु म से कौन-सा/से कथन सही है /ह?

A) केवल 1

B) केवल 2

C) केवल 1 और 2

D) 1, 2 और 3
Hide Answer -
उ रः (a)

ा ाः उपयु म से कथन (2) और (3) सही नहीं ह।

सावजिनक उप म सिमित सं सद की एक थायी िव ीय सिमित है , िजसका गठन ितवष

सं सद ारा समानु पाितक ितिनिध णाली के आधार पर एकल सं मणीय मत के आधार

पर िकया जाता है ।

कोई मं ी इस सिमित का सद नहीं बन सकता। इस सिमित का अ लोकसभा अ

ारा लोकसभा सद ों म से िनयु िकया जाता है ।

इस सिमित का गठन कृ मे नन सिमित की िसफा रश पर वष 1964 म िकया गया।

ारं भ म इस सिमित म 15 सद थे , िजसम 10 लोकसभा के और 5 रा सभा के थे ।

वष 1974 म इस सिमित की सद सं ा बढ़ाकर 22 कर दी गई, िजसम 15 सद

लोकसभा के और 7 सद रा सभा के होते ह।

इस सिमित का कायकाल एक वष का होता है ।

[13]लोक ले खा सिमित के सं दभ म िन िल खत म से कौन-सा/से कथन सही है /ह?

1. इस सिमित ारा िनयं क एवं महाले खा परी क के वािषक ितवे दनों की


जाँ च सं सद म ु त करने से पू व की जाती है ।
2. यह सिमित अपनी रपोट भारत के रा पित को सौंपती है और रा पित
उसके ितवे दनों को सं सद के सम रखवाता है ।
3. इस सिमित के ारा क सरकार के िविनयोग तथा िव ले खाओं की जाँ च
की जाती है ।

नीचे िदये गए कूट का योग कर सही उ र का चयन कीिजयेः

A) केवल 1

B) केवल 3

C) केवल 1 और 3

D) 1, 2 और 3
Hide Answer -
उ रः (b)

ा ाः उपयु म से कथन (1) और (2) गलत ह।

लोक ले खा सिमित भारत के िनयं क एवं महाले खा परी क के वािषक ितवे दनों की जाँ च

करती है , िकंतु रा पित ारा सं सद म रखवाने के प ात् , न िक पू व।

यह सिमित अपना ितवे दन लोकसभा म या लोकसभा अ को सौंपती है न िक रा पित

को। िनयं क एवं महाले खा परी क (कैग) अपनी रपोट रा पित को सौंपता है और

रा पित उसकी रपोट को सं सद म रखवाता है । सं सद उस ितवे दन को लोक ले खा सिमित

को सौंपती है ।

इस सिमित के ारा क सरकार के िविनयोग तथा िव ले खाओं की जाँ च के साथ-साथ

लोकसभा म ु त िकसी अ ले खा की भी जाँ च की जाती है ।

[14]िन िल खत म से कौन-सा एक कथन ा लन सिमित की अिधका रता को नहीं


बताता?

A) यह शासन म द ता एवं िम यता लाने की वै क क नीितयाँ सु झाती है ।

यह परी ा करती है िक ा लन म जो नीित िनधा रत की गई है , उसकी


B)
सीमाओं म रहते ए धन अ ी तरह लगाया गया है या नहीं।

C) यह सु झाव दे ती है िक सं सद म अनु मान िकस तरह ु त िकया जाए।

यह सरकारी अित ियता को रोकने म सहायक है , ोंिक इसकी रपोट पर


D)
सं सद म िवचार-िवमश िकया जाता है ।

Hide Answer -
उ रः (d)

ा ाः उपयु म िवक (d) सही नहीं है ।

ा लन सिमित के ितवे दन पर सदन म चचा नहीं होती। सरकार की अनु दान की मां ग

इस सिमित के ितवे दन की ती ा नहीं करती है । यह उपयोगी सु झाव दे ती है और

आगामी िव ीय वष म सरकार को बढ़ी-चढ़ी मां ग करने से रोकती है । यह सिमित वष भर

काय करती है और अपने िवचार सदन के सम रखती है ।

सरकार ारा ािवत य की समी ा के िलये एक ा लन सिमित गिठत की जाती है ।

इसका गठन लोकसभा म बजट ु त िकये जाने के प ात् होता है ।

यह सिमित सु झाव दे ती है िक िकस कार िमत ियता की जा सकती है और सं गठन म

द ता लाने के िलये कौन से सु धार या शासिनक सु धार िकये जा सकते ह, जो अनु मानों से

सं बंिधत नीित के सं गत हों।

इस सिमित को िमत ियता सिमित भी कहते ह।

[15]िन िल खत म से कौन-सी एक सबसे बड़ी सं सदीय सिमित है ?


A) लोक ले खा सिमित

B) ा लन सिमित

C) सरकारी उप म सिमित

D) यािचका सिमित

Hide Answer -
उ रः (b)

ा ाः उपयु म से िवक (b) सही है ।

ा लन सिमितः इसम कुल 30 सद होते ह, जो सभी लोकसभा के सद होते ह।

लोक ले खा सिमितः इसम कुल 22 सद होते ह, िजसम लोकसभा के 15 और रा सभा

के 7 सद होते ह।

सरकारी उप म सिमितः इसम कुल 22 सद होते ह, िजसम लोकसभा के 15 और

रा सभा के 7 सद होते ह।

यािचका सिमितः इसम लोकसभा सिमित म 15 तथा रा सभा सिमित म 10 सद होते

ह। यह एक जाँ च सिमित है । यह सिमित िवधे यकों, सावजिनक मह के मामलों पर दायर

यािचकाओं एवं आवे दनों पर िवचार करती है । इसे आवे दन सिमित भी कहते ह।

[16]सं सद के िवभाग-सं बंधी थायी सिमितयों के सं दभ म िन िल खत म से कौन-सा

कथन सही नहीं है ?

लोकसभा की िनयम सिमित की अनु शंसाओं पर सं सद म 1990 के दशक म


A)
िवभागीय सिमितयों का गठन िकया गया।

गठन के समय इनकी सं ा 17 थी, जबिक वतमान म इनकी सं ा 24 हो


B)
गई है ।

े क िवभागीय थायी सिमित म लोकसभा और रा सभा के सद शािमल


C)
रहते ह।

े क िवभागीय थायी सिमित म लोकसभा और रा सभा के सद ों का


D)
चु नाव लोकसभा अ ारा िकया जाता है ।
Hide Answer -
उ रः (d)

ा ाः उपयु म से िवक (d) सही नहीं है ।

सं सद की े क िवभागीय थायी सिमित म लोकसभा के सद लोकसभा अ के ारा

और रा सभा के सद रा सभा के सभापित ारा चु ने जाते ह

लोकसभा की िनयम सिमित की अनु शंसाओं पर वष 1993 म 17 िवभागीय सिमितयों का

गठन िकया गया और वष 2004 म इनकी सं ा बढ़ाकर 24 हो गई।कोई भी मं ी

िवभागीय सिमितयों का सद नहीं बन सकता। यिद कोई सद मं ी बन जाता है तो

सिमित से उसकी सद ता समा हो जाती है ।

इन सिमितयों का कायकाल एक वष का होता है ।

8 सिमितयाँ रा सभा और 16 सिमितयाँ लोकसभा के अं तगत काय करती ह।

[17]सदन के पटल पर िकसी मं ी के ारा की गई वचनब ता की प रपू णता हो गई है


अथवा नहीं, इसका िनणय िकया जाएगा-

A) सं बंिधत मं ालय से सं ब परामश सिमित ारा।

B) सं बंिधत मं ालय के काय से सं बंिधत वर सिमित ारा।

C) सं बंिधत मं ालय की राजनीितक मामलों सं बंधी सिमित ारा।

D) लोक आ ासन सिमित ारा।

Hide Answer -
उ रः (d)

ा ाः उपयु म से िवक (d) सही है ।

लोक/सरकारी आ ासन सिमित जाँ च एवं िनयं ण के िलये सं सद की थायी सिमित है । यह

सिमित मं ि यों ारा सदन म समय-समय पर िदये गए आ ासनों, वचनों एवं ित ाओं की

जाँ च करती है और िकस पर िकतना ि या यन आ है , इस पर ितवे दन दे ती है ।

[18]िन िल खत म से िकसके सं बंध म सं सदीय सिमित का गठन नहीं िकया गया है ?

A) अनु सूिचत जाित एवं जनजाित क ाण सिमित।

B) मिहला सश ीकरण सिमित।

C) अ सं कों के क ाण से सं बंिधत सिमित।

D) िवशे षािधकार सिमित।


Hide Answer -
उ रः (c)

ा ाः उपयु म से िवक (c) सही नहीं है ।अ सं कों के क ाण से सं बंिधत िकसी

सिमित का गठन नहीं िकया गया है ।

अनु सूिचत जाित एवं जनजाित के क ाण से सं बंिधत तथा रा ीय अनु सूिचत जाित एवं

जनजाित आयोग के ितवे दनों पर िवचार करने के िलये अनु सूिचत जाित एवं जनजाित

क ाण सिमित का गठन िकया गया है । इसम वतमान म 30 सद ह, 20 लोकसभा से

तथा 10 रा सभा से । यह जाँ च एवं िनयं ण से सं बंिधत सिमित है ।

रा ीय मिहला आयोग के ितवे दनों तथा क सरकार ारा मिहलाओं की थित, ग रमा

तथा सभी े ों म समानता के िलये ा कदम उठाए गए ह, की जाँ च के िलये मिहला

सश ीकरण सिमित का गठन वष 1997 म िकया गया। इसके 20 सद लोकसभा से

एवं 10 सद रा सभा से चु ने जाते ह। यह एक जाँ च एवं िनयं ण से सं बंिधत सिमित है ।

िवशे षािधकार सिमित एक जाँ च सिमित है । यह सदन एवं उसके सद ों के िवशे षािधकार

हनन सं बंधी मामलों की जाँ च करती है और उपयु कायवाही की अनु शंसा करती है ।

एक अ जाँ च सिमित आचार सिमित है । यह सिमित सं सद सद ों के िलये आचार सं िहता

लागू करवाती है ।

[19]परामशदा ी/सलाहकार सिमित के सं दभ म िन िल खत कथनों पर िवचार कीिजयेः

1. यह सं सदीय सिमित नहीं है ।


2. यह सिमित सं सदीय काय मं ालय ारा गिठत की जाती है ।
3. इस सिमित के सद ों को लोकसभा अ नािमत करता है ।
4. इस सिमित का अ लोकसभा अ ारा िनयु िकया जाता है ।

उपयु म से कौन-से कथन सही ह?

A) केवल 1 और 2

B) केवल 3 और 4

C) केवल 1, 2 और 3

D) 1, 2, 3 और 4
Hide Answer -
उ रः (a)

ा ाः उपयु म से कथन (3) और (4) गलत ह।

परामशदा ी सिमित के सद ों को लोकसभा अ नािमत नहीं करता, ब सं सदीय

काय मं ालय, भारत सरकार ारा िविभ मं ालयों म थािपत सिमितयों, प रषदों, बोड ,

आयोगों आिद म सद ों को मनोनीत िकया जाता है ।

इस सिमित का अ उस मं ालय का मं ी अथवा भारी रा -मं ी होता है न िक

लोकसभा अ ारा िनयु होता है ।

परामशदा ी सिमित म सं सद के सद होते ए भी सं सदीय सिमित नहीं है । ोंिक यह

सिमित सं सदीय सिमित के िनयमों का पालन नहीं करती है । इस सिमित की अिधकतम

सद सं ा 30 एवं ू नतम सं ा 10 होती है । इस सिमित की सद ता ै क होती

है , इसे सं सद सद ों एवं ने ताओं की िच पर छोड़ िदया जाता है ।

यह सिमित सं सदीय काय मं ालय ारा गिठत की जाती है । सं सदीय काय मं ालय सं सदीय

काय मं ी के िनदशन म काय करता है ।

[20]सू ची-I को सू ची-II से सु मेिलत कीिजये तथा नीचे िदये गए कूट का योग कर
सही उ र का चयन कीिजयेः

सू ची-I सू ची-II

A. लोक ले खा सिमित 1. तदथ सिमित

B. यािचका सिमित 2. िडं ग सिमित

C. ॉक बाज़ार- ै म सं यु सिमित 3. िव ीय सिमित

D. िवभागीय सिमित 4. कायकारी सिमित

कूट :

A B C D

A) 3 4 1 2

B) 3 1 4 2

C) 4 1 2 3

D) 4 2 1 3
Hide Answer -
उ रः (a)

ा ाः

लोक ले खा सिमितः यह एक िव ीय सिमित है । इसके अलावा दो अ िव ीय सिमितयाँ

ह- ा लन (अनु मान सिमित) एवं सरकारी उप म सिमित।

यािचका सिमितः यािचका/आवे दन सिमित एक जाँ च/कायकारी सिमित है । अ जाँ च

सिमितयाँ - िवशे षािधकार सिमित एवं आचार सिमित।

ॉक बाज़ार- ै म सं यु सिमितः यह एक तदथ सिमित है । तदथ सिमित का गठन

िकसी िवशे ष उ े ों की पू ित के िलये िकया जाता है और जब उन िवशे ष उ े ों की पू ित

हो जाती है तो सिमित समा हो जाती है ।

िवभागीय सिमितः यह एक िडं ग सिमित होती है ।


होम › सामा अ यन टे Hits: 443

[1] भारतीय सं सद िकस रीित से शासन पर िनयं ण करती है ।

A) सं सदीय सिमितयों के मा म से ।

B) िविभ मं ालयों की परामशदा ी सिमितयों के मा म से ।

C) शासकों से आविधक ितवे दन िभजवाकर।

D) कायपािलका को रट जारी करने के िलये बा करके।

Hide Answer -
उ रः (a)

ा ाः भारतीय सं सद सं सदीय सिमितयों के मा म से शासन पर िनयं ण रखती है ।

[2] रा ों के रा पाल के सं दभ म िन िल खत कथनों पर िवचार कीिजएः

1. भारत के सं िवधान म उपबं ध है िक े क रा के िलये एक रा पाल


होगा।
2. भारत म एक ही को एक ही समय म दो या दो से अिधक रा ों का
रा पाल िनयु नहीं िकया जा सकता।
3. रा ों के रा पाल की िनयु भारत के रा पित ारा की जाती है , िक ु
ज ू एवं क ीर के रा पाल की िनयु रा पित नहीं करता है ।
4. रा ों के रा पाल उ ायालय के मु ायमू ित या उसकी
अनु प थित म उस ायालय के उपल व र तम ायाधीश के सम
शपथ या ित ा ले ते ह। शपथ या ित ा का ा प सं िवधान की तीसरी
अनू सूची म िदया गया है ।

उपरो कथनों म से कौन-सा/से सही है /ह?

A) केवल 1

B) केवल 1 और 4

C) केवल 1, 3 और 4

D) 1, 2, 3 और 4
Hide Answer -
उ रः (a)

ा ाः उपयु म से कथन- 2, 3 और 4 गलत ह।

7व सं िवधान सं शोधन अिधिनयम, 1956 के ारा सं िवधान के अनु े द-153 म ावधान

िकया गया िक एक ही को दो या दो से अिधक रा ों का रा पाल िनयु िकया

जा सकता है ।

सं िवधान के अनु े द-155 म उपबं ध है िक रा ों के रा पाल की िनयु भारत के

रा पित ारा की जाएगी। सं िवधान के अनु े द-370 के तहत ज ू -क ीर रा को

िवशे ष रा का दज़ा ा है । 1965 से पहले वहाँ की कायपािलका के धान का चु नाव

रा की िवधानसभा ारा िकया जाता था, िजसे सदर-ए- रयासत के नाम से जाना जाता

था, िक ु 1965 म यह व था समा कर दी गई और सदर-ए- रयासत का नाम

बदलकर रा पाल कर िदया गया, िजसकी िनयु अब भारत के रा पित ारा की जाती

है ।

सं िवधान के अनु े द-159 म उपबं ध है िक रा ों के रा पाल उ ायालय के मु

ायाधीश के सम और उसकी अनु प थित म उस ायालय के उपल े तम

ायाधीश के सम शपथ या ित ा लगे । शपथ या ित ा का ा प इसी अनु ेद म

िदया गया है , न िक सं िवधान की तीसरी अनु सूची म।

सं िवधान के अनु े द-153 म उपबं ध है िक े क रा के िलये एक रा पाल होगा।

[3] भारत के रा पाल के सं दभ म िन िल खत म से कौन-सा कथन सही नहीं है ?

रा पाल िनयु होने के िलये ू नतम आयु 35 वष होनी चािहये तथा उसे
A)
ज से भारत का नाग रक होना चािहये ।

रा पाल अपने पद हण करने की तारीख से पाँ च वष तक पद धारण


B) करे गा। पद की अविध के समा होने के प ात् भी तब तक पद धारण करता
रहे गा, जब तक उसका उ रािधकारी पद हण नहीं कर ले ता।

रा पाल रा पित के सादपयत पद धारण करता है और भारत के सं िवधान


C)
म उसको पद से हटाने हे तु िकसी कार की ि या का उ े ख नहीं है ।

सं िवधान के अनु े द-150 म उपबं ध है िक सं सद के िकसी भी सदन का


कोई सद या रा के िवधानमं डल के िकसी सदन का कोई सद
D)
रा पाल िनयु हो जाता है तो रा पाल के प म पद हण करने की
तारीख से उसकी उस सदन की सद ता समा हो जाएगी।
Hide Answer -
उ रः (a)

ा ाः उपयु म से िवक -(a) सही नहीं है ।

सं िवधान के अनु े द-157 म उपबं ध है िक िकसी के रा पाल िनयु होने के

िलये ू नतम आयु 35 वष होनी चािहये तथा उसे भारत का नाग रक होना चािहये न िक

ज जात नाग रक।

सं िवधान के अनु े द-156(3) म उपबं ध है िक रा पाल पद हण करने की तारीख से

पाँ च वष तक पद धारण करे गा। पद की अविध की समा के प ात भी तब तक पद

धारण करता रहे गा, जब तक उसका उ रािधकारी पद हण न कर ले ।

सं िवधान के अनु े द-156 म उपबं ध है िक रा पाल रा पित के सादपयत पद धारण

करता है । अथात् रा पित कभी भी उसे पद से हटा सकता है या रा पाल रा पित को

अपना ाग-प दे सकता है । सं िवधान म उसे पद से हटाने हे तु िकसी ि या का उ ेख

नहीं िकया गया है ।

सं िवधान के अनु े द-158 म उपबं ध है िक सं सद के िकसी सदन का कोई सद या

रा िवधानमं डल के िकसी सदन का कोई सद रा पाल िनयु हो जाता है तो पद

हण करने की तारीख से वह सदन का सद नहीं रहे गा।

अनु े द-158 म उपबं ध है िक रा पाल को सं िवधान की दू सरी अनु सूची के तहत

उपल याँ , वे तन, भ ों एवं िवशे षािधकार ा होगा और जहाँ दो या दो से अिधक रा ों

का रा पाल िनयु िकया गया हो, वहाँ रा पित के आदे श ारा अवधा रत होगा।

[4] िन िल खत कथनों पर िवचार कीिजएः

1. रा ों के उ ायालय के ायाधीशों की िनयु रा पाल ारा की


जाती है , ठीक वै से ही जै से उ तम ायालय के ायाधीशों की िनयु
रा पित ारा की जाती है ।
2. रा ों के रा पालों के वे तन व भ े सं िवधान की दू सरी अनु सूची के तहत
भारत की सं िचत िनिध पर भा रत होते ह।
3. रा पित उसी को िकसी रा का रा पाल िनयु कर सकेगा,
िजसम लोकसभा या रा िवधानसभा का सद बनने की यो ता होगी।
4. यिद सं बंिधत रा की व थािपका उसे पद से हटाए जाने का ाव
ीकार करती है तो वह पदाविध से पू व भी पदमु िकय जा सकता है ।

उपरो कथनों म से कौन-सा/से स है /ह?

A) केवल 2

B) केवल 1, 2 और 3

C) केवल 2, 3 और 4

D) उपयु म से कोई नहीं।


Hide Answer -
उ रः (d)

ा ाः उपयु म से कोई कथन सही नहीं है ।

रा ों के उ ायालय के ायाधीशों की िनयु रा पित ारा की जाती है न िक

रा पाल के ारा, िक ु शपथ या ित ा रा पाल के सम ले ते ह।

रा ों के रा पालों के वे तन व भ े सं िवधान के दू सरी अनु सूची के तहत रा की सं िचत

िनिध पर भा रत होते ह न िक भारत की सं िचत िनिध पर। वष 2008 म रा पाल का

वे तन ₹ 36,000 से बढ़ाकर ₹ 1.10 लाख ितमाह कर िदया गया है

सं िवधान के अनु े द-157 म उपबं ध है िक रा पित उस को िकसी रा का

रा पाल िनयु कर सकेगा, िजसकी ू नतम आयु 35 वष हो और भारत का नाग रक

हो। अ कोई उपबं ध नहीं है । इसकी िनयु म दो अ पर राएँ जु ड़ी ह;

िजस रा म उसे रा पाल िनयु िकया गया है , वह उस रा से सं बंिधत नहीं होगा।

रा पित रा पाल के िनयु के समय उस रा के मु मं ी से परामश करे ।

सं िवधान म रा पाल को हटाने सं बंधी िकसी ि या का उ े ख नहीं है । वह रा पित के

सादपयत/इ ानु सार पद धारण करता है । रा पित कभी भी उसे पद से हटा सकता है ।

रा पाल रा पित के ित उ रदायी होता है और उसके ितिनिध के प म काय करता

है ।

[5] िन िल खत म से िकसकी िनयु रा के रा पाल ारा नहीं की जाती है ?

A) रा िनवाचन आयु की िनयु ।

B) रा लोक से वा आयोग के अ और सद ों की िनयु ।

C) रा के महािधव ा (Advocate-General of State) की िनयु ।

D) रा िव िव ालय के चां सलर की िनयु ।


Hide Answer -
उ रः (d)

ा ाः उपयु म से िवक (d) सही नहीं है ।

रा के िव िव ालयों का कुलािधपित (चां सलर) रा पाल यं होता है । रा पाल ारा

कुलपित (वाइस चासं लर) की िनयु की जाती है ।

रा पाल ारा िन िल खत की िनयु की जाती है :

1. रा िनवाचन आयु (अनु े द 243 ट)

2. रा लोक से वा आयोग के अ एवं सद (अनु े द-316)

3. रा के महािधव ा (एडवोकेट-जनरल) (अनु े द-165)

4. रा के मु मं ी तथा मु मं ी के सलाह पर अ मं ि यों की िनयु

(अनु े द-164)

5. छ ीसगढ़, म दे श, झारख एवं उड़ीसा म जनजाित क ाण मं ी (194व सं िवधान

सं शोधन अिधिनयम, 2006 के ारा िबहार रा को इस ावधान से बाहर कर िदया

गया।

6. रा िव आयोग की िनयु (अनु े द-243 झ)

7. रा के िजला ायाधीशों की िनयु (अनु े द-233)

[6] िन िल खत म से कौन रा पाल के सादपयत पद धारण करता/करते है /ह?

1. रा के मं ी
2. रा के महािधव ा
3. रा िनवाचन आयु
4. रा लोक से वा आयोग के अ और सद

नीचे िदये गए कूट का योग कर सही उ र का चयन कीिजयेः

A) केवल 1

B) केवल 1 और 2

C) केवल 1, 2 और 3

D) 1, 2, 3 और 4
Hide Answer -
उ रः (b)

ा ाः उपयु म से कथन (3) और (4) गलत ह।

रा िनवाचन आयु की िनयु रा पाल ारा तो की जाती है , िक ु उसे पद से उसी

रीित से हटाया जा सकता है , जै से िकसी उ ायालय के िकसी ायाधीश को हटाया

जाता है ।

रा पाल रा लोक से वा आयोग के अ एवं सद ों की िनयु तो करता है , िक ु

उसे केवल रा पित ही हटा सकता है । रा पाल को हटाने की श नहीं है ।

रा के मु मं ी एवं मु मं ी के सलाह पर अ मं ि यों की िनयु रा पाल ारा की

जाती है और मं ी रा पाल के सादपयत पद धारण करते ह।

रा पाल उ ायालय के ायाधीश िनयु होने के यो िकसी को रा का

महािधव ा िनयु करता है । महािधव ा रा पाल के सादपयत पद धारण करता है ।

[7] रा पाल की श यों के सं दभ म िन िल खत कथनों पर िवचार कीिजये -

1. सं िवधान म यह उपबं ध है िक रा की कायपािलका श रा पाल म


िनिहत होगी और वह इसका योग यं या अपने अधीन थ अिधका रयों
के ारा करे गा।
2. वह सािह , कला, िव ान, सहका रता आ ोलन एवं समाज से वा के े
म ान रखने वालों को िवधान प रषद के िलये मनोनीत कर सकेगा और
यह सं ा िवधान प रषद के कुल सद ों की 1/6 भाग होगी।
3. वह रा की िवधानसभा म दो आं -भारतीय सद ों को मनोनीत कर
सकता है । यिद उसे ऐसा लगे िक रा िवधानसभा म उनका पया
ितिनिध नहीं है ।

उपरो कथनों म से कौन-सा/से कथन सही है /ह?

A) केवल 1

B) केवल 2

C) केवल 1 और 2

D) 1, 2 और 3
Hide Answer -
उ रः (c)

ा ाः उपयु म से कथन (3) गलत है ।

सं िवधान के अनु े द-333 के तहत रा पाल रा की िवधानसभा म एक आं -भारतीय

सद को नािमत कर सकता है । यिद उसे ऐसा लगे िक रा िवधानसभा म आं -

भारतीयों का ितिनिध पया नहीं है , जबिक रा पित लोकसभा के िलये दो आं -

भारतीयों को नािमत कर सकता है ।

सं िवधान के अनु े द-154 म उपबं ध है िक रा की कायपािलका श रा पाल म भी

िनिहत होगी और वह इसका योग सं िवधान के अनु सार यं या अपने अधीन थ

अिधका रयों ारा करे गा।

सं िवधान के अनु े द-171 के तहत रा पाल िवधान प रषद के िलये सािह , कला,

िव ान, समाज से वा एवं सहकारी आ ोलन से सं बंिधत ान रखने वालों को मनोनीत कर

सकेगा। यह सं ा िवधान प रषद की कुल सद सं ा का 1/6 भाग होगी।

[8] रा पाल के अ ादे श के सं दभ म िन िल खत कथनों पर िवचार कीिजयेः

1. सं िवधान के अनु े द-123 के तहत रा के रा पाल को आ ादे श जारी


करने की श है ।
2. रा पाल िकसी भी समय अ ादे श को वापस ले सकता है ।
3. छः माह की समा पर अ ादे श वतन म नहीं रहे गा, यिद उससे पहले
रा के िवधानमं डल ारा ीकृित न िमली हो।

उपयु म से कौन-सा/से कथन सही है /ह?

A) केवल 2

B) केवल 1 और 2

C) केवल 2 और 3

D) 1, 2 और 3

Hide Answer -
उ रः (a)

ा ाः उपयु म से कथन- (1) और (3) गलत ह।

रा पाल को सं िवधान के अनु े द-213 के तहत अ ादे श जारी करने की श है ।

रा पाल के अ ादे श की समय-सीमा छः स ाह की है (न िक 6 माह)। यिद छः स ाह

के भीतर रा के िवधानमं डल ारा ीकृित न दी तो अ ादे श वतन म नहीं रहे गा।

रा पाल िकसी भी समय अ ादे श को वापस ले सकता है ।

[9] िन िल खत कथनों पर िवचार कीिजये -

1. िवधायी श के सं बंध म रा पाल रा िवधानमं डल का अं ग होता है ,


उसी कार जै से रा पित सं सद का अं ग होता है ।
2. रा पाल के पास आपातकालीन श नहीं है ।
3. मादान श के मामले म रा पाल एवं रा पित की श एक समान
ह।

उपयु म से कौन-सा/से कथन सही है /ह?

A) केवल 1

B) केवल 1 और 2

C) केवल 2 और 3

D) 1, 2 और 3

Hide Answer -
उ रः (b)

ा ाः उपयु म से कथन-3 गलत है ।

सं िवधान के अनु े द-72 के तहत रा पित को मादान की श है , जबिक रा पाल को

अनु े द-161 के तहत मादान की श है । रा पित मृ ु द को मा कर सकता है ,

िक ु रा पाल को यह श ा नहीं है । अ सभी मामलों म रा पाल की श

रा पित के समान है ।

िवधायी श के सं बंध म रा पाल िवधानमं डल का अं ग होता है उसी कार जै से रा पित

सं सद का अं ग होता है ।

आपातकालीन श रा पाल को नहीं ा है । ये श भारत के रा पित म िनिहत है ।

[10]िकसी रा के रा पाल की िवधायी श यों के सं दभ म िन िल खत कथनों पर


िवचार कीिजयेः

1. रा पाल रा िवधानसभा को आ त कर सकता है , स ावसान कर


सकता है , और भं ग कर सकता है ।
2. रा पाल रा िवधानसभा की बै ठकों को थिगत कर सकता है ।
3. रा पाल नविनवािचत िवधानसभा के थम स म अिभभाषण करता है ।
4. रा पाल वािषक बजट को रा िवधानसभा के सम रखवाता है ।

उपयु म से कौन-से कथन सही ह?

A) केवल 1 और 3

B) केवल 1, 3 और 4

C) केवल 2, 3 और 4

D) 1, 2, 3 और 4
Hide Answer -
उ रः (b)

ा ाः उपयु म से कथन-2 सही नहीं है ।

रा की िवधानसभा को थिगत करने की श िवधानसभा के अ के पास होती है ।

यह थगन घं टों, िदनों या स ाह का हो सकता है या िवधानसभा को अिनि तकाल के िलये

थिगत कर सकता है ।

रा िवधानमं डल के े क सदन की (िजस रा म िवधान प रषद है ) रा पाल समय-

समय पर बै ठक आ त कर सकता है । दो स ों के बीच छः माह से अिधक का समय

अं तराल नहीं होना चािहये । रा िवधानमं डल को एक वष म कम-से -कम दो बार ज़ र

िमलना चािहये । एक स म िवधानमं डल की कई बठक हो सकती ह। [अनु े द-174

(1)]

रा पाल समय-समय पर सदन या िकसी एक सदन का स ावसान कर सकता है एवं

िवधानसभा का िवघटन कर सकता। [अनु े द-174 (2)]

िवधान प रषद (िजस रा म है ) एक थायी सदन है , उसका िवघटन नहीं हो सकता है ।

रा पाल े क साधारण िनवाचन के प ात् थम स के आरं भ म और े क वष के

थम स के ारं भ म िवधानसभा म अिभभाषण करे गा और िवधान प रषद वाले रा की

दशा म एक साथ समवे त दोनों सदनों म अिभभाषण करे गा और िवधानमं डल को उसके

बु लाने का कारण बताएगा। [अनु े द-176 (1)]

रा पाल िवधानसभा म या िवधान प रषद वाले रा की दशा म उस रा के िवधानमं डल

के िकसी एक सदन म या एक साथ समवे त दोनों सदनों म अिभभाषण कर सकेगा और

इस उ े के िलये सद ों की उप थित की अपे ा कर सकेगा। [अनु े द-175 (1)]

रा पाल, रा के िवधानमं डल म उस समय लं िबत िकसी िवधे यक के सं बंध म सं देश उस

रा के िवधानमं डल के सदन या सदनों को भे ज सकेगा और िजस सदन को कोई सं देश

भे जा गया है उस पर शी ता से िवचार करे गा। [अनु े द-175 (2)]

[11]रा का रा पाल िन िल खत म से कौन-सी प र थित म अ ादे श जारी कर


सकता है ?

A) जब के सरकार उसे ऐसा करने के िलये कहे ।

B) जब रा म कानू न एवं व था की सम ा हो।

जब भी रा की िवधानसभा स म न हो और रा पाल सं तु हो िक ऐसी


C)
प र थितयाँ िव मान ह, िजसके कारण तु र कारवाई आव क है ।

D) जब भी ायपािलका ऐसा करने की सलाह दे ।


Hide Answer -
उ रः (c)

ा ाः उपयु म से िवक - (c) सही है ।

सं िवधान के अनु े द-213 के तहत िकसी रा का रा पाल अ ादे श जारी कर सकता

है , यिद रा पाल को िकसी समय यह समाधान हो जाता है िक ऐसी प र थितयाँ िव मान

ह, िजसके कारण तु र कारवाई करना उनके िलये आव क हो गया है । यह अ ादे श

तब जारी नहीं िकया जा सकता है , जब रा की िवधानसभा स म है या िवधान प रषद

वाले रा म िवधानमं डल के दोनों सदन स म ह।

[12]िकसी रा का रा पाल िन िल खत म से िकन ितवे दनों को रा की


िवधानसभा के सम ु त करता है ः-

1. रा िव आयोग के ितवे दन।


2. रा लोक से वा आयोग के ितवे दन।
3. िनयं क एवं महोलखा परी क के ितवे दन।

नीचे िदये गए कूट का योग कर सही उ र का चयन कीिजयेः

A) केवल 1

B) केवल 2

C) केवल 1 और 2

D) 1, 2 और 3

Hide Answer -
उ रः (d)

ा ाः उपयु सभी कथन सही ह।

सं िवधान के अनु े द-243 (क) म रा िव आयोग के सं बंध म उपबं ध िकया गया है ।

इसकी िनयु रा पाल ारा की जाती है और रा पाल रा िव आयोग की रपोट को

रा के िवधानमं डल के सम रखवाएगा।

सं िवधान के अनु े द-315 म उपबं ध िकया गया है िक े क रा के िलये एक लोक

से वा आयोग होगा और सं िवधान के अनु े द-323(2) के तहत रा पाल, रा लोक से वा

आयोग के ितवे दनों को रा के िवधानमं डल के सम रखवाएगा।

सं िवधान के अनु े द 148 म िनयं क एवं महाले खा परी क का उपबं ध है और

अनु े द-151(2) के तहत रा पाल, िनयं क एवं महाले खा परी क के ितवे दनों को

रा के िवधानमं डल के सम रखवाएगा।

[13]रा के गवनर ारा एक िवधे यक रा पित के िवचाराथ आरि त िकया जाता है ,


तब रा पितः

1. िवधे यक पर अनु मित दे सकता है ।


2. िवधे यक पर अनु मित रोक सकता है
3. रा पाल को िनदश दे गा िक िवधे यक को रा के िवधानमं डल के सदन
या सदनों को पु निवचार के िलये लौटा दे , िक ु सदन या सदनों ारा
सं शोधन या िबना सं शोधन के पा रत कर रा पित के सम पु नः ुत
िकया जाता है तो रा पित अनु मित दे ने के िलये बा है ।

उपयु म से कौन-सा/से कथन सही है /ह?

A) केवल 1

B) केवल 1 और 2

C) केवल 1 और 3

D) 1, 2 और 3

Hide Answer -
उ रः (b)

ा ाः उपयु म से िवक -3 सही नहीं है ।

रा पाल, सं िवधान के अनु े द-201 के तहत रा के िवधानमं डल के ारा पा रत

िवधे यक को रा पित के िलये आरि त रखता है और इस सदं भ म रा पितः

1. िवधे यक पर अपनी ीकृित दे सकता है , या

2. िवधे यक पर अपनी ीकृित रोक सकता है , या

3. रा पाल को िनदश दे गा िक िवधे यक को रा के िवधानमं डल के सदन या सदनों को

पु निवचार के िलये लौटा दे , िक ु सदन या सदनों ारा सं शोधन या िबना सं शोधन के पा रत

कर पु नः रा पित के सम ु त िकया जाता है तो भी रा पित अनु मित दे ने के िलये बा

नहीं है ।

सदन ारा 6 माह के भीतर इस पर पु निवचार करना आव क है ।

धन िवधे यक के सं बंध म रा पित के पास दो िवक होते ह;

1. िवधे यक को अनु मित दे दे या

2. िवधे यक पर अनु मित रोक ले , िक ु िवधे यक को पु निवचार के िलये वापस नहीं लौटा

सकता है ।

[14]रा िवधानमं डल ारा पा रत िवधे यक, उसके िविध बन जाने पर, रा पाल की
राय म उ ायालय की श यों का अ ीकरण होगा, ऐसी थित म रा पालः

A) िवधे यक पर ीकृित नहीं दे गा।

B) िवधये क पर ीकृित दे दे गा।

C) िवधे यक को पु निवचार के िलये रा के िवधानमं डल को वापस लौटा दे गा।

D) िवधे यक को रा पित के िवचार के िलये आरि त नहीं रखे गा।


Hide Answer -
उ रः (a)

ा ाः उपयु म से िवक - (a) सही है ।

सं िवधान के अनु े द-200 के तहत रा िवधानमं डल ारा पा रत िवधे यक, उसके िविध

बन जाने पर, रा पाल की राय म, उ ायालय की श यों का अ ीकरण होगा, तो

रा पाल िवधे यक पर अनु मित नहीं दे गा और उसे रा पित के िलये आरि त रखे गा।

सं िवधान के अनु े द-200 के तहत रा के िवधानमं डल के ारा पा रत िवधे यक के सं बंध

म रा पाल िन कदम उठा सकते ह-

1.िवधे यक पर ीकृित दे दे ।

2. िवधे यक पर ीकृित रोक दे ।

3. िवधे यक को पु निवचार के िलये रा के िवधानमं डल को वापस भे ज दे , िक ु रा के

िवधानमं डल ारा सं शोधन या िबना सं शोधन के रा पाल के पास पु नः ु त िकया जाता

है तो रा पाल उस पर अनु मित दे ने के िलये बा है ।

4. रा पाल िवधे यक को रा पित के िलये सु रि त रख सकता है ।

धन िवधे यक के सं बंध म रा पाल िवधे यक को पु निवचार के िलये रा के िवधानमं डल के

पास वापस नहीं भे ज सकता है । उसके पास उपयु 1, 2 और 4 िवक बचते ह।

[15]रा पाल के सं दभ म िन िनखत कथनों पर िवचार कीिजयेः

1. रा पित अ ादे श का योग मं ि प रषद की सलाह पर करता है , जबिक


रा पाल के अ ादे श की श ै क है ।
2. कोट माशल (सै अदालत) के तहत सज़ा ा यों की सज़ा को
रा पित सं िवधान के अनु े द-72 के तहत और रा पाल सं िवधान के
अनु े द-161 के तहत माफ कर सकता है ।

उपयु म से कौन-सा/से कथन सही है /ह?

A) केवल 1

B) केवल 2

C) 1 और 2 दोनों

D) न तो 1 और न ही 2
Hide Answer -
उ रः (d)

ा ाः उपयु दोनों कथन गलत ह।

रा पित सं िवधान के अनु े द-123 के तहत और रा पाल सं िवधान के अनु े द-213 के

तहत अ ादे श जारी कर सकते ह, िक ु दोनों इस अ ादे श का योग अपनी-अपनी

मं ि प रषद के परामश पर ही कर सकते ह। अथात उनकी अ ादे श की श ै क

नहीं है ।

रा पित को सं िवधान के अनु े द-72 के तहत और रा पाल को सं िवधान के अनु ेद

161 के तहत मादान की श है । रा पित कोट माशल (सै अदालत) के तहत सज़ा

ा यों की सज़ा को माफ कर सकता ह िक ु रा पाल को यह श ा नहीं

है ।

रा पित मृ ु -द ा सज़ा को मा कर सकता है , कम कर सकता है या थगित कर

सकता है या बदल सकता है , पर ु रा पाल को मृ ु -द की सज़ा को मा करने की

श नहीं है , पर ु उसे थिगत कर सकता है या पु निवचार के िलये कह सकता है ।

[16]रा ों के रा पाल के िववे काधीन श यों के सं दभ म िन िल खत कथनों पर


िवचार कीिजयेः

1. मादान की श ।
2. अ ादे श ािपत करने की श ।
3. रा पित के िवचाराथ िकसी िवधे यक को आरि त करना।
4. रा ों म रा पित शासन की िसफा रश करना।

उपयु म से कौन-से कथन सही ह?

A) केवल 1, 2 और 3

B) केवल 3 और 4

C) केवल 1, 3 और 4

D) 1, 2, 3 और 4
Hide Answer -
उ रः (b)

ा ाः उपयु म से कथन-1 और 2 गलत ह।

रा पाल सं िवधान के अनु े द-161 के तहत मादान की श एवं अनु े द-213 के

तहत अ ादे श की श का योग मं ि प रषद के सलाह पर करता है ।

रा पाल को िन िल खत मामलों म िववे क की श है ः

रा पित के िवचार के िलये िकसी िवधे यक को आरि त करना। (अनु े द-200)

रा ों म रा पित शासन की िसफा रश करना। (अनु े द-356)

रा के िवधान िवषयों एवं शासिनक मामलों म मु मं ी से जानकारी ा करना।

सं िवधान के अनु े द-239 (2) के तहत पड़ोसी के -शािसत रा ों म शासक

(अित र कार की थित म) के प म काय करते समय।

रा पाल रा के सं वैधािनक मु ख के साथ-साथ रा पित के ितिनिध के प म भी

काय करता है । इसिलये रा पित ारा सौंपे गए काय को रा पाल िववे काधीन के तहत

करता है ।

रा पाल के कुछ िववे काधीन काय प र थितवश उ होते ह:

िवधानसभा चु नाव म िकसी भी दल को पू ण ब मत न िमलने की थित म या कायकाल के

दौरान अचानक मु मं ी की मृ ु हो जाने पर या उसके िनि त उ रािधकारी न होने पर

मु मं ी िनयु के मामले म।

रा िवधानसभा म िव ास मत हािसल न करने पर मं ि प रषद की बखा गी के मामले

म।

मं ि प रषद के अ मत म आने पर रा िवधानमं डल का िवघटन करना।

[17]रा पाल के सं दभ म िन िल खत म से कौन-सा कथन सही नहीं है ?

A) रा पाल की िववे काधीन श की चचा सं िवधान म कहीं नहीं की गई है ।

B) िववे काधीन श की सं बंध म रा पाल का िनणय अ म होता है ।

रा पाल की िववे काधीन श के सं बंध म िनणय को िकसी ायालय म


C)
गत नहीं िकया जा सकता है ।

रा पाल रा का सं वैधािनक मु ख होने के साथ-साथ रा पित का ितिनिध


D)
भी होता है ।
Hide Answer -
उ रः (a)

ा ाः उपयु म से िवक -(a) सही नहीं है ।

रा पाल को िववे काधीन की श दो कार से ा है ः (1) सं िवधान म इसके िलये

उपबं ध है और (2) प र थितवश उ थित की अनु सार। सं िवधान के

अनु े द-163(2) म रा पाल के िववे काधीन श के सं बंध म चचा की गई है । इस

सं बंध म रा पाल का िनणय अ म होता है । और उसके िनणय को िकसी ायालय म

गत नहीं िकया जा सकता।

रा पाल सं वैधािनक मु खया होने के साथ-साथ के (रा पित) का ितिनिध भी होता है ।

रा पित रा पाल को कुछ िवशे ष उ रदािय ों के िनवहन के िलये समय-समय पर िनदश

दे ता है और रा पाल उस िनदश के अनु सार काय करता है । ऐसे मामलों म रा पाल

रा के मं ि प रषद से सलाह ले ते ह, िक ु अ म िनणय उसका गत िनणय होता

है ।

[18]रा पाल के चयन के िवषय पर सरका रया आयोग ने िन िल खत म से कौन-सा


एक सु झाव नहीं िदया है ?

A) के म स ाधारी दल के िकसी राजनियक की िनयु नहीं होगी।

ऐसे िवयोिजत िजसकी रा की थानीय राजनीित से गहरी घिन ता न


B)
हो, की िनयु नहीं होगी।

C) रा पाल के िनयु की सं बंध म मु मं ी से परामश करना अिनवाय होना।

D) रा पाल नामधारी शासक रहे न िक के का अिभक ा या ितिनिध।

Hide Answer -
उ रः (b)

ा ाः उपयु म से िवक - (b) सही नहीं है ।

के - रा सं बंधों की ा ा करने हे तु सन् 1983 म रं जीत िसं ह सरका रया की

अ ता म एक आयोग का गठन िकया गया, िजसे सरका रया आयोग के नाम से जाना

जाता है । इसने जनवरी 1988 म अपनी रपोट सौंपी। इस रपोट म रा पाल के चयन के

िवषय म िन सु झाव िदयेः

रा पाल िजस रा म िनयु िकया गया हो उस रा से नहीं होना चािहये ।

रा पाल की िनयु के सं बंध म रा पित उस रा के मु मं ी से परामश करे गा।

के म स ाधारी दल के िकसी राजनियक की िनयु रा पाल के प म नहीं की

जाएगी।

रा पाल नामधारी शासक रहे न िक के का अिभक ा या ितिनिध।

[19]िन िल खत कथनों पर िवचार कीिजये -

सं सद िविध ारा िकसी रा के रा ापाल को िकसी िनकटवत सं घ रा


े का शासक िनयु कर सकती है ।
यिद कोई रा का रा पाल िनकटवत सं घ रा के शासक के प म
काय करने हे तु िनयु होता है तो वह ऐसे शासक के प म अपने
क ों का योग अपनी मं ि प रषद से तं रहते ए करे गा।

उपयु कथनों म से कौन-सा/से कथन सही है /ह?

A) केवल 1

B) केवल 2

C) 1 और 2 दोनों

D) न तो 1 और न ही 2

Hide Answer -
उ रः (b)

ा ाः उपयु म से कथन-1 गलत है ।

सं िवधान के अनु े द-239 (2) म उपबं ध है िक रा पित िकसी रा के रा पाल को

िकसी िनकटवत सं घ रा े के शासक के प म िनयु कर सकेगा, न िक सं सद

िविध ारा िनयु करे गी।

वहाँ रा पाल ऐसे शासक के प म अपने काय का योग अपनी मं ि प रषद से तं

प म करे गा।

[20]सं िवधान के मसौदे म िनवािचत रा पालों के ावधानों की मू ल योजना को छोड़


िदया गया, ोिकः

1. इसका ता य होगा दू सरा िनवाचन।


2. िनवाचन मु ख राजनीितक मु ों पर लड़ा जाता है ।
3. िनवािचत रा पाल अपने को मु मं ी से बड़ा समझता।
4. रा पाल को सं सदीय णाली के अधीन ही काय करना था।

नीचे िदये गये कूट का योग कर सही उ र का चयन कीिजयेः

A) केवल 1 और 3

B) केवल 1, 2 और 3

C) केवल, 1, 3 और 4

D) 1, 2, 3 और 4
Hide Answer -
उ रः (c)

ा ाः उपयु म से कथन-2 गलत है ।

ा प सं िवधान म योजना थी िक रा पाल िनवािचत हो, िक ु सं िवधान सभा म िनवाचन

के थान पर रा पित ारा िनयु की व था की गई, ोंिक:

इसका ता य होगा दू सरा िनवाचन।

िनवाचन के मु े िब ुल गत होंगे, मतदाताओं की सं ा ब त अिधक होगी। इसम

जो क और य होगा, उसकी तु लना म फायदा कुछ नहीं होगा।

िनवािचत रा पाल अपने को मु मं ी से बड़ा समझे गा। अतः मु मं ी एवं रा पाल के

बीच सं घष की थित पै दा हो सकती है ।

रा पाल को सं सदीय णाली के अ गत ही काय करना था और रा पाल का सीधा

िनवाचन सं सदीय णाली के िव हो सकता है ।

रा पाल िसफ सं वैधािनक मु ख होता है ।

रा पाल के िनवाचन से अलगाववाद को बढ़ावा िमले गा।

रा पित ारा िनयु के मा म से सं घ सरकार, रा पर िनयं ण बनाए रख सकेगी।

एक िनवािचत रा पाल भािवक प से िकसी राजनीितक दल से जु ड़ा होगा और वह

िन एवं िनः ाथ मु खया नहीं बन पाएगा।

रा पाल का सीधा िनवाचन रा म आम चु नाव के समय एक गं भीर सम ा उ कर

सकता है ।
[1] िन िल खत म से कौन-सा कथन सही है ?

भारत के सं िवधान के भाग-6 म रा सरकार के सं बंध म उपबं ध है । यह


A)
उपबं ध भारत के सभी रा ों पर लागू होता है ।

B) कोई भी एक से अिधक बार रा पाल नहीं बन सकता।

C) तं भारत की पहली मिहला रा पाल सु चेता कृपलानी थी।

D) ितवष 13 फरवरी को भारत म मिहला िदवस मनाया जाता है ।

Hide Answer -
उ रः (d)

ा ाः उपयु म से िवक -(d) सही है ।

भारत म ितवष 13 फरवरी को भारत की थम रा पाल सरोजनी नायडू की ृ ित म

मिहला िदवस मनाया जाता है ।

भारत के सं िवधान के भाग-6 म रा सरकार के सं बंध म उपबं ध िकया गया है । यह

उपबं ध ज ू -क ीर रा को छोड़कर भारत के सभी रा ों पर लागू होता है । सं िवधान

के अनु े द 370 के तहत ज ू -क ीर रा को िवशे ष दज़ा िदया गया है । उसका अपना

अलग सं िवधान है ।

कोई भी एक से अिधक बार भारत के िकसी रा का रा पाल बन सकता है ।

तं भारत की पहली मिहला रा पाल या भारत की थम मिहला रा पाल सरोजनी

नायडू थी।

[2] रा के मु मं ी के सं दभ म िन िल खत म से कौन-सा कथन सही है ?

A) रा का मु मं ी रा एवं सरकार का मु खया होता है ।

सं िवधान म मु मं ी की िनयु और िनवाचन के िलये िवशे ष ि या की


B)
चचा की गई है ।

मु मं ी की िनयु रा पाल के ारा की जाती है और वह उसके


C)
सादपयत पद धारण करता है ।

मु मं ी िनयु होने के िलये िवधानमं डल के दोनों सदनों म से (यिद िकसी


D) रा म दू सरा सदन है ) िकसी एक सदन का सद होना अिनवाय है ।
Hide Answer -
उ रः (c)

ा ाः उपयु म से िवक (c) सही है ।

सं िवधान के अनु े द-164(1) के अनु सार मु मं ी की िनयु रा पाल ारा तथा अ

मं ि यों की िनयु मु मं ी से सलाह पर रा पाल करता है । मु मं ी तथा अ मं ी

रा पाल के सादपयत ही पद धारण करते ह।

सं िवधान म मु मं ी की िनयु एवं िनवाचन के िलये कोई िवशे ष ि या नहीं है ।

रा का रा पाल रा का एवं रा का मु मं ी सरकार का मु खया होता है ।

रा पाल िकसी ऐसे को मु मं ी िनयु कर सकता है , जो िवधानमं डल के िकसी

सदन का सद न हो, िकंतु छः माह के अ र िवधानमं डल के िकसी एक सदन की

सद ता अिनवाय है , अ था 6 माह प ात् वह मु मं ी नहीं रहे गा।

[3] सं िवधान के अनु े द-167 के अनु सार े क रा के मु मं ी का यह क


होगा िक वह-

1. रा के काय एवं शासन सं बंधी और िवधान िवषयक थापनाओं सं बंधी


मं ि प रषद के सभी िनणयों से रा पाल को सू िचत करे ।
2. रा के काय एवं शासन सं बंधी और िवधान िवषयक थापनाओं सं बंधी
जो जानकारी रा पाल मां गे, वह दे ।
3. िकसी िवषय को िजस पर िकसी मं ी ने िनणय कर िदया है , िकंतु
मं ि प रषद ने िवचार नहीं िकया है , रा पाल ारा अपे ा िकये जाने पर
मं ि प रषद के सम िवचार के िलये रखे ।

उपयु म से कौन-से कथन सही ह?

A) केवल 1 और 2

B) केवल 1 और 3

C) केवल 2 और 3

D) 1, 2 और 3
Hide Answer -
उ रः (d)

ा ाः उपयु सभी कथन सही ह।

मु मं ी, रा पाल एवं मं ि प रषद के बीच कड़ी का काम करता है ।

सं िवधान के अनु े द-167 म रा पाल के ित मु मं ी के क ों के सं बंध म उपबं ध

है । इस अनु े द के अनु सार े क रा के मु मं ी का यह क होगा िक वहः

रा के काय एवं शासन सं बंधी और िवधान िवषयक थापनाओं सं बंधी मं ि प रषद के

सभी िनणयों से रा पाल को सू िचत करे ।

रा के काय एवं शासन सं बंधी एवं िवधान िवषयक थापनाओं सं बंधी जो जानकारी

रा पाल मां गे, वह दे ।

िकसी िवषय को िजस पर िकसी मं ी ने िनणय कर िदया है , िकंतु मं ि प रषद ने िवचार नहीं

िकया है , रा पाल ारा अपे ि त िकये जाने पर मं ि प रषद के सम िवचार के िलये रखे ।

[4] सं िवधान के अनु े द-163 म िकसका उपबं ध िकया गया है ?

रा पाल को अपने काय का योग करने म सहायता एवं सलाह दे ने के िलये


A)
एक मं ि प रषद होगी, िजसका धान मु मं ी होगा।

मु मं ी की िनयु रा पाल करे गा और अ मं ि यों की िनयु


B) रा पाल, मु मं ी की सलाह पर करे गा तथा मं ी, रा पाल के सादपयत
पद धारण करगे ।

मु मं ी सिहत मं ि यों की कुल सं ा उस रा की िवधानसभा के सद ों


C)
की कुल सं ा के 15% से अिधक नहीं होगी।

उ ायालय का ायाधीश िनयु होने के यो िकसी को


D)
रा पाल रा का महािधव ा िनयु करे गा।
Hide Answer -
उ रः (a)

ा ाः उपयु म से िवक (a) सही है ।

सं िवधान के अनु े द-163 म िन िल खत उपबं ध िकये गए ह:

रा पाल की िववे काधीन श को छोड़कर रा पाल को सहायता एवं सलाह दे ने के िलये

एक मं ि प रषद होगी, िजसका धान मु मं ी होगा।

रा पाल के िववे काधीन काय के सं बंध म उसका िनणय अ म होगा और उसे कहीं

गत नहीं िकया जा सकता।

मं ि यों ारा रा पाल को दी गई सलाह की िकसी ायालय म जाँ च नहीं की जा सकती।

सं िवधान के अनु े द-164 म िन िल खत उपबं ध िकये गए ह-

मु मं ी की िनयु रा पाल करे गा और अ मं ि यों की िनयु रा पाल मु मं ी

की सलाह पर करे गा तथा मं ी, रा पाल के सादपयत पद धारण करगे ।

रा की मं ि प रषद म मु मं ी सिहत मं ि यों की कुल सं ा रा की िवधानसभा के

सद ों की कुल सं ा के 15% से अिधक नहीं होगी, परं तु मु मं ी सिहत मं ि यों की

कुल सं ा 12 से कम नहीं होगी। इसे 91व सं िवधान सं शोधन अिधिनयम, 2003 के ारा

जोड़ा गया।

सं िवधान के अनु े द-165 म रा के महािधव ा की िनयु के सं बंध म उपबं ध है ।

[5] िन िल खत म से िकस- िकस का अ रा का मु मं ी होता है ?

1. रा योजना बोड
2. े ीय प रषद
3. अं तरा ीय प रषद
4. रा ीय िवकास प रषद

नीचे िदये गए कूट का योग कर सही उ र का चयन कीिजयेः

A) केवल 1

B) केवल 1 और 2

C) केवल 1, 2 और 3

D) 1, 2, 3 और 4
Hide Answer -
उ रः (a)

ा ाः उपयु म से कथन (2), (3) और (4) गलत ह।

े ीय प रषदः सभी े ीय प रषदों का अ क ीय गृ हमं ी होता है । मु मं ी सं बंिधत

े ीय प रषद के मवार उपा के प म काय करता है । एक समय म इसका

कायकाल एक वष का होता है ।

अं तरा ीय प रषदः इसका अ धानमं ी होता है । मु मं ी इसका सद होता है ।

रा ीय िवकास प रषदः इसका अ भारत का धानमं ी होता है । मु मं ी इसका

सद होता है ।

रा योजना बोडः इसका अ मु मं ी होता है ।

[6] रा के उप-मु मं ी के सं दभ म िन िल खत कथनों पर िवचार कीिजयेः

1. उप-मु मं ी की िनयु सामा तया थानीय राजनीितक कारणों से की


जाती है ।
2. उप-मु मं ी कभी भी मं ि प रषद का सद नहीं हो सकता है ।

उपयु म से कौन-सा/से कथन सही है /ह?

A) केवल 1

B) केवल 2

C) 1 और 2 दोनों

D) न तो 1 और न ही 2

Hide Answer -
उ रः (a)

ा ाः उपयु म से कथन (2) गलत है ।

उप-मु मं ी को मं ि प रषद म शािमल िकया जा सकता है ।

उप-मु मं ी की िनयु सामा तया थानीय राजनीितक कारणों से की जाती है ।

रा मं ि प रषद के तीन वग ह-

► कैिबने ट मं ी

► रा मं ी

► उपमं ी

[7] िन िल खत म से कौन-सा कथन सही नहीं है ?


सं िवधान म उपबं ध है िक रा के े क मं ी एवं रा के महािधव ा को
रा की िवधानसभा व िवधान प रषद (जहाँ दोनों सदन ह) दोनों सदनों म
A)
और उसके िकसी सिमित म, िजसका वह सद है , बोलने , उसकी कायवाही
म भाग ले ने और मत दे ने का अिधकार है ।

सु चेता कृपलानी भारत की पहली मिहला मु मं ी थी। वह उ र दे श की


B)
मु मं ी थी।

ज ू -क ीर का मु मं ी अपने पद पर छः वष की अविध तक रह सकता


C)
है ।

6ठे सं िवधान सं शोधन अिधिनयम, 1965 के ारा ज ू -क ीर के कायकारी


D)
अ का पदनाम सदर-ए- रयासत से बदलकर रा पाल कर िदया गया।

Hide Answer -
उ रः (a)

ा ाः उपयु म से िवक (a) सही नही ं है ।

सं िवधान के अनु े द-177 के तहत े क मं ी या रा के महािधव ा को रा की

िवधानसभा या िवधान प रषद वाले रा की दशा म, दोनों सदनों म और उसके िकसी सिमित म,

िजसम उसका नाम सद के प म िदया गया है , बोलने और उसकी कायवाही म भाग ले ने का

अिधकार है , िकंतु वे मत नहीं दे सकते ।

सु चेता कृपलानी वष 1963 म उ र दे श की मु मं ी बनी थी। वह भारत की बनने वाली

पहली मिहला मु मं ी थी।

मु मं ी का कायकाल िनि त नहीं होता है । वह िवधानसभा की कायिविध तक पद पर बना

रहता है । चूं िक ज ू -क ीर िवधानसभा की कायाविध छः वष है , इसिलये वहाँ का मु मं ी छः

वष तक रहता है ।

6ठे सं िवधान सं शोधन अिधिनयम, 1965 के ारा ज ू -क ीर के कायकारी अ सदर-ए-

रयासत का पदनाम बदलकर ‘रा पाल’ कर िदया गया और धानमं ी का पदनाम बदलकर

मु मं ी कर िदया गया।

[8] सं िवधान के अनु े द-164 म छ ीसगढ़, झारखं ड, म दे श और उडीसा म


जनजाितयों के क ाण हे तु एक पृ थक मं ी का उपबं ध है । िन िल खत म से िकस
सं िवधान सं शोधन अिधिनयम के ारा िबहार को इस दािय से मु कर
छ ीसगढ़ एवं झारखं ड के िलये ऐसा करना आव क बना िदया गया?

A) 91व सं िवधान सं शोधन अिधिनयम, 2003

B) 92व सं िवधान सं शोधन अिधिनयम, 2003

C) 93व सं िवधान सं शोधन अिधिनयम, 2006

D) 94व सं िवधान सं शोधन अिधिनयम, 2006


Hide Answer -
उ रः (d)

ा ाः उपयु म से िवक (d) सही है ।

सं िवधान के अनु े द-164(1) म छ ीसगढ़, झारखं ड, म दे श और उडीसा म

जनजाितयों के क ाण के िलये एक पृ थक मं ी की व था की गई है । ारं भ म यह

व था िबहार, म दे श और ओिड़शा के िलये थी, िकंतु 94व सं िवधान सं शोधन

अिधिनयम, 2006 के ारा िबहार को इस दािय से मु कर िदया गया और छ ीसगढ़

एवं झारखं ड के िलये ऐसा करना आव क बना िदया गया।

91व सं िवधान सं शोधन अिधिनयम, 2003 के ारा दल-बदल अिधिनयम को सश

बनाया गया तथा मं ि प रषद म मं ि यों की कुल सं ा सीिमत की गई।

92व सं िवधान सं शोधन अिधिनयम, 2003 के ारा आठवीं अनु सूची म चार नई भाषाएँ

जोड़ी गईं ।

93व सं िवधान सं शोधन अिधिनयम, 2006 के ारा िनजी िश ण सं थानों म थानों के

आर ण के सं बंध म उपबं ध िकया गया ।

[9] रा के िवधानमं डल के सं दभ म िन िल खत कथनों पर िवचार कीिजयेः

1. रा पाल, रा के िवधानमं डल का अिभ अं ग नहीं होता, जबिक रा पित


सं सद का अिभ अं ग होता है ।
2. रा की िवधान प रषद का सृ जन या उ ादन करने की श रा की
िवधानसभा म िनिहत है ।

उपयु म से कौन-सा/से कथन सही है /ह?

A) केवल 1

B) केवल 2

C) 1 और 2 दोनों

D) न तो 1 और न ही 2

Hide Answer -
उ रः (d)

ा ाः उपरो दोनों कथन गलत ह।

रा का िवधानमं डल िवधानसभा, िवधान प रषद वाले रा की दशा म िवधान प रषद और

रा पाल से िमलकर बनता है , जै से सं सद, लोकसभा, रा सभा और रा पित से िमलकर बनती

है । अतः रा पाल िवधानमं डल का अिभ अं ग होता है ।

िजस रा म िवधान प रषद नहीं है , उस रा म उसका सृ जन और िजस रा म िवधान

प रषद है उस रा म उसका उ ादन करने की श सं सद म िनिहत है न िक रा की

िवधानसभा म।

[10]
रा ों म िवधान प रषद का सृ जन या उ ादन के सं दभ म िन िल खत म से
कौन-सा कथन सही है ?

यिद रा की िवधानसभा अपनी कुल सद सं ा के ब मत से या उप थत


या मत दे ने वाले सद ों की सं ा के कम-से -कम दो-ितहाई ब मत ारा
A)
सं क पा रत करे , तो सं सद िविध ारा िकसी रा म इस कार की व था
कर सकती है ।

यिद रा िवधानमं डल अपनी कुल सद सं ा के ब मत से या उप थत


एवं मत दे ने वाले सद ों की कुल सं ा के कम-से -कम एक-ितहाई ब मत
B)
ारा सं क पा रत करे तो सं सद िविध ारा िकसी रा म इस कार की
व था कर सकती है ।

यिद रा िवधानमं डल अपनी कुल सद सं ा के ब मत या उप थत और


मत दे ने वाले सद ों की सं ा के कम-से -कम तीन-चौथाई ब मत ारा
C)
सं क पा रत करे तो सं सद िविध ारा िकसी रा म इस कार की व था
कर सकती है ।

यिद रा िवधानमं डल अपनी कुल सद सं ा के ब मत या उप थत एवं


D) मत दे ने वाले सद ों की सं ा के ब मत ारा सं क पा रत करे तो सं सद
िविध ारा िकसी रा म इस कार की व था कर सकती है ।

Hide Answer -
उ रः (a)

ा ाः उपयु म से िवक (a) सही है ।

सं िवधान के अनु े द-169 म िकसी रा म िवधान प रषद के सृ जन या उ ादन के सं बंध

म उपबं ध है िक यिद िकसी रा की िवधानसभा अपने कुल सद सं ा के ब मत या

उप थत और मतदान दे ने वाले सद ों की सं ा के कम-से -कम 2/3 ब मत से सं क

पा रत करे तो सं सद िविध ारा िजस रा म िवधान प रषद नहीं है , वहाँ उसका सृ जन

और िजस रा म िवधान प रषद है , वहाँ उसका उ ादन/समापन कर सकेगी।

सं सद ारा िकसी रा म िवधान प रषद का सृ जन या उ ादन सं िवधान के अनु े द-368

के तहत सं शोधन नहीं माना जाएगा, ोंिक सं सद इसे साधारण ब मत से पा रत करती है ,

िक ु रा िवधानसभा ारा सं क िवशे ष ब मत ारा पा रत िकया जाता है ।

[11]िन िल खत कथनों पर िवचार कीिजयेः

1. भारत के सं िवधान म उपबं ध है िक े क रा की िवधानसभा 450 से


अनिधक और 60 से अ ू न सद ों से िमलकर बने गी।
2. कोई िकसी रा की िवधानसभा के िकसी थान को भरने के िलये
चु ने जाने यो नहीं होगा, यिद उसकी आयु 21 वष से कम हो।

उपयु म से कौन-सा/से कथन सही है /ह?


A) केवल 1

B) केवल 2

C) 1 और 2 दोनों

D) न तो 1 और न ही 2

Hide Answer -
उ रः (d)

ा ाः उपयु दोनों कथन सही नहीं ह।

भारत के सं िवधान के अनु े द-170 म रा की िवधानसभा की सं रचना के सं बंध म

उपबं ध िकया गया है िक 500 से अनिधक और 60 से अ ू न सद ों से िमलकर बने गी,

पर ु कुछ रा इसका अपवाद ह-

(i) गोवा - 40 सद

(ii) िस म - 32 सद

(iii) िमज़ोरम - 40 सद

(iv) पु दुचेरी - 30 सद

सं िवधान के अनु े द-173 म ावधान है िक िवधानसभा का सद बनने के िलये ू नतम

आयु 25 वष होनी चािहये , न िक 21 वष।

[12]रा की िवधान प रषद के सं बंध म िन िल खत म से कौन-सा/से कथन सही है /


ह?

1. सं िवधान म उपबं ध है िक िवधान प रषद वाले रा की िवधान प रषद के


सद ों की कुल सं ा उस रा की िवधानसभा के सद ों की कुल
सं ा के एक-ितहाई से अिधक नहीं होगी, िक ु िकसी भी दशा म 60 से
कम नहीं होगी।
2. िवधान प रषद वाले रा ों म िवधान प रषद का सद बनने के िलये
ू नतम आयु 35 वष से कम नहीं होनी चािहये ।
3. िवधान प रषद एक थायी सदन है , िजसके एक-ितहाई सद े क तीन
वष की समा पर िनवृ होते ह।

नीचे िदये गए कूट का योग कर सही उ र का चयन कीिजये ।

A) केवल 1

B) केवल 1 और 3

C) 1, 2 और 3

D) उपरो म से कोई नहीं।


Hide Answer -
उ रः (d)

ा ाः उपयु तीनों कथन सही नहीं ह।

सं िवधान के अनु े द-171 म रा िवधान प रषद की सं रचना के सं बंध म उपबं ध िकया

गया है िक िवधान प रषद वाले रा ों म िवधान प रषद के सद ों की कुल सं ा उस

रा की िवधानसभा के सद ों की कुल सं ा के 1/3 से अिधक नहीं होगी, िक ु िकसी

भी दशा म 40 से कम नहीं होगी। रा िवधान प रषद की ू नतम सं ा का िनयम

ज ू -क ीर पर लागू नहीं होता। वहाँ इसके 36 सद ह।

सं िवधान के अनु े द-173 म ावधान है िक िवधान प रषद का सद बनने के िलये

ू नतम आयु 30 वष होनी चािहये ।

सं िवधान के अनु े द-172 म ावधान है िक रा की िवधान प रषद (उ सदन) एक

थायी सदन है । इसका कभी िवघटन नहीं होता। इसके 1/3 सद ित दो वष की

समा पर िनवृ होते ह।

[13]िवधान प रषद वाले रा ों म िवधान प रषद के सद ों के िनवाचन एवं मनोनयन


के सं दभ म िन िल खत कथनों पर िवचार कीिजयेः

1. 1/6 सद थानीय िनकायों ारा चु ने जाते ह।


2. 1/12 सद ातकों ारा चु ने जाते ह।
3. 1/12 सद िश कों ारा चु ने जाते ह।
4. 1/3 सद िवधानसभा के ारा चु ने जाते ह।
5. 1/12 सद रा पाल ारा मनोनीत िकये जाते ह।

उपयु म से कौन-से कथन सही ह?

A) केवल 1, 2 और 4

B) केवल 2, 3 और 4

C) केवल 2, 3, 4 और 5

D) 1, 2, 3, 4 और 5
Hide Answer -
उ रः (b)

ा ाः उपयु म से कथन (1) एवं (5) सही नहीं ह।

सं िवधान के अनु े द-171(3) म िवधान प रषद के सद ों के िनवाचन एवं मनोनयन के सं बंध

म उपबं ध िकया गया है ः

1/3 सद थानीय िनकायों ारा चु ने जाते ह। इसम नगरपािलका, िजला बोड आिद आते

ह।

1/12 सद ऐसे ातकों ारा चु ने जाते ह, जो भारत के रा े म िकसी िव िव ालय

से कम से कम तीन-वष य ातक ह।

1/12 सद 3 वष से अ ापन कर रहे िश कों के ारा चु ने जाते ह। ये अ ापक

मा िमक ू लों से कम के नहीं होने चािहये ।

1/3 सद िवधानसभा के सद ों ारा ऐसे यों म से िनवािचत िकये जाएं गे , जो

िवधानसभा के सद न हों।

शे ष सद रा पाल ारा नाम-िनदिशत िकये जाएं गे । िवधान प रषद के कुल सद ों म से

5/6 सद ों का िनवाचन अ तरीके से एकल सं मणीय मत के ारा आनु पाितक

ितिनिध णाली के आधार पर होता है और शे ष 1/6 सद रा पाल ारा नाम-

िनदिशत होते ह। रा पाल सािह , िव ान, कला, सहकारी आ ोलन और समाज से वा के

े म िवशे ष ान या ावहा रक अनु भव रखने वाले यों को मनोनीत करता है ।

नोटः रा पित रा सभा के िलये 12 सद ों का नाम-िनदिशत करता है , िजसम िव ान, कला

एवं समाज से वा के े म िवशे ष ान या ावहा रक अनु भव रखने वाले होते ह। इसम

सहकारी आ ोलन को शािमल नहीं िकया गया है ।

[14] िन िल खत म से िकन रा ों म िवधानमं डल का उ सदन (िवधान प रषद)


अ म ह?

1. कनाटक
2. आं दे श
3. तिमलनाडु
4. ते लंगाना
5. महारा
6. ज ू एवं क ीर

नीचे िदये गएकूट का योग कर सही उ र का चयन कीिजयेः

A) केवल 2, 3, 4 और 5

B) केवल 1, 2, 3, 4 और 5

C) केवल 2, 3, 4, 5 और 6

D) केवल 1, 2, 4, 5 और 6
Hide Answer -
उ रः (d)

ा ाः उपयु म से कथन (3) सही नहीं है ।

तिमलनाडु रा म िवधानमं डल का उ सदन (िवधान प रषद) अ म नहीं है । वष

2010 म िवधानसभा ारा एक ाव लाया गया, िजसके आधार पर सं सद ने तिमलनाडु

िवधान प रषद अिधिनयम, 2010 अिधिनयिमत िकया, िक ु इसके लागू होने से पहले ही

वष 2011 म िवधानसभा ारा इसके उ ादन के सं बंध म ाव पा रत कर िदया गया।

वतमान म सात रा ों म िवधान प रषद अ म है -

1. उ र दे श

2. िबहार

3. कनाटक

4. आं दे श

5. ते लंगाना

6. महारा

7. ज ू एवं क ीर

आं दे श म िवधान प रषद को आं दे श िवधान प रषद अिधिनयम, 2005 के ारा

पु न थािपत िकया गया।

आं दे श से अलग होकर 2 जू न 2014 को ते लंगाना रा अ म आया। यहाँ

िवधान प रषद की कुल सद सं ा 43 है ।

[15] िन िल खत कथनों पर िवचार कीिजयेः

1. रा ीय आपातकाल के समय सं सद रा िवधानसभा का कायकाल एक


बार म एक वष के िलये बढ़ा सकती है , िक ु िकसी भी दशा म 3 वष से
अिधक नहीं बढ़ा सकती।
2. रा िवधान प रषद की कायाविध छः वष की होती है और इसके एक-
ितहाई सद े क दो वष बाद िनवृ होते रहते ह।
3. रा िवधान प रषद के से वािनवृ सद पु नचु नाव और पु ननामां कन के
िलये यो नहीं होते ।

उपयु म से कौन-सा/से कथन सही है /ह?

A) केवल 1

B) केवल 1 और 3

C) 1, 2 और 3

D) उपयु म से कोई नहीं।


Hide Answer -
उ रः (d)

ा ाः उपयु सभी कथन गलत ह।

रा ीय आपातकाल की थित म सं सद रा िवधानमं डल का कायकाल एक बार म एक

वष के िलये बढ़ा सकती है और एक-एक वष कर सं सद िकतने भी समय के िलये बढ़ा

सकती है । इसकी समय सीमा िनधा रत नहीं है , िक ु आपातकाल समा होने के प ात्

इसका िव ार छः माह से अिधक नहीं हो सकता।

रा िवधान प रषद एक थायी सदन है और इसके 1/3 सद े क दो वष बाद

से वािनवृ त होते रहते ह। इसके सद ों का कायकाल छः वष का होता है ।

रा िवधान प रषद के से वािनवृ सद पु नचु नाव एवं पु ननामां कन के िलये यो होते ह।

[16]रा की िवधानसभा के सद िन िल खत के िनवाचन म भाग ले ते हः-

1. भारत के रा पित के िनवाचन म।


2. भारत के उपरा पित के िनवाचन म।
3. रा सभा के सद ों के िनवाचन म।
4. रा िवधान प रषद के सद ों के िनवाचन म।

उपयु म से कौन-से कथन सही ह?

A) केवल 1 और 4

B) केवल 1, 2 और 3

C) केवल 1, 3 और 4

D) 1, 2, 3 और 4

Hide Answer -
उ रः (c)

ा ाः उपयु म से िवक (2) सही नहीं है ।

भारत के उपरा पित के िनवाचन म रा की िवधानसभा के सद भाग नहीं ले ते ह।

इसके िनवाचन म केवल सं सद के दोनों सदनों के सभी सद भाग ले ते ह।

भारत के रा पित के िनवाचन म सं सद के दोनों सदनों के िनवािचत सद , रा

िवधानसभा के िनवािचत सद एवं िजस के शािसत दे श म िवधानसभा है , उसके

िनवािचत सद भाग ले ते ह।

रा सभा के सद ों का िनवाचन रा की िवधानसभा के सद ों ारा होता है ।

रा िवधान प रषद वाले रा ों म िवधान प रषद के 1/3 सद िवधानसभा के सद ों

ारा िनवािचत होते ह।

[17]रा िवधानसभा के सं दभ म िन िल खत म से कौन-सा कथन सही है ।


रा िवधानसभा का सद िनवािचत होने के िलये उस का सं बंिधत
A)
रा की मतदाता सू ची म नाम होना आव क नहीं है ।

दो या दो से अिधक वष की कैद की सज़ा काट चु का रा िवधानसभा


B)
सद िनवािचत होने के िलये यो नहीं हो सकता।

िकसी रा िवधानसभा का कोई सद अयो ता से है या नहीं, इस


सं बंध म रा पाल का िनणय अ म होगा, िक ु िनणय करने से पहले
C)
रा पाल रा िनवाचन आयोग से राय ले गा और उसी की राय के अनु सार
काय करे गा।

सं िवधान की दसवीं अनु सूची के तहत रा िवधानसभा का कोई सद


D) अयो है या नहीं, इसका िनणय रा िवधानसभा अ करे गा और उसका
िनणय अ म होगा।

Hide Answer -
उ रः (b)

ा ाः उपयु म से िवक (b) सही है ।

जन ितिनिध अिधिनयम, 1954 के तहत उपबं ध िकया गया है िक दो या दो से अिधक

वष की कैद की सज़ा काट चु का रा िवधानसभा का सद बनने के िलये यो

नहीं है ; या िकसी को अ ृ ता, दहे ज़ था, सती था जै से अपराधों म सं िल

पाया जाता है तो वह रा िवधानसभा का सद नहीं बन सकता; या िनधा रत समय सीमा

के अ र चु नावी खच सं बंिधत िववरण ु त करने म िवफल रहा है तो रा

िवधानसभा का सद बनने के यो नहीं होगा।

जन ितिनिध अिधिनयम, 1951 म उपबं ध है िक िवधानसभा सद बनने के िलये

सं बंिधत रा की मतदाता सू ची म नाम होना आव क है ।

सं िवधान के अनु े द-192 म उपबं ध है िक रा िवधानमं डल के िकसी सदन का कोई

सद अयो है या नहीं, तो इसका िनणय रा पाल करे गा और रा पाल का िनणय

अ म होगा, िकंतु िनणय ले ने से पू व रा पाल भारत के िनवाचन आयोग (न िक रा

िनवाचन आयोग) से राय ले गा और उसी की राय के अनु सार काय करे गा।

सं िवधान के अनु े द-191(2) म उपबं ध है िक अनु े द -191(1) के तहत उठता है

िक रा िवधानसभा या िवधान प रषद का कोई सद अयो है या नहीं, इसका िनणय

िवधानसभा अ /सभापित करता है , िक ु उसका िनणय अ म नहीं होता है । उसके

िनणय की ायालय म समी ा की जा सकती है

(िकहोतो होलोहन वाद, 1992)।

[18]रा िवधानमं डल के सदं भ म िन ां िकत म से कौन-सा कथन सही है ।


रा के िवधानमं डल के िकसी सदन का अिधवे शन गिठत करने के िलये
कोरम दस सद या सदन के सद ों के कुल सं ा का दसवां भाग, इसम
A)
से जो भी अिधक हो, होना आव क है , अ था रा पाल सदन की बै ठक
को थिगत या िनल त कर सकता है ।

रा िवधानसभा के अ या उपा तथा रा िवधान प रषद के सभापित


या उपसभापित के वे तन और भ े सं िवधान की दू सरी अनु सूची के तहत भारत
B)
की सं िचत िनिध पर भा रत होंगे और उसे रा के िवधानमं डल म मतदान के
िलये नहीं रखा जाएगा।

रा िवधानसभा के अ या उपा को उसके पद से िवधानसभा म


उप थत एवं मत दे ने वाले सद ों तथा रा िवधान प रषद के सभापित या
C) उपसभापित को उसके पद से िवधान प रषद म उप थत एवं मत दे ने वाले
सद ों के कम से कम दो-ितहाई ब मत से पा रत सं क के ारा हटाया जा
सकेगा।

कोई िवधे यक धन िवधये क है या नहीं रा िवधानसभा अ का िनणय


D) अ म होगा, िक ु यह श रा िवधान प रषद के सभापित को नहीं ा
है ।

Hide Answer -
उ रः (d)

ा ाः उपयु म से िवक (d) सही है ।

सं िवधान के अनु े द-199(3) म उपबं ध है िक िवधानमं डल म पु रः थािपत कोई िवधे यक

धन िवधे यक है या नहीं, उस पर रा िवधानसभा अ का िनणय अ म होगा, पर ु

यह श रा िवधान प रषद के सभापित को ा नहीं है ।

सं िवधान के अनु े द-189(3) म कोरम/गणपू ित के सं बंध म उपबं ध है । िवधानमं डल के

िकसी सदन का अिधवे शन गिठत करने के िलये गणपू ित 10 सद या सदन के सद ों

की कुल सं ा का 10वां भाग, जो भी अिधक हो, होना आव क है , अ था अ या

सभापित सदन की बै ठक को थिगत या िनलं िबत कर सकता है ।

सं िवधान के अनु े द 186 के तहत िवधानसभा के अ या उपा तथा िवधान प रषद

के सभापित या उपसभापित के वे तन व भ े सं िवधान की दू सरी अनु सूची के तहत रा की

सं िचत िनिध पर भा रत होंगे और रा िवधानमं डल म इस पर मतदान नहीं होगा।

सं िवधान के अनु े द-179 म रा िवधानसभा के अ एवं उपा को हटाने तथा

अनु े द-183 म रा िवधान प रषद के सभापित एवं उपसभापित को हटाने सं बंधी

ावधान है । िवधानसभा के त ालीन सम सद ों के ब मत से तथा िवधान प रषद के

त ालीन सम सद ों के ब मत से हटाया जा सकता है , िक ु दोनों ही थितयों म 14

िदन पू व इसकी सू चना दे ना आव क है ।

[19]िवधानसभा िवघटन के प ात् भी िवधानसभा अ अपने पद पर बना रहता है ।


िवधानसभा के िवघटन के प ात् होने वाले िवधानसभा के थम अिधवे शन के
A)
ठीक पहले तक।

िवधानसभा के िवघटन के प ात् होने वाले िवधानसभा के थम अिधवे शन के


B)
कुछ िदन बाद तक।

C) नए लोकसभा अ के चु ने जाने तक।

D) जब तक रा पाल उसे हटने के िलये न कहे तब तक।

Hide Answer -
उ रः (a)

ा ाः उपयु म से िवक (a) सही है ।

सं िवधान के अनु े द-179 (ग) के पर ु म यह ावधान है िक िवधानसभा के िवघटन के

प ात् भी िवधानसभा अ अपने पद पर िवधानसभा के थम अिधवे शन के ठीक पहले

तक बना रहता है ।

[20]सू ची-I को सू ची-II से सु मेिलत कीिजयेः

सू ची-I
सू ची-II
(सं िवधान के
( ावधान )
अनु े द)

1. रा िवधानमं डल के सदनों एवं सिमितयों के सं बंध म


A. अनु े द-177 रा के मं ी एवं महािधव ा के अिधकार के सं बंध म
उपबं ध।

B. अनु े द-187 2. रा िवधानमं डल के सिचवालय के सं बंध म उपबं ध।

3. अ या सभापित को िनणायक मत (Casting


C. अनु े द-189
vote) दे ने का अिधकार।

4. रा िवधानमं डल के िकसी सदन की अनु मित


के िबना लगातार साठ िदनों की अविध तक
D. अनु े द-190
उसकी सभी बै ठकों से अनु प थित रहने पर सद ों
की अयो ता के सं बंध म उपबं ध।

नीचे िदये गएकूट का योग कर सही उ र का चयन कीिजयेः

A B C D

A) 1 2 3 4

B) 1 3 2 4

C) 4 3 2 1

D) 4 2 3 1
Hide Answer -
उ रः (a)

ा ाः उपयु म से िवक (a) सही है ।

सं िवधान के अनु े द-177: े क मं ी और रा के महािधव ा को रा िवधानसभा या

रा िवधान प रषद वाले रा की दशा म दोनों सदनों म और िवधानमं डल की िकसी सिमित म,

िजसम उसका नाम सद के प म िदया गया है , म बोलने और उसकी कायवाही म भाग ले ने

का अिधकार है , िक ु मत दे ने का अिधकार नहीं है ।

सं िवधान के अनु े द-187: रा िवधानमं डल के सिचवालय के सं बंध म उपबं ध है ।

सं िवधान के अनु े द 189: रा िवधानसभा के अ या रा िवधान प रषद के सभापित

थमतः मत नहीं दे ते , िक ु मत बराबर होने की दशा म िनणायक मत (Casting Vote) दे ते

ह।

इसी अनु े द म सदन की गणपू ित के सं बंध म उपबं ध है ।

सं िवधान के अनु े द-190: रा िवधानसभा या िवधान प रषद वाले रा म िवधान प रषद

का कोई सद सदन के अनु मित के िबना लगातार 60 िदनों तक उसकी सभी बै ठकों से

अनु प थत रहता है , तो उस सदन से उसकी सद ता समा हो जाएगी। इन 60 िदनों की

अविध की गणना म उस अविध को शािमल नहीं िकया जाएगा, िजसके दौरान सदन का

स ावसान या िनर र चार से अिधक िदनों के िलये थिगत रहता है ।


[1] रा िवधानमं डल के साधारण िवधे यक के सं दभ म िन िल खत कथनों पर िवचार
कीिजयेः

1. रा िवधानसभा ारा पा रत साधारण िवधे यक को रा िवधान प रषद


अिधकतम चार माह तक रोके रख सकती है ।
2. दोनों सदनों के बीच गितरोध होने पर रा पाल सं यु बै ठक बु लाता है ।
3. िवधान प रषद म पु रः थािपत िवधे यक को िवधानसभा ारा अ ीकृत कर
दे ने पर िवधे यक समा हो जाता है ।

उपयु म से कौन-सा/से कथन सही है /ह?

A) केवल 1

B) केवल 3

C) केवल 1 और 3

D) 1, 2 और 3

Hide Answer -
उ रः (c)

ा ाः उपयु म से कथन (2) गलत है ।

रा िवधानमं डल के सं बंध म सं यु बै ठक का ावधान नहीं है ।

सं िवधान के अनु े द-197 म ावधान है िक रा िवधान प रषद िकसी साधारण िवधे यक

को अिधकतम 4 माह तक रोक सकती है । थम बार म तीन माह एवं दू सरे बार म एक

माह।

िकसी साधारण िवधे यक का ारं भ रा िवधान प रषद म होता है और रा िवधानसभा

ारा अ ीकृत कर िदया जाता है तो िवधे यक समा हो जाता है ।

[2] सं िवधान के अनु े द 201 के सं दभ म िन िल खत म से कौन-सा कथन सही नहीं


है ?

रा िवधानमं डल ारा पा रत िकसी िवधे यक को रा पाल रा पित के िलये


A)
आरि त रख सकता है ।

रा िवधानमं डल ारा पा रत िकसी िवधे यक को रा पाल के सम ुत


B)
करना।

रा पित ारा पु निवचार के िलये लौटाए िकसी साधारण िवधये क पर रा


C)
िवधानमं डल छः माह के अ र पु निवचार करे गी।

रा िवधानमं डल िकसी साधारण िवधे यक पर पु निवचार कर रा पित को


D)
लौटाती है तो रा पित उस पर ीकृित दे ने के िलये बा नहीं है ।
Hide Answer -
उ रः (b)

ा ाः उपयु म से िवक (b) सही नहीं है ।

सं िवधान के अनु े द-200 म उपबं ध है िक रा िवधानमं डल ारा पा रत िवधे यक को

रा पाल के सम ु त करे गा।

सं िवधान के अनु े द-201 के तहत रा पाल रा िवधानमं डल ारा पा रत िकसी

िवधे यक को रा पित के िलये आरि त रख सकेगा।

सं िवधान के अनु े द-201 के तहत रा पित, रा पाल ारा आरि त िवधये क पर अपना

िनणय दे ता है , जो िन ह-

अनु मित दे सकता है ; या

अनु मित रोक सकता है ; या

धन िवधये क नहीं है तो िवधे यक को िवधानमं डल के पास पु निवचार के िलये लौटा सकता है

और िवधानमं डल छःमाह के अ र इस पर पु निवचार कर रा पित को भे जता है तो,

रा पित इस पर ीकृित दे ने के िलये बा नहीं है ।

[3] िन िल खत कथनों पर िवचार कीिजयेः

1. उ ायालय के ायाधीशों की पशन रा की सं िचत िनिध पर भा रत


होती है ।
2. गोवा, भारत का एकमा ऐसा रा है , जहाँ कॉमन िसिवल कोड लागू है ।
3. रा ों के िव ले खाओं पर िनयं ण भारत के िनयं क एवं महाले खा
परी क (कैग) ारा िकया जाता है ।

उपयु म से कौन-सा/से कथन सही है /ह?

A) केवल 1

B) केवल 2

C) केवल 2 और 3

D) 1, 2 और 3

Hide Answer -
उ रः (c)

ा ाः उपयु म से िवक (1) गलत है ।

सं िवधान के अनु े द-112 (3) के तहत उ ायालय के ायाधीशों की पशन भारत की

सं िचत िनिध पर भा रत होती है , जबिक सं िवधान के अनु े द-202 (3) के तहत उ

ायालय के ायाधीशों के वे तन व भ े रा की सं िचत िनिध पर भा रत होते ह।

गोवा भारत का एकमा ऐसा रा है , जहाँ पर कॉमन िसिवल कोड लागू है । यहाँ 19वीं

शता ी से ही पु तगीज िसिवल कोड चला आ रहा है ।

के के साथ-साथ रा िव ले खाओं पर िनयं ण भारत के िनयं क एवं महाले खा परी क

ारा िकया जाता है ।


[4] भारत के सं िवधान के अनु े द-200 के अधीन रा पालः

1. रा िवधानमं डल ारा पा रत िवधे यक पर अपनी ीकृित रोक सकेगा।


2. रा िवधानमं डल ारा पा रत िवधे यक को रा पित के िवचाराथ आरि त
रखता है ।
3. धन िवधे यक से िभ िकसी िवधे यक को पु निवचार के िलये िवधानमं डल को
लौटा सकेगा।

उपयु म से कौन-सा/से कथन सही है /ह?

A) केवल 2

B) केवल 1 और 3

C) केवल 2 और 3

D) 1, 2 और 3

Hide Answer -
उ रः (b)

ा ाः उपयु म से कथन (2) गलत है ।

रा पाल, रा िवधानमं डल ारा पा रत िकसी िवधे यक को सं िवधान के अनु े द-201 के

तहत रा पित के िलये आरि त रखता है ।

सं िवधान के अनु े द-200 के तहत रा पाल रा िवधानमं डल ारा पा रत िकसी

िवधये क पर अपनी अनु मित दे सकता है या रोक सकता है या धन िवधे यक से िभ िकसी

िवधे यक को पु निवचार के िलये रा िवधानमं डल को लौटा सकता है , िकंतु यिद रा

िवधानमं डल िवधे यक म सं शोधन या िबना सं शोधन के रा पाल के पास पु नः ु त करता

है तो रा पाल अनु मित दे ने के िलये बा है ।

रा पाल िवधे यक को रा पित के िलये आरि त रख सकता है । रा पाल की राय म

िवधे यक पर अनु मित दे ने पर उ ायालय की श यों का अ ीकरण होगा, तो िवधे यक

पर अनु मित नहीं दे गा। उसे रा पित के िलये आरि त रखे गा।

[5] भारत के सव ायालय के सं दभ म िन िल खत म से कौन-सा कथन सही है ?

भारत म सं घीय ायालय की थापना भारत शासन अिधिनयम, 1919 के


A)
तहत ई थी।

अमे रकी ायालय के समान भारत म भी एकीकृत ायालय की थापना की


B)
गई है ।

मू लतः सव ायालय के ायाधीशों की सं ा आठ िनि त की गई थी,


C)
िजसम एक मु ायाधीश और सात अ ायाधीश थे ।

वतमान म ायाधीशों की कुल सं ा 26 है , िजसम एक मु ायाधीश


D)
और 25 अ ायाधीश ह।
Hide Answer -
उ रः (c)

ा ाः उपयु म से िवक - c सही है ।

मू लतः सव ायालय के ायाधीशों की कुल सं ा आठ थी, िजसम एक मु

ायाधीश तथा सात अ ायाधीश थे । वतमान म इनकी सं ा 31 है , िजसम एक मु

ायाधीश और 30 अ ायाधीश ह।

भारत म सं घीय ायालय की थापना भारत शासन अिधिनयम, 1935 के तहत 1 अ ू बर

1937 म की गई थी। वतमान का सव ायालय भारत शासन अिधिनयम, 1935 की

दे न है । 28 जनवरी 1950 को भारतीय सव ायालय का उद् घाटन िकया गया।

अमे रका म ै ध ायालय की व था है , जबिक भारत म एकीकृत ायालय की व था

है ।

सं िवधान के भाग V के अ ाय-IV (अनु े द-124 से 147 तक) म इसकी चचा की गई

है ।

सव ायालय ( ायाधीशों की सं ा) सं शोधन अिधिनयम, 2008 के तहत फरवरी

2009 म ायाधीशों की सं ा 26 से बढ़ाकर 31 कर दी गई।

[6] भारत के उ तम ायालय म ायाधीशों की सं ा म वृ के सं दभ म


िन िल खत म से कौन-सा/से कथन सही है /ह?

1. ायाधीशों की सं ा म वृ करने की श रा पित म िनिहत है ।


2. 99व सं िवधान सं शोधन अिधिनयम का सं बंध रा ीय ाियक िनयु
आयोग से है ।
3. ायाधीशों की िनयु से सं बंिधत कॉले िजयम णाली म भारत के मु
ायाधीश सिहत ायाधीशों की कुल सं ा चार है ।

नीचे िदये गए कूट का योग कर सही उ र का चयन कीिजये ।

A) केवल 2

B) केवल 1 और 2

C) केवल 2 और 3

D) 1, 2 और 3
Hide Answer -
उ रः (a)

ा ाः उपयु म से कथन -1 और 3 सही नहीं ह।

सं िवधान के अनु े द-124 के तहत भारत के सव ायालय के ायाधीशों की सं ा म

वृ करने की श सं सद म िनिहत है । सं सद अभी तक पाँ च बार ायाधीशों की सं ा

म वृ कर चु की है , जो िन ानु सार है

⇒ मू ल सं ा = 8 (मु ायाधीश + 7 अ ायाधीश)

⇒ वष 1956 = ⇒ 11 (मु ायाधीश + 10 अ ायाधीश)

⇒ वष 1960 = 14 (मु ायाधीश + 13 अ ायाधीश)

⇒ वष 1977 = 18 (मु ायाधीश + 17 अ ायाधीश)

⇒ वष 1986 = 26 (मु ायाधीश + 25 अ ायाधीश)

⇒ वष 2009 = 31 (मु ायाधीश + 30 अ ायाधीश)

उ तम ायालय और उ ायालय के ायाधीशों की िनयु के िलये 99व सं िवधान

सं शोधन अिधिनयम, 2014 के ारा एक रा ीय ाियक िनयु आयोग (NJAC) का

गठन िकया गया, िक ु वष 2015 म सव ायालय ने इसे असं वैधािनक घोिषत कर

िदया।

सं िवधान के अनु े द-124(2) के तहत भारत का रा पित उ तम ायालय एवं उ

ायालय के ायाधीशों से परामश के प ात् उ तम ायालय के ेक ायाधीश की

िनयु करे गा। मु ायाधीश से िभ िकसी ायाधीश की िनयु की दशा म भारत

के मु ायाधीश से सदै व परामश ले गा और मु ायाधीश 4 व र ायाधीशों से

परामश के प ात् रा पित को िसफा रश भे जेगा। इसे ‘कॉले िजयम णाली’ के नाम से जाना

जाता है , िजसम मु ायाधीश के अित र 4 अ वर ायाधीश होते ह।

[7] िन िल खत कथनों पर िवचार कीिजयेः

1. उ तम ायालय रा पित की मं जूरी के बाद ायालय की ि या और


सं चालन हे तु िनयम बना सकता है ।
2. सं िवधान म उ तम ायालय के ायाधीशों का कायकाल छः वष या 65
वष की आयु तक, जो भी पहले हो, िनधा रत की गई है ।

उपयु म से कौन-सा/से कथन सही है /ह?

A) केवल 1

B) केवल 2

C) 1 और 2 दोनों

D) न तो 1 और न ही 2
Hide Answer -
उ रः (a)

ा ाः उपयु म से कथन (2) गलत है ।

सं िवधान म उ तम ायालय के ायाधीशों का कायकाल तय नहीं िकया गया है । सं िवधान

के अनु े द 124(2) म उपबं ध है िक उ तम ायालय का ायाधीश तब तक पद

धारण करे गा, जब तक 65 वष की आयु ा नहीं कर ले ता।

उ तम ायालय रा पित की मं जूरी के बाद ायालय की ि या और सं चालन हे तु िनयम

बना सकता है ।

[8] ायाधीश जाँ च अिधिनयम, 1968 उ तम ायालय के ायाधीशों को हटाने के


सं बंध म महािभयोग की ि या का उपबं ध करता है । इस सं दभ म िन िल खत
कथनों पर िवचार कीिजये -

1. िन ासन ाव पर लोकसभा या रा सभा के कम से कम 50 सद ों


का ह ा र आव क है ।
2. इस महािभयोग की जाँ च तीन सद ीय सिमित ारा की जाती है ।
3. महािभयोग का ाव सं सद के े क सदन ारा उप थत और मत दे ने
वाले सद ों के ब मत ारा पा रत होना चािहये ।
4. इस कार के महािभयोग का योग केवल एक बार आ है ।

उपयु म से कौन-सा/से कथन सही है /ह?

A) केवल 2

B) केवल 2 और 4

C) केवल 1, 2 और 4

D) 1, 2, 3 और 4
Hide Answer -
उ रः (a)

ा ाः उपयु म से कथन 1, 3 और 4 गलत ह।

उ तम ायालय के िकसी ायाधीश पर महािभयोग चलाने के िलये िन ासन ाव पर

लोकसभा के 100 या रा सभा के 50 सद ों का ह ा र होना आव क है ।

इसकी जाँ च के िलये अ /सभापित ारा तीन सद ीय सिमित का गठन िकया जाता है ,

िजसम उ तम ायालय का मु ायाधीश या उ तम ायालय का कोई ायाधीश,

िकसी उ ायालय का मु ायाधीश और िति त ायवादी शािमल होते ह।

महािभयोग का ाव सं सद के े क सदन ारा अपनी कुल सद सं ा के ब मत

ारा या उप थत एवं मत दे ने वाले सद ों के कम-से -कम दो-ितहाई ब मत (िवशे ष

ब मत) ारा पा रत होना आव क है । सं सद ारा पा रत ाव सं सद के उसी स म

रा पित के सम ु त करना आव क है । इसके बाद रा पित उस ायाधीश को हटाने

सं बंधी आदे श जारी कर दे ता है ।

अभी तक उ तम ायालय के िकसी ायाधीश पर महािभयोग नहीं चलाया गया है ।

इसका पहला मामला ायाधीश वी. रामा ामी (1991-1993) के सं बंध म उठा था,

िकंतु यह लोकसभा म पा रत न हो सका।

[9] िन िल खत म से कौन-सा कथन सही नहीं है ?

उ तम ायालय के िकसी ायाधीश को उसके पद से सािबत कदाचार या


A)
असमथता के आधार पर हटाया जा सकता है ।

उ तम ायालय के िकसी ायाधीश को उसके पद से सं सद की िसफा रश


B)
पर रा पित ारा हटाया जाता है ।

उ तम ायालय के ायाधीशों के वे तन का िनधारण सं सद िविध ारा


C)
समय-समय पर करती है ।

उ तम ायालय म सं िवधान के िनवचन से सं बंिधत मामलों की सु नवाई के


D)
िलये कम से कम सात ायाधीशों का होना आव क है ।
Hide Answer -
उ रः (d)

ा ाः उपयु म से िवक (d) सही नहीं है ।

उ तम ायालय म सं िवधान की ा ा से सं बंिधत मामलों की सु नवाई के िलये कम-से -

कम पाँ च ायाधीश होने चािहये , जबिक अ मामलों का िनणय सामा तया तीन

ायाधीशों की पीठ करती है । सं िवधान म इसे पीठ के नाम से जाना जाता है ।

सं िवधान के अनु े द-124(4) म ावधान है िक उ तम ायालय के िकसी ायाधीश

को सािबत कदाचार या असमथता के आधार पर उसके पद से हटाया जा सकता है ।

उ तम ायालय के िकसी भी ायाधीश को उसके पद से भारत का रा पित, सं सद की

िसफा रश के आधार पर हटा सकता है । जब सं सद िवशे ष ब मत ारा हटाने सं बंधी

ाव को रा पित के सम उसी स म रखती है तो रा पित हटाने का आदे श जारी कर

सकता है ।

सं िवधान के अनु े द -125(1) म ावधान है िक सं सद िविध ारा समय-समय पर

उ तम ायालय के ायाधीशों के िलये वे तन का ावधान करे गी।

[10]िन िल खत कथनों पर िवचार कीिजयेः

1. उ तम ायालय का ेक ायाधीश अपनी शपथ या ित ान रा पित


के सम सं िवधान की दू सरी अनु सूची म िदये गए ा प के अनु सार ले गा।
2. उ तम ायालय के ायाधीशों का वे तन भारत की सं िचत िनिध पर
भा रत होता है , िजसे सं सद म मतदान के िलये रखा जाता है ।
3. उ तम ायालय का ायाधीश से वािनवृ ि के प ात् भारत के रा े
के भीतर िकसी ायालय म या िकसी ािधकारी के सम वकालत या
काय नहीं कर सकता।

उपयु म से कौन-सा/से कथन सही है /ह?

A) केवल 3

B) केवल 1 और 3

C) केवल 2 और 3

D) 1, 2 और 3
Hide Answer -
उ रः (a)

ा ाः उपयु म से कथन (1) और (2) गलत ह।

सं िवधान के अनु े द-124(6) म उपबं ध है िक उ तम ायालय का ेक ायाधीश

रा पित के सम सं िवधान की तीसरी अनु सूची (न िक दू सरी अनु सूची) म िदये गए ा प

के अनु सार शपथ ले गा।

सं िवधान के अनु े द-125 म उ तम ायालय के वे तन आिद के सं बंध म उपबं ध है िक

सं सद िविध ारा समय-समय पर वे तन का िनधारण करे गी। इसके ायाधीशों का वे तन

सं िवधान की दू सरी अनु सूची के तहत भारत की सं िचत िनिध पर भा रत होता है , िकंतु सं सद

म इसे मतदान के िलये नहीं रखा जाता अथात् सं सद के मताधीन नहीं है ।

सं िवधान के अनु े द- 124(7) म ावधान है िक उ तम ायालय के ायाधीश

से वािनवृ ि के प ात् भारत के रा े के भीतर िकसी ायालय या िकसी ािधकारी के

सम वकालत या काय नहीं करगे ।

[11]भारत के सव ायालय के सं दभ म िन िल खत म से कौन-सा/से कथन सही


है /ह?

1. भारत का उ तम ायालय केवल िद ी म ही अिधिव (Sit) हो सकता


है ।
2. भारत का उ तम ायालय िद ी के अित र ऐसे अ थानों पर
अिधिव (Sit) हो सकता है , िजसे भारत का मु ायमू ित, रा पित के
अनु मोदन से िनयत करे ।
3. सं सद िविध ारा उ तम ायालय को अनु पूरक श याँ दान करने के
िलये उपबं ध बना सकती है ।

नीचे िदये गए कूट का योग कर सही उ र का चयन कीिजयेः

A) केवल 2

B) केवल 3

C) केवल 1 और 3

D) केवल 2 और 3
Hide Answer -
उ रः (d)

ा ाः उपयु म से कथन (1) गलत है ।

सं िवधान के अनु े द-130 म उ तम ायालय के थान के सं बंध म उपबं ध िकया गया

है । इसम उपबं ध है िक उ तम ायालय िद ी म या ऐसे अ थान या थानों म

अिधिव हो सकता है , िज भारत के मु ायमू ित, रा पित के अनु मोदन से समय-

समय पर िनयत कर।

सं िवधान के अनु े द-140 के तहत सं सद िविध ारा उ तम ायालय को अनु पूरक

श याँ दान करने के िलये उपबं ध बना सकती है ।

[12]िन िल खत म से कौन-सा कथन सही है ? यिद भारत के मु ायमू ित को


उ तम ायालय की बै ठकों म तदथ (Adhoc) ायाधीश के प म उप थत
रहने के िलये िकसी उ ायालय के ऐसे ायाधीश को अनु रोध करना हो, जो
उ तम ायालय का ायाधीश िनयु होने के िलये स क प से यो है , तोः

A) केवल भारत के रा पित की पू व सहमित आव क है ।

सं बंिधत उ ायालय के मु ायमू ित से परामश तथा भारत के रा पित


B)
की पू व सहमित आव क है ।

C) केवल सं बंिधत उ ायालय के मु ायमू ित से परामश आव क है ।

न तो भारत के रा पित की पू व सहमित और न ही सं बंिधत उ ायालय के


D)
मु ायमू ित से परामश आव क है ।

Hide Answer -
उ रः (b)

ा ाः उपयु म से कथन (b) सही है ।

सं िवधान के अनु े द-127 म तदथ ायाधीशों की िनयु के सं बंध म ावधान है ।

भारत का मु ायमू ित रा पित की पू व सहमित एवं सं बंिधत उ ायालय के मु

ायमू ित से परामश के प ात् तदथ ायाधीशों की िनयु करता है ।

उ तम ायालय का मु ायाधीश उ ायालय के ऐसे ायाधीश को सव

ायालय म तदथ ायाधीश के प म िनयु करता है , जो उ तम ायालय का

ायाधीश िनयु होने के िलये स क प से यो है ।

उ तम ायालय के स को आयोिजत करने या चालू रखने के िलये ायाधीशों की

गणपू ित/कोरम पू रा नहीं है तो तदथ ायाधीशों की िनयु उतनी अविध के िलये की

जाती है । िजस सं दभ म भारत के मु ायामू ित ारा िल खत प से अनु रोध िकया जाता

है ।

तदथ ायाधीशों की िनयु सं बंधी ावधान केवल भारत के उ तम ायालय के सं दभ

म है ।

[13]िन िल खत म से कौन-सा कथन सही है ?


भारत के कायवाहक मु ायाधीश की िनयु भारत के मु ायमू ित के
A)
परामश से भारत के रा पित ारा की जाती है ।

भारत के कायवाहक मु ायमू ित की िनयु रा पित की पू व सहमित से


B)
भारत के मु ायमू ित ारा की जाती है ।

भारत का मु ायमू ित रा पित की पू व सहमित से उ तम ायालय या


C) उ ायालय के से वािनवृ ायाधीश को उ तम ायालय की बै ठक म
उप थत होने के िलये अनु रोध कर सकता है ।

भारत का रा पित भारत के मु ायमू ित के परामश से उ तम ायालय


D) या उ ायालय के से वािनवृ ायाधीश से उ तम ायालय की बै ठक म
उप थत होने के िलये अनु रोध कर सकता है ।

Hide Answer -
उ रः (c)

ा ाः उपयु म से कथन (c) सही है ।

सं िवधान के अनु े द-128 म उ तम ायालय की बै ठकों म से वािनवृ ायाधीशों की

उप थित के सं बंध म ावधान है । इस अनु े द के तहत भारत का मु ायमू ित, िकसी

भी समय, उ तम या उ ायालय के से वािनवृ ायाधीश से उ तम ायालय की

बै ठक म उप थत होने और काय करने के िलये अनु रोध कर सकता है । ऐसा सं बंिधत

और रा पित की पू व अनु मित से िकया जा सकता है । इस कार बै ठने और काय

करने के दौरान वह ऐसे भ ों का हकदार होगा, जो रा पित आदे श ारा अवधा रत करे ।

सं िवधान के अनु े द-126 म भारत के कायवाहक/कायकारी मु ायमू ित की िनयु

के सं बंध म ावधान है । इसकी िनयु भारत के रा पित ारा िन िल खत थितयों म की

जा सकती है -

जब मु ायमू ित का पद र हो।

अ थायी प से मु ायाधीश अनु प थत हो।

मु ायाधीश अपने दािय ों के िनवहन म असमथ हो।

उ तम ायालय के कायवाहक मु ायमू ित (Acting Chief Justice) की िनयु

के सं बंध म रा पित िकसी की सलाह/परामश नहीं ले ता है ।

[14]भारत के सं िवधान म सव ायालय की तं ता एवं िन ता के िलये ावधान


िकये गए ह। इस सं दभ म िन िल खत म से कौन-सा कथन सही नहीं है ?
उ तम ायालय के ायाधीशों की िनयु करते समय भारत के रा पित
A) को उ तम ायालय और उ ायालय के ायाधीश से िवचार-िवमश
करना पड़ता है ।

उ तम ायालय के ायाधीशों का वे तन भारत की सं िचत िनिध पर भा रत


B)
होता है , िजस पर सं सद म मतदान नहीं होता।

भारत के मु ायमू ित को िबना सरकारी ह े प के अिधका रयों एवं


C)
कमचा रयों को िनयु करने का अिधकार है ।

केवल सं सद को यह अिधकार है िक सं सद िविध ारा उ तम ायालय के


D)
ाय े एवं श यों म कटौती एवं वृ कर सकती है ।

Hide Answer -
उ रः (d)

ा ाः उपयु म से िवक (d) गलत है ।

सं िवधान के अनु े द-138 म उ तम ायालय की अिधका रता की वृ के सं बंध म

उपबं ध है । सं सद को उ तम ायालय के ाय े एवं श यों म कटौती का अिधकार

नहीं है , िकंतु सं सद िविध ारा इनम वृ कर सकती है ।सं िवधान के अनु े द-124(2) म

ावधान है िक रा पित उ तम ायालय एवं उ ायालय के ायाधीशों से परामश के

प ात् उ तम ायालय के ेक ायाधीश को िनयु करे गा तथा मु ायाधीश से

िभ िकसी ायाधीश की िनयु म रा पित मु ायमू ित से परामश अव करे गा।

सं िवधान के अनु े द-146 म उ तम ायालय के अिधका रयों एवं से वकों की िनयु

एवं शासिनक य का ावधान है । भारत के मु ायमू ित को िबना सरकारी ह ेप

के अिधका रयों एवं कमचा रयों को िनयु करने का अिधकार है और इसका शासिनक

य भारत की सं िचत िनिध पर भा रत होता है ।

उ तम ायालय के ायाधीशों की िनयु तो रा पित करता है , िकंतु उसे उसके पद

से िबना महािभयोग के नहीं हटाया जा सकता।

िव ीय आपातकाल को छोड़कर उसके वे तन म कटौती नहीं की जा सकती।

महािभयोग के अित र सं सद या रा िवधानमं डल म इसके आचरण पर चचा नहीं की

जा सकती।

से वािनवृ ि के प ात् भारत के रा े के भीतर िकसी ायालय म या ािधकरण म

वकालत या काय नहीं कर सकता।

उ तम ायालय उस को दं िडत कर सकता है , जो उसकी अवमानना करे ।

ायपािलका को कायपािलका से पृ थक् रखा गया है ।

[15]िन िल खत करणों की े िणयों म से िकन-िकन म भारत के उ तम ायालय


को िनणय करने की श ा है ?

1. कानू न एवं त के िकसी पर रा पित ारा भे जे गए सं दभ म।


2. सं िवधान की ा ा से जु ड़े ए करण म।
3. सामा मह के और िविध के सारवान ों से जु ड़े ए करण म।
4. ऐसे िकसी करण म जहाँ कानू न की सं वैधािनकता को चु नौती दी गई हो।
नीचे िदये गए कूट का योग कर सही उ र का चयन कीिजयेः

A) केवल 1, 2 और 3

B) केवल 1, 3 और 4

C) केवल 2, 3 और 4

D) 1, 2, 3 और 4

Hide Answer -
उ रः (c)

ा ाः उपयु म से कथन (1) गलत है ।

सं िवधान के अनु े द-143 म उ तम ायालय से परामश करने की रा पित की श

के सं बंध म ावधान है । यिद िकसी समय रा पित को तीत होता है िक िविध या त का

कोई ऐसा उ आ है या उ होने की सं भावना है , जो ऐसी कृित और ऐसे

ापक मह का है , िजस पर उ तम ायालय की राय ा करना आव क है , तो

रा पित राय ले सकता है , िजस पर उ तम ायालय रा पित को सलाह दे ता है , न िक

उस पर िनणय करता है ।

सव ायालय को सं िवधान का अिभभावक या सं र क (Guardian or Custodian)

माना जाता है । सं िवधान की ा ा करने का अं ितम अिधकार उसी के पास है ।

सं िवधान की ा ा, कानू न की सं वैधािनकता की जाँ च एवं िविध के सारवान ों पर िदये

गए िनणय सबके िलये मा होते ह।

[16]िन िल खत कथनों पर िवचार कीिजयेः

1. सं सद, उ तम ायालय की अिधका रता का िव ार नहीं कर सकती,


ोंिक उसकी अिधका रता वही है , जो सं िवधान ने दान की है ।
2. उ तम ायालय और उ ायालयों के अिधकारी और से वक सं ब
मु ायमू ित ारा िनयु िकये जाते ह और ायालयों का शासिनक
य भारत की सं िचत िनिध पर भा रत होता है ।

उपयु म से कौन-सा/से कथन सही है /ह?

A) केवल 1

B) केवल 2

C) 1 और 2 दोनों

D) न तो 1 और न ही 2
Hide Answer -
उ रः (d)

ा ाः उपयु दोनों कथन गलत ह।

सं िवधान के अनु े द-138 के तहत सं सद, उ तम ायालय की अिधका रकता एवं

श यों म िव ार कर सकती है ।

उ तम ायालय का शासिनक य भारत की सं िचत िनिध पर भा रत होता है , जबिक

उ ायालय का शासिनक य रा की सं िचत िनिध पर भा रत होता है ।

[17]सव ायालय की आरं िभक अिधका रता (Original Jurisdiction) के सं दभ


म िन िल खत कथनों पर िवचार कीिजयेः

1. दो या अिधक रा ों के िनवािसयों के बीच िववाद।


2. भारत सरकार और एक रा या अिधक रा ों के बीच िववाद।
3. एक तरफ भारत सरकार और िकसी रा या रा ों और दू सरी तरफ एक
या अिधक अ रा ों के बीच िववाद।
4. दो या अिधक रा ों के बीच िववाद।

उपयु म से कौन-से कथन सही ह?

A) केवल 1, 2 और 3

B) केवल 1, 3 और 4

C) केवल 2, 3 और 4

D) 1, 2, 3 और 4

Hide Answer -
उ रः (c)

ा ाः उपयु म से कथन (1) सही नहीं है ।

सं िवधान के अनु े द-131 म उ तम ायालय के आरं िभक/मौिलक े ािधकार (Original

Jurisdiction) के सं बंध म ावधान है । यह सव ायालय का अन अिधकार है । इसकी

सु नवाई िकसी और ायालय म नहीं की जा सकती है । इसम िन शािमल है :

भारत सरकार और एक या अिधक रा ों के बीच िववाद।

एक तरफ भारत सरकार और िकसी रा या रा ों और दू सरी तरफ एक या अिधक अ

रा के बीच िववाद।

दो या अिधक रा ों के बीच िववाद।

िकसी िववाद म यिद िविध या त का कोई ऐसा अं तिनिहत है , िजस पर िकसी िविधक

अिधकार का अ या िव ार िनभर है , तो वे उ तम ायालय के ारं िभक े ािधकार के

अं तगत आते ह।

[18]िन िल खत कथनों पर िवचार कीिजयेः


1. सं िवधान के अनु े द-32 के तहत भारत के सव ायालय और रा ों
के उ ायालयों को रट जारी करने की श है ।
2. मू ल अिधकारों के उ ं घन पर रट जारी करने की श केवल भारत के
सव ायालय और रा ों के उ ायालयों म िनिहत है ।
3. रट जारी करने के सं बंध म सव ायालय की अिधका रता की तु लना म
रा ों के उ ायालयों की अिधका रता ादा ापक है ।
4. मू ल अिधकारों के उ ं घन पर नाग रकों को सबसे पहले उ ायालय म
अपील करनी पड़ती है ।

उपयु म से कौन-से कथन सही ह?

A) केवल 1 और 2

B) केवल 2 और 3

C) केवल 1, 2 और 3

D) 1, 2, 3 और 4

Hide Answer -
उ रः (b)

ा ाः उपयु म से कथन (1) और (4) गलत ह।

सं िवधान के अनु े द-32 के तहत सव ायालय को और अनु े द-226 के तहत

रा ों के उ ायालयों को रट जारी करने की श है ।

मू ल अिधकारों के उ ं घन पर नाग रक सीधे सव ायालय या उ ायालय म अपील

कर सकते ह।

मू ल अिधकारों के उ ं घन पर रट जारी करने की श केवल सव ायालय एवं उ

ायालय के पास है

सव ायालय केवल मू ल अिधकारों के उ ं घन पर रट जारी कर सकता है , जबिक

उ ायालय को मू ल अिधकारों के अित र अ मामलों म रट जारी करने की श

है । इस तरह उ ायालय को रट जारी करने के सं बंध म अिधका रता सव ायालय

से ादा ापक है ।

उ तम ायालय को मू ल अिधकारों का सं र क माना जाता है ।

[19]भारत के सव ायालय के अपीलीय े ािधकार के सं बंध म िन िल खत कथनों


पर िवचार कीिजयेः

1. सं घ एवं रा ों के बीच िववाद के मामलों म।


2. सं िवधान के अनु े द-143 के मामलों म।
3. िसिवल एवं आपरािधक मामलों म।
4. सं वैधािनक मामलों म।
5. सं िवधान के अनु े द-136 के मामलों म।

उपयु म से कौन-से कथन सही ह?


A) केवल 1, 2 और 3

B) केवल 3, 4 और 5

C) केवल 1, 3, 4 और 5

D) केवल 2, 3, 4 और 5

Hide Answer -
उ रः (b)

ा ाः उपयु म से कथन (1) और (2) सही नहीं ह।

उ तम ायालय दे श का सव अपीलीय ायालय है । सं िवधान के अनु े द-132 के

तहत सव ायालय को िन मामलों म अपीलीय े ािधकार ा ह-

सं वैधािनक मामलों म-अनु े द-132

दीवानी/िसिवल मामलों म - अनु े द-133

आपरािधक/फौज़दारी मामलों म-अनु े द-134

उ तम ायालय ारा दी गई िवशे ष इज़ाज़त से अपील-अनु े द-136

सं घ एवं रा ों के बीच िववाद उ तम ायालय के ारं िभक े ािधकार (Original

Jurisdiction) के अं तगत आता है । यह सव ायालय का अन े ािधकार है ।

सं िवधान के अनु े द-143 म उ तम ायालय से परामश करने की रा पित की श

का ावधान है ।

[20]सं िवधान के अनु े द-143 म सव ायालय के िलये परामशदा ी/सलाहकारी


अिधका रता का उपबं ध िकया गया है । इस सं दभ म िन िल खत कथनों पर िवचार
कीिजयेः

1. भारत के रा पित ारा सव ायालय को िविध अथवा त के िकसी


पर परामश के िलये मामला भे जा जा सकता है ।
2. सव ायालय ारा दी गई सलाह को रा पित मानने के िलये बा नहीं
है ।
3. सं िवधान पू व की सं िधयों एवं समझौतों से उ ए िववादों को भी रा पित
के ारा सव ायालय को भे जा जा सकता है ।

उपयु म से कौन-से कथन सही ह?

A) केवल 1 और 2

B) केवल 1 और 3

C) केवल 2 और 3

D) 1, 2 और 3
Hide Answer -
उ रः (d)

ा ाः उपयु तीनों कथन सही ह।

सं िवधान के अनु े द-143(1) के तहत रा पित िविध या त के पर सव ायालय

से परामश ले सकता है , िकंतु सव ायालय परामश दे ने के िलये बा नहीं है और न

ही रा पित सव ायालय के परामश को मानने के िलये बा है ।

सं िवधान के अनु े द 143(2) के तहत रा पित सं िवधान पू व की गई सं िधयों एवं समझौतों

से उ िववादों को सव ायालय को परामश के िलये भे ज सकता है । इस मामले म

सव ायालय को परामश दे ना अिनवाय है , िकंतु रा पित ऐसे परामश को मानने के

िलये बा नहीं है । अभी तक इस कार का मामला रा पित ारा िनिद नहीं िकया गया

है ।

सं िवधान के अनु े द 143(1) के तहत अब तक सव ायालय को 15 मामले भे जे जा

चु के ह, िजसम से एक केरल िश ा िवधे यक, 1958 है ।


[1] सं िवधान के अनु े द-129 के तहत भारत का सव ायालय एक अिभले ख
ायालय (Court of Record) है । इस सं दभ म िन िल खत कथनों पर िवचार
कीिजयेः

1. इसके सभी िनणयों का सा ा क मू होता है और इस पर िकसी भी


ायालय म िच नहीं लगाया जा सकता।
2. इसे अपनी अवमानना करने वालों को द त करने की श है ।

उपयु म से कौन-सा/से कथन सही है /ह?

A) केवल 1

B) केवल 2

C) 1 और 2 दोनों

D) न तो 1 और न ही 2

Hide Answer -
उ रः (c)

ा ाः उपयु दोनों कथन सही ह।

सं िवधान के अनु े द-129 के तहत सव ायालय को और अनु े द-215 के तहत

रा के उ ायालयों को अिभले ख ायालय (Court of Record) माना गया है ।

अनु े द-129 म ावधान है िक सव ायालय की कायवाही और उसके फैसले

सावकािलक अिभले ख व सा के प म रखे जाएं गे । इस पर िकसी भी ायालय म

िच नहीं लगाया जा सकता।

सव ायालय को ायालय की अवमानना करने वालों को द त करने की श है ।

इसके तहत 6 वष के िलये सामा जे ल या ` 2000 तक अथद या दोनों हो सकता

है । वष 1991 म उ ायालय ने यह िनणय िदया िक द दे ने की यह श न केवल

उ तम ायालय म िनिहत है , ब उ ायालयों, अधीन थ ायालयों एवं पं चाटों को

भी ा है ।

[2] सू ची-I को सू ची-II से सु मेिलत कीिजयेः

सू ची-I
(भारत के सं िवधान सू ची-II
के अनु े द) ( ावधान)

1. उ तम ायालय ारा घोिषत िविध भारत के


A. अनु े द-127
सभी ायालयों पर आब कर।

2.उ तम ायालय ार सु नाये गए िनणय या िदये


B. अनु े द-131
गए आदे श का पु निवलोकन।
C. अनु े द-137 3. उ तम ायालय की ारं िभक अिधका रता।

D. अनु े द-141 4. तदथ ायाधीश।

5. अिभले ख ायालय।

कूटः
A B C D

A) 5 3 2 1

B) 4 3 5 1

C) 4 3 2 1

D) 4 3 1 2

Hide Answer -
उ रः (c)

ा ाः

अनु े द-129 - अिभले ख ायालय।

अनु े द-127 - तदथ ायाधीशों की िनयु ।

अनु े द-131 - ारं िभक े ािधकार।

अनु े द-137 - यं के िनणय या िदये गए आदे श का पु निवलोकन।

अनु े द-141 - उ तम ायालय ारा घोिषत िविध।

भारत के रा े के भीतर सभी ायालयों पर आब कर ।

[3] भारतीय सं िवधान की मू ल सं रचना (Basic Structure) के सं दभ म िन िल खत

म से कौन-सा कथन सही है ?

A) भारत का सं िवधान अपनी मू ल सं रचना को प से थािपत करता है ।

सव ायालय ने सं िवधान की मू ल सं रचना को िव ार पू वक प रभािषत


B)
िकया है ।

भारत के िविध आयोग ने भारत के महा ायवादी की सहायता से भारतीय


C)
सं िवधान की मू ल सं रचना को प रभािषत िकया है ।

न ही सव ायालय ने और न ही सं सद ने सं िवधान की मू ल सं रचना को


D)
प रभािषत िकया है ।
Hide Answer -
उ रः (d)

ा ाः उपयु म से िवक (d) सही है ।

सं िवधान की मू ल सं रचना/ढाँ चा को न तो ायालय ारा और न सं सद ारा पा रत िकया

गया है । मू ल-ढाँ चा का िस ां त सव थम केशवान भारती बनाम केरल रा , 1973 वाद

म िदया गया। इसके बाद िमनवा िम बनाम भारत सं घवाद, 1980 वाद म ायालय ने

िनणय िदया िक ायालय समय-समय पर घोिषत करे गा िक सं िवधान का मू ल-ढाँ चा ा

है ।

[4] उ तम ायालय की परामश अिधका रता के िवषय म िन िल खत कथनों पर


िवचार कीिजयेः

1. भारत का उ तम ायालय े रण के आधार पर िविध अथवा त ों के


िवषय पर रा पित को परामश दे ता है ।
2. परामश अिधका रता की श के अधीन ा िकसी िनदश पर उ तम
ायालय की पू ण पीठ सु नवाई करती है ।
3. उ तम ायालय को उसकी परामश अिधका रता की श के अधीन
एक बार म केवल एक ही िनदश भे जा जा सकता है ।

उपयु म से कौन-सा/से कथन सही है /ह?

A) केवल 2

B) केवल 1 और 3

C) केवल 2 और 3

D) 1, 2 और 3
Hide Answer -
उ रः (a)

ा ाः उपयु म से कथन (1) और (3) गलत ह।

सं िवधान के अनु े द-143 के तहत रा पित उ तम ायालय से िविध या त के सं बंध

म परामश मां गता है , तो सव ायालय इस पर अपनी सलाह दे ता है , न िक े रण के

आधार पर सलाह दे ता है ।

यह आव क नहीं है िक रा पित एक बार म केवल एक ही िनदश सव ायालय के

पास परामश के िलये भे जेगा।

परामश अिधका रता की श के अधीन ा िनदशों की सु नवाई सव ायालय की

पाँ च सद ीय पू ण पीठ करती है ।

सं िवधान के िनवचन/ ा ा के सं बंध म िविध के सारवान से सं बंिधत िवषय पर िवचार

करने के िलये उ तम ायालय के ू नतम 5 या 5 से अधीक ायाधीशों की सं िवधान

पीठ गिठत की जाती है । अ मामलों के िनणय सामा तया तीन ायाधीशों की पीठ

करती है ।

[5] सं िवधान की ा ा से सं बंिधत सभी िववाद सव ायालय के िकस अिधकार


े के अं तगत लाया जा सकता है ?

A) अन े ािधकार

B) अपीलीय े ािधकार

C) परामशदायी े ािधकार

D) उपयु म से कोई नहीं।

Hide Answer -
उ रः (b)

ा ाः उपयु म से िवक (b) सही ह।

भारत के सव ायालय को मू ल(अन ), अनु े द-32 के तहत रट िनकालने ,

अपीलीय एवं परामश सं बंधी े ािधकार ा ह।

सं िवधान की ा ा से सं बंिधत सभी िववाद सव ायालय के अपीलीय े ािधकार के

अं तगत आते ह। अपीलीय े ािधकार के अं तगत अ शािमल हः- िसिवल, आपरािधक एवं

सं िवधान के अनु े द-136 से सं बंिधत अपील।

[6] भारत के सव ायालय के पास ाियक पु निवलोकन (Judicial Review)


की श है , िजसका आशय है ः
A) रा उ ायालयों के िनणयों के पु निवलोकन की श ।

B) मं ि प रषद के कायकरण के पु निवलोकन की श ।

C) भारत के रा पित को परामश दे ने की श ।

िवधानमं डल ारा पा रत एक अिधिनयम (अथवा उसके िकसी भाग) को


D)
सं वैधािनकता का िनणय दे ने की श ।

Hide Answer -
उ रः (d)

ा ाः उपयु म से िवक (d) सही है ।

भारत के सव ायालय के पास ाियक पु निवलोकन या ाियक समी ा की श है ।

इसके ारा सव ायालय के या रा दोनों रों पर िवधायी एवं कायकारी आदे शों

की सं वैधािनकता की जाँ च करता है । सं सद या रा िवधानमं डल ारा पा रत कोई िवधे यक

सं वैधािनक ावधानों का उ ं घन करता है तो ऐसी िविध/कानू न को सव ायालय

उ ं घन की सीमा तक अवै ध/शू घोिषत कर दे ता है ।

[7] िन िल खत म से कौन-सा एक कथन भारत म ाियक पु निवलोकन के वहार


के िवषय म सही नहीं है ?

िकसी मु कदमे को उसकी कानू नी वै धता के िलये उ तम ायालय के सम


A)
लाया जाना चािहये ।

िकसी कानू न को अकृत और शू घोिषत करने के िलये सभी ायाधीशों का


B)
एकमत होना आव क है ।

िवधायी अिधिनयमों और कायकारी आदे शों को उ तम ायालय र कर


C)
सकता है ।

D) यह श सं िवधान के अनु े द-13 के ावधानों म अ िनिहत है ।


Hide Answer -
उ रः (b)

ा ाः उपयु म से कथन (b) सही नहीं है ।

एक कानू न को अकृत/अवै ध/शू घोिषत करने के िलये सभी ायाधीशों का एक मत होना

आव क नहीं है । वे अपने ब मत के िनणय से िकसी भी कानू न को अकृत/अवै ध/शू

घोिषत कर सकते ह।

िकसी मु कदमे को उसकी कानू नी वै धता के िलये उ तम ायालय के सम लाया जाना

चािहये ।

सं सद या रा िवधानमं डल ारा पा रत कोई कानू न या कायकारी आदे शों की सं वैधािनकता

की जांँ च सव ायालय ारा की जा सकती है और िजस सीमा तक वह अिधिनयम/

आदे श सं िवधान का उ ं घन करता है , उस सीमा तक वह िविध या आदे श अवै ध या शू

होगा।

सं िवधान म कहीं भी प से ाियक पु निवलोकन की चचा नहीं िक गई है , िक ु

यह श सं िवधान के अनु े द-13 के ावधानों के तहत अ िनिहत है । इसम ावधान है

िक रा ऐसी कोई िविध नहीं बनाएगा, जो भाग-3 ारा द अिधकारों को छीनता हो या

ू न करता हो। यिद इसका उ ं घन करता है तो उ ं घन की सीमा तक िविध अवै ध या

शू होगी।

[8] ाियक पु निवलोकन के सं बंध म िन िल खत कथनों पर िवचार कीिजयेः

1. ाियक पु निवलोकन का ितपादन सव थम मारबरी बनाम मे िडसन वाद


म िकया गया।
2. भारत म ाियक पु निवलोकन के िस ां त को अमे रकी सं िवधान से े रत
होकर अपनाया गया।

उपयु म से कौन-सा/से कथन सही है /ह?

A) केवल 1

B) केवल 2

C) 1 और 2 दोनों

D) न तो 1 और न ही 2
Hide Answer -
उ रः (c)

ा ाः उपयु दोनों कथन सही ह।

ाियक समी ा/पु निवलोकन िस ां त की उ ि एवं िवकास अमे रका म आ। इस

िस ां त का ितपादन पहली बार मारबरी बनाम मे िडसन वाद, 1803 म आ। उस समय

अमे रका का मु ायमू ित जॉन माशल था।

भारत म ाियक पु निवलोकन के िस ां त को अमे रकी सं िवधान से े रत होकर हण

िकया गया। इसका तः उ े ख सं िवधान म कहीं नहीं है , िक ु सं िवधान के कुछ

अनु े दों के ावधानों म अ िनिहत है ।

[9] भारतीय सं िवधान म ाियक पु निवलोकन का िस ां त आधा रत है ः

A) िविध ारा थािपत ि या के अनु प।

B) िविध की स क ि या के अनु प।

C) िविध स त शासन पर आधा रत।

D) पू व िनणयों एवं पर राओं पर आधा रत।

Hide Answer -
उ रः (a)

ा ाः उपयु म से कथन (a) सही है ।

भारत म ाियक पु निवलोकन के िस ां त को िवशु प से िविध ारा थािपत ि या के

अनु प अपनाया गया है , ोंिक भारत के सं िवधान म जापान की इसी श ावली को

अपनाया गया है । िविध की उिचत ि या को अमे रका के सं िवधान से अपनाया गया है

[10]कोई भी सं िवधान (सं शोधन) कानू न भारतीय सव ायालय ारा असं वैधािनक
घोिषत िकया जा सकता है , यिद वहः

वतमान ि - रीय सं घीय व था के थान पर तीन- रीय व था लागू


A)
करता है ।

िविध के सम समानता के अिधकार को भाग-3 से हटाकर सं िवधान म कहीं


B)
और रखता है ।

कायका रणी की सं सदीय व था के थान पर अ ा क व था रखता


C)
है ।

सव ायालय के भार को कम करने हे तु एक सं घीय अपीलीय ायालय


D)
थािपत करता है ।
Hide Answer -
उ रः (b)

ा ाः उपयु म से िवक (b) सही है ।

केशवान भारती बनाम केरल रा वाद, 1973 म सव ायालय ने िनणय िदया िक

सं सद को सं िवधान के िकसी भी भाग म सं शोधन करने की श है , िकंतु सं िवधान के

मू ल/आधा रक ढाँ चे म सं शोधन नहीं कर सकती है । सं िवधान के भाग-3 के अनु े द-14

म िविध के सम समानता के अिधकार का ावधान है और ायालय ारा इसे सं िवधान

का मू ल ढाँ चा घोिषत िकया गया है । इसिलये िविध के सम समानता के अिधकार को

भाग-3 से हटाकर सं िवधान के िकसी अ थान पर रखता है , तो सव ायालय ऐसे

सं िवधान सं शोधन को असं वैधािनक/अवै ध/शू घोिषत कर सकता है ।

िमनवा िम बनाम भारतसं घ वाद, 1980 म सव ायालय ने ाियक समी ा या

पु निवलोकन को सं िवधान का मू ल ढाँ चा माना है । सं सद सं िवधान सं शोधन के ारा इसे छीन

नहीं सकती।

वतमान म (वष 2015) सव ायालय ने 99व सं िवधान सं शोधन अिधिनयम, 2014

(रा ीय ाियक िनयु आयोग) को असं वैधािनक घोिषत कर िदया है ।

[11]िन िल खत कथनों पर िवचार कीिजयेः

1. पहले सं िवधान सं शोधन अिधिनयम के ारा सं िवधान म अनु े द-31(ख)


एवं नौवीं अनु सूची जोड़ी गई।
2. वतमान म नौवीं अनु सूची म शािमल िकसी िवषय की ाियक समी ा
सव ायालय नहीं कर सकता।

उपयु म से कौन-सा/से कथन सही है /ह?

A) केवल 1

B) केवल 2

C) 1 और 2 दोनों

D) न तो 1 और न ही 2
Hide Answer -
उ रः (a)

ा ाः उपयु म से कथन (2) गलत है ।

वतमान म सं िवधान की 9वीं अनु सूची म शािमल िकसी िवषय/कानू न की ाियक समी ा

की जा सकती है । कोए ो बनाम तिमलनाडु रा वाद, 2007 म सव ायालय ने

िनणय िदया िक ायालय 24 अ ै ल, 1973 के प ात् डाले गए सभी िवषयों/िविधयों की

ाियक समी ा कर सकेगी। 24 अ ै ल, 1973 को पहली बार केशवान भारती बनाम

केरल रा वाद म ायालय ने मू ल ढाँ चा के िस ां त को िदया।

सरकार ने पहले सं िवधान सं शोधन के ारा अनु े द-31 (ख) एवं 9वीं अनु सूची को जोड़ा,

तािक िविध को ाियक समी ा से सु रि त िकया जा सके। मू ल प से 9वीं अनु सूची म

13 अिधिनयमों को शािमल िकया गया था और वष 2016 तक इसकी सं ा बढ़कर

282 हो गई है ।

[12]जनिहत यािचका (Public Interest Litigation - PIL) के सं दभ म

िन िल खत कथनों पर िवचार कीिजयेः

1. इसकी सं क ना की उ ि एवं िवकास सं यु रा अमे रका म आ।


2. भारत म इसकी उ ि 1990 के दशक म ई।
3. भारत म इसके वतक ायमू ित वी.आर. कृ ा अ र एवं मु
ायमू ित पी. एन. भगवती माने जाते ह।
4. इसे केवल सव ायालय म ु त िकया जा सकता है ।

उपयु म से कौन-सा/से कथन सही है /ह?

A) केवल 1

B) केवल 1, 2 और 3

C) केवल 1 और 3

D) केवल 1, 3 और 4

Hide Answer -
उ रः (c)

ा ाः उपयु म से कथन (2) और (4) गलत ह।

भारत म इसकी उ ि 1980 के दशक के म के ई,न िक 1990 के दशक म।

भारत म जनिहत यािचका के वतक ायमू ित वी.आर. कृ ा अ र एवं मु ायमू ित

पी.एन. भगवती माने जाते ह।

वी.आर. कृ ा अ र उस समय भारत के मु ायमू ित नहीं थे ।

जनिहत यािचकाएंँ सव एवं उ दोनों ायालयों म ु त की जा सकती ह।

जनिहत यािचका िस ां त की उ ित एवं िवकास सं यु रा अमे रका म 1960 के दशक

म आ।
[13]जनिहत यािचका की व था भारत म कैसे ारं भ की गई?

A) सं िवधान म सं शोधन के मा म से ।

B) सं सदीय अिधिनयम के ारा।

C) ाियक पहल के ारा।

D) राजनीितक दलों के ारा।

Hide Answer -
उ रः (c)

ा ाः उपयु म से कथन (c) सही है ।

भारत म 1980 के दशक म सव ायालय की पहल पर जनिहत यािचका/लोकिहतवाद

की शु आत ई। त ालीन सव ायालय के मु ायमू ित पी.एन. भगवती एवं

ायमू ित वी.आर. कृ ा अ र ारा जनिहत यािचका को पहली बार ीकार िकया गया।

पीपु यू िनयन फॉर डे मो े िटक राइट् स (ऐिशया वकस केस), 1982 से जनिहत यािचका

को आधार िमला।

[14]जनिहत यािचका के सं बंध म नीचे िदये गए कथनों पर िवचार कीिजयेः

1. सार प से कोई तृ तीय प ायालय के सम जनिहत के मामले ुत


कर सकता है ।
2. सव ायालय, िकसी नाग रक ारा प अथवा पो काड ारा अपने
मौिलक अिधकारों के सं र ण के िलये अनु रोध करने पर कायवाही कर
सकता है ।

उपयु म से कौन-सा/से कथन सही है /ह?

A) केवल 1

B) केवल 2

C) 1 और 2 दोनों

D) न तो 1 और न ही 2
Hide Answer -
उ रः (c)

ा ाः उपयु दोनों कथन सही ह।

इसके अ गत कोई भी जनभावना वाला या सामािजक सं गठन िकसी भी /

यों के समू हों को अिधकार िदलाने के िलये ायालय जा सकता है । अथात् कोई भी

तृ तीय प ायालय के सम जनिहत के मामले ला सकता है ।

सव ायालय िकसी नाग रक के प अथवा पो काड ारा अपने सं वैधािनक या कानू नी

अिधकारों के सं र ण के िलये अनु रोध करने पर कायवाही कर सकता है ।

जनिहत यािचका को िकसी एक के अ के खलाफ अपने अिधकार का

दावा और इसे लागू करने के िलये दा खल नहीं िकया जा सकता, अिपतु इसका आशय

सावजिनक िहत को आगे बढ़ाना और र ा करना होता है ।

[15]सावजिनक िहत के िलये ायालय ारा यािचका ीकार करने के सं बंध म


िन िल खत कथनों पर िवचार कीिजये । यिद उसे ािवत िकया गया है ः

1. एक ारा, जब उसके िहत भािवत हो रहे हों।


2. एक समू ह ारा, जब उसके सद ों के िहत भािवत हो रहे हों।
3. एक अथवा समू ह ारा, जब आमतौर पर जनसाधारण के िहत
भािवत हो रहे हों।
4. एक ारा एक समू ह की सहायता से , जब समू ह के िहत भािवत हो
रहे हों।

उपयु म से कौन-सा/से कथन सही है /ह?

A) केवल 3

B) केवल 1, 3 और 4

C) केवल 4

D) 1, 2, 3 और 4
Hide Answer -
उ रः (a)

ा ाः उपयु म से कथन (1), (2) और (4) गलत ह।

भारत के सव ायालय ने जनिहत यािचका के सं बंध म िन िल खत िस ां त िवकिसत

िकये ह-

लोक क ाण म िच रखने वाले िकसी की यािचका को ीकार िकया जा सकता

है , ोंिक समाज का कमज़ोर वग इस थित म नहीं है िक वह यं अदालत का दरवाजा

खटखटा सके।

सावजिनक मह के मु े , िजनसे बड़ी सं ा म लोगों के मू ल अिधकार भािवत होते ह,

ायालय एक प अथवा तार को जनिहत यािचका (पी.आई.एल.) के प म ीकार कर

सकता है ।

ायालय, गरीबों, िनर रों तथा िनःश ों की ओर से दायर िशकायतों की सु नवाई करता

है , ोंिक ये लोग अपने सं वैधािनक या वै धािनक अिधकारों के उ ं घन के िलये वै धािनक

अघात के िनवारण म यं स म नहीं होते ।

[16]लोकिहत/जनिहत मु कदमों के अिभ ाय के सं बंध म िन िल खत कथनों पर िवचार

कीिजयेः

1. लोक ित का िनवारण।
2. लोक कर् त ों का वतन।
3. िवस रत अिधकारों (Defused Rights) का सं र ण।
4. िविध वतक अिभकरण की भ ना।

उपयु म से कौन-से कथन सही ह?

A) केवल 1, 2 और 3

B) केवल 1, 3 और 4

C) केवल 2, 3 और 4

D) 1, 2 और 4

Hide Answer -
उ रः (b)

ा ाः उपयु म से कथन (2) सही नहीं है ।

जनिहत यािचका के तहत ायालय ारा ऐसे कारणों, िजनके तहत बड़ी सं ा म लोगों के

िहत भािवत होते ह तथा उनके अिधकार व िहत दाँं व पर लगे होते ह, िकंतु गरीबी और

अ ानता के कारण ायालय तक नहीं प ँ च पाते ह, यािचका ीकार की जाती है ।

जनिहत यािचका के ारा जनसाधारण के िवस रत अिधकारों का सं र ण हो पाता है ।

इसके ारा िविध वतक अिभकरणों की भ ना होती है , ोंिक ये अपने कत ों का पालन

सही तरीके से नहीं कर पाते ।


[17]िन िल खत म से िकस/िकन का सं बंध भारत म ाियक सि यतावाद (Judicial
Activism) से रहा है /ह?

1. ायमू ित वी.आर. कृ ाअ र
2. ायमू ित पी.एन. भगवती
3. ायू मित ओ. िच ा रे ी
4. ायमू ित डी.ए. दे साई

नीचे िदये गए कूट का योग कर सही उ र का चयन कीिजयेः

A) केवल 1

B) केवल 1 और 2

C) केवल 1, 2 और 4

D) 1, 2, 3 और 4

Hide Answer -
उ रः (d)

ा ाः उपयु सभी का सं बंध ाियक सि यतावाद से रहा है ।

भारत म ाियक सि यता का िवकास 1970 के दशक म आ। भारत म इसकी नींव

ायमू ित वी.आर. कृ ा अ र, ायमू ित पी.एन. भगवती, ायमू ित ओ. िच ा रे ी

एवं ायमू ित डी.ए. दे साई के ारा रखी गई।

[18] ाियक सि यतावाद का सं बंध िन िल खत म से िकससे है ?

A) ाियक पु नरावलोकन

B) ाियक तं ता

C) ाियक ितब ता

D) लोकिहतवाद

Hide Answer -
उ रः (d)

ा ाः उपयु म से कथन (d) सही है ।

ाियक सि यतावाद का सं बंध जनिहत यािचका/लोकिहतवाद से है । भारत म ाियक

सि यता का िस ां त 1970 के दशक म आया

[19]भारत म ाियक सि यता के िवषय म िन िल खत कथनों पर िवचार कीिजयेः

1. यह मु तः तब अ म आता है , जब िवधानमं डल और सरकार के


अिधशासी ं ध सं िवधान के अ गत अपने कत ों के िनवहन म असफल
होते ह।
2. ज स ए. एच. अहमदी ने इसकी शु आत की,जब उ ोंने एक पीिड़त
नाग रक ारा िलखे गए प को लोकिहत यािचका के प म बदल िदया।
3. ाियक सि यता के अिधकां श मामले िनचली अदालतों के िनणयों के
िव अपीलों के मा म से अ म आए।
4. ाियक सि यतावाद म ायालय िवचार के दौरान वाद की ि या की
मॉिनट रं ग करता है और जाँ च अिभकरणों को आव क िनदश दे ता है ।

उपयु म से कौन-से कथन सही है /ह?

A) केवल 1 और 2

B) केवल 1 और 4

C) केवल 1, 2 और 4

D) 1, 3 और 4

Hide Answer -
उ रः (b)

ा ाः उपयु म से कथन (2) और (3) सही नहीं ह।

भारत म ाियक सि यता की नींव 1970 के दशक म ायमू ित वी.आर. कृ ा अ र,

ायमू ित पी.एन. भगवती, ायमू ित ओ. िच ा रे ी तथा ायमू ित डी.ए. दे साई ने रखी।

ाियक सि यता की अवधारणा का िवकास अमे रका म आ और इस श ावली का

योग वष 1947 म अमे रकी इितहासकार आथर शले िसं गर जू िनयर ने िकया।

ाियक सि यता का अथ ायालय ारा अपने अिधकार े से बाहर िनकलकर

कायपािलका एवं िवधाियका के अिधकार े म वे श करना है । यह तब होता है , जब

िवधानमं डल और सरकार के अिधशासी ं ध सं िवधान के अ गत अपने कत ों का

िनवाह करने म असफल रहते ह।

ाियक सि यतावाद म ायालय िवचार के दौरान जनिहत यािचका की ि या की

मॉिनट रं ग करता है और जाँ च अिभकरणों को आव क िनदश दे ता है ।

जब जनसाधारण अपनी गरीबी, िनर रता और अ ानता की वज़ह से अपने अिधकारों एवं

िहतों की र ा नहीं कर पाता और न ही ायालय तक प ँ च पाता है , तो ऐसी थित म

कोई तीसरा प एक पो काड या तार के मा म से सव ायालय म यािचका दायर

कर सकता है । अतः यह गलत है िक ाियक सि यता के अिधकां श मामले िनचली

अदालतों के िनणय के िव अपील के मा म से अ म आये ह।

[20] ायालय की ‘प अिधका रता’ वा ां श का िनिहताथ ा है ?

िकसी औपचा रक ितवे दन के न होते ए भी े रणा से कारवाई करने की


A)
श ।

B) ाियक ि या म ढील।

C) रट यािचका के काय- े म िव ार।

D) िकसी मु कदमे का सं वैधािनक ख पीठ को थाना रण।


Hide Answer -
उ रः (c)

ा ाः ायालय, जनिहत यािचका मामले म िकसी तीसरे प के ारा भे जे गए प को मा ता

दे ता है ,िजसके कारण ायालय की रट यािचका के काय े म िव ार आ है ।


[1] भारतीय सव ायालय के िवषय म िन िल खत म से कौन-से कथन सही ह?
1. इसे भारत के भीतर िकसी भी ायालय या ायािधकरण के िनणय के िव
अपील पर िवचार करने का अिधकार है ।
2. इसे रा पित ारा े िषत िकसी िविध या त के पर परामश दे ने का
अिधकार है ।
3. सव ायालय के ायाधीशों का वे तन सं सद के मतदान के अधीन है ।
4. इसकी अिधका रता भारत के भीतर सभी ायालयों के िलये बा कारी है ।
नीचे िदये गए कूट का योग कर सही उ र का चयन कीिजयेः

A) केवल 1, 2 और 3

B) केवल 1, 2 और 4

C) केवल 1, 3 और 4

D) 1, 2, 3 और 4

Hide Answer -
उ रः (b)
ा ाः उपयु म से कथन (3) सही नहीं है ।

सव ायालय के ायाधीशों का वे तन भारत की सं िचत िनिध पर भा रत होता है


और इसे सं सद म मतदान के िलये नहीं रखा जाता।
सव ायालय म भारत के भीतर िकसी भी ायालय या ायािधकरण के िनणय
के िव अपील की जा सकती है ।
अपील िन िल खत मामलों म की जा सकती है ः

सं वैधािनक
दीवानी
फौज़दारी
अनु े द-136 के तहत सव ायालय की िवशे ष अनु मित ारा।

अनु े द-143 के तहत सव ायालय िविध या त के पर रा पित को


परामश दे ता है , िकंतु रा पित के मां गने पर।
अनु े द-141 के तहत इसकी अिधका रता भारत के भीतर सभी ायालयों के िलये
बा कारी है ।

[2] भारतीय सव ायालय के सं दभ म िन िल खत कथनों पर िवचार कीिजयेः


1. उ तम ायालय का ायाधीश बनने के िलये िकसी उ ायालय म कम-
से -कम 10 वष तक ायाधीश होना चािहये ।
2. कोट माशल की अपील सव ायालय म नहीं की जा सकती है ।
3. उ तम ायालय कोट माशल के साथ अ उ ायालयों/अदालतों की
सु नवाई कर सकता है ।
4. उ तम ायालय ारा केशवान भारती केस म गिठत सं वैधािनक पीठ अब
तक की सबसे बड़ी सं वैधािनक पीठ है ।
उपयु कथनों म से कौन-से सही ह?

A) केवल 1 और 2

B) केवल 1 और 3

C) केवल 2 और 4

D) केवल 3 और 4

Hide Answer -
उ रः (d)
ा ाः उपयु म से कथन (1) और (2) सही नहीं ह।

उ तम ायालय का ायाधीश बनने के िलये उसे भारत का नाग रक होना


चािहये ।
िकसी उ ायालय का या ऐसे दो या अिधक ायालयों का लगातार कम-से -कम
5 वष तक ायाधीश रहा हो।
िकसी उ ायालय का या ऐसे दो या अिधक ायालयों का लगातार कम-से -कम
10 वष तक अिधव ा (Advocate) रहा हो।
रा पित की राय म पारं गत िविधवे ा हो।
सश से ना ायािधकरण अिधिनयम, 2007 के ावधान के अनु सार कोट माशल
की अपील सव ायालय म की जा सकती है ।
केशवान भारती बनाम केरल रा केस, 1973 म उ तम ायालय ने 13
ायाधीशों की ख पीठ का गठन िकया। यह अब तक की सबसे बड़ी सं वैधािनक
ख पीठ है , िजसम 7 ायाधीश प एवं 6 ायाधीश िवप म थे । उ तम
ायालय ारा दू सरी सबसे बड़ी ख पीठ गोलकनाथ बनाम पं जाब रा केस,
1967 म गिठत की गई। यह 11 ायाधीशों की ख पीठ थी, िजसम 6 ायाधीश
प म एवं 5 ायाधीश िवप म थे ।

[3] भारत के उ तम ायालय की सभी कायवािहयाँ अं े ज़ी भाषा म होंगी, का


ावधान िकया गया है -

A) उ तम ायालय कानू न, 1966 के ारा।

B) भारतीय सं िवधान के अनु े द-145 के ारा।

C) भारतीय सं िवधान के अनु े द-348 के ारा।

D) सं सद ारा बनाए गए अिधिनयम के ारा।


Hide Answer -
उ रः (c)

ा ाः उपयु म से िवक (c) सही है ।

सं िवधान के अनु े द-348 म उपबं ध िकया गया है िक उ तम ायालय और उ

ायालयों की सभी कायवािहयाँ अं े ज़ी भाषा म होंगी।

[4] भारत के उ ायालय के सं दभ म िन िल खत कथनों पर िवचार कीिजयेः


1. भारत म उ ायालय का गठन सव थम तीन ां तों कलक ा, ब ई एवं
बं गलौर म िकया गया।
2. वतमान म भारत म उ ायालयों की कुल सं ा 25 है ।
3. िकसी भी सं घ रा े का अपना उ ायालय नहीं है ।
उपयु कथनों म से कौन-सा/से सही है /ह?

A) केवल 3

B) केवल 1 और 2

C) 1, 2 और 3

D) उपयु म से कोई नहीं।

Hide Answer -
उ रः (d)
ा ाः उपयु म से कोई भी कथन सही नहीं है ।

भारत म सव थम उ ायालय का गठन 1862 म तीन ां तों कलक ा, ब ई


और म ास (न िक बं गलौर) म िकया गया। वष 1866 म इलाहाबाद म चौथे उ
ायालय की थापना की गई।
वतमान म भारत म उ ायालयों की सं ा 24 है । वष 2013 म पू व र के तीन
रा ों म तीन उ ायालयों की थापना की गई, िजसके फल प उ
ायालयों की कुल सं ा 21 से बढ़कर 24 हो गई। िन तीन रा ों म उ
ायालयों की थापना की गईः

1. मिणपु र - 25 माच, 2013


2. मे घालय - 23 माच, 2013
3. ि पु रा - 26 माच 2013

सं घ रा े िद ी का अपना उ ायालय है । इसकी थापना 31 अ ू बर,


1966 को की गई। िद ी एकमा ऐसा सं घ रा े है , िजसका अपना उ
ायालय है ।

[5] िन िल खत कथनों पर िवचार कीिजयेः

1. पं जाब, ह रयाणा और के -शािसत दे श चं डीगढ़ का एक सामू िहक उ


ायालय है ।
2. अ मान एवं िनकोबार ीप समू ह पर म ास उ ायालय की अिधका रता
है ।
3. ल ीप पर केरल उ ायालय की अिधका रता है ।
उपयु कथनों म से कौन-सा/से सही है /ह?

A) केवल 1

B) केवल 1 और 3

C) केवल 2 और 3

D) 1, 2 और 3

Hide Answer -
उ रः (b)

ा ाः उपयु म से कथन (2) सही नही ं है ।

अ मान िनकोबार ीप समू ह पर कलक ा उ ायालय की अिधका रता है , न िक

म ास उ ायालय की।

पं जाब, ह रयाणा और सं घ-शािसत दे श चं डीगढ़ का एक ही सामू िहक उ ायालय है ।

ल ीप पर केरल उ ायालय की अिधका रता है ।

पु डुचे री पर म ास उ ायालय की अिधका रता है ।

दमन और दीव, दादर एवं नागर हवे ली तथा गोवा पर ब ई उ ायालय की

अिधका रता है ।

नागालड, िमज़ोरम और अ णाचल दे श गु वाहाटी उ ायालय के अं तगत आते ह।

आं दे श एवं ते लंगाना का एक ही उ ायालय है ।

सात उ ायालयों का े ािधकार एक से अिधक रा ों एवं क -शािसत दे शों पर है ।

[6] िन िल खत कथनों पर िवचार कीिजयेः

1. भारत म तीन उ ायालयों की अिधका रता एक से अिधक रा ों पर है ।


2. ब ईउ ायालय की अिधका रता एक से अिधक सं घ-रा े ों पर है ।
उपयु कथनों म से कौन-सा/से सही है /ह?

A) केवल 1

B) केवल 2

C) 1 और 2 दोनों

D) न तो 1 और न ही 2
Hide Answer -
उ रः (b)

ा ाः उपयु म से कथन (1) गलत है ।

भारत म चार उ ायालयों की अिधका रता एक से अिधक रा ों पर है , िजसम िन शािमल

ह-

1. पं जाब और ह रयाणा उ ायालय

2. ब ई उ ायालयः महारा और गोवा

3. गु वाहाटी उ ायालयः अ णाचल दे श, असम, नागालड एवं िमज़ोरम

4. आं दे श एवं ते लंगाना उ ायालय

ब ई उ ायालय की अिधका रता के अं तगत सबसे अिधक के शािसत दे श आते ह-

दमन एवं दीव तथा दादर एवं नागर हवे ली।

कलक ा उ ायालय के अं तगत एक सं घ शािसत दे श आता है ।

म ास उ ायालय के अं तगत एक सं घ रा े आता है ।

पं जाब एवं ह रयाणा उ ायालय के अं तगत एक सं घ रा े आता है ।

िद ी सं घ रा े का अपना उ ायालय है ।

[7] िन िल खत कथनों पर िवचार कीिजयेः

1. भारत के सं िवधान म उपबं ध है िक े क रा के िलये एक उ ायालय


होगा।
2. भारत के सं िवधान म उपबं ध है िक रा पित दो या अिधक रा ों के िलये
अथवा दो या अिधक रा ों और िकसी सं घ-रा े के िलये एक ही उ
ायालय थािपत कर सकता है ।
उपयु कथनों म से कौन-सा/से सही है /ह?

A) केवल 1

B) केवल 2

C) 1 और 2 दोनों

D) न तो 1 और न ही 2

Hide Answer -
उ रः (a)

ा ाः उपयु म से कथन (2) गलत है ।

सं िवधान के अनु े द-231 म ावधान है िक सं सद (न िक रा पित) िविध ारा दो या

अिधक रा ों के िलये अथवा दो या अिधक रा ों और िकसी सं घ-रा े के िलये एक

ही उ ायालय थािपत कर सकती है ।

सं िवधान के अनु े द-214 म उपबं ध है िक े क रा के िलये एक उ ायालय

होगा।

[8]
िन िल खत कथनों पर िवचार कीिजयेः
1. सं िवधान म उपबं ध है िक ेक उ ायालय एक मु ायमू ित और ऐसे
अ ायाधीशों से िमलकर बने गा, िज सं सद िविध ारा समय-समय पर तय
करे गी।
2. सं िवधान म उ ायालय के ायाधीशों की सं ा के सं बंध म ावधान नहीं
िकया गया है ।
उपयु कथनों म से कौन-सा/से सही है /ह?

A) केवल 1

B) केवल 2

C) 1 और 2 दोनों

D) न तो 1 और न ही 2

Hide Answer -
उ रः (b)

ा ाः उपयु म से कथन (1) सही नहीं है ।

सं िवधान के अनु े द-216 म उ ायालय के गठन के सं बंध म ावधान िकया गया है ।

इसम कहा गया है िक ेक उ ायालय एक मु ायमू ित और ऐसे अ

ायाधीशों से िमलकर बने गा, िज रा पित (न िक सं सद) समय-समय पर िनयु करना

आव क समझे ।

सं िवधान म उ ायालय के ायाधीशों की सं ा के सं बंध म कुछ नहीं कहा गया है ।

इसे रा पित के िववे क पर छोड़ िदया गया है । अतः रा पित काय की आव कतानु सार

समय-समय पर इनकी सं ा िनधा रत करता है ।

[9] भारत के उ ायालय के ायाधीशों की िनयु भारत के रा पित ारा की


जाती है । इस सं दभ म िन िल खत कथनों पर िवचार कीिजयेः
1. उ ायालय के मु ायाधीश की िनयु करते समय रा पित भारत के
मु ायमू ित और सं बंिधत रा के रा पाल से परामश ले ता है ।
2. उ ायालय के अ ायाधीशों की िनयु म रा पित सं बंिधत उ
ायालय के मु ायमू ित एवं सं बंिधत रा के रा पाल से परामश ले ता है ।
3. दो या अिधक रा ों के साझा उ ायालयों म अ ायाधीशों की िनयु
म रा पित उ ायालय के मु ायमू ित एवं सभी सं बंिधत रा ों के
रा पालों से परामश करता है ।
4. भारत का मु ायमू ित उ तम ायालय के चार व र तम ायाधीशों से
परामश के प ात् ही रा पित को परामश दे गा।
उपयु कथनों म से कौन-से सही ह?
A) केवल 1 और 3

B) केवल 1, 3 और 4

C) केवल 1, 2 और 4

D) 1, 2, 3 और 4

Hide Answer -
उ रः (a)

ा ाः उपयु म से कथन (2) और (4) सही नहीं ह।

सं िवधान के अनु े द-217 म उ ायालय के ायाधीशों की िनयु के सं बंध म

ावधान िकया गया है ।

उ ायालय के अ ायाधीशों की िनयु के सं बंध म रा पित सं बंिधत उ

ायालय के मु ायमू ित से परामश करे गा, िक ु दो या अिधक रा ों के साझा उ

ायालय के ायाधीशों की िनयु म रा पित सभी सं बंिधत रा ों के रा पालों से भी

परामश करे गा।

उ ायालय के अ ायाधीशों की िनयु म भारत का मु ायमू ित उ तम

ायालय के दो व र तम (न िक चार) ायाधीशों से परामश के प ात् ही रा पित को

परामश दे गा। इसे ‘कॉले िजयम णाली’ कहते ह। 99व सं िवधान सं शोधन अिधिनयम,

2014 के ारा उ तम ायालय एवं उ ायालयों म ायाधीशों की िनयु म

‘कॉले िजयम णाली’ के थान पर रा ीय ाियक िनयु आयोग (NJAC) ित थािपत

िकया गया, िक ु वष 2015 म सव ायालय ने फोथ जजे ज़ केस, 2015 म इसे

असं वैधािनक घोिषत कर िदया।

उ ायालय के मु ायमू ित की िनयु म रा पित भारत के मु ायमू ित और

सं बंिधत रा के रा पाल से परामश करे गा।

[10]भारत के उ ायालय के ायाधीशों की यो ता के सं दभ म िन िल खत कथनों पर


िवचार कीिजयेः
1. भारत के रा े म कम-से -कम पाँ च वष तक ाियक पद धारण कर चु का हो।
2. उ ायालय या ऐसे दो या अिधक ायालयों का लगातार कम-से -कम दस वष
तक अिधव ा (Advocate) रहा हो।
3. रा पित की राय म पारं गत िविधवे ा हो।
उपयु कथनों म से कौन-सा/से सही है /ह?

A) केवल 2

B) केवल 1 और 2

C) केवल 2 और 3

D) 1, 2 और 3
Hide Answer -
उ रः (a)

ा ाः उपयु कथनों म से (1) और (3) सही नही ं ह।

सं िवधान के अनु े द-217 म उ ायालय के ायाधीश बनने के िलये यो ता के सं बंध म

ावधान िन िल खत ह-

उसे भारत का नाग रक होना चािहये ।

भारत के रा े म कम-से -कम 10 वष (न िक 5 वष तक) तक ाियक पद धारण

कर चु का हो।

उ ायालय या ऐसे दो या अिधक ायालयों का लगातार कम-से -कम 10 वष तक

महािधव ा/वकील रहा हो।

रा पित की राय म पारं गत िविधवे ा हो, उ ायालय का ायाधीश बनने के

यो है , के सं बंध म सं िवधान म उ े ख नहीं िकया गया है ।

[11]भारत म उ ायालय के ायाधीश के सं दभ म िन िल खत कथनों म से कौन-सा


सही है ?

उ ायालय का ेक ायाधीश अपना पद हण करने से पहले रा के


A) रा पाल के सम सं िवधान की तीसरी अनु सूची म िदये गए ा प से
अनु सार शपथ या ित ान करे गा।

ेक ायालय का ायाधीश छः वष या 62 वष की आयु , जो भी पहले हो,


B)
तक पद धारण करे गा।

उ ायालय के िकसी ायाधीश की आयु के सं बंध म कोई उठता है ,


C) तो उस का िनणय रा पित भारत के मु िनवाचन आयु से परामश
करने के बाद करे गा और उसका िनणय अ म होगा।

उ ायालय के ायाधीशों के वे तन, भ े और पशन रा की सं िचत िनिध


D)
पर भा रत होते ह।
Hide Answer -
उ रः (a)
ा ाः उपयु म से िवक (a) सही है ।

सं िवधान के अनु े द-219 म उ ायालय के ायाधीशों के शपथ या ित ान के


सं बंध म उपबं ध है । इसम ावधान है िक ेक उ ायालय का ायाधीश
सं िवधान की तीसरी अनु सूची म िदये गए ा प के अनु सार रा के रा पाल के
सम शपथ या ित ान करे गा।
सं िवधान के अनु े द-217 के तहत ेक उ ायालय का ायाधीश 62 वष
की आयु तक पद धारण करे गा।15व सं िवधान सं शोधन अिधिनयम, 1963 के ारा
से वािनवृ ि की आयु 60 वष से बढ़ाकर 62 वष कर दी गई। इसके ायाधीशों की
से वािनवृ ि की आयु 62 वष से बढ़ाकर 65 वष करने हे तु 114वाँ सं िवधान
सं शोधन अिधिनयम, 2010 सं सद म िवचाराधीन है ।
अनु े द-217 के तहत उ ायालय के िकसी ायाधीश की आयु के सं बंध म
उठता है तो उस का िनणय रा पित भारत के मु ायमू ित (न िक भारत
के िनवाचन आयु ) से परामश करने के प ात् करे गा और रा पित का िनणय
अ म होगा।
सं िवधान के अनु े द-221 म उ ायालय के ायाधीशों के वे तन, भ े एवं
पशन के सं बंध म ावधान है । इसके ायाधीशों के वे तन व भ े रा की सं िचत
िनिध [अनु े द-202 (3)(घ)] पर भा रत होते ह, िक ु पशन भारत की सं िचत
िनिध [अनु े द-112 (3)(घ)] पर भा रत होती है । रा की सं िचत िनिध या भारत
की सं िचत िनिध पर भा रत य पर रा िवधानमं डल या सं सद म मतदान नहीं
होता है ।
इसके ायाधीशों के वे तन,भ े एवं पशन सं सद िविध ारा समय-समय पर िनधा रत
करे गी, तक तब तक सं िवधान की दू सरी अनु सूची के तहत दे य होंगे।

[12]िन िल खत कथनों पर िवचार कीिजयेः

1. भारत म उ ायालय के ायाधीश को हटाने की रीित, उ तम ायालय


के ायाधीश को हटाने की रीित के समान है ।
2. उ ायालय का कोई थायी ायाधीश अपने पद से से वािनवृ ि के प ात्
भारत के िकसी भी ायालय या िकसी ािधकारी के सम अिभवचन या काय
नहीं कर सकता।
उपयु कथनों म से कौन-सा/से सही है /ह?

A) केवल 1

B) केवल 2

C) 1 और 2 दोनों

D) न तो 1 और न ही 2
Hide Answer -
उ रः (a)

ा ाः उपयु म से कथन (2) सही नही ं है ।

सं िवधान के अनु े द-220 म ावधान है िक उ ायालय का कोई थायी ायाधीश

अपने पद से से वािनवृ ि के प ात् उ तम ायालय और उ ायालयों (िजस उ

ायालय म िनयु थे उससे िभ उ ायालय) के िसवाय भारत म िकसी ायालय या

िकसी ािधकारी के सम अिभवचन या काय नहीं करे गा।

सं िवधान के अनु े द-217(1)(ख) म ावधान है िक उ ायालय के ायाधीश को

अनु े द-124 (4) म उपबं िधत रीित से उसके पद से रा पित ारा हटाया जा सकेगा।

इसे पद से हटाने सं बंधी दो आधार ह- िस कदाचार एवं असमथता।

ायाधीश जाँ च अिधिनयम, 1968 म उ ायालय के िकसी ायाधीश को महािभयोग

के ारा हटाने की ि या का ावधान है ।

अब तक उ ायालय के िकसी ायाधीश पर महािभयोग नहीं लगाया गया है । वष

2011 म कलक ा उ ायालय के त ालीन ायाधीश सौिम से न के खलाफ

महािभयोग का ाव लाया गया था, िकंतु उसने ागप दे िदया था। तं भारत म

िकसी ायमू ित के खलाफ ऐसा ाव पे श होने का यह पहला मामला है ।

[13]भारत म चिलत ायालय (Mobile Court) िन िल खत म से िकसका मानसपु है ?

A) ायमू ित पी. एन. भगवती

B) ायमू ित वी. आर. कृ ाअ र

C) डॉ. ए.पी.जे . अ ु ल कलाम

D) राजीव गां धी

Hide Answer -
उ रः (c)

ा ाः उपयु म से िवक (c) सही है ।

भारत म चिलत ायालय (Mobile Court) की प रक ना सव थम डॉ. ए.पी.जे .

अ ु ल कलाम ने की थी। 4 अग , 2007 को ह रयाणा के मे वात िज़ले म थत पु ाना

म दे श की पहली मोबाइल (सघन) अदालत का उद् घाटन िकया गया। इसका शु भारं भ

भारत के त ालीन मु ायाधीश के. जी. बालाकृ न ने िकया। इसका उ े सु दूर

एवं िपछड़े इलाकों तक ाियक व था का लाभ प ँ चाना है ।

[14]सू ची-I को सू ची-II से सु मेिलत कीिजयेः

सू ची-I
सू ची-II
(भारतीय सं िवधान के
( ावधान)
अनु े द)

A. अनु े द-222 1. उ ायालयों की बै ठकों म से वािनवृ ायाधीशों


की िनयु ।

2. उ ायालय म अित र और कायकारी


B. अनु े द-223
ायाधीशों की िनयु ।

3. उ ायालय म कायकारी मु ायमू ित


C. अनु े द-224
की िनयु ।

4. िकसी ायाधीश का एक उ ायालय से दू सरे


D. अनु े द-224(क)
उ ायालय म थानां तरण।

नीचे िदये गए कूट का योग कर सही उ र का चयन कीिजयेः


A B C D

A) 4 3 2 1

B) 4 2 3 1

C) 1 2 3 4

D) 1 3 2 4

Hide Answer -
उ रः (a)
ा ाः

अनु े द-222: िकसी ायाधीश को एक उ ायालय से दू सरे उ ायालय


म थानां तरण करने सं बंधी ावधान है । यह काय भारत का रा पित भारत के मु
ायमू ित से परामश के बाद करता है । भारत के मु ायमू ित उ तम ायालय
के चार व र ायाधीशों एवं दो उ ायालयों (एक वहाँ के जहाँ से ायाधीश का
थानां तरण हो रहा है और एक वहाँ के जहाँ थानां त रत हो रहा हो) के मु
ायाधीशों से परामश करने के प ात् रा पित को परामश दे गा।
अनु े द-223: िकसी उ ायालय म कायकारी मु ायाधीश की िनयु के
सं बंध म ावधान है । यह काय भारत के रा पित ारा िकया जाता है ।
अनु े द-224: िकसी उ ायालय म अित र और कायकारी ायाधीशों की
िनयु के सं बंध म उपबं ध है । भारत का रा पित दो वष की अविध के िलये
अित र ायाधीशों तथा कायकारी ायाधीश की िनयु करता है तथा कायकारी
ायाधीश की िनयु रा पित ारा तब तक के िलये की जाती है , जब तक थायी
ायाधीश अपना पद हण न कर ले ।
अनु े द-224(क): इसम िकसी उ ायालय म से वािनवृ ायाधीशों की
िनयु के सं बंध म ावधान है । उ ायालय के मु ायाधीश रा पित की पू व
अनु मित तथा सं बंिधत की मं ज़ूरी के प ात् उस उ ायालय के से वािनवृ
ायाधीश या िकसी अ उ ायालय के से वािनवृ ायाधीश की िनयु
अ थायी तौर पर कर सकेगा।

[15]
िन िल खत कथनों पर िवचार कीिजयेः
1. उ ायालय के अिधका रयों और से वकों के वे तन व भ े रा की सं िचत
िनिध पर भा रत होते ह, जबिक उनकी पशन भारत की सं िचत िनिध पर।
2. उ ायालय के अिधका रयों एवं से वकों की िनयु उस उ ायालय का
मु ायमू ित रा पित से पू व परामश के बाद करे गा।
उपयु कथनों म से कौन-सा/से सही है /ह?

A) केवल 1

B) केवल 2

C) 1 और 2 दोनों

D) न तो 1 और न ही 2

Hide Answer -
उ रः (d)

ा ाः उपयु दोनों कथन गलत ह।

सं िवधान के अनु े द-229 म उ ायालय के अिधका रयों और से वकों की िनयु यों

और उनके वे तन एवं भ ों एवं पशन के सं बंध म ावधान है ।

उ अिधका रयों और से वकों की िनयु सं बंिधत उ ायालय का मु ायमू ित

करता है । िनयु यों म रा पित का पू व परामश नहीं ले ता है ।

उ ायालय के अिधका रयों और से वकों के वे तन, भ े व पशन (अथात् उ ायालय

के शासिनक य) सं बंिधत रा की सं िचत िनिध पर भा रत होते ह, िक ु उ

ायालय के ायाधीशों की पशन भारत की सं िचत िनिध पर भा रत होती है ।

उ ायालय के अिधका रयों एवं से वकों की से वा शत रा िवधानमं डल ारा बनाई गई

िविध के अं तगत रहते ए उ ायालय के मु ायाधीश िनधा रत करे गा।

[16]उ ायालय के े ािधकार के सं दभ म िन िल खत पर िवचार कीिजयेः


1. परामश सं बंधी े ािधकार
2. ारं िभक े ािधकार
3. िनरी ण सं बंधी े ािधकार
4. अपीलीय े ािधकार
उपयु म से कौन-से े ािधकार उ ायालय के े ािधकार के अ गत आते
ह?

A) केवल 1 और 4

B) केवल 1, 2 और 4

C) केवल 2, 3 और 4

D) 1, 2, 3 और 4
Hide Answer -
उ रः (c)
ा ाः उपयु म से कथन (1) सही नहीं है ।

परामश सं बंधी े ािधकार सं िवधान के अनु े द-143 के तहत केवल भारत के


सव ायालय को ा है ।
उ ायालय को िन िल खत े ािधकार ा ह-
ारं िभक े ािधकारः इस अिधकार के तहत उ ायालय िकसी िववाद को सीधे
सु न सकता है , जै से- मू ल अिधकार (अनु े द-226 के ारा) सं सद और रा
िवधानमं डल के सद ों के िनवाचन सं बंधी िववाद आिद।
िनरी ण/पयवे ण सं बंधी े ािधकारः सं िवधान के अनु े द-227 के तहत उ
ायालय अपने अिधकार े के सभी ायालयों व अिधकरणों का अधी ण करता
है । (सै अिधकरण को छोड़कर)।
अपीलीय े ािधकारः उ ायालय को िसिवल और आपरािधक दोनों मामलों म
अपील सु नने की श है ।
ाियक समी ा की श ः उ ायालय रा िवधानमं डल एवं कायकारी आदे शों
की सं वैधािनकता का परी ण कर सकता है और सं िवधान के उ ं घन की थित म
उसे असं वैधािनक शू /घोिषत कर सकता है ।
अिभले ख ायालयः उ ायालय एक अिभले ख ायालय है तथा इसके फैसले ,
कायवाही और काय को िकसी अधीन थ ायालय म चु नौती नहीं दी जा सकती।
अपनी अवमानना के िलये उसे द त करने की श है । ायालय की अवमानना
को सं िवधान म प रभािषत नहीं िकया गया है । इसे ायालय के अिधिनयम, 1971
म प रभािषत िकया गया है ।

[17]जब िकसी उ ायालय का मु ायाधीश शासिनक है िसयत से काम करता


है तो वह अधीन होता है -

A) उ ायालय के अ ायाधीशों म से िकसी भी रट अिधका रता के।

B) भारत के मु ायमू ित ारा यु िवशे ष िनयं ण के।

C) रा के रा पाल की िववे क श यों के।

D) इस िवषय म मु मं ी को द िवशे ष श यों के।

Hide Answer -
उ रः (a)

ा ाः उपयु म से िवक (a) सही है ।

उ ायालय का मु ायाधीश ाियक भू िमका के साथ-साथ शासिनक भू िमका भी

िनभाता है । शासिनक है िसयत से काय करते ए वह िनचली अदालतों का पयवे ण,

अधी ण, शासन व िनयं ण करता है । इस आधार पर मु ायाधीश उ ायालय के

अ ायाधीशों के रटों की अिधका रता म आता है ।


[18]िन िल खत कथनों पर िवचार कीिजयेः

1. भारतीय नाग रकों के अिधकारों को सं रि त करने म उ ायालय सव


ायालय की अपे ा अ ी थित म ह।
2. सव ायालय केवल मौिलक अिधकारों के सं र ण के िलये परमादे श जारी
कर सकता है ।
उपयु कथनों म से कौन-सा/से सही है /ह?

A) केवल 1

B) केवल 2

C) 1 और 2 दोनों

D) न तो 1 और न ही 2

Hide Answer -
उ रः (c)

ा ाः उपयु दोनों कथन सही ह।

भारत का सव ायालय सं िवधान के अनु े द-32 के तहत केवल मू ल अिधकारों के

सं दभ म रट िनकाल सकता है , जबिक उ ायालय सं िवधान के अनु े द-226 के

तहत नाग रकों के मू ल अिधकारों के साथ-साथ अ मामलों म भी रट िनकाल सकते ह।

अतः उ ायालय, उ तम ायालय से इस सं दभ म अ ी थित म है ।

सव ायालय म केवल मू ल अिधकारों के सं र ण के िलये परमादे श जारी करने का

ावधान है , जबिक उ ायालय मू ल अिधकारों के अित र सं वैधािनक, वै धािनक आिद

मामलों म परमादे श िनकाल सकते ह।

[19]िन िल खत म से कौन िकसी उ ायालय की अिधका रता का िकसी सं घ रा


े पर िव ार या उससे अपवजन करता है ?

A) सं सद िविध ारा।

B) िजस रा मउ ायालय थत है , वहाँ का िवधानमं डल िविध ारा।

C) भारत के रा पित ारा।

D) उस रा का रा पाल िजस रा मउ ायालय थत है ।

Hide Answer -
उ रः (a)

ा ाः उपयु म से िवक (a) सही है ।

सिवधान के अनु े द-230 म रा उ ायालय की अिधका रता का सं घ रा े पर

िव ार या अपवजन का उपबं ध है । सं सद, िविध ारा ऐसा कर सकती है ।

[20]िन िल खत कथनों पर िवचार कीिजयेः

1. भारत के सं िवधान के उपबं धों के अनु सार उ तम ायालय के िकसी


ायाधीश के आचरण के सं बंध म िकसी रा के िवधानमं डल म कोई चचा नहीं
की जा सकती।
2. भारतीय सं िवधान के उपबं धों के अनु सार रा के उ ायालय के िकसी
ायाधीश के आचरण के सं बंध म रा के िवधानमं डल म चचा िकये जाने पर
कोई िनबधन नहीं है ।
3. भारतीय सं िवधान के उपबं धों के अनु सार ायालय िवधानमं डलों की
कायवािहयों की जाँ च नहीं कर सकता।
उपयु कथनों म से कौन-सा/से सही है /ह ?

A) केवल 1

B) केवल 1 और 3

C) केवल 2 और 3

D) 1, 2 और 3

Hide Answer -
उ र: (b)

ा ाः उपयु म से कथन (2) गलत है ।

सं िवधान म उपबं ध है िक एक उ ायालय के ायाधीश के आचारण पर सं सद या रा

िवधानमं डल म चचा नहीं की जा सकती। केवल उस थित म की जा सकती है , जब

िकसी- ायाधीश के खलाफ महािभयोग का ाव सं सद म िवचारधीन है ।

सं िवधान के अनु सार ायालय िवधानमं डलों की कायवािहयों की जाँ च नहीं कर सकता।
[1] िन िल खत म से कौन-सा एक मामला उ एवं उ तम ायालय दोनों की
अिधका रता म आता है ?

A) के व रा ों के बीच िववाद।

B) रा ों के बीच िववाद।

C) मू ल अिधकारों का सं र ण।

D) सं िवधान के उ ं घन से सं र ण।

Show Answer +

[2] िज़ला ायाधीश के सं दभ म िन िल खत म से कौन-सा कथन सही नहीं है ?

A) िज़ला ायाधीश िज़ले का सबसे बड़ा ाियक अिधकारी होता है ।

िज़ला ायाधीशों की िनु य , पद थापना एवं पदो ित भारत के रा पित ारा


B)
रा के उ ायालय के परामश से की जाती है ।

ऐसे को िज़ला ायाधीश िनयु िकया जाता है , जो क या रा


C)
सरकार म से िकसी म भी सरकारी से वा म कायरत न रहा हो।

िज़ला ायाधीश िनयु होने के िलये िकसी के पास कम-से -कम सात
D)
वष का अिधव ा (Advocate) के काय का अनु भव हो।

Hide Answer -
उ रः (b)

ा ाः उपयु म से कथन (b) सही नहीं है ।

सं िवधान के अनु े द-233 म िज़ला ायाधीशों की िनयु के सं बंध म उपबं ध िकया गया

है । िज़ला ायाधीशों की िनयु , पद थापना एवं पदो ित रा के रा पाल ारा रा

के उ ायालय के परामश से की जाती है ।

कोई िज़ला ायाधीश बनने के िलये तभी पा होगा, जबः-

वह क या रा सरकार म िकसी सरकारी से वा म कायरत न हो।

कम-से -कम सात वष तक अिधव ा या ीडर रहा हो।

उसकी िनयु के िलये उ ायालय ारा िसफा रश की गई हो।

िज़ला ायाधीश, िज़ले का सबसे बड़ा ाियक अिधकारी होता है । उसे िसिवल और

आपरािधक मामलों म मू ल एवं अपीलीय अिधकार ा ह।िज़ला ायाधीश के पास

ाियक एवं शासिनक दोनों कार की श यां ह। उसके पास अधीन थ ायालयों का

िनरी ण करने की श है ।

उसके फैसले के िव उ ायालय म अपील की जा सकती है ।

िज़ला ायाधीश को उ कैद से ले कर मृ ु दंड दे ने तक का अिधकार ा है ।

[3] भारत की ाियक णाली के सं दभ म िन िल खत कथनों पर िवचार कीिजयेः


1. भारत के रा ों म अधीन थ ायालयों के ितमान और पदनाम एक से
नहीं ह।
2. मुं िसफ ायालयों को िसिवल और दा क दोनों अिधकार ा ह।
3. कुछ रा ों म पं चायत ायालय काय कर रहे ह।

उपयु म से कौन-सा/से कथन सही है /ह?

A) केवल 1 और 3

B) केवल 1, 2 और 4

C) केवल 2, 3 और 4

D) 1, 2, 3 और 4

Hide Answer -
उ रः (b)

ा ाः उपयु सभी कथन सही ह।

भारत के रा ों म अधीन थ ायालयों के ितमान और पदनाम एक से नहीं ह।

मुं िसफ ायालयों को िसिवल और दा क दोनों अिधकार ा ह।

दे श कुछ रा ों म पं चायत ायालय काय कर रहे ह।

[4] भारत के सं िवधान के अनु सार िन िल खत म से कौन िज़ला ायाधीश की


अिभ के अं तगत स िलत नहीं होता?

A) मु े सीडसी मिज े ट

B) से शन ायाधीश

C) अिधकरण ायाधीश

D) लघु वाद ायालय का मु ायाधीश

Hide Answer -
उ रः (c)

ा ाः उपयु म से िवक (c) सही है ।

सं िवधान के अनु े द-236(क) के अं तगत ावधान है िक िज़ला ायाधीश के अं तगत

ा- ा शािमल है । इसम अिधकरण ायाधीश शािमल नहीं है ।

[5] रा ीय िविधक से वा ािधकरण (NALSA) के सं दभ म िन िल खत कथनों पर


िवचार कीिजयेः

1. इसका गठन िविधक से वा ािधकरण अिधिनयम, 1997 के तहत िकया


गया।
2. यह दे श भर म िविधक काय म और योजनाओं को लागू करने के िलये
रा िविधक से वा ािधकरणों को िदशा-िनदश जारी करता है ।
3. इसका उ े समान अवसरों के आधार पर समाज के कमज़ोर वग को
िनःशु एवं स म िविधक से वाएँ उपल कराना है ।

उपयु म से कौन-सा/से कथन सही है /ह?

A) केवल 3

B) केवल 1 और 3

C) केवल 2 और 3
D) 1, 2 और 3

Hide Answer -
उ रः (c)

ा ाः उपयु म से कथन (1) गलत है ।

रा ीय िविधक से वा ािधकरण (ना ा) का गठन िविधक से वा ािधकरण अिधिनयम,

1987 के तहत िकया गया।

ना ा दे श भर म कानू नी से वा काय मों को काया त करने के िलये रा िविधक से वा

ािधकरणों के िलये नीित, िस ां त एवं िदशा-िनदश िनधा रत करता है तथा भावी एवं

स ीयोजनाएँ बनाता है ।

सं िवधान के अनु े द-39(A) म समाज के कमज़ोर वग के िलये िनःशु िविधक सहायता

का उपबं ध िकया गया है । इसी उ े को पू रा करने के िलये सं सद ारा कानू नी से वा

ािधकरण अिधिनयम, 1987 पा रत िकया गया, जो 5 नवं बर, 1995 से लागू है ।

इसका उ े समान अवसरों के आधार पर समाज के कमज़ोर वग को िनःशु एवं

स म िविधक से वाएँ उपल कराना, िववादों को सौहा पू ण ढं ग से िनपटाने के िलये लोक

अदालतों का गठन करना, ामीण े ों म कानू नी जाग कता िशिवरों का आयोजन करना

आिद है ।

[6] लोक अदालत के सं दभ म िन िल खत कथनों म से कौन-सा कथन सही है ?

लोक अदालत की अिधका रता मु कदमा दायर करने से पहले ही मामलों का


A)
िनपटारा करने की है , उन मामलों की नहीं जो िकसी ायालय म लं िबत ह।

े क लोक अदालत म केवल से वारत या से वािनवृ ाियक अिधकारी ही


B)
िनयु हो सकते ह।

लोक अदालत ऐसे मामलों का िनपटारा नहीं कर सकती है , जो िसिवल एवं


C)
आपरािधक कृित के हों।

लोक अदालत ारा िदया गया फैसला अं ितम तथा सभी प ों पर बा कारी
D)
होगा तथा इनके फैसलों के िव िकसी ायालय म कोई अपील नहीं होगी।
Hide Answer -
उ रः (d)

ा ाः उपयु म से िवक (d) सही है ।

िविधक से वा ािधकरण अिधिनयम, 1987 के ारा लोक अदालत को वै धािनक दज़ा दान

िकया गया। लोक अदालत का फैसला िसिवल कोट या िकसी भी अ अदालत के िकसी

भी आदे श की तरह मा होगा।

लोक अदालत ारा िदया गया फैसला अं ितम तथा सभी प ों पर बा कारी होगा और

इसके फैसले के िव िकसी भी अदालत म कोई अपील नहीं होगी।

लोक अदालत म ायालय के लं िबत मामलों के साथ-साथ ऐसे मामलों को भी लाया जा

सकता है , जो अभी ायालय म दायर नहीं िकये गए ह।

े क लोक अदालत म उतनी ही सं ा म से वारत तथा से वािनवृ ाियक अिधकारी तथा

उस इलाके के अ शािमल होंगे, िजतनी की लोक अदालत को आयोिजत करने

वाली एजसी िनिद कर।

लोक अदालत म िसिवल तथा कितपय आपरािधक मामलों (Compoundable

Offences) पर भी िवचार िकया जाता है ।

लोक अदालत को वही श याँ ा होती ह, जो िसिवल कोट को कोड ऑफ िसिवल

ोसीज़र (Code of Civil Procedure) के अं तगत ा होती ह।

े क लोक अदालत आपरािधक ि या सं िहता (1973) के उ े से एक िसिवल कोट

माना जाएगा।

कानू नी से वा ािधकरण अिधिनयम, 1987 म वष 2012 म सं शोधन िकया गया एवं थायी

लोक अदालत का ावधान िकया गया।

थायी लोक अदालत म एक अ या सभापित एवं दो अ होंगे, िज

सावजिनक से वाओं का पया अनु भव हो।

थायी लोक अदालत का िव ीय े ािधकार ₹10 लाख तक का होगा, िकंतु क सरकार

इसे समय-समय पर बढ़ा सकती है ।

थायी लोक अदालत ारा िदया गया िनणय अं ितम होगा एवं सम प ों पर बा कारी

होगा।

[7] भारत म प रवार ायालय के सं दभ म िन िल खत म से कौन-सा कथन सही नहीं


है ?

प रवार ायालय की थापना के िलये प रवार ायालय अिधिनयम, 2005


A)
पा रत िकया गया।

प रवार ायालय की थापना का ावधान रा सरकार उ ायालय की


B)
सहमित के आधार पर करती है ।

रा सरकार एक लाख से अिधक जनसं ा वाले े क नगर म प रवार


C)
ायालय थािपत करने के िलये बा है ।

D) प रवार ायालय की अपील उ ायालय म की जा सकती है ।


Hide Answer -
उ रः (a)

ा ाः उपयु म से िवक (a) सही नहीं है ।

प रवार ायालय की थापना के िलये प रवार ायालय अिधिनयम, 1984 पा रत िकया

गया। इस अिधिनयम की िन िवशे षताएँ ह:

⇒ रा सरकार उ ायालय की सहमित से प रवार ायालय की थापना का ावधान

करती है ।

⇒ यह अिधिनयम रा सरकार को एक लाख से अिधक जनसं ा वाले े क नगर म

एक प रवार ायालय की थापना के िलये बा कारी बनाता है ।

⇒ यह अिधिनयम केवल एक अपील का अिधकार दे ता है और वह अपील उ ायालय

म की जा सकती है ।

वष 2016 तक दे श भर म 438 प रवार ायालय सं चािलत ह।

[8] ाम ायालय अिधिनयम, 2008 के सं दभ म िन िल खत कथनों पर िवचार


कीिजयेः

1. इसका उ े नाग रकों को उनके ार पर ाय सु लभ कराना है ।


2. ाम ायालय एक मोबाइल कोट है ।
3. ाम ायालय को केवल दीवानी ायालय की श ा है ।
4. े क पं चायत के िलये एक ाम ायालय की थापना की जाएगी।

उपयु म से कौन-से कथन सही ह?

A) केवल 1 और 4

B) केवल 1, 2 और 4

C) केवल 1, 3 और 4

D) केवल 2, 3 और 4
Hide Answer -
उ रः (b)

ा ाः उपयु म से कथन (3) गलत है ।

ाम ायालय अिधिनयम, 2008 के तहत ाम ायालय को दीवानी और फौज़दारी दोनों

ायालयों की श याँ ा ह।

इसका उ े नाग रकों को उनके ार पर ाय सु लभ करवाना है और यह सु िनि त

करना है िक कोई नाग रक सामािजक, आिथक एवं अ मताओं के कारण ाय से वं िचत न

रह जाए।

यह एक मोबाइल कोट (चिलत ायालय) है ।

यह अिधिनयम े क पं चायत के िलये एक ाम ायालय की थापना का ावधान करता

है ।

ाम ायालय अिधिनयम, 2008 मोटे तौर पर िविध आयोग की अनु शंसाओं पर आधा रत

है । इसने अपनी 114वीं रपोट म ाम ायालयों की थापना का सु झाव िदया है ।

ाम ायालय थम े णी के द ािधकारी (Magistrate) की अदालत होगी तथा इसके

ायािधकारी की िनयु उ ायालय की सहमित से रा सरकार ारा की जाएगी।

ाम ायालय ाकृितक ाय के िस ां तों से िनदिशत होता है । यह ायालय भारतीय

सा अिधिनयम, 1872 से बं धा नहीं है ।

ाम ायालय की थापना रा सरकार उ ायालय के परामश से करे गी।

[9] िन िल खत म से कौन-सा कथन सही नहीं है ?

सं िवधान के भाग-XII म सं घ और रा ों के बीच िवधायी सं बंधों का उ ेख


A)
िकया गया है ।

रा पित को यह श है िक सं घ रा े अ मान एवं िनकोबार ीप


B) समू ह, ल ीप, दादर एवं नागर हवे ली तथा दमन एवं दीव के सं बंध म सं सद
के िकसी अिधिनयम को िनरिसत या सं शोिधत कर दे ।

रा पाल को यह श है िक वह सं सद के िकसी िवधे यक को सू चीब े


C)
म लागू न करे या कुछ सं शोधनों के साथ लागू करे ।

असम के रा पाल को यह श है िक सं सद के िकसी िवधे यक को


D) जनजातीय े ( ाय िज़लों) म लागू न करे या कुछ िविश सं शोधनों के
साथ लागू करे ।
Hide Answer -
उ रः (a)

ा ाः उपयु म से िवक (a) सही नहीं है ।

सं घ और रा ों के बीच िवधायी सं बंधों का उ े ख सं िवधान के भाग-XI के अ ाय-1 म

(अनु े द-245 से 255 तक) िकया गया है और शासिनक सं बंधों का उ े ख सं िवधान

के भाग-XI के अ ाय-2 (अनु े द-256 से ले कर 263 तक) म िकया गया है ।

सं िवधान के भाग-12 म सं घ और रा ों के बीच िव ीय सं बंधों का उ े ख है ।

सं िवधान के अनु े द-240 के तहत भारत के रा पित को चार सं घ रा े ों: अ मान

एवं िनकोबार ीप समू ह, ल ीप, दादर एवं नागर हवे ली और दमन एवं दीव के सं बंध म

सं सद के िकसी अिधिनयम को िनरिसत या सं शोिधत करने की श है । रा पित इन े ों

म शां ित, उ ित एवं अ ी सरकार के िलये िनयम बना सकता है ।

सं िवधान की पाँ चवी अनु सूची के तहत रा पाल को यह श है िक वह सं सद के िकसी

िवधये क को अनु सूिचत े म लागू न करे या कुछ िवशे ष सं शोधन कर लागू करे ।

सं िवधान की छठी अनु सूची के तहत असम के रा पाल को यह श है िक वह सं सद के

िकसी िवधे यक को जनजातीय े ( ाय िज़लों) म लागू न करे या कुछ िविश प रवतन

के साथ लागू करे । इसी कार की श भारत के रा पित को जनजातीय े ों ( ाय

िज़लों): मे घालय, ि पु रा एवं िमज़ोरम के िलये ा है ।

[10]िन िल खत म से िकन आधारों पर सं घ और रा ों के बीच सं बंध िनभर करते ह?

1. सं वैधािनक ावधानों के आधार पर


2. िनयोजन ि याओं के आधार पर
3. ाियक ा ाओं के आधार पर
4. पर राओं तथा वहारों के आधार पर

नीचे िदये गए कूट का योग कर सही उ र का चयन कीिजयेः

A) केवल 1 और 2

B) केवल 1, 2 और 3

C) केवल 1, 2 और 4

D) 1, 2, 3 और 4
Hide Answer -
उ रः (d)

ा ाः उपयु सभी कथन सही ह।

भारत म के -रा ों के बीच सं बंध िन िल खत आधारों पर िनभर करता है -

⇒ सं वैधािनक ावधानों पर।

⇒ िनयोजन ि या के आधार पर।

⇒ ाियक ा ाओं के आधार पर।

⇒ पर राओं तथा वहारों के आधार पर।

⇒ राजनीितक िहतों के अ िवरोध के आधार पर।

⇒ बातचीत के िलये यं िव ास पर।

⇒ चलाने की इ ा की बलता से ।

[11]िन िल खत म से कौन-सा/से यु सही सु मेिलत है /ह?

1. सं घ सू ची - 97 िवषय
2. रा सू ची - 67 िवषय
3. समवत सू ची - 47 िवषय

नीचे िदये गए कूट का योग कर सही उ र का चयन कीिजयेः

A) केवल 1

B) केवल 1 और 2

C) केवल 1 और 3

D) 1, 2 और 3
Hide Answer -
उ रः (c)

ा ाः उपयु म से यु (2) सही सु मेिलत नहीं है ।

सं िवधान की 7वीं अनु सूची म तीन कार की सू िचयों का उपबं ध िकया गया है ः सं घ सू ची,

रा सू ची और समवत सू ची।

इन तीनों सू िचयों म िवि यों की सं ा (मू ल प म) िन िल खत है ः

1. सं घ सू ची- 97 िवि याँ /िवषय। वतमान म इसम कुल िवि यों की सं ा 100 है । इसम

िन िल खत िवि यों को जोड़ा गया है ः

िवि सं ा 2 (क) : 42 व सं िवधान सं शोधन अिधिनयम 1976 के ारा।

िवि सं ा 92(क): 6 ठ सं िवधान सं शोधन अिधिनयम, 1956 के ारा।

िवि सं ा 92(ख): 46 वाँ सं िवधान सं शोधन अिधिनयम, 1982 के ारा।

िवि सं ा 92 (ग): 88 वाँ सं िवधान सं शोधन अिधिनयम, 2003 के ारा।

- िवि सं ा 33 को 7व सं िवधान सं शोधन ारा लोप कर िदया गया।

2. रा सू चीः 66 िवषय/ िवि । वतमान म इसम िव यों की सं ा घटकर 61 रह गई है ।

इसम 11, 19, 20, 29 को 42व सं िवधान सं शोधन के ारा लोप कर िदया गया है

3. समवत सू चीः 47 िवि याँ । वतमान म इसम 52 िवि याँ ह। इसम 11(क), 17 (क)

एवं (ख), 20(क) को 42व सं िवधान सं शोधन एवं 33(क) को तीसरे सं िवधान सं शोधन

अिधिनयम के ारा जोड़ा गया।

[12]भारत म अविश िवधायी श यों (Residuary powers of legislation) के


सं दभ म िन िल खत कथनों पर िवचार कीिजयेः

1. भारत के सं िवधान म अविश श यों का उ ेख प से कहीं नहीं


िकया गया है ।
2. भारत शासन अिधिनयम, 1935 म अविश श भारत के गवनर
जनरल म िनिहत थी।
3. भारत के सं िवधान ने अमे रकी सं िवधान की भाँ ित अविश श यां के
म िनिहत की ह।

उपयु म से कौन-सा/से कथन सही है /ह?

A) केवल 2

B) केवल 1 और 2

C) केवल 2 और 3

D) 1, 2 और 3
Hide Answer -
उ रः (a)

ा ाः उपयु म से कथन (1) और (3) गलत ह।

सं िवधान के अनु े द-248 म अविश िवधायी श यों के सं बंध म उपबं ध िकया गया है ।

भारत ने कनाडा के सं िवधान की भां ित अविश श याँ के म िनिहत की ह। अमे रका

तथा ऑ े िलया म अविश श याँ रा ों को सौंप दी गई ह।

कोई िवषय अविश श यों के अं तगत है या नहीं, इसका िनणय ायालय करता है और

उसका िनणय अं ितम होता है ।

[13]भारत का सं िवधान सं सद को रा सू ची के िवषय म िविध बनाने की श सौंपती


है यिदः

1. रा सभा म उप थत और मत दे ने वाले सद ों म से कम-से -कम एक-


ितहाई सद ों के ब मत ारा सं क पा रत कर िदया गया है ।
2. रा ीय आपात की उद् घोषणा वतन म रहने पर।
3. कम-से -कम आधे रा ों के िवधानमं डल ाव पा रत कर िक रा सू ची
म िविध बनाई जाए।
4. अं तरा ीय करारों को भावी करने के िलये ।

उपयु म से कौन-से कथन सही ह?

A) केवल 2 और 4

B) केवल 1, 2 और 3

C) केवल 1, 2 और 4

D) केवल 2, 3 और 4
Hide Answer -
उ रः (a)

ा ाः उपयु म से कथन (1) और (3) गलत ह।

सं िवधान के अनु े द-249 के तहत रा सभा उप थत व मत दे ने वाले सद ों म से

कम-से -कम दो-ितहाई (न िक एक-ितहाई) ब मत से सं क पा रत कर दे िक रा ीय

िहत म यह आव क समीचीन है । इस आधार पर सं सद भारत के स ू ण रा े या

उसके िकसी भाग के िलये िविध बना सकती है । यह िविध एक वष के िलये होगी। इस

िविध को एक-एक वष के िलये असं बार बढ़ाया जा सकता है ।

सं िवधान के अनु े द-252 के तहत दो या अिधक रा ों के िवधानमं डल ाव पा रत

कर िक सं सद रा सू ची म िविध बनाए। यह िविध उ ीं रा ों म लागू होगी, िज ोंने िविध

बनाने के सं बंध म ाव पा रत िकया है , िक ु कोई रा बाद म िवधानमं डल म पा रत

ाव के मा म से इसे लागू कर सकता है । इस कार के िविध का सं शोधन या इस पर

पु निवचार केवल सं सद ही कर सकती है ।

सं िवधान के अनु े द-250 म ावधान है िक सं सद रा ीय आपात की उद् घोषणा वतन म

रहने पर भारत के स ू ण रा े या िकसी भाग के िलये रा -सू ची म िविध बना सकती

है । उद् घोषणा वतन म नहीं रहने पर छःमाह की समा पर िविध भावी नहीं रहे गी।

सं िवधान के अनु े द-253 के तहत सं सद अ रा ीय करारों/ समझौतों को लागू करने के

िलये भारत के स ू ण रा े या उसके िकसी भाग के िलये रा सू ची के िवषय पर

िविध बना सकती है ।

सं िवधान के अनु े द-356 के तहत रा ों म रा पित शासन के दौरान सं सद रा सू ची म

िविध बना सकती है ।

[14]िन िल खत पर िवचार कीिजयेः

1. आिथक और सामािजक िनयोजन।


2. से वाओं पर कर।
3. जनसं ा िनयं ण और प रवार िनयोजन।
4. िश ा।

उपयु म से कौन-से िवषय समवत सू ची से सं बंिधत ह?

A) केवल 1 और 3

B) केवल 3 और 4

C) केवल 1, 3 और 4

D) केवल 2, 3 और 4
Hide Answer -
उ रः (c)

ा ाः उपयु म से िवक (2) गलत है ।

से वाओं पर कर सं घ सू ची का िवषय है । 88व सं िवधान सं शोधन अिधिनयम 2003 के ारा

सं घ सू ची म िवि सं ा 92(ग) जोड़ा गया और इसम यह ावधान िकया गया।

समवत सू ची म िन िवषय शािमल ह;

1. आिथक एवं सामािजक िनयोजनः िवि सं ा 20 म इसका उपबं ध है ।

2. जनसं ा िनयं ण एवं प रवार िनयोजनः 42व सं िवधान सं शोधन अिधिनयम, 1976 के

ारा िवि सं ा 20 (क) जोड़ा गया और इसम इसका उपबं ध िकया गया।

3. िश ाः पहले यह रा सू ची का िवषय था, िक ु 42व सं िवधान सं शोधन अिधिनयम के

ारा इसे समवत सू ची म डाल िदया गया।

[15]भारतीय सं िवधान के अ गत िवषय तथा सं बंिधत सू ची के सं बंध म िन िल खत

यु ों म से कौन-सा एक सही सु मेिलत नहीं है ः


िवषय सू ची

A) वन : समवत सू ची

B) शे यर बाज़ार : समवत सू ची

C) डाकघर बचत बक : सं घ सू ची

D) जन ा : रा सू ची

Hide Answer -
उ रः (b)

ा ाः उपयु म से िवक (b) सही नहीं है ।

सं िवधान की 7वीं अनु सूची म तीन कार की सू िचयाँ ह - सं घ सू ची, रा सू ची एवं समवत

सू ची। सं घ सू ची म के सरकार, रा सू ची म रा सरकार तथा समवत सू ची म दोनों

िविध बना सकते ह।

सं घ सू चीः शे यर बाज़ार, डाकघर बचत बक, जनगणना, बै िकंग, बीमा, र ा, रे लवे ,

वै देिशक मामले , िनगम कर आिद िवषय आते ह।

रा सू चीः पु िलस, लोक ा एवं ता, िनखात िनिध, गै स, कृिष, रे लवे पु िलस,

पं चायती राज, भू िम सु धार, कारागार, लोक व था आिद िवषय आते ह।

समवत सू चीः वन, व जीव, ज ु ओं व पशु ओं का सं र ण, पशु ओं के ित ू रता का

िनवारण, िववाह और िववाह िव े द, गोद ले ना, आपरािधक मामले आिद िवषय आते ह।

वन को समवत सू ची म 42व सं िवधान सं शोधन अिधिनयम 1976 के ारा जोड़ा गया।

[16]सू ची-I को सू ची-II से सु मेिलत कीिजये तथा नीचे िदये गए कूट का योग कर
सही उ र का चयन कीिजयेः
सू ची-I सू ची-II
(िवषय) (सू ची)

A.के सू ची 1. पु िलस एवं लोक व था

B.रा सू ची 2. अ र अनु संधान

C.समवत सू ची 3. जनगणना

D.अविश िवषय 4. जनसं ा िनयं ण एवं प रवार िनयोजन

कूटः
A B C D

A) 3 1 4 2

B) 3 4 1 2

C) 2 4 1 3

D) 2 1 4 3

Hide Answer -
उ रः (a)

ा ाः उपयु म से िवक (a) सही है ।

1. सं घ सू चीः जनगणना।

2. रा सू चीः पु िलस और लोक व था।

3. समवत सू चीः जनसं ा िनयं ण एवं प रवार िनयोजन।

4. अविश िवषयः अं त र अनु संधान।

7वीं अनु सूची से सं बंिधत िकसी सू ची म कोई िवषय शािमल नहीं ह तो ऐसे िवषय को अविश

िवषय की े णी म रखते ह। सं िवधान के अनु े द-248 म इसका ावधान िकया गया है । भारत

म इस पर के िविध का िनमाण करता है ।

[17]भारतीय सं िवधान म रा सू ची म िन िल खत म से कौन-सा एक शािमल नहीं है ?

A) पु िलस

B) कानू न और व था

C) कारागार

D) द ि या सं िहता
Hide Answer -
उ रः (d)

ा ाः उपयु म से िवक (d) सही नहीं है ।

द ि या सं िहता समवत सू ची का िवषय है । इसका उपबं ध िवि सं ा 2 म िकया

गया है ।

[18]िन िल खत म से िकस/िकन आयोग/सिमित का सं बंध के -रा सं बंधों से नहीं


रहा है ?

1. सरका रया आयोग


2. राजम ार सिमित
3. इ जीत गु ा सिमित
4. पुं छी आयोग

नीचे िदये गए कूट का योग कर सही उ र का चयन कीिजयेः

A) केवल 3

B) केवल 1 और 4

C) केवल 1, 2 और 4

D) केवल 1, 3 और 4
Hide Answer -
उ रः (a)

ा ाः उपयु म से इ जीत गु ा सिमित का सं बंध के -रा सं बंधों से नहीं रहा है ।

इ जीत गु ा सिमित का गठन वष 1998 म चु नाव सु धार एवं जन ितिनिध कानू न,

1951 म सं शोधन म सु झाव दे ने के िलये गिठत िकया गया था। इस सिमित का मु

उ े चु नाव काय म रा की ओर से धन उपल कराए जाने सं बंधी वहारों का िवशे ष

अ यन करना और उस पर अपना िन ष दे ना था।

सरका रया आयोगः जू न 1983 म के सरकार के ारा के -रा सं बंधों पर उ तम

ायालय के से वािनवृ ायाधीश रं जीत िसं ह सरका रया (आर.एस. सरका रया) की

अ ता म तीन सद ीय (दो अ सद बी. िशवराम और एस.आर.से न) आयोग का

गठन िकया गया। इसने अपनी फाइनल रपोट जनवरी 1988 म सरकार को सौंपी।

राजम ार सिमितः के -रा सं बंधों की समी ा एवं रा ों को ाय ता िदलाने के िलये

तिमलनाडु सरकार (डी.एम.के.) ने डॉ. वी.पी. राजम ार की अ ता म तीन सद ीय

(दो अ सद ः डॉ. ल ण ामी मु दािलयर एवं पी.सी. च ा रे ी) सिमित का गठन

िकया। इसने वष 1971 म तिमलनाडु सरकार को अपनी रपोट सौंपी।

पुं छी आयोगः के -रा की समी ा के िलये के सरकार ारा अ ै ल 2007 म उ तम

ायालय के भू तपू व मु ायमू ित मदन मोहन पुं छी की अ ता म पाँ च सद ीय एक

आयोग का गठन िकया गया। आयोग के चार अ सद ः धरे िसं ह, िवनोद कुमार

गु ल, ो. एन.आर. माधव मे नन एवं डॉ. अमरे श बागची थे । बागची के िनधन के प ात्

उसके थान पर िवजय शं कर को वष 2008 म एक सद के प म शािमल िकया गया।

इसने अ ै ल 2010 को अपनी रपोट के सरकार को सौंपी।

शासिनक सु धार आयोग की रपोट के अ यन के िलये एम.सी. शीतलवाड़ की अ ता

म एक दल का गठन िकया गया। इसने अपनी रपोट वष 1969 म के सरकार को

सौंपी, िजसम के -रा सं बंधों को सु धारने के िलये 22 िसफ रश ु त की।

[19]भारत म रा ों के बीच सहयोग एवं सम य ा करने के िलये िन िल खत


सं थाओं म से कौन-सी सं िवधाने र सं था/सं थाएँ है /ह?

1. रा ीय िवकास
2. प रषदरा पालों का स े लन
3. े ीय प रषद
4. अं तरा ीय प रषद

नीचे िदये गए कूट का योग कर सही उ र का चयन कीिजयेः

A) केवल 1

B) केवल 1 और 2

C) केवल 1, 2 और 3

D) केवल 1, 3 और 4
Hide Answer -
उ रः (b)

ा ाः उपयु म से े ीय प रषद एवं अं तरा ीय प रषद एक सं िवधाने र सं थाएँ नहीं है ।

े ीय प रषदः यह एक वै धािनक/सां िविधक सं था है । रा पु नगठन अिधिनयम, 1956

के तहत भारतीय रा े ों को पाँ च े ों म बां टा गया और इसके िलये पाँ च े ीय

प रषदों का गठन िकया गया। इन प रषदों के अित र पू व र प रषद अिधिनयम,

1971 के ारा एक पू व र प रषद का भी गठन िकया गया।

अं तरा ीय प रषदः यह एक सं वैधािनक प रषद है । इसका उ े ख सं िवधान के

अनु े द-263 म िकया गया है । सरका रया आयोग की िसफा रश पर 28 मई, 1990 को

(वी.पी. िसं ह के ने तृ वाली जनता दल सरकार ने ) अं तरा ीय प रषद का गठन िकया।

सं िवधाने तर एवं गै र-सां िविधक सं थाएँ िन ह:

1. नीित (NITI) आयोग, (योजना आयोग के थान पर): इसका गठन के सरकार ारा

1 जनवरी, 2015 को एक आदे श के ारा िकया गया। 8 फरवरी, 2015 को इसकी

पहली बै ठक ई।

2. र ीय िवकास प रषद (NDC): इसकी थापना 6 अग , 1952 को भारत सरकार

के एक कायपािलकीय सं क के ारा की गई। रा ीय िवकास प रषद को भं ग या समा

करने सं बंधी सं क अभी तक पा रत नहीं आ।

3. रा ीय एकता प रषदः वष 1961 म के सरकार ारा एक िनणय के साथ आ है ।

4. रा पालों का स े लनः रा पित की अ ता म।

5. मु मं ि यों का स े लनः धानमं ी की अ ता म।

6. मु सिचवों का स े लनः कैिबने ट सिचव की अ ता म।

7. पु िलस महािनदे शकों का स े लन।

8. मु ायाधीशों का स े लन।

9. कुलपितयों का स े लन।

10. गृ हमं ि यों का स े लन।

11. िविध मं ि यों का स े लन।

आं चिलक प रषदः एक वै धािनक प रषद है । इसे सं सद के अिधिनयमों के ावधानों के तहत

गिठत िकया गया है ।

[20]िन िल खत म से कौन-सा कथन सही नहीं है ?

सं िवधान म उपबं ध है िक सं सद िविध ारा अं तरा ीय नदी या नदी-दू न के या


A) उसम जल के योग, िवतरण या िनयं ण के सं बंध म िकसी िववाद के ाय-
िनणय के िलये उपबं ध कर सकेगी।

सं िवधान म उपबं ध है िक सं सद िविध ारा अं तरा ीय िववादों की जाँ च करने


B)
और सलाह दे ने के िलये अं तरा ीय प रषद का गठन कर सकेगी।

C) सं सद ने वष 1956 म अं तरा ीय जल िववाद अिधिनयम पा रत िकया।

D) सं सद ने वष 1956 म नदी बोड अिधिनयम पा रत िकया।


Hide Answer -
उ रः (b)

ा ाः उपयु म से िवक (b) सही नहीं है ।

सं िवधान के अनु े द-263 म रा ों के म तथा के व रा ों के म सम य के िलये

अं तरा ीय प रषद के गठन का उपबं ध िकया गया है । इसका गठन भारत के रा पित ारा

िकया जाता है । वष 1990 म सरका रया आयोग की िसफा रश पर रा पित ारा

अं तरा ीय प रषद का गठन िकया गया।

सं िवधान के अनु े द-262 म ावधान है िक सं सद िविध ारा अं तरा ीय नदी या नदी-

दू न के या उसम जल के योग, िवतरण या िनयं ण के सं बंध म िकसी िववाद के ाय-

िनणय के िलये उपबं ध कर सकेगी। यह एक ाियकेतर व था है अथात् सं िवधान के

अनु े द-131 के तहत सव ायालय को के और रा एवं रा ों के बीच िववाद

को सु लझाने के िलये अन अिधका रता है , िक ु सं िवधान के अनु े द-262 के तहत इसे

ाियक ह े प से बाहर रखा गया है ।

सं सद ने अनु े द-262 के तहत दो कानू न बनाए ह-

1. नदी बोड अिधिनयम, 1956

2. अं तरा ीय जल िववाद अिधिनयम, 1956

सं सद वष 2016 तक अं तरा ीय जल िववाद के िलये आठ अं तरा ीय जल िववाद

ायािधकरणों की थापना कर चु की है ।
[1] भारतीय सं िवधान के अनु े द-312 म अ खल भारतीय से वाओं के सं बंध म
ावधान िकया गया है । इस सं दभ म िन िल खत पर िवचार कीिजयेः
1. भारतीय शासिनक से वा (आई.ए.एस.)
2. भारतीय पु िलस से वा (आई.पी.एस.)
3. भारतीय वन से वा (आई.एफ.एस.)
4. भारतीय िवदे श से वा (आई.एफ.एस.)
उपयु म से कौन-कौन-सी से वाओं को अ खल भारतीय से वा म शािमल िकया
गया है ?

A) केवल 1 और 2

B) केवल 1, 2 और 3

C) केवल 1, 2 और 4

D) 1, 2, 3 और 4

Hide Answer -
उ रः (b)

ा ाः उपयु म से भारतीय िवदे श से वा (आई.एफ.एस.) अ खल भारतीय से वाओं म शािमल

नहीं है ।

सं िवधान के अनु े द-312 म उपबं ध है िक रा सभा उप थत और मत दे ने वाले सद ों

म से कम-से -कम 2/3 सद ों ारा सं क पा रत कर दे िक रा ीय िहत म ऐसा करना

आव क या समीचीन है तो सं सद िविध ारा सं घ और रा ों के िलये स िलत एक या

अिधक अ खल भारतीय से वाओं का सृ जन कर सकेगी।

अ खल भारतीय से वाओं के अ गत शािमल ह:-

1. भारतीय शासिनक से वा (IAS)

2. भारतीय पु िलस से वा (IPS)

3. भारतीय वन से वा (IFS)

[2] िन िल खत कथनों पर िवचार कीिजयेः

1. भारत म थम शासिनक सु धार आयोग का गठन हनु मनतै या की अ ता म


वष 1966 म िकया गया।
2. भारत म ि तीय शासिनक सु धार आयोग का गठन वीर ा मोइली की
अ ता म वष 2005 म िकया गया।
उपयु म से कौन-सा/से कथन सही है /ह?

A) केवल 1

B) केवल 2

C) 1 और 2 दोनों

D) न तो 1 और न ही 2
Hide Answer -
उ रः (b)

ा ाः उपयु म से कथन (1) गलत है ।

भारत म थम शासिनक सु धार आयोग का गठन 5 जनवरी, 1966 को मोरारजी दे साई

की अ ता म िकया गया, िक ु बाद म मोरारजी दे साई के थान पर के. हनु मनतै या को

इसका अ बनाया गया, ोंिक मोरारजी दे साई को उप- धानमं ी बना िदया गया था।

ि तीय शासिनक सु धार आयोग का गठन अग 2005 म वीर ा मोइली की अ ता

म गिठत िकया गया। अ ै ल 2009 म वीर ा माइली के ाग-प दे ने पर आर.

रामच न को इसका अ बनाया गया।

जय काश नारायण जो ि तीय शासिनक सु धार आयोग के सद थे ने वष 2007 म

ाग-प दे िदया था।

[3] अ रा ीय प रषद के सं दभ म िवचार कीिजयेः


1. रा पित
2. धानमं ी
3. सभी रा ों के मु मं ी
4. सभी रा ों के रा पाल
5. िवधानसभा वाले के शािसत दे शों के मु मं ी
6. उन के शािसत दे शों के शासक, जहाँ िवधानसभाएँ नहीं ह।
उपयु म से कौन इसके सद होते ह?

A) केवल 1, 2, 3 और 4

B) केवल 2, 3, 4 और 5

C) केवल 2, 3, 5 और 6

D) केवल 2, 3, 4, 5 और 6
Hide Answer -
उ रः (c)
ा ाः उपयु म से (1) एवं (4) सही नहीं ह।

अ रा ीय प रषद के िन सद होते ह:

1. धानमं ी: अ
2. सभी रा ों के मु मं ी
3. िवधानसभा वाले के शािसत दे शों के मु मं ी
4. उन के शािसत दे शों के शासक, जहाँ िवधानसभाएँ नहीं ह।
5. रा पित शासन वाले रा ों के रा पाल (न िक सभी रा ों के रा पाल)।
6. धानमं ी ारा नािमत छः के ीय कैिबने ट मं ी (गृ हमं ी सिहत)।

अ रा ीय प रषद की बै ठक एक वष म कम-से -कम तीन बार होती ह।


अ रा ीय प रषद की थायी सिमितः इसकी थापना वष 1996 म की गई। इसके
िन िल खत सद होते ह:

(a) के ीय गृ हमं ी : अ
(b) पाँ च के ीय कैिबने ट मं ी
(c) नौ मु मं ी

अं तरा ीय प रषद का सिचवालयः इसकी थापना वष 1991 म की गई। इसका


मु ख भारत सरकार का एक सिचव होता है । वष 2011 से यह सिचवालय े ीय
प रषद के सिचवालय के प म भी काय कर रहा है ।

[4] भारत के अ रा ीय प रषद के े ािधकार से िन िल खत म से कौन-सा एक


िवषय बाहर रखा गया है ?

रा ों के बीच जो िववाद उ हो गया है उसकी जाँ च पड़ताल करना और


A)
उन पर परामश दे ना।

ऐसे िवषयों का अ े षण और उन पर िवचार-िवमश करना, िजसम कुछ या


B)
सभी रा ों अथवा सं घ और एक या अिधक रा ों का सामा िहत हो।

ऐसे िकसी िवषय पर िसफा रश करने और िवशे षतया उस िवषय के सं बंध म


C)
नीित और कायवाही के अिधक अ े सम य के िलये िसफा रश करना।

िकसी अ रा क नदी या नदी घाटी के या उसके जल के योग, िवतरण या


D)
िनयं ण के सं बंध म िकसी िववाद या प रवाद का अिधिनणय।
Hide Answer -
उ रः (d)

ा ाः उपयु म से िवक (d) अ रा ीय प रषद के े ािधकार से बाहर है ।

सं िवधान के अनु े द-262 म उपबं ध है िक सं सद िविध ारा िकसी अ रा क नदी या

नदी घाटी के या उसके जल के योग, िवतरण या िनयं ण के सं बंध म िकसी िववाद या

प रवाद के ाय-िनणयन के िलये उपबं ध कर सकेगी।

अ रा ीय प रषद के सं बंध म उपबं ध सं िवधान के अनु े द-263 म िकया गया है ।

भारत का रा पित इस प रषद का गठन करता है । इसके कत सं गठन एवं ि या को

प रभािषत या िनधा रत कर सकता है । अनु े द-263 म अ रा ीय प रषद के

े ािधकार की चचा की गई है ।

[5] े ीय प रषद के सं दभ म िन िल खत कथनों पर िवचार कीिजयेः


1. े ीय प रषद की थापना भारतीय सं िवधान के एक अनु े द के अ गत की
गई है ।
2. रा पु नगठन अिधिनयम, 1956 के तहत भारत म छः े ीय प रषदों का
गठन िकया गया।
3. सभी े ीय प रषदों का अ भारत सरकार का गृ हमं ी होता है ।
उपयु म से कौन-सा/से कथन सही है /ह?

A) केवल 3

B) केवल 1 और 2

C) केवल 1 और 3

D) 1, 2 और 3

Show Answer +

[6] े ीय प रषद की सं रचना के िवषय म िन िल खत पर िवचार कीिजयेः


1. भारत सरकार का गृ हमं ी।
2. उस े के रा ों म से े क के मु मं ी।
3. उस े के रा ों म से े क के रा पाल।
4. उस े के े क रा से दो मं ी।
5. उस े म थत े क के शािसत दे श के शासक।
उपयु म से कौन े ीय प रषद म शािमल होते ह।

A) केवल 1, 2 और 3

B) केवल 1, 2 और 5

C) केवल 1, 2, 3 और 4

D) केवल 1, 2, 4 और 5
Hide Answer -
उ रः (d)

ा ाः उपयु म से रा पाल े ीय प रषद का सद नहीं होता है ।

सभी े ीय प रषदों का अ भारत सरकार का गृ हमं ी होता है ।

रा पु नगठन अिधिनयम, 1956 ारा पाँ च े ीय प रषदों का गठन िकया गया तथा

पू व र प रषद अिधिनयम, 1971 के ारा पू व र प रषद का गठन िकया गया।

[7] िन िल खत कथनों पर िवचार कीिजयेः

1. सं सद िविध ारा यह उपबं ध कर सकती है िक न तो सव ायालय और न


ही अ कोई ायालय िकसी अ रा क नदी या नदी घाटी के जल के उपयोग,
िवतरण या िनयं ण के सं बंध म िकसी िववाद या िशकायत के िवषय म
अिधका रता का योग करे गा।
2. अ रा क जल िववाद अिधिनयम, 1956 िकसी अ रा क नदी िववाद
को जल िववाद अिधकरण ारा म थता के िलये िनदश करने का उपबं ध करता
है ।
उपयु म से कौन-सा/से कथन सही है /ह?

A) केवल 1

B) केवल 2

C) 1 और 2 दोनों

D) न तो 1 और न ही 2

Hide Answer -
उ रः (c)

ा ाः उपयु दोनों कथन सही ह।

सं िवधान के अनु े द-262 म अ रा क निदयों या नदी घािटयों के जल सं बंधी िववादों

के ाय-िनणयन के सं बंध म उपबं ध िकया गया है । इस अनु े द के तहत िववादों का

िनपटारा ाियकेतर िविध के ारा िकया जाता है अथात् सव ायालय या अ कोई

ायालय इसम ह े प नहीं कर सकता। सं सद िविध ारा इस कार के िववादों को

िनपटाने के िलये तदथ/अ थायी ायािधकरण की थापना कर सकेगी। इस ायािधकरण

का िनणय अ म एवं िववाद से सं बंिधत सभी प ों के िलये बा कारी होता है ।

इस अनु े द के तहत सं सद ने नदी बोड अिधिनयम, 1956 एवं अ रा क जल िववाद

अिधिनयम, 1956 पा रत िकया है ।

अ रा क जल िववाद अिधिनयम, 1956 नदी िववादों को जल िववाद अिधकरण ारा

म थता के िलये िनदश करने का उपबं ध करता है । इस अिधकरण का अिधिनणय

अनु े द-262(2) के तहत अ म होता है ।

नदी बोड अिधिनयम, 1956 अ रा क नदी या नदी घाटी के िविनयमन या िवकास के

सं बंध म िहतब सरकार को सलाह दे ने के िलये नदी बोड की थापना का उपबं ध करता

है ।
[8] े ीय प रषदों से सं बंिधत िन िल खत कथनों पर िवचार कीिजयेः
1. े ीय प रषद का गठन रा पु नगठन अिधिनयम के अ गत िकया गया है ।
2. े ीय प रषद के और दे शों को सामािजक एवं आिथक मामलों से सं बंिधत
एक समान नीितयों के िनधारण म सहायता प ँ चाती है ।
3. े ीय प रषद आिथक एवं सामािजक योजनाओं, सीमा िववाद, अ रा ीय
प रवहन के पार रक मह वाले िभ िवषयों पर िवचार-िवमश करती है ।
4. े ीय प रषद का िनणय के व रा ों पर बा कारी है ।
उपयु म से कौन-से कथन सही ह?

A) केवल 1, 2 और 3

B) केवल 1, 3 और 4

C) केवल 2, 3 और 4

D) 1, 2, 3 और 4

Hide Answer -
उ रः (a)

ा ाः उपयु म से कथन (4) गलत है ।

े ीय प रषद चचा क एवं परामशदा ी िनकाय है । यह केवल परामश या सलाह दे

सकती है ।

रा पु नगठन अिधिनयम, 1956 के ारा भारत म पाँ च े ीय प रषदों का गठन िकया

गया।

े ीय प रषद के , रा ों तथा सं घशािसत दे शों की सामािजक तथा आिथक मामलों से

सं बंिधत एकसमान नीितयों के िनधारण म सहायता करती है ।

े ीय प रषद सामािजक एवं आिथक योजना, भाषायी अ सं क, सीमा िववाद, नदी का

जल, अ रा ीय यातायात के पार रक मह वाले िभ िवषयों पर िवचार-िवमश तथा

सं ु ित करती है ।

[9] के -रा सं बंधों को िवशे ष प से िकस सं ग म ‘ ु िनिसपल सं बंध’ कहा गया


है ?

A) िवधायन के े म रा पर के के िनयं ण के सं ग म।

B) शासिनक े म रा पर के के िनयं ण के सं ग म।

C) योजना म म रा ों पर के के िनयं ण के सं ग म।

D) िव ीय मामलों म रा पर के के िनयं ण के सं ग म।
Hide Answer -
उ रः (d)

ा ाः उपयु म से कथन (d) सही है ।

सं िवधान के भाग-11 म के -रा के िवधायी एवं शासिनक सं बंधों के िवषय म उपबं ध

है , जबिक सं िवधान के भाग-12 म के -रा के बीच िव ीय सं बंधों के िवषय म उपबं ध

िकया गया है । इन तीनों थितयों म के -रा के बीच श यों का बँ टवारा िकया गया है ,

िक ु िव ीय मामलों म के का रा ों पर अिधक वच है । िव ीय सं बंधों म रा , के

के ऊपर अिधक िनभर हो जाते ह। इसिलये िव ीय मामलों म रा ों की तु लना

‘ ु िनिसपल’ (नगरपािलका) से कर दी जाती है ।

[10]िन िल खत कथनों पर िवचार कीिजयेः

1. भारत म िव ीय सौदों पर ा शु सं घ ारा उद् ृ गहीत एवं सं गृहीत िकया


जाता है , िकंतु रा ों ारा िविनयोिजत िकया जाता है ।
2. भारत म औषधीय एवं साधन की व ु ओं पर उ ाद शु सं घ ारा
उद् ृ गहीत, िक ु रा ों ारा सं गृहीत और िविनयोिजत िकया जाता है ।
उपयु म से कौन-सा/से कथन सही है /ह?

A) केवल 1

B) केवल 2

C) 1 और 2 दोनों

D) न तो 1 और न ही 2

Hide Answer -
उ रः (b)

ा ाः उपयु म से कथन (1) गलत है ।

सं िवधान के अनु े द-268 म सं घ ारा उद् ृ गहीत, िक ु रा ों ारा सं गृहीत एवं

िविनयोिजत िकये जाने वाले शु के सं बंध म उपबं ध है ः

(i) िव ीय सौदों पर ा शु ।

(ii) औषधीय एवं साधन की व ु ओं पर उ ाद शु ।

101व सं िवधान सं शोधन अिधिनयम, 2016 के ारा अनु े द-268 म सं शोधन िकया गया

है ।

ये दोनों सं घ सू ची का िवषय ह, िक ु ये भारत की सं िचत िनिध के भाग नहीं होते । इ रा ों को

सौंप िदया जाता है ।

[11]िन िल खत कथनों पर िवचार कीिजयेः

1. भारत म से वा कर सं घ ारा उद् ृ गहीत, िक ु सं घ और रा ों ारा सं गृहीत एवं


िविनयोिजत िकया जाता है ।
2. भारत म सं घ के उ े ों के िलये कुछ शु ों एवं करों पर सं सद को अिधभार
(Surcharge) लगाने की श है , िक ु इसम रा ों का कोई िह ा नहीं होता
है ।
उपयु म से कौन-सा/से कथन सही है /ह?

A) केवल 1

B) केवल 2

C) 1 और 2 दोनों

D) न तो 1 और न ही 2

Hide Answer -
उ रः (c)

ा ाः उपयु दोनों कथन सही ह।

सं िवधान के अनु े द-268(क) म से वा कर के सं बंध म उपबं ध है । से वा कर सं घ ारा

उद् ृ गहीत, िक ु सं घ व रा ों ारा सं गृहीत एवं िविनयोिजत िकया जाता है ।

अनु े द-268(क) को 88व सं िवधान सं शोधन अिधिनयम, 2003 के ारा जोड़ा गया था,

िकंतु 101व सं िवधान सं शोधन अिधिनयम, 2016 के ारा इसका लोप कर िदया गया है ।

सं िवधान के अनु े द 271 के तहत सं सद को कुछ शु ों व करों पर सं घ के उ े ों के

िलये अिधभार (Surcharge) लगाने की श है । यह अिधभार सीधे के सरकार को

ा होता है और इसम रा ों का कोई िह ा नहीं होता।

101व सं िवधान सं शोधन अिधिनयम, 2016 के ारा अनु े द, 271 म सं शोधन िकया

गया है ।

[12]सू ची-I को सू ची-II से सु मेिलत कीिजये तथा नीचे िदये गए कूट का योग कर
सही उ र का चयन कीिजयेः

सू ची-I सू ची-II

A. कृिष से िभ
1. एकमा रा सरकार का कर।
आयकर

B. रे ल िकरायों तथा 2. के ारा लगाए गए, वसू ले गए तथा ितधा रत


भाड़ों पर कर कर।

3. के ारा लगाए गए, वसू ले गए, िक ु रा ों


C. िनगम कर
को समनु देिशत िकये गए कर।

4. के ारा लगाए गए और वसू ले गए, िक ु सं घ


D. िब ी कर
व रा ों के बीच िवत रत कर।

कूटः
A B C D
A) 4 3 2 1

B) 4 2 3 1

C) 1 2 3 4

D) 1 3 2 4

Hide Answer -
उ रः (a)

ा ाः

कृिष से िभ आयकरः क ारा लगाए गए, वसू ले गए, िक ु सं घ और रा ों के बीच

िवत रत ।

रे ल िकराए तथा भाड़े पर करः के ारा लगाए गए, वसू ले गए, िक ु रा ों को दे िदया

जाता है ।

िनगम करः के ारा लगाए गए, वसू ले गए तथा ितधा रत कर।

ि की करः एकमा रा सरकार का कर।

आयकर पर अिधभारः एकमा सं घ सरकार का कर।

[13]रा सरकारों को कृिष आयकर कौन समनु देिशत करता है ?

A) िव आयोग

B) नीित आयोग

C) रा ीय िवकास प रषद

D) भारत का सं िवधान

Hide Answer -
उ रः (d)

ा ाः उपयु म से िवक (d) सही है ।

सं िवधान के अनु े द-274 म इसका ावधान िकया गया है ।

[14] ा सं िवधान के अ गत भारतीय सं घ के रा िवदे शी ऋण सीधे ले ने की श रखते


ह?

A) हाँ

B) नहीं

C) हाँ , िक ु केवल अं तरा ीय सं गठन से ।

D) हाँ , िक ु भारतीय रज़व बक की अनु मित से ।


Hide Answer -
उ रः (b)

ा ाः उपयु म से िवक (b) सही है ।

सं िवधान के अनु े द-293 म रा ारा उधार ले ने के सं बंध म ावधान िकया गया है ।

भारतीय सं घ के रा िवदे शी ऋण सीधे ा नहीं कर सकते । रा ारा िवदे शी ऋण

भारत सरकार के मा म से (भारत सरकार की गार ी पर) ा िकया जा सकता है ।

रा सरकार भारत म (बाहर नहीं) रा की सं िचत िनिध की गार ी दे कर ऋण ले

सकता है ।

के सरकार िकसी रा सरकार को ऋण दे सकती है या िकसी रा ारा ले ने पर

गार ी दे सकती है । ऋण के ऐसे उ े हे तु आव क धनरािश भारत की सं िचत िनिध पर

भा रत होगी।

सं िवधान के अनु े द-292 म ावधान है िक के सरकार भारत म या इसके बाहर से

भारत की सं िचत िनिध की गारं टी दे कर ऋण ले सकती है ।

[15]सं िवधान के अनु े द-268(क) म से वा कर से सं बंिधत ावधान िकया गया है ।


इसे सं घ ारा लगाया जाता है , िक ु सं घ और रा ों ारा िविनयोिजत एवं
सं किलत िकया जाता है । यह उपबं ध सं िवधान के िकस सं शोधन अिधिनयम के
ारा जोड़ा गया?

A) 80व सं िवधान सं शोधन अिधिनयम, 2000

B) 88व सं िवधान सं शोधन अिधिनयम, 2003

C) 92व सं िवधान सं शोधन अिधिनयम, 2003

D) 93व सं िवधान सं शोधन अिधिनयम, 2006

Hide Answer -
उ रः (b)

ा ाः 88व सं िवधान सं शोधन अिधिनयम, 2003 के ारा सं िवधान म अनु े द-268(क)

जोड़ा गया, िजसम से वा कर से सं बंिधत ावधान है । इसके ारा सं घ सू ची म िव सं ा

92(ग) (से वाओं पर कर) जोड़ा गया। से वाओं पर कर सं घ ारा लगाया जाता है , िक ु सं घ और

रा ों ारा सं किलत एवं िविनयोिजत िकया जाता है ।

101व सं िवधान सं शोधन अिधिनयम, 2016 के ारा अनु े द-268(क) तथा सं घ सू ची म

िव सं ा 92 एवं 92(ग) को हटा िदया गया है ।

[16]िन िल खत म से कौन-सा एक सही सु मेिलत नहीं है ?


अनु े द-265 - कोई कर िविध के ािधकार से ही अिधरोिपत या सं गृहीत
A)
िकया जाएगा, अ था नहीं।

B) अनु े द-266 - भारत की सं िचत िनिध।

C) अनु े द-267 - भारत की लोक ले खा िनिध।

D) अनु े द-275 - कुछ रा ों को सं घ से अनु दान।

Hide Answer -
उ रः (c)

ा ाः उपयु म से (c) सही सु मेिलत नहीं है ।

अनु े द-266 : भारत की सं िचत िनिध एवं लोक ले खा िनिध।

अनु े द-267 : भारत की आक क िनिध।

अनु े द-275 : कुछ रा ों को सं घ से अनु दान।

अनु े द-265 : िविध के ािधकार के िबना करों का अिधरोपण न िकया जाना।

[17]िन िल खत म से कौन-से कर ह, जो भारत सरकार ारा लगाए और वसू ल िकये जाते


ह, िक ु रा ों को दे िदये जाते ह?
1. रे ल भाड़ों और माल भाड़ों पर कर।
2. समाचार प ों के य या िव य पर और उनम कािशत िव ापनों पर कर।
3. कृिष आय पर लगाए जाने वाले कर।
नीचे िदये गए कूट का योग कर सही उ र का चयन कीिजयेः

A) केवल 1

B) केवल 2

C) केवल 1 और 2

D) 1, 2 और 3

Hide Answer -
उ रः (c)

ा ाः उपयु म से कथन (3) गलत है ।

सं िवधान के अनु े द-269 म सं घ ारा उद् ृ गहीत और सं गृहीत, िक ु रा ों को सौंपे जाने

वाले कर के सं बंध म ावधान है । इसके अ गत िन कर आते ह:

रे ल भाड़ों और माल भाड़ों पर कर।

समाचार प ों के य या िव य कर और उनम कािशत िव ापनों पर कर।

कृिष आय पर लगाए जाने वाले कर का रा ों ारा उद् ृ गहीत, सं गृहीत एवं उपभोग िकया

जाता है ।

101व सं िवधान सं शोधन अिधिनयम, 2016 के ारा अनु े द-269 म सं शोधन िकया गया

एवं एक नया अनु े द-269(क) जोड़ा गया।

[18]
भारत म शे यर बाज़ार तथा ू चस बाज़ार म ई ले न-दे न पर करः
1. सं घ सरकार ारा लगाए जाते ह।
2. रा ों ारा एकि त िकये जाते ह।
नीचे िदये गए कूट का योग कर सही उ र का चयन कीिजयेः

A) केवल 1

B) केवल 2

C) 1 और 2 दोनों

D) न तो 1 और न ही 2

Hide Answer -
उ रः (c)

ा ाः उपयु दोनों कथन सही ह।

सं िवधान के अनु े द-268 म ऐसे करों के सं बंध म ावधान है , जो सं घ ारा उद् ृ गहीत,

िक ु रा ों ारा सं गृहीत एवं िविनयोिजत िकये जाते ह। भारत म शे यर बाज़ार तथा ू चस

बाज़ार म ई ले न-दे न पर कर इसी अनु े द के तहत आता है ।

101व सं िवधान सं शोधन अिधिनयम ारा अनु े द-268 म सं शोधन िकया गया है ।

[19]भारत के िनवाचन आयोग के िवषय म िन िल खत म से कौन-सा कथन सही है ?

सं सद, रा के िवधानमं डलों, रा पित, उप-रा पित, पं चायत एवं नगर-िनगम


के िनवाचनों के िलये िनवाचक नामावली तै यार करने और िनवाचनों के
A)
सं चालन, िनदशन एवं िनयं ण की िज़ े दारी भारत के िनवाचन आयोग के
ऊपर होती है ।

सं िवधान म उपबं ध है िक भारत के मु िनवाचन आयु की िनयु


B) रा पित तथा अ िनवाचन आयु ों की िनयु भारत का मु िनवाचन
आयु रा पित के परामश से करे गा।

भारत के िनवाचन आयु ों और ादे िशक आयु ों की से वा-शत व पदाविध


C)
सं सद िविध ारा तय करे गी।

िनवाचन आयोग को सौंपे गए काय के पालन म आयोग की सहायता के िलये


D) रा पित िनवाचन आयोग के परामश से उतने ादे िशक आयु ों की िनयु
कर सकेगा, िजतने वह आव क समझे ।
Hide Answer -
उ रः (d)

ा ाः उपयु म से कथन (d) सही है ।

सं िवधान के भाग-15 के अनु े द-324 म भारत के िनवाचन आयोग हे तु ावधान है ।

सं िवधान के अनु े द-324 (4) म उपबं ध है िक भारत का रा पित, िनवाचन आयोग के

परामश से िनवाचन आयोग को सौंपे गए काय म सहायता के िलये उतने ादे िशक

आयु ों की िनयु कर सकेगा, िजतने वह आव क समझे ।

सं िवधान के अनु े द-324 (1) म उपबं ध है िक सं सद, रा के िवधानमं डलों, रा पित

और उप-रा पित के िनवाचनों के िलये िनवाचक-नामावली तै यार कराने और िनवाचन के

सं चालन, िनदशन व िनयं ण आिद की िज़ े दारी भारत के िनवाचन आयोग के ऊपर है ।

पं चायतों एवं नगरपािलकाओं के िनवाचन की िज़ े दारी रा िनवाचन आयोग के ऊपर है ।

सं िवधान के अनु े द-324 (2) के तहत भारत के मु िनवाचन आयु एवं अ

िनवाचन आयु ों की िनयु भारत के रा पित ारा की जाती है ।

सं िवधान के अनु े द-324(5) म उपबं ध है िक भारत के िनवाचन आयु ों एवं ादे िशक

आयु ों की से वा-शत एवं पदाविध रा पित ारा िनधा रत की जाएगी।

[20]िन िल खत कथनों पर िवचार कीिजयेः

1. भारत म मतदान दे ने का अिधकार सं वैधािनक अिधकार है ।


2. 61व सं िवधान सं शोधन अिधिनयम के ारा मतदान करने की ू नतम आयु म
वृ की गई है ।
उपयु म से कौन-सा/से कथन सही है /ह?

A) केवल 1

B) केवल 2

C) 1 और 2 दोनों

D) न तो 1 और न ही 2

Hide Answer -
उ रः (a)

ा ाः उपयु म से कथन (2) गलत है ।

61व सं िवधान सं शोधन अिधिनयम, 1988 के ारा वष 1989 म मतदान करने की

ू नतम आयु म कमी की गई। अथात 21 वष से घटाकर 18 वष कर दी गई है ।

मतदान का अिधकार एक सं वैधािनक एवं िविधक अिधकार है । सं िवधान के अनु े द -326

एवं जन ितिनिध अिधिनयम, 1951 म उपबं ध है िक लोकसभा और रा की

िवधानसभा का चु नाव मतािधकार के आधार पर िकया जाएगा, िजसम ू नतम आयु

18 वष िनधा रत की गई है ।
[1] िन िल खत म से कौन-सा कथन सही है ?

सं िवधान म उपबं ध है िक भारत का िनवाचन आयोग तीन सद ों से िमलकर बने गा,


A)
िजसम एक मु िनवाचन आयु तथा दो अ िनवाचन आयु होंगे।

सं िवधान म उपबं ध है िक भारत का िनवाचन आयोग एक मु िनवाचन


B) आयु एवं उतने अ िनवाचन आयु ों से , िजतने सं सद िविध ारा समय-
समय पर िनयत करे , िमलकर बने गा।

सं िवधान म उपबं ध है िक भारत का िनवाचन आयोग एक मु िनवाचन


C) आयु तथा उतने अ िनवाचन आयु ों से , िजतने रा पित समय-समय पर
िनयत करे , से िमलकर बने गा।

सं िवधान म उपबं ध है िक भारत का िनवाचन आयोग पाँ च सद ों से िमलकर


D) बने गी, िजसम एक मु िनवाचन आयु तथा चार अ िनवाचन आयु
होंगे।

Hide Answer -
उ रः (c)

ा ाः उपयु म से कथन (c) सही है ।

सं िवधान के अनु े द-324 (2) म ावधान है िक भारत का िनवाचन आयोग मु

िनवाचन आयु और उतने अ िनवाचन आयु ों से , िजतने रा पित समय-समय पर

िनयत करे , िमलकर बने गा।

वष 1950 से 15 अ ू बर 1989 तक भारत का िनवाचन आयोग एक सद ीय था,

िजसम केवल मु िनवाचन आयु होता था, िकंतु 16 अ ू बर 1989 म रा पित ारा

दो अ िनवाचन आयु ों की िनयु कर इसे ब सद ीय बना िदया गया। 1990 म

पु नः इसे एक सद ीय बना िदया गया। अ ू बर, 1993 म पु नः दो आयु ों की िनयु

कर इसे ब सद ीय बना िदया गया। वतमान म इसम एक मु िनवाचन आयु एवं दो

अ िनवाचन आयु ह।

[2] भारत के िनवाचन आयोग के सं दभ म िन िल खत कथनों पर िवचार कीिजयेः

1. मु िनवाचन आयु और अ िनवाचन आयु ों की श याँ एक समान


होती ह, िक ु उ िमलने वाले वे तन व भ े अलग-अलग होते ह।
2. मु िनवाचन आयु का वे तन भारत के सव ायालय के ायाधीश के
समान है जबिक अ िनवाचन आयु ों का वे तन उ ायालय के ायाधीशों के
समान।
3. मु िनवाचन आयु और अ िनवाचन आयु ों के बीच मतभे द की थित
म िनणय ब मत के आधार पर िलया जाता है ।
4. मु िनवाचन आयु का कायकाल 6 वष या 65 वष की आयु , जो भी पहले
हो, जबिक अ िनवाचन आयु ों का कायकाल 6 वष या 62 वष की आयु , जो
भी पहले हो, होता है ।
उपयु म से कौन-सा/से कथन सही है /ह?
A) केवल 3

B) केवल 3 और 4

C) केवल 2, 3 और 4

D) केवल 1, 3 और 4

Hide Answer -
उ रः (a)

ा ाः उपयु म से कथन (3) सही है ।

िनवाचन आयोग ारा िनणय ब मत के आधार पर िलया जाता है । मु िनवाचन आयु

और अ िनवाचन आयु ों के बीच मतभे द की थित म िनणय ब मत के आधार पर

िलया जाता है ।

मु िनवाचन आयु एवं अ िनवाचन आयु ों की श याँ तथा वे तन व भ े समान

होते ह। उ भारत के सव ायालय के ायाधीशों के समान वे तन िमलता है । वतमान

म ायालय के ायाधीशों का वे तन ₹ 90,000 ितमाह है ।

सं िवधान म इस बात का उ े ख नहीं है िक िनवाचन आयोग के सद ों की पदाविध

िकतनी होगी। इसकी पदाविध रा पित िनयम ारा अवधा रत करता है । इनका कायकाल

छः वष या आयु 65 वष जो भी पहले हो, होता है । मु िनवाचन आयु एवं अ

िनवाचन आयु ों की पदाविध समान होती है ।

[3] िन िल खत म से िकसके ारा भारत के मु िनवाचन आयु को उसके पद से हटाया


जा सकता है :

A) भारत के मु ायाधीश ारा उसके िव दोषारोपण िस होने पर।

B) सं घीय मं ि मं डल की सहमित के आधार पर भारत के रा पित के ारा।

एक सिमित ारा, िजसके सद होंगे-भारत के मु ायाधीश, भारत के


C)
िविध मं ी और भारत के उप-रा पित।

D) रा पित ारा सं सद म दो-ितहाई ब मत से पा रत ाव के आधार पर।


Hide Answer -
उ रः (d)

ा ाः उपयु म से िवक (d) सही है ।

सं िवधान के अनु े द-324(5) के पर ु म ावधान िकया गया है िक भारत के मु

िनवाचन आयु को उसी आधार पर हटाया जा सकता है , िजस आधार पर उ तम

ायालय के ायाधीश को हटाते ह। अथात् सािबत कदाचार या अ मता के आधार पर

सं सद के दोनों सदनों ारा िवशे ष ब मत (सं सद के े क सदन की कुल सद सं ा

का ब मत और उप थत एवं मतदान करने वाले सद ों का दो-ितहाई ब मत से सं क

पा रत) से सं क पा रत करके रा पित ारा हटाया जा सकता है ।

भारत के मु िनवाचन आयु की िनयु रा पित ारा तो की जाती है , िक ु वह

रा पित के सादपय पद धारण नहीं करता।

सं िवधान म उपं बध है िक अ िनवाचन आयु ों या ादे िशक आयु ों को उसके पद से

मु िनवाचन आयु की िसफा रश पर ही रा पित हटा सकता है , अ था नहीं।

[4] िन िल खत कथनों पर िवचार कीिजयेः

1. सं िवधान म भारत के िनवाचन आयोग के सद ों की यो ता के सं बंध म कोई


ावधान नहीं िकया गया है ।
2. सं िवधान म उपबं ध है िक भारत का िनवाचन आयु से वािनवृ ि के प ात्
भारत सरकार की िकसी अ से वा म िनयु नहीं हो सकेगा।
उपर् यु म से कौन-सा/से कथन सही है /ह?

A) केवल 1

B) केवल 2

C) 1 और 2 दोनों

D) न तो 1 और न ही 2

Hide Answer -
उ रः (a)

ा ाः उपयु म से कथन (2) गलत है ।

सं िवधान म भारत के िनवाचन आयु के सं बंध म से वािनवृ ि के प ात् भारत सरकार की

िकसी अ िनयु पर रोक नहीं लगाई गई।

सं िवधान म भारत के िनवाचन आयु ों की यो ता के सं बंध म ावधान नहीं िकया गया।

सं िवधान म िनवाचन आयोग के सद ों की पदाविध का उ े ख नहीं िकया गया है ।

[5] िन िल खत म से कौन-सा कथन सही है ?


भारत म िनवाचन की िसफा रश एवं उसकी अिधसू चना िनवाचन आयोग ारा
A)
जारी की जाती है ।

भारत म िनवाचन की िसफा रश सरकार ारा तथा उसकी अिधसू चना िनवाचन
B)
आयोग ारा जारी की जाती है ।

भारत म िनवाचन की िसफा रश िनवाचन आयोग ारा तथा उसकी अिधसू चना
C)
रा पित या रा पाल ारा जारी की जाती है ।

भारत म िनवाचन की िसफा रश गृ ह मं ालय तथा उसकी अिधसू चना िनवाचन


D)
आयोग ारा जारी की जाती है ।

Hide Answer -
उ रः (c)

ा ाः उपयु म से िवक (c) सही है ।

भारत म िनवाचन की अिधसू चना (Notification) िनवाचन आयोग की िसफा रश पर

रा पित या रा पाल के ारा जारी की जाती है । इसके प ात् िनवाचन आयोग िनवाचन की

ितिथ एवं समय-सारणी का िनधारण करता है । लोकसभा एवं रा सभा के िलये रा पित

तथा िवधानसभा एवं िवधान प रषद के िलये रा पाल िनवाचन की अिधसू चना जारी करता

है ।

[6] भारत के िनवाचन आयोग के काय के सं दभ म िन िल खत म से कौन-सा/से

कथन सही है /ह?


1. लोकसभा के अ एवं उपा तथा रा सभा के सभापित एवं उप-
सभापित के पदों के िलये िनवाचन करवाना।
2. पं चायतों, नगरपािलकाओं एवं नगर-िनगमों के िलये िनवाचन करवाना।
3. रा पित एवं उप-रा पित के िलये िनवाचन करवाना।
4. सं सद एवं रा के िवधानमं डलों के सभी चु नाव करवाना।
नीचे िदये गए कूट का योग कर सही उ र का चयन कीिजयेः

A) केवल 3 और 4

B) केवल 1, 3 और 4

C) केवल 1 और 4

D) 1, 2, 3 और 4
Hide Answer -
उ रः (a)

ा ाः उपयु म से कथन (1 ) और( 2) गलत ह।

लोकसभा के अ एवं उपा तथा रा सभा के सभापित एवं उप-सभापित का चु नाव

सदन के सद ों ारा िकया जाता है ।

पं चायतों, नगरपािलकाओं एवं नगर-िनगमों का चु नाव रा िनवाचन आयोग करवाता है ।

भारत के रा पित एवं उप-रा पित तथा सं सद एवं रा के िवधानमं डलों के िनवाचनों के

िलये िनवाचक-नामावली तै यार करने , सभी िनवाचनों के सं चालनों का अधी ण, िनदशन

एवं िनयं ण करने की िज़ े दारी भारत के िनवाचन आयोग के ऊपर होती है ।

[7] िन िल खत कथनों पर िवचार कीिजयेः

1. चु नाव से उ सभी सं देहों एवं िववादों का िनणय भारत के िनवाचन आयोग


ारा िकया जाता है ।
2. सं सद सद ों की अयो ता से सं बंिधत िनणय भारत का िनवाचन आयोग
रा पित की सलाह से करता है ।
उपयु म से कौन-सा/से कथन सही है /ह?

A) केवल 1

B) केवल 2

C) केवल 1 और 2

D) न तो 1 और न ही 2

Hide Answer -
उ रः (d)

ा ाः उपयु दोनों कथन गलत ह।

चु नाव से उ सभी िववादों का िनणय ायालय ारा िकया जाता है ।

सं सद सद ों की अयो ता के सं बंध म कोई िववाद है तो उसका िनणय रा पित ारा

िकया जाता है और रा पित का िनणय अ म होता है , िक ु रा पित ऐसा िनणय भारत के

िनवाचन आयोग के परामश से करे गा। रा िवधानमं डलों के सं दभ म रा पाल िनणय

करता है और उसका िनणय अ म होता है , िकंतु रा पाल ऐसा िनणय भारत के िनवाचन

आयोग से परामश के प ात् करे गा।

दल-बदल से सं बंिधत सं सद सद ों की अयो ता से सं बंिधत िववाद का िनणय अ /

सभापित एवं रा िवधानमं डल के अ /सभापित ारा िकया जाता है ।

[8] भारत के िकसी चु नाव म िनवािचत होने वाला कोई ाशी अपनी ज़मानत रािश
(िनि रािश) खो दे ता है तो उसका अथ है ः
A) मतदान कम आ है ।

B) ब सद ीय िनवाचन े के िलये िनवाचन था।

िनवाचन ाशी की अपने िनकटतम ित ी पर िवजय ब मत कम मतों से


C)
थी।

D) िनवाचन लड़ने वाले ािशयों की सं ा ब त अिधक थी।

Hide Answer -
उ रः (d)

ा ाः उपयु म से िवक (d) सही है ।

िकसी चु नाव म ेक ाशी को एक िनि त ज़मानत/िनि रािश रखनी पड़ती है । उसे

चु नाव म कुल वै ध पड़े मतों का 1/6 भाग लाना पड़ता है । यिद कोई ाशी कुल वै ध पड़े

मतों का 1/6 मत नहीं ा कर पाता तो वह अपनी ज़मानत रािश खो दे ता है । यिद कोई

ाशी िकसी चु नाव म िवजयी घोिषत रहता है और वह कुल वै ध पड़े मतों का 1/6 भाग

नहीं ला पाता है तो इसका कारण है िक चु नाव लड़ने वाले ािशयों की सं ा ब त

अिधक थी।

[9] िन िल खत कथनों पर िवचार कीिजयेः

1. राजनीितक दलों को मा ता दान करना और उसे चु नाव िच आवं िटत


करना।
2. िनवाचक-नामावली तै यार करना और यो मतदाताओं को पं जीकृत करना।
3. नामां कन ि या शु करने की ितिथ, मतदान की ितिथ, मतगणना की ितिथ
एवं चु नाव प रणामों की घोषणा।
4. राजनीितक दलों और उसके उ ीदवारों के िलये आचार सं िहता लागू करना।
उपयु म से कौन-सा/से काय भारत के िनवाचन आयोग का है /ह?

A) केवल 1

B) केवल 1 और 3

C) केवल 1 और 4

D) 1, 2, 3 और 4
Hide Answer -
उ रः (d)
ा ाः उपयु सभी कथन सही ह।

भारत के िनवाचन आयोग के काय िन िल खत ह:

- राजनीितक दलों को मा ता दे ना और उ चु नाव िच आवं िटत करना।


- िनवाचन-नामावली तै यार करना और सभी यो मतदाताओं को पं जीकृत करना।
- सव थम चु नाव की अिधसू चना रा पित (रा ों म रा पाल) ारा जारी की
जाती है । इसके प ात् िनवाचन आयोग के ारा नामां कन ि या शु करने की
ितिथ, मतदान की ितिथ, मतगणना की ितिथ और चु नाव प रणामों की घोषणा की
जाती है ।
- राजनीितक दलों एवं उनके उ ीदवारों के िलये आचार सं िहता का िनमाण करना
तथा उसे लागू करना।
- चु नाव थिगत करना, र करना, दोबारा चु नाव कराना, दोबारा मतगणना
कराना आिद।
- राजनीितक दलों को पं जीकृत करना तथा उ रा ीय या रा रीय पािटयों का
दज़ा दे ना।
- रा पित को सलाह दे ना िक रा पित-शासन वाले रा ों म एक वष समा होने
के प ात् िनवाचन कराया जाए या नहीं।

[10]भारत की िनवाचन णाली के सं दभ म िन िल खत कथनों पर िवचार कीिजयेः


1. वय मतािधकार के आधार पर िनवाचन णाली।
2. एकल सं मणीय मत के ारा आनु पाितक ितिनिध णाली।
3. आनु पाितक ितिनिध की सू ची णाली।
4. अ िनवाचन की सं चयी मतदान णाली।
उपयु म से कौन-सा/से कथन सही है /ह?

A) केवल 1 और 2

B) केवल 1, 2 और 3

C) केवल 1, 2 और 4

D) 1, 2, 3 और 4

Hide Answer -
उ रः (a)

ा ाः उपयु म से कथन (3) और (4) गलत ह।

भारत की िनवाचन णाली म एवं अ दोनों तरीके अपनाए गए ह। लोकसभा

एवं रा ों की िवधानसभाओं का चु नाव यानी फ -पा द-पो िस म तथा

रा सभा, रा की िवधान प रषद, रा पित एवं उप-रा पित का चु नाव अ एवं

आनु पाितक ितिनिध णाली के आधार पर एकल सं मणीय मत ारा िकया जाता है ।

भारत के चु नाव म सू ची णाली तथा सं चयी मतदान णाली को नहीं अपनाया गया है ।
[11]िन िल खत कथनों पर िवचार कीिजयेः

1. ायालय ारा घोिषत अपराधी चु नाव लड़ने के िलये अयो होगा,


इसका िनणय भारत के रा पित ारा िकया जाता है ।
2. मतािधकार एवं िनवािचत होने का अिधकार सं वैधािनक अिधकार है ।
उपयु म से कौन-सा/से कथन सही है /ह?

A) केवल 1

B) केवल 2

C) 1 और 2 दोनों

D) न तो 1 और न ही 2

Hide Answer -
उ रः (d)

ा ाः उपयु दोनों कथन गलत ह।

जन ितिनिध अिधिनयम, 1951 म सं सद ारा उपबं ध िकया गया है िक ायालय ारा

िजस को अपराधी घोिषत करते ए दो वष की सज़ा सु नाई है , वह सज़ा के दौरान

तथा अगले छः वष की अविध के िलये चु नाव लड़ने के िलये अयो होगा।

मतदान का अिधकार सं वैधािनक अिधकार के साथ-साथ वै धािनक अिधकार है । मतदान के

अिधकार का उ े ख सं िवधान के अनु े द-326 एवं जन ितिनिध कानू न, 1951 म

िकया गया है , िक ु िनवािचत होने का अिधकार एक वै धािनक अिधकार है , िजसकी चचा

सं सद ारा जन ितिनिध अिधिनयम, 1951 म की गई है । इसिलये दोनों साथ होने की

थित म इसे वै धािनक अिधकार माना जाएगा, िक ु केवल मतदान के अिधकार की बात

हो तो इसे सं वैधािनक अिधकार माना जाएगा।

[12]उस दे श म आनु पाितक ितिनिध णाली आव क नहीं है , जहाँ :

A) कोई आरि त िनवाचन े है ।

B) ि -दलीय णाली िवकिसत ई है ।

C) फ -पा -द-पो िस म चिलत है ।

D) रा पित और सं सदीय शासन णाली का सिम ण है ।

Hide Answer -
उ रः (b)

ा ाः आनु पाितक ितिनिध णाली का उ े समाज के े क समू ह को समु िचत अनु पात

म शासन म ितिनिध िदलाना है । इस णाली म अ सं यक समू हों को उनकी सं ा के

अनु पात म ितिनिध ा होता है ।

इस णाली म िविभ समू हों का ितिनिध करने वाले राजनीितक दलों को उसके ा

मत के अनु पात म स ा म िह े दारी िमलती है । यह णाली वहाँ मह हीन हो जाती है ,

जहाँ ि -दलीय णाली िवकिसत ई है ।


[13]भारत के प रसीमन आयोग के सं दभ म िन िल खत कथनों पर िवचार कीिजयेः

1. भारत म प रसीमन आयोग का गठन रा पित ारा िकया जाता है ।


2. प रसीमन आयोग के आदे श जब लोकसभा या रा िवधानसभा के स ु ख
रखे जाते ह, तब उन आदे शों म कोई सं शोधन नहीं िकया जा सकता।
3. प रसीमन आयोग के आदे श को िकसी ायालय म चु नौती नहीं दी जा सकती।
उपयु म से कौन-सा/से कथन सही है /ह?

A) केवल 3

B) केवल 1 और 3

C) केवल 2 और 3

D) 1, 2 और 3

Hide Answer -
उ रः (d)

ा ाः उपयु तीनों कथन सही ह।

सं सद अब तक चार प रसीमन आयोग अिधिनयम बना चु की है ः- 1952, 1962, 1972,

एवं 2002

रा पित ारा वष 1952, 1963, 1973 एवं 2002 म प रसीमन आयोग का गठन िकया

गया।

प रसीमन आयोग के आदे श जब लोकसभा या रा िवधान सभा म रखे जाते ह, तब उन

आदे शों म कोई सं शोधन नहीं िकया जा सकता।

इसके आदे शों को िकसी ायालय म चु नौती नहीं दी जा सकती।

प रसीमन आयोग एक तं सं था है , जो िनवाचन आयोग के साथ िमलकर काय करती

है ।

इसका गठन पू रे दे श म िनवाचन े ों की सीमा खींचने के उ े से िकया जाता है ।

प रसीमन के बाद प रसीमन आयोग े क िनवाचन े म जनसं ा की सं रचना दे खता

है । 87व सं िवधान सं शोधन अिधिनयम, 2003 के ारा िनवाचन े के प रसीमन का

आधार जनगणना-2001 को बनाया गया है ।

वतमान म प रसीमन आयोग का अ सव ायालय के से वािनवृ ायाधीश कुलदीप

िसं ह है ।

[14]िन िल खत म से कौन, लोकसभा चु नावों म डाले गए मतों की पु नगणना का

आदे श दे सकता है , यिद कोई ाशी इस तरह की मां ग करता है ?

A) भारत का िनवाचन आयोग

B) रा का िनवाचन आयोग

C) चु नाव े का िनवाचन अिधकारी

D) भारत का रा पित
Hide Answer -
उ रः (c)

ा ाः उपयु म से िवक (c) सही है ।

चु नाव े के िनवाचन अिधकारी को इस बात का अिधकार होता है िक वह मतों की

पु नगणना करा सके।

सं िवधान के अनु े द-324(6) म उपबं ध है िक जब िनवाचन आयोग ऐसा अनु रोध करे ,

तब रा पित या िकसी रा का रा पाल िनवाचन आयोग या ादे िशक आयु को उतने

कमचारी उपल कराए, िजतने उसके काय के िलये आव क हो।

भारत का रा पित िनवाचन आयोग को सौंपे गए काय के पालन म उसकी सहायता के

िलये उतने ादे िशक आयु ों की िनयु कर सकेगा, िजतने वह आव क समझे ।

[15]िन िल खत कथनों पर िवचार कीिजयेः-

1. भारत के सं िवधान म ावधान है िक सं सद िविध ारा सं िवधान ारं भ होने के


दो वष के भीतर और उसके प ात् े क पाँ च वष पर िव आयोग का गठन
करे गी।
2. िव आयोग कुल पाँ च सद ों के िमलकर बने गा, िजसम एक अ एवं चार
सद होंगे।
3. भारत का िव मं ी िव आयोग की िसफा रश को सं सद के े क सदन म
रखवाएगा।
उपयु म से कौन-सा/से कथन सही है /ह?

A) केवल 2

B) केवल 1 और 2

C) 1, 2 और 3

D) उपयु म से कोई नहीं।

Hide Answer -
उ रः (a)

ा ाः उपयु म से कथन (1) और (3) गलत ह।

सं िवधान के अनु े द-280 म िव आयोग के सं बंध म ावधान है ।

सं िवधान के अनु े द-280(1) म ावधान है िक रा पित (न िक सं सद) इस सं िवधान के

ारं भ से दो वष के भीतर और उसके बाद े क पाँ च वष पर या आव कतानु सार उससे

पहले िव आयोग का गठन करे गा।

िव आयोग कुल पाँ च सद ों से िमलकर बने गा, िजसम एक अ और चार सद

होंगे।

सं िवधान के अनु े द-281 म ावधान है िक िव आयोग की िसफा रश को रा पित (न

िक सं घ के िव मं ी) सं सद के े क सदन म रखवाएगा।

सं सद को िव आयोग के सद ों की यो ता एवं चयन िविध िनधा रत करने की श है ।


[16]सं िवधान के अनु े द-280(3) म िव आयोग के क ों/काय के सं बंध म
ावधान है । इस सं दभ म िन िल खत कथनों पर िवचार कीिजयेः
1. भारत की सं िचत िनिध से धन िनकालने की अनु मित दे ना।
2. ा करों को रा ों के भागों म बाँ टना।
3. सहायता अनु दान के िलये रा ों के आवे दनों पर िवचार करना।
4. रा िव आयोग की िसफा रशों के आधार पर रा की सं िचत िनिध के
सं व न के िलये आव क उपाय सु झाना।
उपयु म से कौन- से कथन सही ह?

A) केवल 1 और 2

B) केवल 2 और 3

C) केवल 2, 3 और 4

D) 1, 2, 3 और 4

Hide Answer -
उ रः (c)

ा ाः उपयु म से कथन (1) सही नहीं है ।

सं िवधान के अनु े द-280(3) म िव आयोग के क ों/काय के सं बंध म ावधान ह,

जो िन िल खत ह -

करों के शु आगमन का सं घ व रा ों के बीच िवभाजन तथा रा ों के भागों म बँ टवारे के

सं बंध म।

भारत की सं िचत िनिध म से रा ों को सहायता अनु दान दे ने के सं बंध म।

रा के िव आयोग की िसफा रशों के आधार पर रा की सं िचत िनिध के सं व न के

िलये आव क उपाय के सं बंध म।

सु ढ़ िव के िहत म रा पित ारा आयोग को िनिद िकये गए िकसी अ िवषय के बारे

म।

िव आयोग की िसफा रशों की कृित सलाहकारी होती है । इसे मानने के िलये सरकार

बा नहीं है ।

भारत की सं िचत िनिध से धन िनकालने की अनु मित दे ना सं सद का काय है , न िक िव

आयोग का।

[17]िन िल खत म से कौन के की समे िकत/सं िचत िनिध से रा ों को राज की


सहायता हे तु अनु दान दे ने वाले िस ां तों की अनु शंसा करता है ।

A) िव मं ी

B) नीित आयोग

C) िव आयोग

D) सं सद
Hide Answer -
उ रः (c)

ा ाः उपयु म से िवक (c) सही है ।

सं िवधान के अनु े द-280(3) म िव आयोग के क ों के सं बंध म उपबं ध है । इसके

तहत के ीय िव आयोग भारत की सं िचत िनिध से रा ों को राज की सहायता हे तु

अनु दान दे ने वाले िस ा ों की अनु शंसा करता है ।

[18]भारत के िव आयोग के सं दभ म िन िल खत म से कौन-सा कथन सही है ?

A) वह अवसं रचना िवकास हे तु िवदे शी पूं जी अ वाह को ो ािहत करता है ।

B) वह सावजिनक े के ित ानों म समु िचत िव िवतरण को सु गम बनाता है ।

C) वह िव शासन म पारदिशता सु िनि त करता है ।

D) उपयु म से कोई नहीं।

Hide Answer -
उ रः (d)

ा ाः उपयु म से कोई कथन सही नही ं है ।

सं िवधान के अनु े द-280(3) म िव आयोग के क ों/काय के सं बंध म ावधान है ।

उपयु म से कोई भी काय िव आयोग के काय म शािमल नहीं है ।

[19]िन िल खत कथनों पर िवचार कीिजयेः

1. सं घीय िव आयोग पं चायतों को िव ीय सहायता दे ने के िलये सं ु ित नहीं कर


सकता।
2. चौदहव िव आयोग की कायविध वष 2020 तक होगी।
3. के ीय करों म रा ों की िह े दारी 42% कर दी गई है ।
उपयु म से कौन-सा/से कथन सही है /है ?

A) केवल 2

B) केवल 3

C) केवल 2 और 3

D) केवल 1 और 3
Hide Answer -
उ रः (c)

ा ाः उपयु म से कथन (1) गलत है ।

सं िवधान के अनु े द-280(3) म उपबं ध है िक के ीय िव आयोग, रा िव आयोग

की िसफा रशों के आधार पर रा ों म पं चायतों के सं साधनों की आपू ित के िलये आव क

उपायों का सु झाव दे सकेगी।

14व िव आयोग का गठन रा पित के ारा 2 जनवरी, 2013 को िकया गया। इसकी

अविध 1 अ ै ल, 2015 से 31 माच, 2020 तक है । वाई.वी. रे ी (भारतीय रज़व बक

के से वािनवृ त गवनर) को इसका अ बनाया गया है । इसने अपनी रपोट 15 िदस र,

2014 को रा पित को सौंपी। इसकी िसफा रशों के आधार पर के ीय करों म रा ों की

िह े दारी 32% से बढ़ाकर 42% कर दी गई है ।

[20]लोकसभा अ के सं दभ म िन िल खत म से कौन-सा कथन सही है ?

A) इसका अ मनोनीत िकया जाता है ।

B) इसका अ िनवािचत िकया जाता है ।

C) इसका अ िनयु िकया जाता है ।

D) इसका अ चयिनत िकया जाता है ।

Hide Answer -
उ रः (d)

ा ाः उपयु म से िवक (d) सही है ।

सं िवधान के अनु े द-93 म उपबं ध है िक लोकसभा अपने दो सद ों को अपना अ

और उपा चु नेगी। जब-जब अ या उपा का पद र होगा तब लोकसभा

िकसी अ सद को अ या उपा चु नेगी।


[1] िन िल खत कथनों पर िवचार कीिजयेः

1. सं िवधान के भाग-XX म आपातकाल का उपबं ध िकया गया है ।


2. 44व सं िवधान सं शोधन अिधिनयम,1978 के ारा ‘सश िव ोह’ को
‘आं त रक अशां ित’ श से अ ः थािपत िकया गया।
3. रा ीय आपातकाल की उद् घोषणा रा पित के ारा केवल मं ि मं डल की
िल खत सलाह पर ही की जा सकती है ।

उपयु म से कौन-सा/से कथन सही है /ह?

A) केवल 3

B) केवल 1 और 2

C) केवल 1 और 3

D) केवल 2 और 3

Hide Answer -
उ रः (a)

ा ाः उपयु म से कथन-1 और 2 गलत ह।

सं िवधान के भाग-18 म आपातकाल सं बंधी ावधान िकया गया है । सं िवधान म तीन कार

के आपातकाल का ावधान है ः-

1. रा ीय आपात → अनु े द 352

2. रा पित शासन/ सं वैधािनक आपातकाल → अनु े द -356

3. िव ीय आपातकाल → अनु े द-360

44व सं िवधान सं शोधन अिधिनयम, 1978 के ारा आं त रक अशां ित को ‘सश िव ोह’

श से अ ः थािपत िकया गया है । 1975 म ‘आं त रक अशां ित’ के आधार पर रा ीय

आपात लगाया गया था।

रा ीय आपातकाल की उद् घोषणा के तीन आधार है ः यु , बा आ मण या सश

िव ोह (आं त रक अशां ित के थान पर)।

रा पित रा ीय-आपात की घोषणा तभी कर सकता है , जब मं ि मं डल िल खत प से

िसफा रश करे । स ू ण सं िवधान म केवल अनु े द-352 म ही ‘मं ि मं डल’/‘कैिबने ट’ श

का उपयोग िकया गया है , अ था सभी जगह ‘मं ि प रषद’ श का योग िकया गया है ।

[2] िन िल खत कथनों पर िवचार कीिजयेः

1. रा ीय आपातकाल की घोषणा सं पूण भारतवष या उसके िकसी भाग के


िलये की जा सकती है ।
2. रा ीय आपातकाल की उद् घोषणा जारी होने के छः माह के भीतर सं सद के
दोनों सदनों के ारा अनु मोिदत नहीं िकये जाने पर समा हो जाएगी।
3. रा ीय आपातकाल की उद् घोषणा का ाव सं सद के े क सदन ारा
कुल सद सं ा के ब मत ारा पा रत होना आव क है ।

उपयु म से कौन-सा/से कथन सही है /ह?


A) केवल 1

B) केवल 1 और 2

C) केवल 1 और 3

D) 1, 2 और 3

Hide Answer -
उ रः (a)

ा ाः उपयु म से कथन (2) और (3) गलत ह।

रा ीय आपातकाल की उद् घोषणा जारी होने के एक महीने के अ र सं सद के दोनों सदनों

ारा अनु मोिदत होना आव क है अ था आपातकाल की उद् घोषणा समा हो जाएगी।

एक माह का ावधान 44व सं िवधान सं शोधन अिधिनयम, 1978 के ारा िकया गया है ।

इससे पू व दो माह का ावधान था। यिद दोनों सदनों ारा अनु मोदन हो गया है तो रा ीय

आपातकाल छः माह तक जारी रहे गा। इसके बाद े क छः माह पर सं सद इसे अनं तकाल

तक बढ़ा सकती है ।

यिद लोकसभा का िवघटन एक माह के अदं र िबना अनु मोिदत िकये हो जाता है तो

लोकसभा के पु नगठन के प ात् पहली बै ठक से 30 िदनों तक जारी रहे गा। इसकी कार

लोकसभा ारा छः माह के अ र िबना अनु मोिदत िकये िवघटन हो जाता है तो लोकसभा

के पु नगठन के प ात पहली बै ठक से 30 िदनों तक जारी रहे गा।

रा ीय आपातकाल की उद् घोषणा अथवा इसे जारी रखने का ेक ाव सं सद के

े क सदन ारा िवशे ष ब मत ( े क सदन की कुल सद सं ा का ब मत तथा

उप थत एवं मत दे ने वाले सद ों का 2/3 ब मत ारा पा रत) से पा रत होना आव क

है ।

रा ीय आपातकाल की घोषणा स ू ण भारत म अथवा उसके िकसी भाग के िलये की जा

सकती है ।

[3] िन िल खत कथनों पर िवचार कीिजयेः

1. रा पित ारा रा ीय आपात की उद् घोषणा को िकसी भी समय एक दू सरी


उद् घोषणा से समा िकया जा सकता है । ऐसी उद् घोषणा के िलये सं सद
के अनु मोदन की आव कता नहीं होती।
2. रा ीय आपातकाल की उद् घोषणा के समय सं सद का कायकाल एक बार
म एक वष के िलये बढ़ाया जा सकता है , िक ु अिधकतम तीन बार से
अिधक नहीं।

उपयु म से कौन-सा/से कथन सही है /ह?

A) केवल 1

B) केवल 2

C) 1 और 2 दोनों

D) न तो 1 और न ही 2
Hide Answer -
उ रः (a)

ा ाः उपयु म से कथन (2) गलत है ।

रा ीय आपातकाल की उद् घोषणा के समय सं सद का कायकाल एक बार म एक वष के

िलये बढ़ाया जा सकता है । सं सद का कायकाल एक-एक वष के िलये िकतनी भी बार

बढ़ाया जा सकता है ,अथात बढ़ाने की कोई सीमा िनधा रत नहीं है , िक ु आपात की

उद् घोषणा की समा के प ात् इसका िव ार छः माह से ादा नहीं हो सकता। इसी

तरह सं सद रा िवधानसभा का कायकाल एक बार म एक वष के िलये (िकतनी भी बार)

बढ़ा सकती है , िक ु आपातकाल की समा के प ात् इसका िव ार छः माह से

अिधक नहीं हो सकता।

रा पित रा ीय आपातकाल की उद् घोषणा को िकसी भी समय एक दू सरी उद् घोषणा ारा

समा कर सकता है । ऐसी उद् घोषणा को सं सदीय अनु मोदन की आव कता नहीं होती।

[4] रा ीय आपातकाल की उद् घोषणा से सं बंिधत सं िवधान सं शोधन अिधिनयम, 1978

भारत के रा पित से िन िल खत के अनु सार काय करने की अपे ा करता है ः

A) स ू ण मं ि प रषद की सामू िहक सलाह।

B) के ीय मं ि मं डल की सलाह।

C) भारत के मा धानमं ी की सलाह।

D) भारत के सव ायालय की सलाह।

Hide Answer -
उ रः (b)

ा ाः उपयु म से कथन (b) सही है ।

रा ीय आपातकाल की उद् घोषणा से सं बंिधत 44व सं िवधान सं शोधन अिधिनयम, 1978

के ारा रा ीय आपातकाल की उद् घोषणा रा पित तभी करे गा, जब मं ि मं डल/कैिबने ट

िल खत प से सलाह दे ता है । अनु े द 352 म ‘मं ि मं डल’ को प रभािषत िकया गया है ।

[5] िन िल खत म से िकन प र थितयों म भारत के रा पित के ारा रा ीय


आपातकाल की उद् घोषणा की जा सकती है ?

1. यु
2. बा आ मण
3. आतं रक अशां ित
4. सश िव ोह

नीचे िदये गए कूट का योग कर सही उ र का चयन कीिजये -


A) केवल 1, 2 और 3

B) केवल 1, 2 और 4

C) केवल 1, 3 और 4

D) केवल 2, 3 और 4

Hide Answer -
उ रः (b)

ा ाः भारत का रा पित सं िवधान के अनु े द-352 के तहत रा ीय आपात की उद् घोषणा

कर सकता है । इसके तीन आधार हो सकते ह-

1. यु

2. बा आ मण

3. सश िव ोह

नोटः मू ल सं िवधान म ‘आं त रक अशां ित’ श को शािमल िकया गया था, िक ु 44व सं िवधान

सं शोधन अिधिनयम, 1978 के ारा आं त रक अशां ित को ‘सश िव ोह’ श से अ ः थािपत

कर िदया है । रा पित केवल वा िवक घटनाओं के आधार पर ही उद् घोषणा नहीं कर सकता,

ब आस खतरे के आधार पर इसके पहले भी रा ीय आपातकाल की उद् घोषणा कर सकता

है ।

[6] िन िल खत कथनों पर िवचार कीिजयेः

1. रा ीय आपातकाल की उद् घोषणा के समय रा मं ि प रषद का िवघटन


हो जाता है ।
2. रा ीय आपातकाल की उद् घोषणा के समय रा की िवधाियका िनलं िबत
हो जाती है ।
3. सं सद ारा रा ीय आपातकाल के समय बनाई गई कोई भी िविध
आपातकाल की समा के प ात् एक वष तक भावी रहती है ।

उपयु म से कौन-सा/से कथन सही है /ह?

A) केवल 3

B) केवल 2 और 3

C) 1, 2 और 3

D) उपयु म से कोई नहीं


Hide Answer -
उ रः (d)

ा ाः उपयु म से कोई कथन सही नहीं है ।

रा ीय आपातकाल के समय के की कायपािलका श का िव ार रा की

कायपािलका श पर हो जाता है । के , रा को िकसी भी िवषय पर कायकारी िनदश

दे सकता है । अथात रा के के पू ण िनयं ण म हो जाता है िक ु रा मं ि प रषद

िनलं िबत/िवघिटत नहीं होती।

रा ीय आपातकाल के समय के -रा श िवभाजन का िवतरण िनलं िबत हो जाता है ।

सं सद रा सू ची पर िविध बना सकती है , िक ु रा की िवधाियका श िनलं िबत नहीं

होती और न ही रा की िवधाियका िनलं िबत/िवघिटत होती है ।

सं सद ारा रा ीय आपातकाल म रा सू ची म बनाई गई िविध आपातकाल की समा के

प ात् छः माह तक भावी रहती है । रा ीय आपातकाल के समय यिद सं सद का स नहीं

चल रहा होता है तो रा पित अ ादे श जारी कर रा सू ची के िवषय म कानू न बना सकता

है ।

रा ीय आपात के समय रा पित के ारा रा ों को िदये जाने वाले धन/िव को कम

अथवा समा कर सकता है । रा पित के ऐसे े क आदे श को सं सद के दोनों के सम

रखा जाता है ।

[7] िन िल खत कथनों पर िवचार कीिजयेः

1. जब रा ीय आपातकाल की उद् घोषणा वतन म हो तो सं िवधान के


अनु े द-359 के तहत अनु े द-19 के ारा द छः मू ल अिधकारों का
तः ही िनलं बन हो जाता है ।
2. जब रा ीय आपातकाल की उद् घोषणा वतन म हो तो सं िवधान के
अनु े द-358 के तहत केवल उसी मू ल अिधकार का िनलं बन होगा,
िजसका रा पित के आदे श म विणत है ।
3. रा ीय आपातकाल की उद् घोषणा पर सं िवधान के अनु े द-20 और 21
का िनलं बन नहीं होगा।

उपयु म से कौन-सा/से कथन सही है /ह?

A) केवल 3

B) केवल 1 और 2

C) केवल 2 और 3

D) 1, 2 और 3
Hide Answer -
उ रः (a)

ा ाः उपयु म से कथन (1) और (2) गलत ह।

जब रा ीय आपातकाल की उद् घोषणा वतन म रहती है , तब सं िवधान के अनु े द-358

के तहत अनु े द-19 के तहत ा छः मू ल अिधकारों का तः ही िनलं बन हो जाता है ।

इसके िनलबं न के िलये अलग से आदे श नहीं िनकालना पड़ता है । जब आपातकाल की

उद् घोषणा वतन म नहीं रहती है तो अनु े द 19 तः ही पु नज िवत हो जाता है ।

44व सं िवधान सं शोधन अिधिनयम 1978 के ारा अनु े द-358 पर ितबं ध लगाया गया

िक अनु े द-19 के तहत मू ल अिधकारों का िनलं बन केवल यु या बा आ मण के

आधार पर ही िनलं िबत िकया जा सकता है । इसका िनलं बन सश िव ोह के आधार पर

नहीं िकया जा सकता।

अनु े द-19 ारा द मू ल अिधकार रा ीय आपातकाल म स ू ण अविध के िलये

िनलं िबत िकये जाते ह।

रा ीय आपातकाल की उद् घोषणा के समय सं िवधान के अनु े द-359 के तहत केवल

उ ीं मौिलक अिधकारों का िनलं बन हो सकता है , िजनका उ े ख रा पित के आदे श म

होता है और उतनी ही अविध के िलये िनलं िबत होते ह, िजतनी अविध का उ े ख रा पित

के आदे श म होता है ।

अनु े द-359 यु , बा आ मण या सश िव ोह (आं त रक आपातकाल) तीनों

प र थितयों म लागू िकया जा सकता है । यह स ू ण अविध या अ अविध के िलये हो

सकता है ।

अनु े द-358 सं पूण दे श म तथा अनु े द-359 सं ू ण दे श या दे श के िकसी भाग पर

लागू िकया जा सकता है ।

अनु े द-359 के तहत अनु े द-20 और 21 का िनलं बन नहीं िकया जा सकता है ,

जबिक अनु े द-358 के तहत अनु े द-19 के तहत ा सभी मू ल अिधकारों का

िनलं बन हो जाता है । 44व सं िवधान सं शोधन से पू व अनु े द- 20 एवं 21 का िनलं बन

िकया जा सकता था, िकंतु 44व सं िवधान सं शोधन अिधिनयम के बाद रा ीय आपातकाल

के समय अनु े द-20 और 21 का िनलं बन नहीं हो सकता अथात इस सं शोधन के ारा

अनु े द -359 यह पर ितबं ध लगाया गया।

[8] यिद कोई रा के के िनदशों का पालन करने या उ भावी बनाने म


असफल रहता है तो रा पित के िलये यह मानना िविधपू ण होगा िक उस रा म
ऐसी प र थितयाँ उ हो गई ह, िजससे वहाँ की शासन व था को सं िवधान के
उपबं धों के अनु सार नहीं चलाया जा सकता है । यह ावधान भारतीय सं िवधान के
िकस अनु े द म िकया गया है ?

A) अनु े द-355

B) अनु े द-356

C) अनु े द-360

D) अनु े द-365
Hide Answer -
उ रः (d)

ा ाः उपयु म से िवक (d) सही है ।

रा ों म रा पित शासन को सं वैधािनक आपातकाल अथवा रा आपात के नाम से भी

जानते ह। वहाँ की शासन व था को रा का रा पाल रा पित के नाम पर रा सिचव

की सहायता से या रा पित ारा िनयु िकसी सलाहकार की सहायता से चलाता है ।

रा ों म रा पित शासन लगाने के दो आधार ह:-

1. अनु े द - 356

2.अनु े द - 365

अनु े द-365 म ावधान है िक यिद कोई रा के के िनदशों का पालन करने या उसे

भावी करने म असफल रहता है तो रा पित ारा यह माना जाएगा िक वहाँ की शासन

व था सं िवधान के उपबं धों के अनु सार नहीं चलाई जा सकती और वहाँ रा पित शासन

लगाया जा सकता है ।

अनु े द-356 म उपबं ध है िक रा पित को रा के रा पाल से ितवे दन िमलने पर या

यह समाधान हो जाता है िक ऐसी थितयाँ उ हो गई ह, िजससे उस रा का शासन

सं िवधान के उपबं धों के अनु सार नहीं चलाया जा सकता, तो रा पित शासन लगाया जा

सकता है ।

[9] रा ों म रा पित शासन के िवषय म िन िल खत म से कौन-सा कथन सही है ?

रा ों म रा पित शासन की उद् घोषणा जारी होने के छःमाह के अ र इसका


A)
अनु मोदन सं सद के दोनों सदनों ारा होना आव क है ।

सं सद ारा रा पित शासन की उद् घोषणा की ीकृित िमलने के प ात्


B)
रा पित शासन छःमाह तक चलता है ।

सं सद ारा े क छःमाह की ीकृित से रा पित शासन को अन समय के


C)
िलये बढ़ाया जा सकता है ।

सं िवधान के अनु े द-356 के अ गत भारत म रा पित शासन सव थम पे ू


D)
(Pepsu) म लगाया गया था।
Hide Answer -
उ रः (d)

ा ाः उपयु म से िवक (d) सही है ।

सं िवधान के अनु े द-356 के तहत भारत म रा पित शासन सव थम पे ू (PEPSU),

अथात पिटयाला ए ई पं जाब े ट यू िनयन म वष 1951 म लगाया गया था।

रा पित शासन की उद् घोषणा जारी होने के 2 माह के अ र इसका अनु मोदन सं सद के

दोनों सदनों ारा होना आव क है । यिद लोकसभा 2 माह के अ र िबना अनु मोदन िकये

िवघिटत हो जाती है तो लोकसभा के पु नगठन के प ात पहली बै ठक से 30 िदन के अ र

अनु मोदन करना आव क है , अ था रा पित शासन वतन म नहीं रहे गा। उसी तरह

सं सद ारा रा पित की ीकृित के प ात् 6 माह के अ र सं सद ारा इसका अनु मोदन

िकया जाना आव क है । यिद लोकसभा िबना अनु मोदन िकए छः माह के अ र िवघिटत

हो जाती है तो लोकसभा के पु नगठन होने एवं पहली बै ठक से 30 िदनों के अ र

अनु मोदन िकया जाना आव क है । यह अनु मोदन सं सद के े क सदन ारा उप थत

एवं मत दे ने वाले सद ों के ब मत (सामा ब मत) ारा िकया जाता है ।

सं सद े क छः माह पर ाव पा रत कर इसे अगले छः माह के िलये बढ़ा सकती है ,

िक ु अिधकतम तीन वष से अिधक नहीं बढ़ा सकती है ।

[10]भारत म रा पित शासन को सं सद ारा एक वष के प ात् छः माह के िलये तभी


बढ़ाया जा सकता है , जबः-

1. पू रे भारत म अथवा पू रे रा या उसके िकसी भाग म रा ीय आपात की


उद् घोषणा की गई हो।
2. चु नाव आयोग यह मािणत करे िक सं बंिधत रा म िवधानसभा चु नाव के
िलये सम ाएँ उप थत ह।
3. उपयु दोनों ावधान 42व सं िवधान सं शोधन अिधिनयम, 1976 के ारा
जोड़े गए ह।

नीचे िदये गए कूट का योग कर सही उ र का चयन कीिजयेः

A) केवल 1 और 2

B) केवल 1 और 3

C) केवल 2 और 3

D) 1, 2, 3 और 4
Hide Answer -
उ रः (a)

ा ाः उपयु म से कथन (3) गलत है ।

44व सं िवधान सं शोधन अिधिनयम, 1970 के ारा (न िक 42व सं िवधान सं शोधन

अिधिनयम, 1976 के ारा) सं सद ारा रा पित शासन को एक वष के प ात् भी जारी

रखने के िलये दो ितबं ध लगाता है ।

सं सद रा पित शासन को एक वष के प ात् छः माह की अविध के िलये तभी बढ़ा सकती

है , जबः

1. पू रे भारत म अथवा पू रे रा या उसके िकसी भाग म रा ीय आपातकाल की

उद् घोषणा की गई हो।

2. चु नाव आयोग यह मािणत करे िक सं बंिधत रा म िवधानसभा चु नाव के िलये

सम ाएँ उप थत ह ।

रा पित ारा, रा पित शासन को िकसी भी समय परवत घोषणा ारा वापस िलया जा

सकता है । ऐसी घोषणा की िलये सं सद की अनु मित आव क नहीं होती।

[11]रा पित शासन की उद् घोषणा के भाव के िवषय म िन िल खत कथनों पर िवचार


कीिजयेः

1. रा पित, मु मं ी के ने तृ वाली मं ि प रषद को भं ग कर दे ता है ।


2. रा पित रा िवधानमं डल को िनलं िबत या िवघिटत कर सकता है ।
3. रा पित, सं बंिधत उ ायालय की थित, र, श याँ एवं काय म
कमी कर सकता है ।

उपयु म से कौन-सा/से कथन सही है /ह?

A) केवल 3

B) केवल 1 और 2

C) केवल 1 और 3

D) 1, 2 और 3
Hide Answer -
उ रः (b)

ा ाः उपयु म से कथन-3 गलत है ।

रा पित शासन की उद् घोषणा के दौरान रा पित सं बंिधत रा के उ ायालय की

थित, र, श यां एवं काय म कमी नहीं कर सकता है । उ ायालय की सभी

थितयां भावी रहगी।

रा पित, मु मं ी के ने तृ वाली मं ि प रषद को भं ग कर दे ता है ।

रा पित, रा िवधानसभा को िवघिटत या िनलं िबत कर सकता है । रा िवधानसभा के

िवघटन व िनलं बन की थित म सं सद, रा के िलये िविध बनाने की श रा पित या

उसके ारा उ े खत िकसी अिधकारी को दे सकती है । लोकसभा के सं म नहीं रहने पर

रा पित रा की सं िचत िनिध के उपयोग को ािधकृत कर सकता है । सं सद के स म

नहीं रहने पर रा पित अ ादे श जारी कर सकता है ।

रा पित घोषणा कर सकता है िक सं सद, रा िवधाियका की श यों का योग करे गी।

रा पित अथवा सं सद या अ िकसी िवशे ष अिधकारी ारा बनाई गई िविध रा पित शासन

के प ात् भी भावी रहे गी, िक ु रा िवधाियका ारा ऐसी िविध को वापस, प रवितत या

पु नः लागू िकया जा सकता है ।

[12]सं िवधान के अनु सार- 356 के िवषय म िन िल खत कथनों पर िवचार कीिजयेः

1. इस अनु े द के अ गत की गई रा पित शासन की उद् घोषणा की


ाियक समी ा की जा सकती है ।
2. इस उद् घोषणा के साथ ही रा िवधानसभा भं ग की जा सकती है ।

उपयु म से कौन-सा/से कथन सही है /ह?

A) केवल 1

B) केवल 2

C) 1 और 2 दोनों

D) न तो 1 और न ही 2
Hide Answer -
उ रः (a)

ा ाः उपयु म से कथन (2) गलत है ।

एस.आर. बो ई बनाम भारत सं घ, 1994 वाद म सव ायालय ने िनणय िदया िक

रा पित शासन की उद् घोषणा के साथ ही रा िवधानसभा को िवघिटत नहीं िकया जा

सकता है , जब तक ऐसी उद् घोषणा को सं सद के दोनों सदनों ारा दो माह के अं दर

अनु मोिदत नहीं कर िदया जाता। रा पित केवल रा िवधानसभा को िनलं िबत कर सकता

है । इस वाद म ावधान िकया गया िक सं वैधािनक तं की िवफलता यथाथ म होनी चािहये

न िक क त आधार पर।

44व सं िवधान सं शोधन अिधिनयम, 1978 म ावधान िकया गया िक रा पित शासन की

उद् घोषणा ाियक समी ा से परे नहीं है । एस.आर. बो ई बनाम भारत सं घ, 1994 वाद

म सव ायालय ने िनणय िदया िक रा पित शासन लागू करने की रा पित की

उद् घोषणा असं वैधािनक एवं अवै ध है तो ायालय को िवघिटत सरकार तथा िवघिटत

अथवा िनलं िबत िवधानसभा को पु नः बहाल करने की श है । इस वाद म कहा गया िक

अनु े द-356 के अधीन िविश श यां ह । इनका योग िवशे ष प र थितयों म यदा-

कदा ही करना चािहये ।

[13]िन िल खत म से सं िवधान के िकस अनु े द म यह ावधान है िक सं घ का यह


क होगा िक वह बा आ मण और आं त रक अशा से े क रा की
सं र ा करे ?

A) अनु े द-352

B) अनु े द-355

C) अनु े द-356

D) अनु े द-365

Hide Answer -
उ रः (b)

ा ाः उपयु म से िवक (b) सही है ।

सं िवधान के अनु े द-355 म उपबं ध है िक सं घ का यह क होगा िक वह बा

आ मण और आं त रक अशां ित से े क रा की सं र ा करे और े क रा की

सरकार का इस सं िवधान के उपबं धों के अनु सार चलाया जाना सु िनि त कर।

[14]सं िवधान का अनु े द-360 भारत म िव ीय आपातकाल के िवषय म उपबं ध


करता है । इस सं दभ म िन िल खत कथनों पर िवचार कीिजयेः

1. िव ीय आपात की उद् घोषणा भारत के रा पित ारा के ारा की जाती है ,


िक ु ऐसी उद् घोषणा को एक माह के अ र सं सद की ीकृित िमलना
आव क है ।
2. िव ीय आपात की अिधकतम समय सीमा सं िवधान म िनधा रत नहीं की गई
है ।
3. रा पित ारा िव ीय आपात को िकसी प ातवत उद् घोषणा ारा वापस
िलया जा सकता है या प रवितत िकया जा सकता है ।

उपयु म से कौन-सा/से कथन सही है /ह?

A) केवल 3

B) केवल 1 और 3

C) केवल 2 और 3

D) 1, 2 और 3

Hide Answer -
उ रः (c)

ा ाः उपयु म से कथन-1 गलत है ।

भारत के रा पित के ारा िव ीय आपातकाल की उद् घोषणा को दो माह के अ र (न िक

एक माह के अदं र) सं सद के दोनों सदनों से ीकृित िमलना अिनवाय है । यिद लोकसभा

का िवघटन दो माह के अ र िबना ीकृित िदये हो जाता है तो लोकसभा का पु नगठन

होने एवं इसकी पहली बै ठक से 30 िदनों के अं दर ीकृित दे ना आव क है , अ था

इसके प ात् उद् घोषणा वतन म नहीं रहे गी। सं सद ारा एक बार अनु मोदन करने के

प ात् िव ीय आपातकाल अिनि त काल के िलये वृ रहे गा, जब तक रा पित ारा इसे

वापस न ले िलया जाता।

सं िवधान म िव ीय आपात की अिधकतम समय सीमा िनधा रत नहीं की गई है । सं सद ारा

एक बार ीकृत हो जाने के प ात् पु नः ीकृित की आव कता नहीं पड़ती है ।

रा पित िव ीय आपात को िकसी प ातवत उद् घोषणा ारा वापस ले सकता है या

प रवितत कर सकता है । ऐसी उद् घोषणा के िलये सं सदीय ीकृित की आव कता नहीं

होती है ।

िव ीय आपातकाल की उद् घोषणा को मं ज़ूरी दे ने वाला ाव सं सद के े क सदन ारा

उप थत एवं मतदान करने वाले सद ों के ब मत (सामा ब मत) ारा पा रत िकया

जाता है , जबिक अनु े द-352 के तहत ाव को सं सद के सदनों के िवशे ष ब मत की

आव कता पड़ती है ।

[15]िव ीय आपातकाल के िवषय म िन िल खत कथनों पर िवचार कीिजयेः

1. िव ीय आपात की थित म रा पित के की से वा म लगे सभी अथवा


िकसी भी े णी के से वकों के वे तन एवं भ ों म कटौती हे तु िनदश जारी कर
सकता है ।
2. िव ीय आपात की थित म भी रा पित उ तम ायालय के ायाधीशों
के वे तन एवं भ े म कटौती नहीं कर सकता।

उपयु म से कौन-सा/से कथन सही है /ह?


A) केवल 1

B) केवल 2

C) 1 और 2 दोनों

D) न तो 1 और न ही 2

Hide Answer -
उ रः (a)

ा ाः उपयु म से कथन (2) गलत है ।

िव ीय आपात की थित म रा पित के की से वा म लगे सभी अथवा िकसी भी े णी के

से वकों के वे तन एवं भ े म कटौती हे तु िनदश जारी कर सकता है तथा सव एवं उ

ायालय के सभी ायाधीशों के वे तन एवं भ ों म कटौती हे तु िनदश जारी कर सकता है ।

रा की से वा म िकसी भी अथवा सभी वग के से वकों के वे तन व भ ों म कटौती हे तु

िनदश जारी कर सकता है ।

रा िवधाियका ारा पा रत सभी धन िवधे यक अथवा अ िव िवधे यक को रा पित के

िलये आ र त रखना।

के िकसी रा को िव ीय औिच सं बंधी िस ां तों के पालन का िनदश दे सकता है ।

[16]िन िल खत कथनों पर िवचार कीिजयेः

1. सं िवधान के अनु े द-352 के अं तगत भारत म अब तक चार बार रा ीय


आपात की उद् घोषणा हो चु की है ।
2. सं िवधान के अनु े द-360 के तहत भारत म केवल एक बार िव ीय
आपात की उद् घोषणा ई है , जब भारत म वष 1991 म आिथक सं कट
उ आ था।

उपयु म से कौन-सा/से कथन सही है /ह?

A) केवल 1

B) केवल 2

C) 1 और 2 दोनों

D) न तो 1 और न ही 2
Hide Answer -
उ रः (d)

ा ाः उपयु दोनों कथन गलत ह।

सं िवधान के अनु े द-352 के तहत भारत म रा ीय आपातकाल की उद् घोषणा अब तक

तीन बार हो चु की है ।

पहली उद् घोषणा वष 1962 म भारत-चीन यु के समय ई। यह उद् घोषणा जनवरी

1968 तक जारी रही। इसिलये वष 1965 म भारत-पािक ान यु के समय इसकी

आव कता नहीं पड़ी।

दू सरी उद् घोषणा वष 1971 म भारत-पािक ान यु के समय ई। यह आपातकाल

भावी ही था िक वष 1975 म ‘आं त रक अशां ित ’ के नाम पर तीसरी उद् घोषणा कर दी

गई। दू सरी एवं तीसरी उद् घोषणा दोनों एक साथ माच 1977 म समा ई।

जनता पाट की सरकार ने वष 1977 म तीसरी आपातकाल की घोषणा की जाँ च के िलये

‘शाह आयोग’ का गठन िकया। आयोग ने वष 1975 के आपातकाल को तकसं गत नहीं

बताया।

सं िवधान के अनु े द-360 के तहत िव ीय आपात की उद् घोषणा अभी तक एक बार भी

नहीं की गई है ।

सं िवधान सभा के सद - दयनाथ कुंज ने िव ीय आपातकाल को रा की िव ीय

सं भु ता के िलये गं भीर खतरा कहा है ।

[17]‘ओमबु ड्समै न’ सं था का गठन करने वाला सव थम दे श कौन है ?

A) नाव

B) िफनलै

C) ीडन

D) डे नमाक
Hide Answer -
उ रः (c)

ा ाः उपयु म से िवक (c) सही है ।

ओमबु ड्समै न सं था पहली बार वष 1809 म ीडन म थािपत की गई। ीडन की

सं सद ारा चार वष के िलये ओमबु ड्समै न िनयु िकया जाता है । उसे सं सद ारा ही

हटाया जा सकता है । यह अपना वािषक ितवे दन सं सद को ही ु त करता है ।

ीडन के प ात यह सं था िफनलै (1919), डे नमाक (1955) और नव (1962) म

गिठत की गई। उपरो सभी दे श िडने िवयन दे श’ कहलाते ह।

रा कुल दे शों (रा मं डलीय दे श) म ू ज़ीलै पहला दे श है , िजसने वष 1962 म

ओमबु ड्समै न णाली को पािलयामे ी किम र फॉर इनवे गे शन के प म अपनाया।

ि टे न ने इसे 1967 म अपनाया।

भारत म ओमबु ड्समै न को लोकपाल/लोकायु के नाम से जाना जाता है ।

ाँ स म शासिनक ायालय की च व था थािपत की गई।

समाजवादी दे शों म मु ार णाली (Procurator System) नाम से सं था सृ िजत की

गई।

[18]ओमबु ड्समै न को भारत म लोकपाल एवं लोकायु के नाम से जानते ह। इस


िवषय म िन िल खत कथनों पर िवचार कीिजयेः

1. भारत म लोकपाल और लोकायु की थापना का सु झाव ि तीय


शासिनक सु धार आयोग के ारा िदया गया था।
2. सं सद म पहला लोकपाल िवधे यक वष 1968 म चौथी लोकसभा म ुत
िकया गया था।
3. लोकपाल और लोकायु िवधे यक, 2011 को 116व सं िवधान सं शोधन
अिधिनयम, 2011 के तहत सं वैधािनक दज़ा ा है ।

उपयु म से कौन-सा/से कथन सही है /ह?

A) केवल 1

B) केवल 2

C) केवल 1 और 2

D) 1, 2 और 3
Hide Answer -
उ रः (b)

ा ाः उपयु म से कथन (1) और (3) गलत ह।

भारत म थम शासिनक सु धार आयोग की िसफा रश पर नाग रकों की सम ाओं के

समाधान हे तु दो िवशे ष अिधका रयों: लोकपाल एवं लोकायु की िनयु की गई। इसकी

थापना िडने िवयन एवं ू ज़ीलै की तज़ पर की गई। भारत म थम शासिनक सु धार

आयोग का गठन 5 जनवरी, 1966 म मोरारजी दे साई की अ ता म गिठत की गई थी।

बाद म इसका अ हनु मंतैया को बनाया गया।

भारत म पहला लोकपाल िवधे यक वष 1968 म चौथी लोकसभा म ु त िकया गया,

िकंतु लोकसभा के िवघटन के कारण प रत न हो सका। लोकपाल िवधे यक- 1968,

1971, 1977, 1985, 1986, 1988 और 2001 लोकसभा के िवघटन के कारण

पा रत न हो सका। लोकपाल िवधे यक 1989 एवं 2011 सरकार ारा वापस ले िलया

गया।

लोकपाल एवं लोकायु िवधे यक, 2011 को सं वैधािनक दज़ा दान करने की बात की

गई। इसके िलये 116वाँ सं िवधान सं शोधन अिधिनयम, 2011 लाया गया। लोकपाल एवं

लोकायु िवधे यक, 2011 कुछ सं शोधनों के साथ पा रत िकया गया, िकंतु 116वाँ

सं िवधान सं शोधन िवधे यक पा रत न हो सका और इसे सं वैधािनक दज़ा न िमल पाया,

जबिक िडने िवयन दे शों म इसे सं वैधािनक दज़ा ा है ।

यह िवधे यक लोकसभा एवं रा सभा म पा रत होने के प ात् 01.01. 2014 से लागू हो

गया। इस िवधे यक को लोकपाल एवं लोकायु िवधे यक, 2013 के नाम से जाना जाता है ।

[19]लोकपाल और लोकायु िवधे यक, 2013 के सं दभ म िन िल खत म से कौन-सा


कथन सही नहीं है ?

A) लोकपाल के े ािधकार से भारत के धानमं ी को बाहर रखा गया है ।

ितवष ₹10 लाख से अिधक िवदे शी सहायता ा सभी इकाइयाँ लोकपाल


B)
के े ािधकार के अं तगत आती ह।

C) लोकपाल म एक अ और अिधकतम 8 सद हो सकते ह।

लोकपाल के 50% सद अनु सूिचत जाित, जनजाित, अ िपछड़ा वग,


D)
अ सं क और मिहला े णी से होंगे।
Hide Answer -
उ रः (a)

ा ाः उपयु म से िवक (a) सही नहीं है । लोकपाल एवं लोकायु ए , 2013 के

अं तगत भारत के धानमं ी, मं ीगण, सं सद सद तथा सभी वग के सरकारी कमचारी आते ह।

िवदे शी अनु दान (िनयमन) कानू न, 2010 के अं तगत ितवष ₹10 लाख से अिधक िवदे शी

सहायता ा सभी इकाईयाँ लोकपाल के े ािधकार के अं तगत आएं गी।

लोकपाल म एक अ एवं अिधकतम 8 सद होंगे, िजसम 50% सद अनु सूिचत

जाित, जनजाित, अ िपछड़ा वग, अ सं क वग तथा मिहलाओं के बीच से होंगे।

लोकपाल के अ एवं सद ों की िनयु एक चयन सिमित ारा होती है िजसम

धानमं ी, लोकसभा अ , लोकसभा म िवप का ने ता, भारत के मु ायाधीश या

उसके ारा नािमत उ ायालय का कायरत ायाधीश और कोई िति त ायवे ता

शािमल होते ह। चयन सिमित की मदद के िलये एक सच सिमित के गठन का ावधान है ।

[20]िन िल खत म से िकस रा म सव थम लोकायु सं था की थापना की गई?

A) महारा

B) उड़ीसा

C) िबहार

D) राज थान

Hide Answer -
उ रः (a)

ा ाः उपयु म से िवक (a) सही है ।

भारत म सव थम लोकायु सं था/कायालय की थापना महारा म वष 1971 म की

गई, िकंतु लोकपाल और लोकयु अिधिनयम सव थम उड़ीसा रा म 1970 म पा रत

िकया गया िजसे वष 1983 म लागू िकया गया। वष 2013 तक 21 रा ों एवं एक क -

शािसत रा िद ी म लोकायु सं था की थापना की जा चु की है ।

रा ों म लोकायु सं था/कायालय की थापना वषः

-महारा - 1971

-राज थान - 1973

-िबहार - 1974

-उ र दे श - 1975
[1] िन िल खत कथनों पर िवचार कीिजयेः

1. लोकपाल और लोकायु अिधिनयम, 2013 रा ों को लोकायु की


बनावट की तं ता दे ता है ।
2. लोकायु एवं उप-लोकायु ों की िनयु म रा पाल तं है ।
3. लोकायु सं बंिधत रा के रा पाल के ित उ रदायी होता है ।
4. लोकायु की िसफा रश सलाहकारी कृित की होती है ।

उपयु म से कौन-से कथन सही ह?

A) केवल 1 और 4

B) केवल 1, 2 और 4

C) केवल 1, 3 और 4

D) केवल 2, 3 और 4

Hide Answer -
उ रः (a)

ा ाः उपयु म से कथन (2) और (3) गलत ह।

लोकायु और उप-लोकायु ों की िनयु सं बंिधत रा के रा पाल ारा की जाती है ,

िकंतु रा पाल िनयु के सं बंध म तं त नहीं है । इनकी िनयु के समय रा पाल को

रा के उ ायालय के मु ायाधीश एवं रा िवधानसभा म िवप के ने ता से

परामश करना अिनवाय है ।

नोटः आं दे श म िवधानसभा म िवप के ने ता से परामश करने की आव कता नहीं होती।

कनाटक म रा िवधान प रषद का सभापित, िवधानसभा अ एवं िवप के ने ता से सलाह की

आव कता होती है

लोकायु रा िवधानमं डल के ित उ रदायी होता है , न िक रा पाल के ित।

लोकपाल एवं लोकायु अिधिनयम, 2013 रा ों को लोकायु की बनावट की तं ता

दान करता है । इसिलये सभी रा ों म लोकायु की बनावट एक जै सी नहीं होती है ।

लोकायु की िसफा रश सलाहकारी कृित की होती ह । इसकी िसफा रश रा सरकार

के िलये बा कारी नहीं होती ह।

अिधकां श रा ों म लोकायु का कायकाल 5 वष या 65 वष की आयु तक, जो भी पहले

हो, होता है । यह पु निनयु का पा नहीं होता है ।

िहमाचल दे श, आं दे श, म दे श और गु जरात के मु मं ि यों को लोकायु की

प रिध म रखा गया है , जबिक िबहार, उ र दे श, महारा , उड़ीसा एवं राज थान के

मु मं ि यों को लोकायु की प रिध से बाहर रखा गया है ।

[2] क ीय सतकता आयोग के सं दभ म िन िल खत म से कौन-सा कथन सही है ?


सं सद ारा पा रत अिधिनयम के तहत क ीय सतकता आयोग का गठन वष
A)
1964 म िकया गया।

क ीय सतकता आयोग एक ब सद ीय िनकाय है , िजसके अ एवं


B)
सद ों की िनयु भारत के रा पित ारा की जाती है ।

इसके अ एवं सद ों का कायकाल पाँ च वष या 65 वष की आयु तक,


C)
जो भी पहले हो, होता है ।

क ीय सतकता आयु का वे तन, भ े एवं अ से वा-शत भारत के मु


D)
ायमू ित के समान होती ह।

Hide Answer -
उ रः (b)

ा ाः उपयु म से िवक (b) सही है ।

क ीय सतकता आयोग एक ब सद ीय िनकाय/सं था है , िजसका एक अ (क ीय

सतकता आयु ) एवं दो या दो से कम सतकता आयु होते ह। इनकी िनयु भारत के

रा पित के ारा तीन सद ीय सिमित की िसफा रशों पर की जाती है ।

क ीय सतकता आयोग का गठन वष 1964 म क ीय सरकार ारा पा रत एक ाव के

अं तगत आ था, िकंतु वष 2003 म सं सद ारा पा रत सतकता आयोग अिधिनयम,

2003 के ारा इसे सां िविधक/वै धािनक दज़ा दान िकया गया। इस कार ारं भ म यह

आयोग न तो सं वैधािनक था और न वै धािनक सं था थी, िकंतु इस अिधिनयम के प ात

आयोग को वै धािनक दज़ा ा हो गया।

ाचार को रोकने के िलये वष 1962 म सं थानम सिमित का गठन िकया गया था। इस

आयोग का गठन इसी सिमित की िसफा रश पर िकया गया था।

आयोग के अ और सद ों का कायकाल 4 वष या 65 वष की आयु तक, जो भी

पहले हो, होता है । अपने कायकाल के प ात् वे क अथवा रा सरकार के िकसी भी पद

के िलये यो नहीं होते ।

क ीय सतकता आयु (अ ) का वे तन, भ े एवं अ से वा-शत सं घ लोक से वा

आयोग के सद के समान होती ह।

[3] क ीय सतकता आयोग के अ एवं सद ों की िनयु भारत के रा पित के


ारा एक सिमित की िसफा रश के आधार पर की जाती है । इस सिमित म शािमल
होते ह-

1. भारत का धानमं ी
2. लोकसभा म िवप का ने ता
3. क ीय गृ हमं ी
4. क ीय मानव सं साधन मं ी

नीचे िदये गए कूट का योग कर सही उ र का चयन कीिजयेः


A) केवल 1 और 2

B) केवल 1, 2 और 3

C) केवल 1, 2 और 4

D) केवल 1, 3 और 4

Hide Answer -
उ रः (b)

ा ाः उपयु म से कथन (4) गलत है ।

क ीय सतकता आयोग के अ एवं सद ों की िनयु के सं बंध म गिठत तीन सद ीय

सिमित म िन शािमल होते ह।

1. भारत का धानमं ी (अ के प म)

2. लोकसभा म िवप का ने ता

3. क ीय गृ हमं ी

[4] िन िल खत कथनों पर िवचार कीिजयेः

1. क ीय सतकता आयोग को िसिवल और फौज़दारी ायालय की श याँ


ा ह।
2. क ीय सतकता आयोग अपना वािषक ितवे दन रा पित को सौंपता है और
रा पित ऐसे ितवे दन को सं सद के दोनों सदनों के सम ु त करता है ।
3. क ीय सतकता आयोग क सरकार और उसके ािधकरणों से कोई भी
जानकारी अथवा रपोट की मां ग कर सकता है ।

उपयु म से कौन-सा/से कथन सही है /ह?

A) केवल 3

B) केवल 1 और 2

C) केवल 1 और 3

D) केवल 2 और 3
Hide Answer -
उ रः (d)

ा ाः उपयु म से कथन (1) गलत है ।

क ीय सतकता आयोग को िसिवल/दीवानी ायालय की श याँ ा ह। उसे फौज़दारी

ायालय की श याँ ा नहीं ह।

क ीय सतकता आयोग अपना वािषक ितवे दन रा पित को सौंपता है और रा पित ऐसे

ितवे दन को सं सद के दोनों सदनों के सम ु त करता है ।

क ीय सतकता आयोग सरकार और उसके ािधकरणों से कोई भी जानकारी या रपोट की

मां ग कर सकता है ।

रा पित, क ीय सतकता आयोग के अ एवं सद ों को दु राचरण या अ मता के

आधार पर हटा सकता है , िकंतु ऐसे आरोप को रा पित को उ तम ायालय के पास

भे जना पड़ता है । जाँ च के उपरां त ायालय आरोप को सही पाता है तो रा पित उनको पद

से हटा सकता है ।

[5] क ीय सतकता आयोग के सं दभ म िन िल खत म से कौन-से कथन सही ह?

1. क सरकार इस आयोग की अनु शंसा के आधार पर वतन िनदे शालय का


िनदे शक िनयु करता है ।
2. यह आयोग मानी लॉिडं ग ए के तहत ले न-दे न सं बंधी सू चना ा कर
सकता है ।
3. यह आयोग ाचार िनवारण अिधिनयम, 1988 के तहत िद ी िवशे ष
पु िलस थापना को िनदश दे सकता है ।

नीचे िदये गए कूट का योग कर सही उ र का चयन कीिजयेः

A) केवल 1 और 2

B) केवल 1 और 3

C) केवल 2 और 3

D) 1, 2 और 3
Hide Answer -
उ रः (d)

ा ाः उपयु सभी कथन सही ह।

क सरकार क ीय सतकता आयोग की अनु शंसा के आधार पर वतन िनदे शालय के

िनदे शक को िनयु करता है ।

इस आयोग को मनी लॉिडं ग ए , 2002 के तहत ले न-दे न सं बंधी सू चना ा करने का

िवशे षािधकार है ।

यह आयोग ाचार िनवारण अिधिनयम, 1988 के तहत िन काय करता है

अपराधों की जाँ च से सं बंिधत िद ी िवशे ष पु िलस थापना के कामकाज की दे ख-रे ख

करना।

अपराधों की जाँ च से सं बंिधत िद ी िवशे ष पु िलस की थापना को िनदश दे ना।

िद ी िवशे ष पु िलस बल ारा की गई जाँ च की समी ा करना।

लोकपाल एवं लोकायु ए , 2013 के ारा क ीय सतकता (िनगरानी) आयोग ए ,

2003 तथा िद ी िवशे ष पु िलस थापना अिधिनयम, 1946 म सं शोधन िकया गया है ।

क ीय सतकता आयोग अपनी कायवाही अपने मु ालय नई िद ी से सं चािलत करता है ।

[6] सल ोउर सं र ण िवधे यक, 2011 के तहत िन िल खत म से िकसे इस ए


से बाहर रखा गया है ?

1. रा ीय सु र ा से जु ड़ी सू चनाओं को
2. ज ू -क ीर को
3. से ना को
4. धानमं ी की सु र ा म लगे िवशे ष सु र ा बल को
5. पू व धानमं ी की सु र ा म लगे बल को

नीचे िदये गए कूट का योग कर सही उ र का चयन कीिजयेः

A) केवल 1, 3 और 4

B) केवल 1, 2 3 और 4

C) केवल 1, 3, 4 और 5

D) 1, 2, 3, 4 और 5
Hide Answer -
उ रः (d)

ा ाः उपयु सभी सही ह।

सल ोअर सं र ण िवधे यक, 2011 के तहत िन िल खत को इस कानू न से बाहर रखा

गया है -

1. रा ीय सु र ा से जु ड़ी सू चना को

2. ज ू -क ीर

3. से ना

4. धानमं ी की सु र ा म लगे िवशे ष सु र ा बल।

5. पू व धानमं ि यों की सु र ा म लगे बल को।

इस ए के तहत सल ोअर ( ाचार की सू चना दे ने वाले ) की पहचान को गोपनीय

रखना।

गलत या दु भावना से े रत सू चना दे ने पर िशकायतकता को दो वष की जे ल तथा

₹30,000 जु माना लगाया जा सकता है ।

सू चना िल खत प से या ई-मे ल के ारा दी जा सकती है ।

9 मई, 2014 को सल ोअर ए , 2011 को रा पित की ीकृित िमल गई है ।

[7] रा ीय मानवािधकार के िवषय म िन िल खत कथनों पर िवचार कीिजये :

1. रा ीय मानवािधकार आयोग का अ केवल भारत के सव ायालय


का से वािनवृ मु ायमू ित ही बन सकता है ।
2. रा ीय मानवािधकार आयोग का गठन पाँ च सद ों से िमलकर होता है
िजसम एक अ एवं चार अ सद होते ह।
3. इसके अ एवं सद ों की िनयु भारत के रा पित ारा की जाती है ।
4. इसके अ एवं सद ों का कायकाल पाँ च वष या 65 वष की आयु तक
जो भी पहले हो, होता है ।

उपयु म से कौन-से कथन सही ह?

A) केवल 3 और 4

B) केवल 1, 3 और 4

C) केवल 1, 2 और 3

D) केवल 2, 3 और 4
Hide Answer -
उ रः (c)

ा ाः उपयु म से कथन (4) गलत है ।

रा ीय मानवािधकार आयोग के अ एवं सद ों का कायकाल पाँ च वष या 70 वष की

आयु तक, जो भी पहले हो, होता है ।

रा ीय मानवािधकार आयोग का अ केवल भारत के सव ायालय के से वािनवृ

मु ायमू ित ही बन सकता है ।

यह रा ीय आयोग अ एवं चार सद ों से िमलकर बनता है और इसके अ एवं

सद ों की िनयु भारत के रा पित ारा की जाती है ।

भारत के रा पित ारा आयोग के अ या उसके सद ों को उसके पदाविध से पहले भी

दु राचार या अ मता के आधार पर हटा सकता है । ऐसे िवषय को रा पित उ तम

ायालय म जाँ च के िलये रखता है । जाँ च के उपरा उ तम ायालय इन आरोपों को

सही पाता है तो उसकी सलाह पर रा पित उसे हटा सकता है ।

[8] िन िल खत म से कौन एक रा ीय मानवािधकार आयोग का अं ग नहीं है ?

A) रा ीय अ सं क आयोग के अ

B) रा ीय िपछड़ा वग आयोग का अ

C) रा ीय अनु सूिचत जाित एवं रा ीय अनु सूिचत जनजाित आयोग के अ

D) रा ीय मिहला आयोग के अ

Hide Answer -
उ रः (b)

ा ाः उपयु म से िवक (b) सही नहीं है ।

रा ीय मानवािधकार आयोग म पू णकािलक सद ों के अित र चार अ पदे न सद भी

होते ह। रा ीय िपछड़ा वग आयोग का अ इसम शािमल नहीं होता है ।

[9] रा ीय मानवािधकार आयोग एक अ और चार सद ों से िमलकर बनता है


िजसे पू णकािलक सद कहते ह। इसके सद ों म शािमल होते ह-

1. एक सद उ तम ायालय का केवल से वािनवृ ायाधीश।


2. एक सद उ ायालय का कायरत या से वािनवृ त मु या अ
ायाधीश।
3. दो अ सद िजसे मानवािधकार से सं बंिधत जानकारी या कायानु भव हो।

नीचे िदए गए कूट का योग कर सही उ र का चयन कीिजये :

A) केवल 3

B) केवल 1 और 3

C) केवल 2 और 3

D) 1, 2 और 3
Hide Answer -
उ रः (a)

ा ाः उपयु म से कथन 1 और 2 गलत ह।

रा ीय मानवािधकार आयोग म एक अ जो केवल भारत के सव ायालय का केवल

से वािनवृ मु ायमू ित तथा सद के प म, एक सद सव ायालय के कायरत

या से वािनवृ ायाधीश, एक सद उ ायालय के कायरत या से वािनवृ मु

ायाधीश और दो अ िज मानवािधकार से सं बंिधत जानकारी हो अथवा

कायानु भव ा हो, शािमल होते ह। इसे रा ीय मानवािधकार आयोग का पू णकािलक

सद कहते ह

[10]रा ीय मानवािधकार आयोग के अ एवं सद ों की िनयु भारत के रा पित


ारा धानमं ी के ने तृ म गिठत छः सद ीय सिमित की िसफा रश पर की
जाती है । इस सिमित के सद ों म शािमल होते ह-

1. लोकसभा का अ
2. रा सभा का सभापित
3. सं सद के दोनों सदनों के मु िवप ी दल के ने ता
4. के ीय गृ हमं ी

नीचे िदए गए कूट का योग कर सही उ र का चयन कीिजये :

A) केवल 1, 2 और 3

B) केवल 1, 3 और 4

C) केवल 1, 2 और 4

D) 1, 2, 3 और 4

Hide Answer -
उ रः (b)

ा ाः उपयु दोनों म से कथन-2 गलत है ।

रा ीय मानवािधकार आयोग के अ और सद ों की िनयु रा पित छः सद ीय

सिमित की िसफा रश पर करता है । यह सिमित धानमं ी के ने तृ म गिठत की जाती है ।

सिमित म िन िल खत स िलत होते ह:

⇒ धानमं ी

⇒ लोकसभा अ

⇒ रा सभा के उप सभापित (न की सभापित)

⇒ लोकसभा के िवप के ने ता

⇒ रा सभा के िवप के ने ता

⇒ के ीय गृ हमं ी

[11]रा ीय मानवािधकार आयोग के िवषय म िन िल खत कथनों पर िवचार कीिजये :

1. आयोग को िसिवल ायालय के अिधकार और श याँ ा ह।


2. आयोग उन घटनाओं की जाँ च नहीं कर सकता िज घिटत ए एक वष से
अिधक का समय हो गया हो।
3. आयोग के पास यं का अपना जाँ च दल होता है ।

उपयु म से कौन-सा/से कथन सही है /ह?

A) केवल 1

B) केवल 1 और 2

C) केवल 1 और 3

D) 1, 2 और 3

Hide Answer -
उ रः (d)

ा ाः उपयु तीनों कथन सही ह।

रा ीय मानवािधकार आयोग के पास िसिवल/दीवानी ायालय के सभी अिधकार एवं

श याँ ह ।

आयोग के पास मानवािधकारों के उ ं घन से सं बंिधत िशकायतों की जाँ च हे तु यं का

जाँ च दल है । इसके अित र आयोग के अथवा रा सरकारों के िकसी भी अिधकारी

या जाँ च एजसी की से वाएँ ले सकता है ।

आयोग िकसी ऐसे मामलों की जाँ च के िलये अिधकृत नहीं है िजसे घिटत ए एक वष से

अिधक हो गया हो।

[12]रा ीय मानवािधकार आयोग के िवषय म िन िल खत म से कौन-सा कथन सही है ?

सं सद ारा पा रत अिधिनयम के अं तगत रा ीय मानवािधकार आयोग का गठन


A)
वष 1990 म िकया गया।

रा ीय मानवािधकार आयोग को मानवािधकार उ ं घन के दोषी को द त


B)
करने की श है ।

मानवािधकार से पीिड़त िकसी को रा ीय मानवािधकार आयोग ारा


C)
आिथक सहायता दान की जाती है ।

रा ीय मानवािधकार आयोग की श याँ केवल सलाहकारी कृित की होती


D)
ह।
Hide Answer -
उ रः (d)

ा ाः उपयु म से िवक (d) सही है ।

रा ीय मानवािधकार आयोग की श याँ सलाहकारी/िसफा रश कृित की होती ह। इसकी

िसफा रश सं बंिधत सरकार या अिधकारी पर बा कारी नहीं होती, िक ु आयोग की

िसफा रश पर की गई कायवाही के बारे म एक महीने के अ र सू िचत करना पड़ता है ।

सश बलों के मामले म आयोग की भू िमका एवं श याँ सीिमत ह। इस िवषय म आयोग

के सरकार से रपोट ा कर अपनी सलाह दे सकता है तथा के सरकार को तीन

महीने के अ र आयोग की िसफा रश पर की गई कायवाही के सं बंध म सू िचत करना

होगा।रा ीय मानवािधकार आयोग का गठन सं सद ारा पा रत अिधिनयम के अं तगत 12

अ ू बर,1993 को िकया गया।

इस आयोग को मानवािधकार उ ं घन के दोषी को द त करने की श ा नहीं है

और न ही िकसी कार की सहायता दे सकता है ।

यह आयोग अपनी वािषक या िवशे ष रपोट के एवं सं बंिधत रा सरकार को भे जती है

तथा इसके ितवे दनों को सं बंिधत िवधाियका के सम रखा जाता है ।

[13]िन िल खत कथनों पर िवचार कीिजये :

1. रा मानवािधकार आयोग का गठन े क रा म रा ीय मानवािधकार


आयोग की इकाई के प म िकया गया है ।
2. रा मानवािधकार आयोग एक ब सद ीय िनकाय है िजसम एक अ
और चार सद होते ह।
3. रा मानवािधकार आयोग के अ और सद ों का कायकाल पाँ च या
70 वष की आयु तक जो भी पहले हो, होता है ।

उपयु म से कौन-सा/से कथन सही है /ह?

A) केवल 3

B) केवल 2

C) केवल 1 और 2

D) केवल 2 और 3
Hide Answer -
उ रः (a)

ा ाः उपयु म से कथन 1 और 2 गलत ह।

मानवािधकार सं र ण अिधिनयम, 1993 न केवल के म ब रा ों म भी

मानवािधकार आयोग की थापना का ावधान करता है । अतः रा मानवािधकार आयोग

का गठन रा ीय मानवािधकार आयोग की इकाई के प म नहीं आ है , ब रा

सरकार ारा इसका गठन िकया गया है ।

रा मानवािधकार आयोग एक ब सद ीय िनकाय है िजसम एक अ और दो अ

सद होते ह । वष 2006 म इसकी सद सं ा पाँ च से घटाकर तीन कर दी गई है ।

रा मानवािधकार आयोग का अ उ ायालय का से वािनवृ मु ायाधीश होता

है तथा सद के प म उ ायालय के से वािनवृ या कायरत ायाधीश एवं रा के

िज़ला ायालय का कोई ायाधीश िजसे सात वष का अनु भव हो या ऐसे िजसे

मानवािधकार आयोग के बारे म िवशे ष अनु भव हो, शािमल िकया जाता है ।

रा मानवािधकार आयोग के अ और सद ों का कायकाल पाँ च वष या 70 वष की

आयु तक, जो भी पहले हो, होता है ।

[14]रा मानवािधकार आयोग के िवषय म िन िल खत म से कौन-सा कथन सही नहीं


है ?

रा मानवािधकार आयोग के अ एवं सद ों की िनयु और उनको पद


A)
से हटाना, सं बंिधत रा के रा पाल ारा िकया जाता है ।

रा मानवािधकार आयोग ऐसे मामलों की सु नवाई नहीं करता है िजसका एक


B)
वष से अिधक का समय बीत गया है ।

आयोग का अ व सद कायकाल पू रा करने के प ात् के सरकार और


C)
रा सरकार के अधीन कोई सरकारी पद हण कर नहीं सकते ह।

आयोग के अ एवं सद ों के वे तन-भ े और अ से वा-शत का िनधारण


D)
रा सरकार ारा िकया जाता है ।
Hide Answer -
उ रः (a)

ा ाः उपयु म से िवक (a) गलत है ।

रा मानवािधकार आयोग के अ और सद ों की िनयु सं बंिधत रा के रा पाल

ारा एक सिमित की अनु शंसा पर की जाती है िक ु पद से केवल रा पित (न िक

रा पाल) ही हटा सकता है । रा पित आयोग के अ या सद को उसी कार से हटा

सकता है िजस कार रा ीय मानवािधकार आयोग के अ या सद को हटाते ह।

रा मानवािधकार आयोग एवं रा ीय मानवािधकार आयोग के अ या सद कायकाल

पू रा करने के प ात् के सरकार और रा सरकार के अधीन कोई सरकारी पद हण

नहीं कर सकता है ।

रा ीय मानवािधकार आयोग के अ एवं सद ों के वे तन एवं भ े तथा अ से वा-शत

के सरकार ारा तथा रा के मामले म रा सरकार ारा िनधा रत की जाती है ।

रा मानवािधकार आयोग मानवािधकार के उ घं न से सं बंिधत ऐसे मामलों की सु नवाई

नहीं कर सकता िजन मामलों को एक वष से अिधक का समय हो गया है ।

रा मानवािधकार आयोग को दीवानी/ िसिवल ायालय की श याँ ा ह। इसका

काय िसफा रश कृित का होता है । यह आयोग मानवािधकार के उ ं घन करने वाले

को सज़ा नहीं दे सकता और न पीिड़त को िकसी कार की सहायता/

मु आवज़ा दे सकता है ।

यह आयोग अपना वािषक या िवशे ष ितवे दन रा सरकार को भे जता है और रा

सरकार इसे रा िवधाियका के पटल पर रखती है ।

[15]रा मानवािधकार आयोग के अ एवं सद ों की िनयु रा पाल ारा एक


सिमित की अनु शंसा पर की जाती है । िन िल खत म से कौन इस सिमित के
सद ों म शािमल होते ह?

1. रा का मु मं ी
2. रा िवधानसभा का अ
3. रा िवधान प रषद (यिद है तो) का अ
4. रा िवधानसभा म िवप का ने ता
5. रा िवधान प रषद (यिद हो तो) म िवप का ने ता

कूटः

A) केवल 1, 3, 4 और 5

B) केवल 1, 2 और 4

C) केवल 1, 2, 4 और 5

D) 1, 2, 3, 4 और 5
Hide Answer -
उ रः (d)

ा ाः उपयु सभी सही ह।

रा मानवािधकार आयोग के अ एवं सद ों की िनयु िजस सिमित की अनु शंसा

पर की जाती है , उसम िन िल खत शािमल होते ह-

रा का मु मं ी (अ के प म)

रा िवधानसभा का अ

रा िवधान प रषद (यिद है तो) का अ

रा िवधानसभा म िवप का ने ता

रा िवधान प रषद (यिद है तो) म िवप का ने ता

रा का गृ हमं ी

[16]सू चना आयोग के िवषय म िन िल खत म से कौन-सा कथन सही नहीं है ?

क ीय सू चना आयोग के सद ों की िनयु रा पित के ारा, जबिक रा


A)
सू चना आयोग के सद ों की िनयु रा पाल ारा की जाती है ।

क ीय सू चना आयोग एवं रा सू चना आयोग के सद ों का कायकाल पाँ च


B)
वष या 65 वष की आयु तक, जो भी पहले हो, होती है ।

क ीय सू चना आयोग एवं रा सू चना आयोग की सद सं ा दस से अिधक


C)
नहीं हो सकती है ।

क ीय सू चना आयोग एवं रा सू चना आयोग के सद ों को रा पित ारा


D)
उसके पद से हटाया जा सकता है ।

Hide Answer -
उ रः (d)

ा ाः उपयु म से कथन (d) गलत है ।

क ीय सू चना आयोग के सद ों को रा पित के ारा, जबिक रा सू चना आयोग के

सद ों को रा पाल ारा हटाया जा सकता है ।

क ीय सू चना आयोग के सद ों को रा पित के ारा एक सिमित की िसफा रश पर

िनयु िकया जाता है , जबिक रा सू चना आयोग के सद ों को एक सिमित की

िसफा रश पर रा पाल के ारा िनयु िकया जाता है ।

क ीय सू चना आयोग और रा सू चना आयोग के सद ों का कायकाल 5 वष या 65 वष

की आयु तक, जो भी पहले हो, होता है । पु निनयु होने के िलये यो नहीं होते ह।

क ीय सू चना आयोग और रा सू चना आयोग के सद ों की सं ा 10 से अिधक नहीं हो

सकती, िजसम एक मु सू चना आयु तथा अ सू चना आयु होते ह। गठन के समय

क ीय सू चना आयोग के सद ों की सं ा छः थी, िकंतु वतमान म इसकी सं ा आठ है ,

जबिक रा सू चना आयोग की सद सं ा े क रा म अलग-अलग है ।

सू चना का अिधकार अिधिनयम, 2005 म क ीय सू चना आयोग और रा सू चना आयोग

का ावधान िकया गया। ये आयोग ािधकारयु एक तं िनकाय ह।


[17]क ीय सू चना आयोग के मु सू चना आयु एवं अ सू चना आयु ों की
िनयु रा पित ारा एक सिमित के िसफा रश पर की जाती है । इस सिमित म
शािमल होते ह-

1. धानमं ी
2. लोकसभा म िवप का ने ता
3. रा सभा म िवप का ने ता
4. धानमं ी ारा मनोनीत एक कैिबने ट मं ी

नीचे िदये गए कूट का योग कर सही उ र का चयन कीिजयेः

A) केवल 1 और 2

B) केवल 1, 2 और 3

C) केवल 1, 2 और 4

D) केवल 2, 3 और 4

Hide Answer -
उ रः (c)

ा ाः उपयु म से कथन (3) गलत है ।

क ीय सू चना आयोग के मु सू चना आयु एवं अ सू चना आयु ों की िनयु से

सं बंिधत सिमित म िन सद शािमल होते ह:

⇒ भारत का धानमं ी-अ के प म

⇒ लोकसभा म िवप का ने ता

⇒ धानमं ी ारा मनोनीत एक कैिबने ट मं ी

रा सू चना आयोग के सद ों की िनयु से सं बंिधत सिमित म िन सद शािमल होते

ह-

⇒ मु मं ी-अ के प म

⇒ िवधानसभा म िवप का ने ता

⇒ मु मं ी ारा मनोनीत एक कैिबने ट मं ी

[18]क ीय सू चना आयोग के िवषय म िन िल खत कथनों पर िवचार कीिजयेः

1. मु सू चना आयु के वे तन, भ े एवं अ से वा-शत भारत के मु


ायमू ित के समान होती ह।
2. भारत का रा पित आयोग के अ या सद ों को िस कदाचार या
अ मता के आधार पर हटा सकता है , िकंतु रा पित को ऐसे मामले
सव ायालय के पास भे जने पड़ते ह।

उपयु म से कौन-सा/से कथन सही है /ह?


A) केवल 3

B) केवल 2

C) 1 और 2 दोनों

D) न तो 1 और न ही 2

Hide Answer -
उ रः (b)

ा ाः उपयु म से कथन (1) गलत है ।

मु सू चना आयु का वे तन, भ े और अ से वा-शत भारत के मु िनवाचन आयु

के समान होती ह, जबिक अ सद ों के वे तन, भ े और से वा-शत अ िनवाचन

आयु ों के समान होती ह। रा सू चना आयु के मु सू चना आयु का वे तन, भ े

एवं अ से वा-शत भारत के िनवाचन आयु के समान होती ह, जबिक अ सद ों का

वे तन, भ े एवं से वा-शत रा के मु सिचव के समान होती ह।

भारत के मु सू चना आयु या सद ों को उसके पद से सािबत कदाचार या अ मता के

आधार पर रा पित ारा हटाया जा सकता है , िकंतु ऐसे मामले को रा पित को उ तम

ायालय के पास भे जना पड़ता, जबिक रा मु सू चना आयु या सद ों को उसके

पद से िस कदाचार या अ मता के आधार पर रा वाल ारा हटाया जा सकता है , िकंतु

ऐसे मामलों को रा पाल को सव ायालय के पास जाँ च के िलये भे जना पड़ता है ।

[19]क ीय सू चना आयोग के काय /श यों के सं दभ म िन िल खत कथनों पर िवचार


कीिजयेः

1. आयोग - े रण के आधार पर िकसी मामले की जाँ च का आदे श दे सकता


है ।
2. िशकायतों की जाँ च करते समय सभी सावजिनक द ावे ज़ों को आयोग के
सम ु त करना अिनवाय है ।
3. आवे दक को सही जानकारी न िमलने या ित होने पर िकसी लोक
पदािधकारी को मु आवज़ा दे ने का आदे श दे सकता है ।
4. सू चना का अिधकार अिधिनयम के तहत आयोग को अथद लगाने की
श ा है ।

उपयु म से कौन-से कथन सही ह?

A) केवल 1, 2 और 3

B) केवल 1, 2 और 4

C) केवल 2, 3 और 4

D) 1, 2, 3 और 4
Hide Answer -
उ रः (d)

ा ाः उपयु सभी कथन सही ह।

आयोग के काय एवं श याँ इस कार ह-

⇔ िकसी ठोस आधार पर कोई मामला ा होता है तो आयोग - े रण के आधार पर

मामले की जाँ च का आदे श दे सकता है ।

⇔ िशकायत की जाँ च करते समय सभी सावजिनक द ावे ज़ो को आयोग के सामने ुत

करना अिनवाय है ।

⇔ आवे दक को सही जानकारी न िमलने या ित की दशा म आयोग लोक ािधकारी को

मु आवज़ा दे ने का आदे श दे सकता है ।

⇔ सू चना का अिधकार अिधिनयम, 2005 के तहत आयोग को अथ-द लगाने की

श ा है । आयोग 250 पए ितिदन के िहसाब से जु माना लगा सकता है जो

अिधकतम 25,000 पए तक हो सकता है ।

आयोग को दीवानी/िसिवल ायालय की श याँ ा ह।

आयोग अपना वािषक ितवे दन क सरकार को सौंपता है और क सरकार उस ितवे दन

को सं सद के दोनों सदनों के पटल पर रखती है ।

[20]नीित आयोग के िवषय म िन िल खत कथनों पर िवचार कीिजयेः

1. नीित आयोग का गठन योजना आयोग के थान पर 1 जनवरी, 2014 को


िकया गया।
2. योजना आयोग एक गै र-सां िविधक एवं गै र-सं वैधािनक सं था थी, िजसका
गठन के ीय मं ि मं डल के ारा सं क पा रत कर िकया गया था, जबिक
नीित आयोग एक गै र-सं वैधािनक सं था है , िक ु गै र-सां िविधक सं था
नहीं।
3. नीित आयोग सहकारी सं घवाद के िस ां त पर आधा रत है ।

उपयु म से कौन-सा/से कथन सही है /ह?

A) केवल 3

B) केवल 1 और 3

C) केवल 2 और 3

D) 1, 2 और 3
Hide Answer -
उ रः (a)

ा ाः उपयु म से कथन-1 और 2 गलत ह।

नीित आयोग का गठन योजना आयोग के थान पर 1 जनवरी, 2015 को िकया गया एवं

इसकी पहली बै ठक 8 फरवरी, 2015 को ई।

13 अग , 2014 को योजना आयोग को भं ग कर उसके थान पर एक नए आयोग के

थापना की घोषणा की गई।

योजना आयोग एवं नीित आयोग दोनों ही गै र-सं वैधािनक (Non - Constitutional) एवं

गै र-सां िविधक (Non - statutory) िनकाय ह, िजसका गठन मं ि मं डलीय सं क के

ारा िकया गया है ।

नीित आयोग सहकारी सं घवाद के िस ां त पर आधा रत है ।

नीित आयोग भारत सरकार की नीित-िनमाण की शीष ‘िथं क टक’ सं था है , जो

िनदे शकीय एवं नीितगत दोनों कार के इनपु ट दान करता है ।

NITI Aayog: National Institution for Transforming India Aayog.


[1] रा ीय िवकास प रषद के िवषय म िन िल खत कथनों पर िवचार कीिजयेः

1. रा ीय िवकास प रषद का गठन योजना आयोग एवं नीित आयोग की भाँ ित


एक मं ि मं डल सं क के ारा िकया गया।
2. नीित आयोग के सिचव रा ीय िवकास प रषद के सिचव के प म काय
करता है ।
3. रा ीय िवकास प रषद एवं योजना आयोग का गठन एक मं ि मं डलीय
सं क के ारा एक साथ आ।
4. नीित आयोग का अ , रा ीय िवकास प रषद का अ होता है ।

उपयु म से कौन-सा/से कथन सही है /ह?

A) केवल 1, 2 और 3

B) केवल 1, 2 और 4

C) केवल 1, 3 और 4

D) केवल 2, 3 और 4

Hide Answer -
उ रः (b)
ा ाः उपयु म से कथन (3) सही नही ह।

रा ीय िवकास प रषद एवं योजना आयोग का गठन एक साथ नहीं ब अलग-


अलग आ है । योजना आयोग का गठन 15 माच, 1950 को जबिक रा ीय िवकास
प रषद (एन.डी.सी) का गठन 6 अग , 1952 को एक मं ि मं डलीय सं क के
ारा िकया गया। एन.डी.सी. गठन की अनु शंसा थम पं चवष य योजना (1952-
57) म की गई। रा ीय िवकास प रषद, योजना आयोग/नीित आयोग की भाँ ित गै र-
सं वैधािनक एवं गै र-सां िविधक सं था है । इसका गठन एक मं ि मं डलीय सं क के
ारा िकया गया है ।
नीित आयोग के सिचव एन.डी.सी. के सिचव के प म काय करता है और नीित
आयोग का अ ( धानमं ी) एन.डी.सी. का अ होता है ।
एन.डी.सी. की आ खरी बै ठक (57वीं बै ठक) 27 िदस र, 2012 को 12वीं
पं चवष य योजना (2012-17) को ीकृत करने के िलये ई थी।
धानमं ी के ारा एन.डी.सी. को भं ग करने का िनणय िलया गया, िक ु अभी तक
ऐसा कोई सं क पा रत नहीं िकया गया है ।

[2] रा ीय िवकास प रषद (एन.डी.सी.) की सं रचना के सं दभ म िवचार कीिजयेः

1. भारत का धानमं ी
2. छः सं घीय कैिबने ट मं ी
3. सभी रा ों के मु मं ी
4. सभी सं घ रा े के मु मं ी/ शासक
5. नीित आयोग के सद

उपयु म से कौन-से इनके सद ों म शािमल होते ह?


A) केवल 1, 2, 3 और 4

B) केवल 1, 3, 4 और 5

C) केवल 1, 2, 3 और 5

D) 1, 2, 3, 4, और 5

Hide Answer -
उ रः (b)

ा ाः उपयु म से कथन (2) गलत है ।

एन.डी.सी. के सद ों म िन शािमल होते ह:

भारत का धानमं ी

सम सं घीय कैिबने ट मं ी, न िक केवल छः सं घीय कैिबने ट मं ी

सभी रा ों के मु मं ी

सभी सं घ रा े ों के मु मं ी/ शासक

नीित आयोग के सद

नीित आयोग के सिचव एन.डी.सी. के सिचव के प म काय करता है ।

[3] रा ीय िवकास प रषद के काय के सदभ म िन िल खत कथनों पर िवचार कीिजयेः

1. रा ीय योजना के िनमाण हे तु िदशा-िनदश िनधा रत करना।


2. योजना आयोग ारा तै यार रा पित योजना पर िवचार करना।
3. रा ीय योजना के उ े ों की ा के उपाय सु झाना।
4. योजनाओं म धन आवं टन हे तु सु झाव।

उपयु म से कौन-से कथन सही ह?

A) केवल 1, 2 और 3

B) केवल 1, 2 और 4

C) केवल 2, 3 और 4

D) 1, 2, 3 और 4
Hide Answer -
उ रः (a)

ा ाः उपयु म से कथन-4 गलत है ।

एन.डी.सी. के काय िन िल खत ह:

- रा ीय योजना की तै यारी के िलये िदशा-िनदश िनधा रत करना।

- योजना आयोग ारा तै यार रा ीय योजना पर िवचार करना।

- रा ीय योजना के उ े ों की ा के उपाय सु झाना।

- रा ीय योजना के काम- काज की समय-समय पर समी ा करना।

- रा ीय िवकास को भािवत करने वाली सामािजक एवं आिथक नीितयों से सं बंिधत

मह पू ण ों पर िवचार करना।

- योजनाओं को काया त करने के िलये ज़ री सं साधनों का आकलन करना और

इसकी सं वृ के आय सु झाना आिद।

- योजनाओं म धन आं वटन हे तु सु झाव दे ना योजना आयोग का काय था।

- नीित आयोग ारा ा प योजना सव थम मं ि मं डल को भे जी जाती है । मं ि मं डल की

ीकृित के प ात् इसे एन.डी.सी. के सम ु त िकया जाता है । एन.डी.सी. की

ीकृित के प ात् इसे सं सद के सम ु त िकया जाता है । सं सद के ीकृित के प ात्

यह आिधका रक योजना मान ली जाती है और सरकारी गजट म इसे कािशत िकया

जाता है ।

- एन.डी.सी. की अनु शंसाएँ बा कारी नहीं होती यह के एवं रा सरकार को अपनी

अनु शंसाएँ भे जती ह । ितवष कम- से - कम इसकी दो बै ठक अिनवाय ह।

[4] िन िल खत कथनों पर िवचार कीिजयेः

1. रा ीय िवकास प रषद का मु सं बंध पं चवष य योजना के अनु मोदन से


है ।
2. रा ीय िवकास प रषद के ारा योजना आयोग (अब नीित आयोग) और
रा सरकारों के बीच सम यक ा के प म काय िकया जाता है ।

उपयु म से कौन-सा/से कथन सही है /ह?

A) केवल 1

B) केवल 2

C) 1 और 2 दोनों

D) न तो 1 और न ही 2
Hide Answer -
उ रः (c)

ा ाः उपयु म से दोनों कथन सही ह।

पं चवष य योजना का अ म अनु मोदन रा ीय िवकास प रषद ारा िकया जाता है । अतः

रा ीय िवकास प रषद इस मामले म सं सद के नीचे उ तम िनकाय है । एन.डी.सी. योजना

आयोग (अब नीित आयोग) एवं रा सरकारों के बीच सम यक ा के प म काय करता

है , ोंिक इसके सद के प म भारत के धानमं ी सम सं घीय कैिबने ट मं ी, सभी

रा ों के मु मं ी एवं सभी सं घ-रा ों े ों के मु मं ी/ शासक नीित आयोग के सद

होते ह। पं चवष य योजना का अं ितम अनु मोदन रा ीय िवकास प रषद ारा िकया जाता है ।

[5] िन िल खत कथनों पर िवचार कीिजये :

1. सं िवधान के भाग-15 म उपबं ध है िक सं घ के िलये एक लोक से वा आयोग


एवं े क रा के िलये एक लोक से वा आयोग होगा।
2. सं िवधान म उपबं ध है िक े क रा का िवधानमं डल िविध ारा दो या
अिधक रा ों के िलये सं यु रा लोक से वा आयोग की िनयु का
उपबं ध कर सकेगी।

उपयु म से कौन-सा/से कथन सही है /ह?

A) केवल 1

B) केवल 2

C) 1 और 2 दोनों

D) न तो 1 और न ही 2

Hide Answer -
उ रः (d)

ा ाः उपयु दोनों कथन गलत ह।

सं िवधान के भाग-14 के अ ाय-2 म (अनु े द-315) उपबं ध है िक सं घ के िलये एक

लोक से वा आयोग एवं े क रा के िलये एक लोक से वा आयोग होगा।

दो या अिधक रा समझौते ारा यह तय कर िक उनके िलये एक ही लोक से वा आयोग

होगा और इस आशय का सं क रा िवधानमं डल ारा पा रत कर दे तो सं सद (न िक

रा के िवधामं डल) सं यु रा लोक से वा आयोग (सं यु आयोग) की िनयु का

उपबं ध कर सकेगी।

[6] िन िल खत कथनों पर िवचार कीिजयेः

1. सं घ लोक से वा आयोग के अ एवं सद ों की िनयु रा पित के ारा


की जाती है ।
2. रा लोक से वा आयोग एवं सं यु रा लोक से वा आयोग के अ एवं
सद ों की िनयु सं बंिधत रा के रा पाल ारा की जाती है , िक ु
सं यु रा लोक से वा आयोग के िवषय म रा पाल रा पित से परामश
अव ले गा।
3. सं घ लोक से वा आयोग एवं रा लोक से वा आयोग की सद सं ा का
उ े ख सं िवधान म नहीं िकया गया है ।

उपयु म से कौन-सा/से कथन सही है /ह?

A) केवल 1

B) केवल 1 और 2

C) केवल 1 और 3

D) 1, 2 और 3

Hide Answer -
उ रः (c)

ा ाः उपयु म से कथन 2 गलत है ।

सं िवधान के अनु े द-316 म सं घ लोक से वा आयोग एवं रा लोक से वा आयोग के

अ एवं सद ों की िनयु के सं बंध म उपबं ध है ।

रा लोक से वा आयोग के अ एवं सद ों की िनयु सं बंिधत रा के रा पाल ारा

की जाती है , िक ु सं यु रा लोक से वा आयोग के अ एवं सद ों की िनयु

रा पित ारा की जाती है ।

सं घ लोक से वा आयोग के अ एवं सद ों की िनयु रा पित ारा की जाती है ।

सं िवधान म सं घ लोक से वा आयोग एवं रा लोक से वा आयोग के अ एवं सद ों की

सं ा का उ े ख नहीं है । यह रा पित एवं रा पाल के िववे क पर छोड़ िदया गया है ।

[7] िन िल खत म से कौन-सा कथन सही नहीं है ?

सं घ लोक से वा आयोग एवं रा लोक से वा आयोग के अ एवं सद पद


A) हण करने की तारीख से छः वष की अविध या 65 वष की आयु तक, इनम
जो भी पहले हो, अपना पद धारण करगे ।

सं िवधान, सं घ लोक से वा आयोग के सं बंध म रा पित और रा लोक से वा


B) आयोग के सं बंध म रा पाल को अ एवं सद ों की से वा-शत िनधा रत
करने का अिधकार सौंपता है ।

सं घ लोक से वा आयोग और रा लोक से वा आयोग के अ एवं सद ों को


C)
केवल कदाचार के आधार पर रा पित हटा सकता है ।

सं घ लोक से वा आयोग एवं रा लोक से वा आयोग के अ या सद ,


D)
कायकाल समा के प ात् पु नः िनयु के यो नहीं होगा।
Hide Answer -
उ रः (a)

ा ाः उपयु म से कथन (a) गलत है ।

सं िवधान के अनु े द-316(2) म ावधान है िक सं घ लोक से वा आयोग के अ पद

हण करने की तारीख से छः वष की अविध या 65 वष की आयु तक, जो भी पहले हो,

पद पर बने रहगे िक ु रा लोक से वा आयोग एवं सं यु लोक से वा आयोग के सं बंध म

छः वष या 62 वष की अविध तक, जो भी पहले हो, का ावधान है ।

भारत का सं िवधान सं घ लोक से वा आयोग के अ एवं सद ों की से वा-शत के सं बंध म

रा पित को, जबिक रा लोक से वा आयोग के सं बंध म रा पाल को यह अिधकार

सौंपता है ।

सं िवधान के अनु े द-317 म उपबं ध है िक सं घ लोक से वा आयोग एवं रा लोक से वा

आयोग के अ या सद ों को केवल कदाचार के आधार पर रा पित हटा सकता है ।

िक ु ऐसे मामले को रा पित जाँ च के िलये उ तम ायालय को भे जता है । यिद उ तम

ायालय जाँ च के प ात् बखा करने के परामश का समथन करता है तो रा पित आयोग

के अ या सद हो हटा सकता है । उ तम ायालय ारा रा पित को दी गई सलाह

बा कारी है ।

रा के लोक से वा आयोग के अ एवं सद ों की िनयु रा पाल करता है , िक ु

इ हटा नहीं सकता। केवल रा पित ही हटा सकता है ।

सं िवधान के अनु े द-316(3) म उपबं ध है िक सं घ लोक से वा आयोग एवं रा लोक

से वा आयोग के अ या सद कायकाल समा के प ात् पु नः िनयु के यो नहीं

होगा।

सं िवधान म सं घ लोक से वा आयोग एवं रा लोक से वा आयोग के सद ों की यो ता के

िवषय म उ े ख नहीं िकया गया है । िक ु आयोग के आधे सद ों को भारत सरकार या

रा सरकार के अधीन कम-से -कम 10 वष काम करने का अनु भव हो।

[8] िन िल खत कथनों पर िवचार कीिजये

1. सं घ लोक से वा आयोग एवं रा लोक से वा आयोग के अ या सद ों


के वे तन, भ े एवं पशन भारत की सं िचत िनिध पर भा रत होते ह और इसे
सं सद म मतदान के िलये नहीं रखा जाता।
2. सं घ लोक से वा आयोग एवं सं यु रा लोक से वा आयोग अपनी रपोट
रा पित को एवं रा लोक से वा आयोग अपनी रपोट सं बंिधत रा के
रा पाल को सौंपता है ।

उपयु म से कौन-सा/से कथन सही है /ह?

A) केवल 1

B) केवल 2

C) 1 और 2 दोनों

D) न तो 1 और न ही 2
Hide Answer -
उ रः (a)

ा ाः उपयु म से कथन 2 गलत है ।

सं िवधान के अनु े द-323 म ावधान है िक सं घ लोक से वा आयोग अपना ितवे दन

रा पित को ितवष दे गा और रा पित उस ितवे दन को सं सद के दोनों सदनों के सम

रखवाएगा तथा रा लोक से वा आयोग एवं सं यु रा लोक आयोग अपना ितवे दन

ितवष सं बंिधत रा पाल को सौंपेगा और रा पाल उस ितवे दन को रा के ेक

िवधानमं डल के सम रखवाएगा।

[9] िन िल खत कथनों पर िवचार कीिजये :

1. भारत म पहली बार लोक से वा आयोग का गठन भारत शासन


अिधिनयम,1935 के अं तगत िकया गया।
2. लोक से वा आयोग के िसफा रश सलाहकारी वृ ि की होती ह।
3. सं सद िविध ारा सं घ लोक से वा आयोग और रा िवधानमं डल िविध ारा
रा लोक से वा आयोग के काय े का िव ार कर सकती है ।

उपयु म से कौन-सा/से कथन सही है /ह?

A) केवल 2

B) केवल 1 और 3

C) केवल 2 और 3

D) 1, 2 और 3

Hide Answer -
उ रः (c)

ा ाः उपयु म से कथन 1 गलत है ।

भारत म पहली बार लोक से वा आयोग का गठन भारत शासन अिधिनयम, 1919 के

अतं गत वष 1926 म िकया गया। भारत शासन अिधिनयम, 1935 के ारा इसका नाम

सं घीय लोक से वा आयोग रखा गया।

लोक से वा आयोग की िसफा रश सलाहकारी वृ ि की होती ह।

सं िवधान के अनु े द-321 म उपबं ध है िक सं सद िविध ारा सं घ लोक से वा आयोग और

रा का िवधानमं डल िविध ारा रा लोक से वा आयोग के काय े का िव ार कर

सकती है ।

[10]भारत के सं घ लोक से वा आयोग के िवषय म िन िल खत म से कौन-सा कथन

सही है ?
A) यह रा लोक से वा आयोग का िनरी ण करता है ।

B) इसका रा लोक से वा आयोग से कोई ले ना दे ना नहीं है ।

C) इसके सारे सद रा लोक से वा आयोग से िलये जाते ह।

D) यह रा लोक से वा आयोग को वािषक िदशा-िनदश भे जता है ।

Hide Answer -
उ रः (b)

ा ाः उपयु म से कथन b सही है ।

सं िवधान के अनु सार सं घ लोक से वा आयोग और रा लोक से वा आयोग अपने -अपने

काय के िलये एक-दू सरे से तं ह।

सं िवधान का अनु े द-315 के के िलये एक सं घ लोक से वा आयोग, रा ों के िलये रा

लोक से वा आयोग और दो या अिधक रा ों के िलये सं यु रा लोक से वा आयोग का

उपबं ध करता है ।

[11]िकसी रा लोक से वा आयोग के काय को िन िल खत म से िकस एक के


अनु मोदन से सं घ लोक से वा आयोग को सौंपा जा सकता है ?

A) भारत के रा पित

B) भारत के मु ायमू ित

C) रा पाल

D) धानमं ी

Hide Answer -
उ रः (a)

ा ाः उपयु म से कथन a सही है ।

सं घ लोक से वा आयोग, रा पाल के अनु रोध एवं रा पित की सं ु ित के बाद रा की

आव कतानु सार भी काय कर सकता है अथात् रा पाल की िसफा रश पर रा पित के

अनु मोदन से िकसी रा लोक से वा आयोग के काय को सं घ लोक से वा आयोग को सौपा

जा सकता है ।

[12]िन िल खत म से कौन-से आयोग/अिधकारी सं वैधािनक आयोग/अिधकारी ह?

1. रा ीय अनु सूिचत जाित आयोग


2. रा ीय अनु सूिचत जनजाित आयोग
3. रा ीय िपछड़ा वग आयोग
4. भाषाई अ सं क वग के िलये िवशे ष अिधकारी

नीचे िदये गए कूट का योग कर सही उ र का चयन कीिजये ।


A) केवल 3 और 2

B) केवल 1, 2 और 3

C) केवल 1, 2 और 4

D) 1, 2, 3 और 4

Hide Answer -
उ रः (c)

ा ाः उपयु म से कथन-3 सही नहीं है ।

रा ीय िपछड़ा वग आयोग एक वै धािनक/सां िविधक आयोग है , न िक सं वैधािनक है , जबिक

रा ीय अनु सूिचत जाित एवं जनजाित आयोग तथा भाषायी अ सं क वग के िलये िवशे ष

अिधकार सं वैधािनक आयोग/अिधकारी ह, जै से-

1. रा ीय अनु सूिचत जाित आयोग- अनु े द 338

2. रा ीय अनु सूिचत जनजाित आयोग-अनु े द 339(A)

3. भाषाई अ सं क वग के िलये - अनु े द-350(B) िवशे ष अिधकारी।

[13]िन िल खत कथनों पर िवचार कीिजयेः

1. रा ीय अनु सूिचत जाित एवं रा ीय अनु सूिचत जनजाित आयोग एक अ ,


एक उपा एवं पाँ च अ सद ों से िमलकर बने गा।
2. 65व सं िवधान सं शोधन अिधिनयम के ारा रा ीय अनु सूिचत जाित एवं
जनजाित आयोग का गठन िकया गया।
3. 89व सं िवधान सं शोधन अिधिनयम ारा रा ीय अनु सूिचत जाित एवं
जनजाित आयोग को अलग िकया गया।

उपयु म से कौन-सा/से कथन सही है /ह?

A) केवल 3

B) केवल 2

C) केवल 2 और 3

D) 1, 2 और 3
Hide Answer -
उ रः (c)

ा ाः उपयु म से कथन 1 गलत है ।

रा ीय अनु सूिचत जाित एवं रा ीय अनु सूिचत जनजाित का गठन एक अ , एक उपा

एवं तीन अ सद ों से िमलकर िकया जाएगा।

65व सं िवधान सं शोधन अिधिनयम, 1990 के ारा रा ीय अनु सूिचत जाित एवं जनजाित

आयोग का गठन िकया गया एवं 1992 से यह अिधिनयम लागू आ। इस सं िवधान

सं शोधन के ारा इस आयोग को सं वैधािनक दजा दान िकया गया। पु नः 89व सं िवधान

सं शोधन अिधिनयम, 2003 के ारा इन आयोगों को अलग-अलग कर िदया गया।

अनु े द-338 म रा ीय अनु सूिचत जाित आयोग एवं अनु े द-338(A) म रा ीय

अनु सूिचत जनजाित आयोग का उपबं ध िकया गया। यह अिधिनयम 2004 से लागू आ।

[14]िन िल खत कथनों पर िवचार कीिजये :

1. रा ीय अनु सूिचत जाित एवं जनजाित आयोग की िनयु भारत के रा पित


ारा की जाती है , िक ु उसकी से वा- शत एवं कायकाल सं सद िविध ारा
िनधा रत करती है ।
2. ससं द ने िविध ारा रा ीय अनु सूिचत जाित आयोग एवं रा ीय अनु सूिचत
जनजाित आयोग के सद ों का कायकाल पाँ च वष िनधा रत िकया है ।
3. रा ीय अनु सूिचत जाित आयोग एवं रा ीय अनु सूिचत जनजाित आयोग
अपना ितवे दन ितवष रा पित को सौंपेगा और रा पित ऐसे सभी
ितवे दनों को सं सद के े क सदन के सम रखवाएगा।

उपयु म से कौन-सा/से कथन सही है /ह?

A) केवल 3

B) केवल 1 और 3

C) केवल 2 और 3

D) 1, 2 और 3

Hide Answer -
उ रः (a)

ा ाः उपयु म से कथन 1 और 2 गलत ह ।

रा ीय अनु सूिचत जाित और रा ीय अनु सूिचत जनजाित आयोगों की िनयु भारत के

रा पित के ारा की जाती है तथा उसकी से वा-शत एवं कायकाल भी रा पित ारा

िनधा रत िकये जाते ह। इनका कायकाल 3 वष का होता है ।

सं िवधान म ावधान है िक रा ीय अनु सूिचत जाित और रा ीय अनु सूिचत जनजाित आयोग

अपना ितवे दन ितवष रा पित को सौंपते ह और रा पित ऐसे ितवे दनों को सं सद के

े क सदन म रखवाता है । कोई ऐसा ितवे दनों या उसका कोई भाग िकसी रा सरकार

से सं बंिधत है तो ऐसे ितवे दन की एक ित उस रा के रा पाल को भे जेगा और

रा पाल उस ितवे दन को िवधानमं डल के सम रखवाएगा।


[15]िन िल खत कथनों पर िवचार कीिजये :

1. रा ीय अनु सूिचत जाित आयोग और रा ीय अनु सूिचत जनजाित आयोग को


दीवानी एवं फौज़दारी ायालय की श याँ ा ह।
2. रा ीय अनु सूिचत जाित और रा ीय अनु सूिचत जनजाित आयोग अनु सूिचत
जाित एवं अनु सूिचत जनजाित के अित र िपछड़ा वग एवं आं -भारतीय
समु दाय के सं वैधािनक सं र ण एवं अ िविधक सं र ण की जाँ च करते ह
और इसके सं बंध म ितवदे न रा पित को सौंपत है ।

उपयु म से कौन-सा/से कथन सही है /ह?

A) केवल 1

B) केवल 2

C) 1 और 2 दोनों

D) न तो 1 और न ही 2

Hide Answer -
उ रः (d)

ा ाः उपयु दोनों कथन गलत ह।

सं िवधान अनु े द-338(B) म उपबं ध है िक रा ीय अनु सूिचत जाित एवं रा ीय अनु सूिचत

जनजाित आयोग को दीवानी/िसिवल ायालय की श याँ ा है , फौजदारी ायालय

की नहीं।

सं िवधान के अनु े द-338(10) म उपबं ध है िक रा ीय अनु सूिचत जाित आयोग

अनु सूिचत जाित के अित र िपछड़ा वग एवं आं -भारतीय समु दाय के सं वैधािनक

सं र ण तथा अ िविध सं र ण की जाँ च करता है और इसके सं बंध म अपनी रपोट

रा पित को सौंपता है , िक ु यह श रा ीय अनु सूिचत जनजाित आयोग को ा नहीं

है ।

[16]भारत का सं िवधान भाषाई अ सं क वग के िलये िवशे ष अिधकारी का उपबं ध


करता है , इस सं दभ म िन िल खत कथनों पर िवचार कीिजये :

1. भारत के सं िवधान के भाग-XVI म इसका उपबं ध िकया गया है ।


2. भाषाई अ सं क वग के िलये िवशे ष अिधकारी की िनयु भारत के
रा पित ारा की जाती है ।
3. यह िवशे ष अिधकारी अपना ितवदे न भारत के रा पित को सौंपता है ।

उपयु म से कौन-सा/से कथन सही है /ह?

A) केवल 2

B) केवल 2 और 3

C) केवल 1 और 3

D) 1, 2 और 3
Hide Answer -
उ रः (b)

ा ाः उपयु म से कथन 1 गलत है ।

सं िवधान के भाग-XVII एवं अनु े द-350(ख) म भाषाई अ सं क वग के िलये एक

िवशे ष अिधकारी का ावधान िकया गया है । इसकी िनयु भारत के रा पित ारा की

जाती है । यह अपना ितवे दन रा पित को सौंपता है और रा पित उस ितवे दन को सं सद

के े क सदन के सम रखवाता है एवं सं बंिधत रा सरकारों को भे जवाता है ।

मू ल सं िवधान म इस अिधकारी के िलये कोई ावधान नहीं था। रा पु नगठन आयोग की

िसफा रश पर 7व सं िवधान सं शोधन अिधिनयम, 1956 के ारा अनु े द-350(ख) जोडा

गया।

इस अिधकारी का पदनाम आयु /किम र िदया गया है एवं इस आयु का मु ालय

इलाहाबाद (उ र दे श) है ।

के ीय र पर यह आयु अ सं क मं ालय के अं तगत काय करता है ।

भाषाई अ सं कों की भाषा एवं सं ृ ित को सं रि त करने के सरकारी उपायों को नया

वे ग दे ने के िलये 10 सू ी काय म की शु आत की गई है ।

[17]सं घ रा े के िवषय म िन िल खत कथनों पर िवचार कीिजयेः

1. सं िवधान म उपबं ध है िक सं घ रा े का शासन रा पित के ारा एक


शासक के मा म से सं चािलत िकया जाएगा।
2. सं घ रा े का शासक रा पित का अिभक ा/एजे होता है , िकंतु
इसकी थित अ रा पालों की तरह रा मु ख की होती है ।
3. सं िवधान म उपबं ध है िक रा पित िकसी रा के रा पाल को िकसी
िनकटवत सं घ रा े का शासक िनयु कर सकेगा, िकंतु वह ऐसे
शासक के प म अपने काय का योग अपनी मं ि प रषद के अधीन
रह कर करे गा।

उपयु म से कौन-सा/से कथन सही है /ह?

A) केवल 1

B) केवल 1 और 3

C) केवल 2 और 3

D) 1, 2 और 3
Hide Answer -
उ र: (a)

ा ाः उपयु म से कथन (2) और (3) गलत ह।

सं िवधान के भाग-VIII (अनु े द-239-241 तक) म सं घ रा े /क -शािसत दे श

के सं बंध म उपबं ध िकया गया है ।

सं घ रा े का शासक रा पित का अिभक ा/एजट होता है । इसिलये इसकी थित

अ रा पालों की भां ित रा मु ख की नहीं होती है ।

सं िवधान के अनु े द-239(2) म उपबं ध है िक रा पित िकसी रा के रा पाल को

िकसी िनकटवत सं घ रा े का शासक िनयु कर सकेगा, िकंतु वह ऐसे शासक

के प म अपने काय का योग अपनी मं ि प रषद से तं रह कर करे गा। जै से: पं जाब

का रा पाल चं डीगढ़ का शासन होता है ।

[18]सं घ रा े िद ी के िवषय म िन िल खत कथनों पर िवचार कीिजयेः

1. 70व सं िवधान सं शोधन अिधिनयम, 1992 के ारा इसे रा ीय राजधानी


े िद ी की है िसयत दान की गई।
2. सं घ रा े िद ी की िवधानसभा रा सू ची के सभी िवषयों पर िविध
बना सकती है ।
3. मु मं ी की िनयु रा पित के ारा की जाती है तथा अ मं ि यों की
िनयु रा पित, मु मं ी की सलाह पर करता है ।
4. 91व सं िवधान सं शोधन अिधिनयम, 2003 के ारा उपबं ध िकया गया िक
मं ि प रषद की सद सं ा िवधानसभा की कुल सद सं ा का 15
ितशत से अिधक नहीं होगी, िकंतु 12 से कम नहीं होगी।

उपयु म से कौन-सा/से कथन सही है /ह?

A) केवल 3

B) केवल 1 और 3

C) केवल 1, 3 और 4

D) केवल 1, 2 एवं 4
Hide Answer -
उ र: (a)

ा ाः उपयु म से कथन 1, 2 और 4 गलत ह।

69व सं िवधान सं शोधन अिधिनयम, 1991 के ारा िद ी को िवशे ष दज़ा दान िकया

गया। िद ी को ‘रा ीय राजधानी े िद ी’ की है िसयत दान की गई, जबिक 70व

सं िवधान सं शोधन अिधिनयम, 1992 के ारा िद ी एवं पु डुचे री िवधानसभा के िनवािचत

सद ों को रा पित के िनवाचक मं डल म शािमल करने सं बंधी उपबं ध िकया गया है ।

िद ी की िवधान सभा लोक व था, पु िलस एवं भू िम सं बंधी िवषयों को छोड़कर रा

सू ची के सभी िवषयों एवं समवत सू ची के िवषयों पर िविध बना सकती है । पु डुचे री

िवधानसभा रा सू ची एवं समवत सू ची के िवषयों पर कानू न बना सकती है ।

िद ी म मं ि प रषद की सद सं ा िवधानसभा के कुल सद सं ा का 10% से

अिधक नहीं होगी । िद ी म िवधानसभा की कुल सद सं ा 70 है , इसिलये

मं ि प रषद की सद सं ा अिधकतम 7 हो सकती है । मं ि प रषद की सद सं ा के

सं बंध म 91व सं िवधान सं शोधन अिधिनयम, 2003 िद ी पर लागू नहीं होता।

िद ी एवं पु डुचे री के मु मं ि यों की िनयु रा पित के ारा की जाती है एवं अ

मं ि यों की िनयु रा पित, मु मं ी की सलाह पर करता है । मु मं ी एवं मं ी रा पित

के सादपयत पद धारण करते ह।

मं ि -प रषद सामू िहक प से िवधानसभा के ित उ रदायी होती है ।

[19]सं िवधान म उपबं ध है िक भारत का रा पित िन िल खत सं घ रा े ों की शां ित,


गित और सु शासन के िलये िविध बना सकता है ः

1. दादर एवं नागर हवे ली


2. ल ीप
3. िद ी
4. अं डमान और िनकोबार ीप समू ह

नीचे िदये गए कूट का योग कर सही उ र का चयन कीिजयेः

A) केवल 1, 2 और 3

B) केवल 1, 2 और 4

C) केवल 2, 3 एवं 4

D) 1, 2, 3 और 4
Hide Answer -
उ र: (b)

ा ाः सं िवधान के अनु े द-240 के तहत भारत का रा पित िन िल खत सं घ रा े की

शां ित, गित एवं सु शासन के िलये िविनयम बना सकता है -

1. अं डमान एवं िनकोबार ीप समू ह

2. ल ीप

3. दादर एवं नागर हवे ली

4. दमन एवं दीव

यिद पु दुचेरी िवधानसभा िवघिटत या बखा हो तो रा पित इसके िलये भी िविनयम बना सकता

है ।

[20]िन िल खत कथनों पर िवचार कीिजयेः

1. भारत का रा पित िकसी क -शािसत दे श म उ ायालय की थापना


कर सकता है या उसे िनकटवत रा के उ ायालय के अधीन कर
सकता है ।
2. भारत के िकसी भी सं घ रा े के पास अपना उ ायालय नहीं है ।

उपयु म से कौन-सा/से कथन सही है /ह?

A) केवल 1

B) केवल 2

C) 1 और 2 दोनों

D) न तो 1 और न ही 2

Hide Answer -
उ र: (d)

ा ाः उपयु दोनों कथन गलत ह।

सं िवधान के अनु े द-241 म उपबं ध है िक सं सद (न िक रा पित) िविध ारा िकसी सं घ

रा े के िलये उ ायालय गिठत कर सकेगी या िकसी सं घ रा े को िनकटवत

उ ायालय के अधीन कर सकती है , जै सेः दादर एवं नागर हवे ली तथा दमन एवं दीव,

ब ई उ ायालय, अं डमान एवं िनकोबार ीप समू ह कलक ा उ ायालय, चं डीगढ़

पं जाब एवं ह रयाणा उ ायालय, ल ीप केरल उ ायालय और पु दुचेरी म ास

उ ायालय के अधीन ह।

िद ी एक मा क शािसत दे श है , िजसके पास अपना उ ायालय (वष 1966 म

थािपत) है
[1] सं घ रा े के िवषय म िन िल खत कथनों पर िवचार कीिजयेः

1. सं घ रा े म अ ादे श ािपत करने की श भारत के रा पित के


पास है ।
2. सं घ रा े म िवधायन, िव एवं बजट से वाएँ तथा उप-रा पाल एवं
शासकों की िनयु सं बंधी सभी मामलों के िलये नोडल एजसी सं घीय गृ ह
मं ालय होता है ।

उपयु म से कौन-सा/से कथन सही है /ह?

A) केवल 1

B) केवल 2

C) 1 और 2 दोनों

D) न तो 1 और न ही 2

Hide Answer -
उ र: (b)

ा ाः उपयु म से कथन (1) गलत है ।

सं िवधान के अनु े द- 239(ख) म उपबं ध है िक सं घ रा े के िवधानमं डल के स म

नहीं होने पर वहाँ के शासक/उप-रा पाल अ ादे श (न िक रा पित) ािपत कर

सकता है , िकंतु िवधानसभा के िवघटन या िनल न की थित म ऐसा नहीं कर सकता है ।

अ ादे श ािपत करने के िलये रा पित से पू वानु मित अिनवाय है ।

भारत सरकार (कायवाही आवं टन) िनयमावली,1961 के अं तगत सं घ का गृ ह मं ी सं घ

रा े ों म िवधायन, िव एवं बजट से वाएँ तथा उप रा पाल एवं शासकों की िनयु

से सं बंिधत सभी मामलों के िलये नोडल एजसी होता है ।

िजन सं घ रा े ों म िवधाियका नहीं है , वहाँ ‘गृ ह-मं ी सलाहकार सिमित’/‘ शासक

सलाहकार सिमित’ सं घ रा े के समािजक आिथक िवकास से जु ड़े सामा मु ों पर

िवचार करती है । सां सद एवं थानीय िनकायों के सद इन सिमितयों के सद होते ह।

‘गृ ह-मं ी सलाहकार सिमित’ की अ ता क ीय गृ ह मं ी एवं ‘ शासक सलाहकार

सिमित’ की अ ता उस े का शासक करता है ।

[2] सू ची-I को सू ची-II से सु मेिलत कीिजयेः

सू ची-I सू ची-II
(भारतीय सं िवधान के अनु े द) ( ावधान)

1. भाषाई अ सं कों के िलये िवशे ष


A. अनु े द 350(क)
अिधकारी

B. अनु े द 350 (ख) 2. ाथिमक र पर मातृ भाषा म िश ा

C. अनु े द 344 3. उ तम एवं उ ायालयों की भाषा

D. अनु े द 348 4. राजभाषा के सं बंध म आयोग


नीचे िदये गए कूट का योग कर सही उ र का चयन कीिजयेः
A B C D

A) 2 1 4 3

B) 1 2 3 4

C) 1 2 4 3

D) 2 1 3 4

Hide Answer -
उ रः (a)

ा ाः उपयु म से िवक (a) सही है ।

अनु े द-344 - राजभाषा के सं बंध म आयोग और सं सद की सिमित के सं बंध म उपबं ध।

अनु े द-348 - उ तम एवं उ ायालयों की भाषा।

अनु े द-350(क) - ाथिमक र पर मातृ भाषा म िश ा।

अनु े द-350(ख) - भाषाई अ सं क वग के िवशे ष अिधकारी।

[3] िन िल खत कथनों पर िवचार कीिजयेः

1. सं िवधान म उपबं ध है िक सं घ की राजभाषा िह ी, िलिप दे वनागरी और


सं घ के शासकीय उ े ों के िलये योग होने वाले अं कों का प दे वनागरी
होगा।
2. सं िवधान के ारं भ होने से पं ह वष की अविध तक सं घ के उन सभी
उ े ों के िलये अं े ज़ी भाषा का योग िकया जाता रहे गा, िकंतु पं ह वष
प ात रा पित ऐसे उ े ों के िलये अं े ज़ी भाषा या अं कों का दे वनागरी
प का उपबं ध कर सकेगा।

उपयु म से कौन-सा/से कथन सही है /ह?

A) केवल 1

B) केवल 2

C) 1 और 2 दोनों

D) न तो 1 और न ही 2
Hide Answer -
उ रः (d)

ा ाः उपयु दोनों कथन गलत ह।

सं िवधान के अनु े द-343 म उपबं ध है िक सं घ की राजभाषा िह ी और िलिप दे वनागरी

होगी। सं घ के शासकीय उ े ों के िलये योग होने वाले अं को का प अं तरा ीय (न िक

दे वनागरी) प होगा।

उपयु बातों के होते ए भी सं िवधान के ारं भ से 15 वष की अविध तक सं घ के उन

सभी शासकीय उ े ों के िलये अं े ज़ी भाषा का योग िकया जाता रहे गा, परं तु रा पित

इस अविध के दौरान, सं घ के शासकीय उ े ों के िलये अं े ज़ी भाषा के अित र िह ी

भाषा का और भारतीय अं कों के अं तरा ीय प के अित र दे वनागरी प का योग

अिधकृत कर सकेगा।

उपयु 15 वष की समा के प ात् सं सद (न िक रा पित) िविध ारा अं े ज़ी भाषा या

अं कों का दे वनागरी प का उपबं ध कर सकेगी। इस आधार पर सं सद ने वष 1963 म

राजभाषा अिधिनयम पा रत िकया, िजसम ावधान िकया गया िक 15 वष के समा

(1965 के प ात् भी) के प ात् भी सं घ के सभी सरकारी काय एवं सं सद की कायवाही म

अं े ज़ी के साथ-साथ िह ी का योग िकया जाएगा। इस अिधिनयम म अं े ज़ी भाषा के

योग के िलये कोई सीमा का िनधारण नहीं िकया गया। वष 1967 म कुछ िविश मामलों

म िह ी के साथ अं े ज़ी भाषा का योग अिनवाय करने के िलये इसम सं शोधन िकया गया।

[4] िन िल खत कथनों पर िवचार कीिजयेः

1. सं िवधान म उपबं ध है िक सं सद िविध ारा सं िवधान के ारं भ से पाँ च वष


की समा पर उसके प ात् दस वष की समा पर राजभाषा के िलये
एक आयोग के सं बंध म उपबं ध कर सकेगी।
2. सं िवधान म उपबं ध है िक राजभाषा के िलये गिठत आयोग की िसफा रशों
की परी ा करने एवं रा पित को उन पर अपनी राय दे ने के िलये एक
सं सदीय सिमित गिठत की जाएगी, िजसम लोकसभा एवं रा सभा के
सद शािमल होंगे।

उपयु म से कौन-सा/से कथन सही है /ह?

A) केवल 1

B) केवल 2

C) 1 और 2 दोनों

D) न तो 1 और न ही 2
Hide Answer -
उ रः (b)

ा ा : उपयु म से कथन (1) गलत है ।

सं िवधान के अनु े द-344 म ावधान है िक रा पित (न िक सं सद) इस सं िवधान के

ारं भ होने से पाँ च वष की समा पर और पु नः उसके प ात् 10 वष की समा पर,

आदे श के ारा, एक आयोग का गठन करे गा, जो एक अ एवं आठवीं अनु सूची म

िनिद भाषाओं का ितिनिध करने वाले ऐसे अ सद ों से िमलकर बने गा, िजसे

रा पित िनयु करे ।

राजभाषा आयोग के िलये गिठत सं सदीय सिमित म कुल 30 सद होंगे,िजसम 20

लोकसभा एवं 10 रा सभा के सद होंगे और इसके सद ों का चु नाव लोकसभा एवं

रा सभा के सद ों ारा आनु पाितक ितिनिध प ित के अनु सार एकल सं मणीय मत

के ारा होगा।

वष 1955 म रा पित ारा बी.जे .खे र की अ ता म एक आयोग का गठन िकया गया।

आयोग ने अपनी रपोट वष 1956 म रा पित को ु त की। वष 1957 म पं िडत गोिव

व व पं त की अ ता म बनी सं सदीय सिमित ने इस रपोट की समी ा की। पर रा के

अनु सार क ीय गृ हमं ी सिमित के अ चु ने जाते ह। वष 1960 म दू सरे आयोग का

गठन होना था, िकंतु इसका गठन नहीं हो सका।

[5] रा की राजभाषा के िवषय म िन िल खत कथनों पर िवचार कीिजयेः

1. रा का िवधानमं डल िविध ारा उस रा म योग होने वाली भाषाओं म


से एक या अिधक भाषाओं को या िह ी को उस रा के सभी या िक ीं
शासकीय उ े ों के िलये योग की जाने वाली भाषा या भाषाओं के प
म व था कर सकेगी।
2. सं घ म शासकीय उ े ों के िलये योग की जाने वाली ािधकृत भाषा,
एक रा एवं दू सरे रा के बीच तथा िकसी रा और सं घ के बीच
राजभाषा होगी।
3. बं िधत रा के रा पाल को यह समाधान हो जाता है िक िकसी रा की
जनसं ा का पया भाग यह चाहता है िक उसके ारा बोली जाने वाली
भाषा को रा ारा मा ता दी जाए तो वह िनदश दे सकेगा िक ऐसी
भाषा को उस रा म सव या उसके िकसी भाग म ऐसे उ े ों के िलये
शासकीय मा ता दी जाए।

उपयु म से कौन-से कथन सही ह ?

A) केवल 1 और 2

B) केवल 1 और 3

C) केवल 2 और 3

D) 1, 2 और 3
Hide Answer -
उ रः (a)

ा ाः उपयु म से कथन (3) गलत है ।

सं िवधान के अनु े द-347 म उपबं ध है िक भारत के रा पित को (न िक रा पाल को)

यह समाधान हो जाता है िक िकसी रा की जनसं ा का पया भाग यह चाहता है िक

उसके ारा बोली जाने वाली भाषा को रा ारा मा ता दी जाए तो रा पित ऐसा िनदश

दे सकता है ।

सं िवधान के अनु े द-345 म ावधान है िक िकसी रा का िवधानमं डल िविध ारा ऐसा

उपबं ध कर सकेगा । रा का िवधानमं डल िविध ारा, ऐसा उपबं ध न कर, तब तक रा

के भीतर उन शासकीय उ े ों के िलये अं े ज़ी भाषा का योग िकया जाता रहे गा।

अनु े द-346 म उपबं ध है िक एक रा एवं दू सरे रा के बीच या िकसी रा और सं घ

के बीच सं घ म शासकीय उ े ों के िलये योग की जाने वाली ािधकृत भाषा राजभाषा

होगी, िकंतु दो या अिधक रा यह समझौता करते ह िक उन रा ों के बीच प आिद की

राजभाषा िह ी भाषा होगी, तो उस भाषा का योग िकया जा सकेगा।

[6] िन िल खत कथनों पर िवचार कीिजयेः

1. सं िवधान म उपबं ध है िक उ तम ायालय और ेक उ ायालय की


सभी कायवािहयाँ , सं सद या रा िवधानमं डल म ु त िकये जाने वाले
िवधे यक या ाव तथा रा पित या रा पाल ारा ािपत सभी
अ ादे शों के ािधकृत पाठ अं े ज़ी या िह ी भाषा म होंगे।
2. सं िवधान म उपबं ध है िक िकसी रा का रा पाल, रा पित की पू व
अनु मित से िह ी अथवा िकसी रा की अ भाषा को उ ायालय की
कायवाही की भाषा का दज़ा दे सकता है िकंतु ायालय ारा िदये गए
िनणय, आ ा अथवा आदे श केवल अं े जी म होग।

उपयु म से कौन-सा/से कथन सही है /ह?

A) केवल 1

B) केवल 2

C) 1 और 2 दोनों

D) न तो 1 और न ही 2
Hide Answer -
उ रः (b)

ा ाः उपयु म से कथन (1) गलत है ।

सं िवधान के अनु े द-348 म ावधान है िक उ तम ायालय और ेक उ ायालय

की सभी कायवािहयाँ , सं सद या रा िवधानमं डल म ु त िकये जाने वाले िवधे यक या

ाव तथा रा पित या रा पाल ारा ािपत सभी अ ादे श के ािधकृत पाठ अं े जी

भाषा म होग इसम िह ी भाषा का उ े ख नहीं है ।

सं िवधान के अनु े द-348(2) म उपबं ध है िक िकसी रा का रा पाल, रा पित की

पू व अनु मित से िह ी अथवा िकसी रा की अ भाषा को उ ायालय की कायवाही

की भाषा का दज़ा दे सकता है , परं तु ायालय ारा िदये गए िनणय, आ ा अथवा आदे श

केवल अं े ज़ी म होंगे।

[7] राजभाषा अिधिनयम, 1963 के िवषय म िन िल खत म से कौन-सा कथन सही

नहीं है ?

यह अिधिनयम उपबं ध करता है िक रा पित के ािधकार से कािशत


A) अिधिनयम, अ ादे श, आदे श, िनयम एवं उप-िनयमों के िह ी अनु वाद
आिधका रक ले ख माने जाएं गे ।

इस अिधिनयम म उपबं ध है िक सं सद म ुत े क िवधे यक के साथ


B)
उसका िह ी अनु वाद आव क है ।

यह अिधिनयम रा पाल को यह अिधकार दे ता है िक वह रा पित की पू व


C) अनु मित से उ ायालय ारा िदये गए फैसलों, िनणयों और पा रत आदे शों
म िह ी या रा के िकसी अ भाषा के योग की अनु मित दे सकता है ।

यह अिधिनयम उ तम ायालय म अं े ज़ी के साथ-साथ िह ी भाषा के


D)
योग की अनु मित दे ता है ।

Hide Answer -
उ रः (d)

ा ाः उपयु म से िवक (d) गलत है ।

सं सद ारा वष 1963 म राजभाषा अिधिनयम पा रत िकया गया। यह अिधिनयम सं घ के

सभी सरकारी काय की कायवाही म अं े जी के योग को जारी रखने के साथ ही िह ी के

योग का उपबं ध करता है , जो इस कार ह।

रा पित के ािधकार से कािशत अिधिनयम, अ ादे श, आदे श, िनयम एवं उप-िनयम।

सं सद म यु े क िवधे यक।

रा पाल, रा पित की पू वानु मित से उ ायालय म।

सं सद ारा उ तम ायालय म अं े ज़ी के साथ-साथ िह ी भाषा के योग के सं बंध म

ऐसा कोई िनयम नहीं बनाया गया है । अथात उ तम ायालय केवल उ ीं यािचकाओं को

सु नता है , जो केवल अं े ज़ी भाषा म होते ह।

[8] िन िल खत कथनों पर िवचार कीिजयेः


1. सं िवधान म ावधान है िक ेक िकसी था के िनवारण के िलये
सं घ या रा के अिधकारी या ािधकारी को सं घ या रा म योग होने
वाली िकसी भाषा म आवे दन दे ।
2. सं िवधान म यह ावधान है िक भाषाई अ सं क वग के बालकों को
ाथिमक र पर मातृ भाषा म िश ा दान की जाए।
3. सं सद ने िविध बनाकर यह व था की है िक भाषाई अ सं क वग के
िलये एक िवशे ष अिधकारी होगा, िजसे रा पित िनयु करे गा।

उपयु म से कौन-से कथन सही ह?

A) केवल 1 और 2

B) केवल 1 और 3

C) केवल 2 और 3

D) 1, 2 और 3

Hide Answer -
उ रः (a)

ा ाः उपयु म से कथन (3) गलत है ।

सं िवधान के अनु े द-350(ख) म उपबं ध है (न िक सं सद ने िविध बनाकर व था की

है ) िक भाषाई अ सं क वग के िलये एक िवशे ष अिधकारी होगा, िजसे रा पित िनयु

करे गा। यह अिधकारी अपना ितवे दन रा पित को सौंपेगा और रा पित ऐसे ितवे दनों को

सं सद के े क सदन के सम रखवाएगा और सं बंिधत रा ों की सरकारों को

िभजवाएगा।

सं िवधान के अनु े द-350 म उपबं ध है िक ेक िकसी था के िनवारण के िलये

सं घ या िकसी रा के िकसी अिधकारी या पदािधकारी को सं घ या रा म योग होने

वाली िकसी भाषा म आवे दन दे ने का हकदार होगा।

सं िवधान के अनु े द-350(क) म उपबं ध है िक े क करा और रा के भीतर

थानीय ािधकारी अ सं क वग के बालकों को िश ा के ाथिमक र पर मातृ भाषा

म िश ा की पया सु िवधाओं की व था करने का यास करे गा और रा पित िकसी रा

को ऐसा िनदश दे सकेगा, जो उिचत समझता है ।

सं िवधान के अनु े द-351 म उपबं ध है िक सं घ का यह क होगा िक वह िह ी भाषा

का सार बढाए एवं उसका िवकास करे ।

[9] िन िल खत भाषाओं पर िवचार कीिजयेः

1. क ड़
2. गु जराती
3. उिड़या
4. सं ृ त
5. मलयालम

उपयु म से िकसे /िक सरकार ने शा ीय(Classical) भाषा/भाषाएँ घोिषत


नहीं िकया है ?
A) केवल 2

B) केवल 1 और 2

C) केवल 2 और 5

D) केवल 2, 3 और 4

Hide Answer -
उ रः (a)

ा ाः उपयु म से कथन (2) गलत है ।

वष 2004 म भारत सरकार ने एक नए भाषा वग शा ीय/ े भाषा को बनाने का िनणय

िलया तथा वष 2006 म शा ीय भाषा का दज़ा दे ने के िलये मापद तै यार िकया गया।

वष 2016 तक छः भाषाओं को शा ीय भाषाओं का दज़ा िमल चु का है :

शा ीय भाषा वष

1. तिमल 2004

2. सं ृ त 2005

3. ते लुगू 2008

4. क ड़ 2008

5. मलयालम 2013

6. उिड़या 2014

[10]िन िल खत कथनों पर िवचार कीिजयेः

1. सं िवधान के भाग-21 एवं आठवीं अनु सूची म राजभाषा म सं बंध म उपबं ध


िकया गया है ।
2. िजस समय भारतीय सं िवधान लागू िकया गया,उस समय आठवीं अनु सूची
म कुल भाषाओं की सं ा 18 थी।
3. 91व सं िवधान सं शोधन अिधिनयम, 2003 के ारा आठवीं अनु सूची म
चार नई भाषाएं जोड़ी गई।

उपयु म से कौन-सा/से कथन सही है /ह?

A) केवल 1

B) केवल 1 और 3

C) केवल 2 और 3

D) उपयु म से कोई नहीं।


Hide Answer -
उ रः (d)

ा ाः उपयु सभी कथन गलत ह।

सं िवधान के भाग-17 एवं 8वीं अनु सूची म राजभाषा के सं बंध म उपबं ध िकया गया है ।

जब भारतीय सं िवधान लागू (26 जनवरी, 1950) आ, उस समय आठवीं अनु सूची म

कुल 14 भाषाएँ थी।

92व सं िवधान सं शोधन अिधिनयम, 2003 के ारा आठवीं अनु सूची म चार नई भाषाएँ

जोड़ी गई और इस तरह इसम भाषाओं की कुल सं ा 18 से बढ़कर 22 हो गई। वतमान

म 8वीं अनु सूची म भाषओं की कुल सं ा 22 ह ।

[11]िन िल खत म से भारतीय सं िवधान के िकस अनु े द म आठवीं अनु सूची के ित िनदश


िदया गया है ?

A) अनु े द-348

B) अनु े द-350(क)

C) अनु े द-350(ख)

D) अनु े द-351

Hide Answer -
उ रः (d)

ा ाः

सं िवधान के अनु े द-344(1) एवं अनु े द-351 म आठवीं अनु सूची के सं बंध म िनदश

िदया गया है ।

अनु े द-344(1): रा पित एक राजभाषा आयोग गिठत करे गा, िजसम एक अ एवं

8वीं अनु सूची म शािमल भाषाओं का ितिनिध करने वाले सद शािमल होंगे।

अनु े द-351: इसम िनदश िदया गया है िक िह ी भाषा का िवकास उसकी कृित म

ह े प के िबना िकया जाए तथा 8वीं अनु सूची म शािमल की गई भारत की अ भाषाओं

म यु प, शै ली और पदों को आ सात करते ए िकया जाए।

[12]िन िल खत म से िकस सं िवधान सं शोधन का सं बंध आठवीं अनु सूची से नही ं है ?

A) 21व सं िवधान सं शोधन अिधिनयम, 1966

B) 24व सं िवधान सं शोधन अिधिनयम, 1971

C) 71व सं िवधान सं शोधन अिधिनयम, 1992

D) 92व सं िवधान सं शोधन अिधिनयम, 2003


Hide Answer -
उ रः (b)

ा ाः उपयु म से िवक (b) का सं बंध सं िवधान की 8वीं अनु सूची से नहीं है ।

24व सं िवधान सं शोधन अिधिनयम, 1971 का सं बंध मौिलक अिधकारों म सं शोधन से

सं बंिधत है ।

21व सं िवधान सं शोधन अिधिनयम, 1966 के ारा िसं धी भाषा को आठवीं अनु सूची म

जोड़ा गया।

71व सं िवधान सं शोधन अिधिनयम, 1992 के ारा 8वीं अनु सूची म तीन भाषाएं जोड़ी

गई: ने पाली, कोंकणी एवं मिणपु री।

92व सं िवधान सं शोधन अिधिनयम, 2003 के ारा 8वीं अनु सूची म चार भाषाएं जोड़ी गई:

मै थली, सं थाली,डोगरी और बोडो।

[13]िन िल खत म से िकसका/िकनका सं बंध सं िवधान की आठवीं अनु सूची से है /ह?

1. िह ी
2. अं े ज़ी
3. उदू
4. सं ृ त
5. ने पाली

नीचे िदये गए कूट का योग कर सही उ र का चयन कीिजयेः

A) केवल 1, 3 और 4

B) केवल 1, 2, 3 और 4

C) केवल 1, 3, 4 और 5

D) 1, 2, 3, 4 और 5

Hide Answer -
उ रः (c)

ा ाः उपयु म से अं े ज़ी भाषा का सं बंध 8वीं अनु सूची से नहीं है ।

8वीं अनु सूची म चार रा ीय र की राजभाषाओं एवं 18 े ीय र की राजभाषाओं का

ावधान है । चार रा ीय र की राजभाषाएं ह - िह ी, उदू , सं ृ त और ने पाली।

[14]सं िवधान के भाग-XVI म कुछ वग के िलये िवशे ष उपबं ध िकया गया है । इस

िवषय म िन िल खत म से कौन-सा कथन सही है ?


भारत के सं िवधान म अनु सूिचत जाित एं व अनु सूिचत जनजाित तथा िपछड़े वग
A)
को प से प रभािषत िकया गया है ।

भारत के सं िवधान म अनु सूिचत जाित, अनु सूिचत जनजाित, िपछड़ा वग और


B)
आं -भारतीय समु दाय को प रभािषत िकया गया है ।

भारत के सं िवधान म उपबं ध है िक अनु सूिचत जाित एवं अनु सूिचत जनजाित
C) की सू ची भारत का रा पित े क रा के रा पाल से परामश करके
बनाएगा।

भारत का सं िवधान रा पित को यह श दे ता है िक वह अनु सूिचत जाित एवं


D) अनु सूिचत जनजाित के िलये जारी सू ची म िकसी अनु सूिचत जाित अथवा
जनजाित को जोड़ या हटा सकता है ।

Hide Answer -
उ र: (c)

ा ाः उपयु म से िवक (c) सही है ।

सं िवधान के भाग-16 (अनु े द-330 से 342 तक) म कुछ वग के िलये िवशे ष उपबं ध

िकया गया है , िजसम शािमल ह :

- अनु सूिचत जाित

- अनु सूिचत जनजाित

- िपछड़ा वग

- आं -भारतीय समु दाय

भारत के सं िवधान म अनु सूिचत जाित, अनु सूिचत जनजाित और िपछड़े वग को प रभािषत

नहीं िकया गया है , िकंतु अनु े द-366(2) म आं -भारतीय को प रभािषत िकया गया

है ।

भारत के सं िवधान ने रा पित को यह श दान की है िक वह े क रा के रा पाल

से परामश करके अनु सूिचत जाित एवं अनु सूिचत जनजाित के िलये एक सू ची बनाए और

अिधसू चना जारी कर, िकंतु रा पित ारा जारी अिधसू चना म िकसी अनु सूिचत जाित अथवा

जनजाित को जोड़ने अथवा हटाने की श भारत की सं सद म िनिहत है । अथात रा पित

ारा जारी सू ची का सं सद पु नरी ण कर सकती है (अनु े द-341 एवं 342)।

सं िवधान के अनु े द-340 म उपबं ध है िक भारत का रा पित सामािजक एवं शै ि क ि

से िपछड़े वग की दशाओं के अ े षण के िलये एक आयोग की िनयु कर सकेगा। यह

आयोग अपना ितवे दन रा पित को सौंपेगा और रा पित ऐसे ितवे दनों को सं सद के दोनों

सदनों के सम रखवाएगा।

[15]भारत म सं वैधािनक आयोग (Constitutional Commission) के िवषय म

िवचार कीिजयेः

1. रा ीय अ सं क आयोग
2. रा ीय िपछड़ा वग आयोग
3. रा ीय अनु सूिचत जाित आयोग
4. रा ीय अनु सूिचत जनजाित आयोग

नीचे िदये गए कूट का योग कर सही उ र का चयन कीिजयेः


A) केवल 1, 2 और 4

B) केवल 2, 3 और 4

C) केवल 3 एवं 4

D) 1, 2, 3 और 4

Hide Answer -
उ र: (c)
ा ाः उपयु म से रा ीय अ सं क आयोग एवं रा ीय िपछड़ा वग आयोग
सं वैधािनक आयोग नहीं ह।

रा ीय अ सं क आयोग अिधिनयम,1992 के तहत वष 1993 म रा ीय


अ सं क आयोग का गठन िकया गया। यह एक वै धािनक/सां िविधक आयोग है ।
भारत के सं िवधान म अ सं क श को प रभािषत नहीं िकया गया है । इसकी
अिधसू चना भारत सरकार के ारा की जाती है । मू ल प म पाँ च धािमक समु दायों
िस , ि न, बौ एवं पारसी (Zoroastrians) को अ सं क समु दाय का
दज़ा िदया गया है , िकंतु 30 जनवरी, 2014 को भारत के ारा जै न समु दाय को
अ सं क समु दाय घोिषत िकया गया। वतमान म भारत म छः अ सं क
समु दाय ह।
रा ीय िपछड़ा आयोग का गठन वष 1993 म रा ीय िपछड़ा वग आयोग
अिधिनयम, 1993 के तहत िकया गया है । यह एक वै धािनक आयोग है ।
भारत म रा पित के ारा अब तक िपछड़े वग के िलये दो आयोगों का गठन िकया
जा चु का है :

- वष 1953 म काका काले लकर की अ ता म पहला िपछड़ा वग आयोग।


इसने वष 1955 म अपनी रपोट दी।
- वष 1979 म बी.पी. मं डल की अ ता म दू सरा िपछड़ा वग आयोग। इसने
वष 1980 म अपनी रपोट दी। इस रपोट के आधार पर वष 1990 म वी.पी.
िसं ह सरकार के ारा सरकारी नौक रयों म 27% आर ण की घोषणा की गई।

सव थम 65व सं िवधान सं शोधन अिधिनयम, 1990 के ारा अनु सूिचत जाित एवं
जनजाित के िलये रा ीय आयोग को सं वैधािनक दज़ा दान िकया गया । 89व सं िवधान
सं शोधन, 2003 के ारा अनु सूिचत जाित एवं अनु सूिचत जनजाित के िलये दो तं प
से रा ीय आयोगों का गठन िकया गया। सं िवधान के अनु े द-338 म रा ीय अनु सूिचत
जाित आयोग का ावधान िकया गया एवं अनु े द 338(क) म रा ीय अनु सूिचत जनजाित
आयोग का ावधान िकया गया।

[16]भारत के सं िवधान के भाग-XVI म उपबं ध िकया गया है :

1. अ सं क वग के िलये ।
2. आं -भारतीय समु दाय के िलये ।
3. िपछड़े वग के िलये ।
4. अनु सूिचत जाित एवं अनु सूिचत जनजाित वग के िलये ।

नीचे िदये गए कूट का योग कर सही उ र का चयन कीिजयेः


A) केवल 2 और 4

B) केवल 3 और 4

C) केवल 2, 3 एवं 4

D) 1, 2, 3 और 4
Hide Answer -
उ र: (c)

ा ाः उपयु म से अ सं क वग के िलये सं िवधान के भाग-XVI म उपबं ध नहीं िकया

गया है ।

सं िवधान के भाग-XVI म िन वग के िलये उपबं ध िकया गया है :

(1) आं -भारतीयः सं िवधान के अनु े द-331, अनु े द-333, अनु े द-336 एवं

अनु े द-337 म उपबं ध िकया गया है ।

अनु े द-331- रा पित की राय म लोकसभा म आं -भारतीय समु दाय का ितिनिध

पया नहीं है तो दो आं -भारतीयों का नाम िनदिशत कर सकेगा।

अनु े द-333- रा पाल की राय म रा िवधानसभा म आं -भारतीय समु दाय का

ितिनिध पया नहीं है तो एक का नाम िनदिशत कर सकेगा।

अनु े द-336- कुछ से वाओं म आं -भारतीय समु दाय के िलये िवशे ष उपबं ध।

अनु े द-337- आं -भारतीय समु दाय के फायदे के िलये शै िणक अनु दान के िलये

िवशे ष उपबं ध।

(2) िपछड़ा वगः सं िवधान के अनु े द-340 म उपबं ध है िक भारत का रा पित सामािजक एवं

शै िणक (आिथक श शािमल नहीं है ) ि से िपछड़े वग की दशाओं एवं किठनाईयों के

अ े षण के िलये एक आयोग की िनयु कर सकेगा।

(3) अनु सूिचत जाित एवं अनु सूिचत जनजाितः सं िवधान के अनु े द-330, अनु े द-332,

अनु े द-335, अनु े द-338, अनु े द-338(क), अनु े द-339, अनु े द-341 एवं

अनु े द-342 म उपबं ध िकया गया है ।

अनु े द-330- लोकसभा म उसकी जनसं ा के अनु पात म थानों का आर ण।

अनु े द-332- रा की िवधानसभाओं म उसकी जनसं ा के अनु पात म आर ण।

अनु े द-335- सं घ या िकसी रा की से वाओं म िवशे ष उपबं ध। 82व सं िवधान सं शोधन।

अिधिनयम, 2000 के ारा सं घ या िकसी रा की से वाओं म पदो ित के सं बंध म

आर ण तथा ू नतम अं क कम करने के िवषय म ावधान िकया गया है ।

अनु े द-338- अनु सूिचत जाित वग के िलये एक रा ीय आयोग का उपबं ध।

अनु े द-338(क)- अनु सूिचत जनजाित वग के िलये एक रा ीय आयोग का उपबं ध।

अनु े द-339- रा पित अनु सूिचत े ों के शासन एवं अनु सूिचत जनजाितयों के क ाण

के बारे म ितवे दन दे ने के िलये एक आयोग की िनयु कर सकेगा। वष 1960 म

यू .एन. ढे बर की अ ता म एक आयोग का गठन िकया गया।

अनु े द-341- रा पित, अनु सूिचत जाित के िलये एक सू ची तै यार करे गा या अिधसू चना

जारी करे गा।

अनु े द-342- रा पित, अनु सूिचत जनजाित के िलये एक सू ची तै यार करे गा या

अिधसू चना जारी करे गा।

अनु े द-334- लोकसभा एवं रा की िवधानसभाओं म अनु सूिचत जाित एवं अनु सूिचत

जनजाित के िलये आर ण एवं आं -भारतीय समु दाय के िलये ितिनिध का ावधान 60

वष (वष 2010 तक) के िलये था, िजसे 95व सं िवधान सं शोधन अिधिनयम 2009 के

ारा 70 वष (वष 2020 तक) के िलये बढ़ा िदया गया है ।

[17]भारतीय सं िवधान म आर ण के िवषय म िन िल खत कथनों पर िवचार कीिजयेः


1. रा सभा म िकसी भी जाित अथवा जनजाित के िलये आर ण का ावधान
नहीं है ।
2. रा िवधान प रषद म िकसी भी जाित अथवा जनजाित के िलये आर ण
का ावधान नहीं है ।

उपर् यु म से कौन-सा/से कथन सही है /ह?

A) केवल 1

B) केवल 2

C) 1 और 2 दोनों

D) न तो 1 और न ही 2

Hide Answer -
उ र: (c)

ा ाः उपरो दोनों कथन सही ह।

रा सभा म िकसी भी जाित अथवा जनजाित के िलये आर ण का ावधान नहीं है । रा

िवधान प रषद म िकसी भी जाित अथवा जनजाित के िलये आर ण का ावधान नहीं है ।

[18]िन िल खत सिमितयों पर िवचार कीिजयेः

1. बलव राय मे हता सिमित


2. अशोक मे हता सिमित
3. वी.के.आर.वी. राव सिमित
4. एल.एम. िसं घवी सिमित
5. थुं गन सिमित

उपयु म से िकन सिमितयों का सं बंध पं चायती राज व था से रहा है ?

A) केवल 1, 2 और 3

B) केवल 1, 2, 3 और 4

C) केवल 1, 2, 4 और 5

D) केवल 1, 2, 3 और 5
Hide Answer -
उ रः (c)

ा ाः उपयु म से वी.के.आर.वी. राव सिमित का सं बंध पं चायती राज व था से नहीं रहा

है । यह मु ख अथशा ी थे , जो िविभ सिमितयों एवं आयोग का सद रहा है । जी.वी.के.राव.

सिमित का सं बंध पं चायती राज व था से रहा है । इस सिमित का गठन वष 1985 म िकया

गया।

िन सिमितयों का सं बंध पं चायती राज सं था से रहा है ः

1. बलव राय मे हता सिमित, 1957

2. अशोक मे हता सिमित, 1977

3. जी.वी.के.राव सिमित, 1985

4. एल.एम. िसं घवी सिमित, 1986

5. थुं गन सिमित, 1988

6. गाडिगल सिमित, 19888

7. सािदक अली सिमित, 1964

[19]भारत म पं चायती राज के िवषय म िन िल खत म से कौन-सा कथन सही है ?

सं िवधान के अनु े द-45 म पं चायती राज के गठन के सं बंध म ावधान िकया


A)
गया है ।

बलव राय मे हता सिमित का गठन वष 1959 म सामु दाियक िवकास


B)
काय म ारा िकये गए काय की जाँ च/समी ा के िलये िकया गया।

73व सं िवधान सं शोधन के ारा सं िवधान म भाग-9(क) जोड़ा गया और


C)
पं चायती राज सं था को सं वैधािनक दज़ा दान िकया गया।

पं चायती राज की थापना सव थम 2 अ ू बर 1959 म राज थान के नागौर िज़ले


D)
म की गई।

Hide Answer -
उ रः (d)

ा ाः उपयु म से िवक (d) सही है ।

दे श म सव थम पं चायती राज की थापना 2 अ ू बर 1959 को राज थान के नागौर

िज़ले म पं . जवाहरलाल ने ह के ारा की गई।

सं िवधान के अनु े द-40 म (न िक अनु े द-45) पं चायती राज के गठन के सं बंध म

ावधान िकया गया है ।

भारत म सामु दाियक िवकास काय म (1952) एवं रा ीय िव ार से वा (1953) के ारा

िकये गए काय की जाँ च एवं समी ा के िलये जनवरी, 1957 म (न िक 1959) बलव

राय मे हता की अ ता म एक सिमित का गठन िकया गया।

73व सं िवधान सं शोधन 1992 के ारा सं िवधान के भाग-9 एवं 11वीं अनु सूची जोड़ी गई

और पं चायती राज सं था को सं वैधािनक दज़ा दान िकया गया।

[20]िन िल खत कथनों पर िवचार कीिजयेः


1. पं चायती रा समवत सू ची का िवषय है ।
2. पं चायती राज को सं वैधािनक दज़ा धानमं ी वी.पी. िसं ह के कायकाल म
दान िकया गया।

उपयु म से कौन-सा/से कथन सही है /ह?

A) केवल 1

B) केवल 2

C) 1 और 2 दोनों

D) न तो 1 और न ही 2

Hide Answer -
उ रः (d)

ा ाः उपयु दोनों कथन गलत ह।

पं चायती राज रा सू ची का िवषय है । इसका उपबं ध िवि सं ा 5 म िकया गया है ।

73व सं िवधान सं शोधन अिधिनयम, 1992 के ारा पं चायती राज को सं वैधािनक दज़ा

दान िकया गया। उस समय भारत के धानमं ी पी.वी. नरिस ा राव थे ।


[1] भारत म पं चायती राज व था के मू ल उ े के िवषय म िन िल खत कथनों पर
िवचार कीिजयेः

1. लोकतां ि क िवक ीकरण सु िनि त करना।


2. िवकास म जन भागीदारी को बढ़ावा दे ना।
3. राजनीितक जवाबदे ही का िवकास करना।
4. िव ीय सं ह को बढ़ावा दे ना।

उपयु म से कौन-सा/से कथन सही है /ह?

A) केवल 1 और 2

B) केवल 1, 2 और 3

C) केवल 1, 2 और 4

D) 1, 2, 3 और 4

Hide Answer -
उ रः (a)

ा ाः उपयु म से कथन (3) और 4 सही नहीं ह।

पं चायती राज व था को 73व सं िवधान सं शोधन अिधिनयम, 1992 (वष 1993 म लागू ) के

ारा सं वैधािनक दज़ा दान िकया गया। इसका मू ल उ े िवकास म जन-भागीदारी को बढ़ावा

दे ना एवं लोकतां ि क िवक ीकरण को सु िनि त करना है । राजनीितक जवाबदे ही पं चायती राज

व था का मू ल उ े नहीं है ।

[2] भारत म पं चायती राज व था को सं वैधािनक दजा िविभ सिमितयों की


िसफा रशों के आधार पर िदया गया। िन िल खत म से वह कौन-सी सिमित है ,
िजसने ऐसी िसफा रश नहीं की थी?

A) जी.वी.के. राव सिमित

B) एल.एम. िसं घवी सिमित

C) थुं गन सिमित

D) गाडिगल सिमित
Hide Answer -
उ रः (a)

ा ाः उपयु म से जी.वी.के. राव सिमित के ारा ऐसी िसफा रश नहीं की गई थी, जबिक

एल.एम. िसं घवी सिमित, थुं गन सिमित एवं गाडिगल सिमित ने पं चायती राज व था को

सं वैधािनक दज़ा दान करने की िसफा रश की थी। इसी आधार पर 73व सं िवधान सं शोधन के

ारा इसे सं वैधािनक दज़ा दान िकया गया।

पं चायती राज को सं वैधािनक दज़ा दान करने के िलये त ालीन धानमं ी राजीव गां धी के

ारा 64वां सं िवधान सं शोधन अिधिनयम वष 1989 म लोकसभा म लाया गया, िकंतु यह

िवधे यक रा सभा म पा रत न हो सका।

[3] िन िल खत म से कौन-सा/से सही सु मेिलत है /ह?

1. अशोक मे हता सिमित - ि - रीय पं चायती राज


2. बलवं त राय सिमित – ि - रीय पं चायती राज
3. थुं गन सिमित - पं चायतों को सं वैधािनक दज़ा

नीचे िदये गए कूट का योग कर सही उ र का चयन कीिजयेः

A) केवल 3

B) केवल 1 और 2

C) केवल 2 और 3

D) केवल 1 और 3
Hide Answer -
उ रः (a)

ा ाः उपयु म से कथन (1) और (2) गलत ह।

अशोक मे हता सिमित का गठन िदसं बर 1977 म जनता पाट की सरकार ारा िकया

गया। इसने अपनी रपोट अग 1978 म सौंपी। इसने ि - रीय पं चायती राज व था

की िसफा रश की थी। िज़ला र पर िज़ला प रषद तथा 15-20 हज़ार जनसं ा के गाँ वों

के समू ह पर मं डल पं चायत। इसने म वत पं चायत की बात नहीं की।

बलवं त राय मे हता सिमितः इसका गठन जनवरी 1957 म िकया गया और नवं बर 1957

म इसने अपनी रपोट सरकार को सौंपी। इसने लोकतां ि क िवक ीकरण की योजना

िसफा रश की, जो ‘पं चायती राज’ के नाम से जाना गया। इसने ि - रीय पं चायती राज

व था की बात कीः

1. ाम र पर ाम पं चायत

2. ॉक/खं ड र पर पं चायत सिमित

3. िज़ला र पर िज़ला प रषद

मे हता ने िसफा रश की िक ाम पं चायत की थापना प से चु ने ए ितिनिधयों

ारा होनी चािहये , जबिक पं चायत सिमित एवं िजला प रषद का गठन अ प से

अथात चु ने ए सद ों ारा होनी चािहये । ये तीनों र आपस म अ चु नाव के ारा

जु ड़े होने चािहये ।

मे हता ने िसफा रश की िक पं चायत सिमित को कायकारी िनकाय तथा िज़ला प रषद को

सलाहकारी सम यकारी और पयवे ण िनकाय होना चािहये । िज़ला प रषद का अ

िज़ला अिधकारी होना चािहये ।

भारत म पं चायती राज व था का वा ु कार/िश ी बलवं त राय मे हता (बी.आर. मे हता)

को कहा जाता है ।

एल.एम. िसं घवी सिमित के अलावा थुं गन एवं गाडिगल सिमित ने भी पं चायती राज को

सं वैधािनक दज़ा दे ने की बात कही।

73वां सं िवधान सं शोधन अिधिनयम, 1992 लोकसभा के ारा 22 िदसं बर, 1992 एवं

रा सभा के ारा 23 िदसं बर, 1992 म पा रत िकया गया तथा 20 अ ै ल, 1993 को

रा पित ारा ीकृित िमली।

[4] 73व सं िवधान सं शोधन अिधिनयम, 1992 के िवषय म िन िल खत कथनों पर

िवचार कीिजयेः

1. इसके ारा सं िवधान म 11वीं अनु सूची जोड़ी गई तथा इसम 18 िवषयों को
शािमल िकया गया।
2. इसके ारा दे श म मज़बू त एवं जीव पं चायती राज सं थाओं की बु िनयाद
रखी गई।
3. इसके ारा पं चायत के चु नावों को आदे शा क तथा लोकसभा एवं
िवधानसभा चु नावों के समक बनाया गया।

उपयु म से कौन-सा/से कथन सही है /ह?


A) केवल 1

B) केवल 1 और 2

C) केवल 2 और 3

D) 1, 2 और 3

Hide Answer -
उ रः (c)

ा ाः उपयु म से कथन (1) गलत है ।

73व सं िवधान सं शोधन, 1992 के ारा सं िवधान म भाग-9 एवं 11वीं अनु सूची जोड़ी गई

तथा इसम 29 िवषयों (न िक 18) को शािमल िकया गया।

इस सं शोधन के ारा पं चायती राज को सं वैधािनक दज़ा दान िकया गया तथा इसके िलये

िनयिमत चु नाव की व था की गई। िन एवं आदे शा क चु नाव कराने के िलये रा

िनवाचन आयोग का गठन िकया गया।

इस सं शोधन के ारा दे श म मज़बू त एवं जीव पं चायती व था की बु िनयाद रखी गई।

[5] िन िल खत म से िकसका सं बंध ‘ ास ट डे मो े सी’ से है ?

A) पं चायती राज व था

B) लोकपाल एवं लोकायु अिधिनयम

C) े ीय राजनीित

D) अ रा ीय प रषद

Hide Answer -
उ रः (a)

ा ाः उपयु म से िवक (a) सही है ।

तृ णमू ल लोकतं (Grassroot Democracy) या ज़मीनी सतह पर लोकतं का सं बंध

लोकतं के िवक ीकरण से है , िजसे पं चायती राज व था के नाम से जानते ह। भारत म

तं ता के प ात् पहली बार राज थान के नागौर िज़ले म 2 अ ू बर, 1959 को पं िडत

जवाहरलाल ने ह के ारा लागू की गई। उसी िदन यह सं पूण राज थान म लागू की गई।

पं चायती राज व था को लागू करने वाला दू सरा रा आं दे श है ।

[6] ‘ ाम सभा’ के िवषय म कौन-सा कथन सही है ?

A) पं चायत े के सभी यों से िमलकर ाम सभा का गठन होता है ।

पं चायत े के वै से यों से िमलकर ाम सभा का गठन होता है िजसका


B)
नाम गाँ व की िनवाचक नामावली म दज़ है ।

गाँ व के िनवाचक नामावली म पं चायत े के ऐसे का नाम दज़ िकया


C)
जाता है , िजसकी आयु 21 वष से कम न हो।

D) उपयु म से कोई नहीं।


Hide Answer -
उ रः (b)

ा ाः ाम पं चायत का चु नाव ाम सभा ारा िकया जाता है । ाम सभा का गठन पं चायत े

के ऐसे यों से होता है , िजनका नाम गाँ व की िनवाचक नामावली म दज़ हो।गाँ व के िनवाचक

नामावली म नाम दज़ कराने के िलये ू नतम आयु 18 वष (न िक 21 वष) िनधा रत की गई है ।

[7] पं चायत सिमित के िवषय म िन िल खत कथनों पर िवचार कीिजयेः

1. पं चायत सिमित के सद अ िविध से चु ने जाते ह।


2. पं चायत सिमित शासिनक ािधकरण के प म काय करती है ।

उपयु म से कौन-सा/से कथन सही है /ह?

A) केवल 1

B) केवल 2

C) 1 और 2 दोनों

D) न तो 1 और न ही 2

Hide Answer -
उ रः (b)

ा ाः उपयु म से कथन (1) गलत है ।

पं चायत सिमित का उपबं ध पं चायती राज व था के म वत र पर िकया गया है ।

म वत र/ ॉक/खं ड र पर पं चायत उन रा ों म की जाएगी, िजसकी जनसं ा

कम से कम 20 लाख हो।

पं चायत सिमित के सद ों का चु नाव जनता ारा प से िकया जाता है । उ र दे श

म पं चायत सिमित को े पं चायत कहा जाता है ।

ॉक/खं ड र पर पं चायत सिमित एक शासिनक ािधकरण के प म काय करती है ।

यह ाम पं चायत एवं िज़ला पं चायत के बीच सं पक थािपत करती है । े पं चायत के

िनवािचत सद ों ारा एक मु ख, एक उप- मु ख एवं एक किन उप- मु ख चु ने जाते ह।

[8] 73व सं िवधान सं शोधन के िवषय म िन िल खत कथनों पर िवचार कीिजयेः

1. इसका अिभपालन करने वाला पहला रा म दे श है ।


2. ाम, म वत एवं िज़ला र पर पं चायत के अ का चु नाव अ
िविध से िकया जाता है ।

उपयु म से कौन-सा/से कथन सही है /ह?

A) केवल 1

B) केवल 2

C) 1 और 2 दोनों

D) न तो 1 और न ही 2
Hide Answer -
उ रः (a)

ा ाः उपयु म से कथन (2) गलत है ।

73वां सं िवधान सं शोधन अिधिनयम, 1992, 24 अ ै ल, 1992 को लागू आ। इसका

अिभपालन करने वाला पहला रा म दे श है ।

ाम र पर पं चायत के अ का चु नाव प से िकया जाता है , जबिक म वत एवं

िज़ला र पर अ का चु नाव अ िविध से िकया जाता है । दू सरी ओर ाम, म वत एवं

िज़ला र पर पं चायत के सभी सद िविध से चु ने जाते ह।

[9] 73व सं िवधान सं शोधन अिधिनयम, 1992 के ारा पं चायत म आर ण का


उपबं ध िकया गया है । इस िवषय म िन िल खत म से कौन-सा कथन सही है ?

पं चायत म मिहलाओं का आर ण एक-ितहाई से कम न हो (अनु सूिचत जाित


A) एवं अनु सूिचत जनजाित की मिहलाओं के िलये आरि त सीटों की सं ा को
छोड़कर)।

पं चायत म मिहलाओं का आर ण एक-ितहाई से कम न हो (इसम वह सं ा


B) भी शािमल है , िजसके तहत अनु सूिचत जाित एवं अनु सूिचत जनजाित की
मिहलाओं का आर ण िदया गया है ।)।

इस सं िवधान के ारा पं चायातों म अ िपछड़ा वग के िलये 27 ितशत थान


C)
आरि त िकया गया है ।

इस सं िवधान सं शोधन के ारा उपबं ध िकया गया है िक पं चायतों म अनु सूिचत


D)
जाित एवं जनजाित के सद ों के िलये 33% का आर ण हो।

Hide Answer -
उ रः (b)

ा ाः उपयु म से कथन (b) सही है ।

सं िवधान के अनु े द-243(घ) म पं चायतों म आर ण के सं बंध म ावधान िकया गया है ।

इसके तहत पं चायतों म मिहलाओं के िलये 1/3 थान (33% थान) आरि त ह (इसके

अं तगत वह सं ा की शािमल है , िजसके तहत अनु सूिचत जाित एवं अनु सूिचत जनजाित

की मिहलाओं को आर ण िदया है ।)। ेक र पर पं चायत के अ पदों के िलये कुल

सं ा का कम-से -कम 1/3 थान (33% थान) यों के िलये आरि त िकया गया है ।

रा का िवधानमं डल िपछड़े वग के िलये आर ण का उपबं ध कर सकता है , िकंतु अभी

तक िपछड़े वग के िलये आर ण का ावधान नहीं िकया गया है ।

इस सं शोधन के ारा पं चायतों म अनु सूिचत जाित एवं अनु सूिचत जनजाित के िलये उसकी

जनसं ा के अनु पात म आर ण का ावधान िकया गया है ( न िक 33% आर ण का

ावधान िकया गया है ), िकंतु इसम से कम-से -कम 1/3 (33%) थान अनु सूिचत जाित

एवं अनु सूिचत जनजाित के मिहलाओं के िलये आरि त होगा। पं चायत म अ पदों के

िलये अनु सूिचत जाित एवं अनु सूिचत जनजाित की िलये उसकी जनसं ा के अनु पात म

आर ण का ावधान िकया गया है ।


[10]िन िल खत कथनों पर िवचार कीिजयेः

1. पं चायत के समय पू व िवघटन की तारीख से छः माह के अ र चु नाव करा


ले ना आव क है ।
2. पं चायत के समय पू व िवघिटत होने के प ात् पु नगिठत पं चायत पू रे पाँ च
वष के िलये काय करे गी।

उपयु म से कौन-सा/से कथन सही है /ह?

A) केवल 1

B) केवल 2

C) 1 और 2 दोनों

D) न तो 1 और न ही 2

Hide Answer -
उ रः (a)

ा ाः उपयु म कथन (2) गलत है ।

सं िवधान के अनु े द-243(ड) म पं चायत की अविध के सं बंध म उपबं ध िकया गया है ।

इसम ावधान है िक पं चायत अपने थम अिधवे शन के िलये िनयत तारीख से पाँ च वष तक बनी

रहे गी, िकंतु पाँ च वष पू व पं चायत का िवघटन कर िदया जाता है तो िवघटन की तारीख से छः

माह के अ र िनवाचन कराना आव क है । िवघटन के प ात् पु नगिठत पं चायत शे ष अविध के

िलये काय करे गी न िक पू ण अविध (5 वष के िलये ) के िलये , िकंतु शे ष अविध छः माह से कम

है तो िनवाचन कराना आव क नहीं है ।

[11]िन िल खत म से कौन-सा/से कथन सही है /ह?

1. िकसी के िलये पं चायत का सद बनने के िलये ू नतम आयु 25


वष होनी चािहये ।
2. पं चायतों के िनवाचन सं बंधी मामलों की वै धता को भारत के िकसी
ायालय म गत नहीं िकया जा सकता।

नीचे िदये गए कूट का योग कर सही उ र का चयन कीिजयेः

A) केवल 1

B) केवल 2

C) 1 और 2 दोनों

D) न तो 1 और न ही 2
Hide Answer -
उ रः (b)

ा ाः उपयु म से कथन (1) गलत है ।

सं िवधान के अनु े द-243(च) म उपबं ध है िक पं चायत का सद बनने के िलये िकसी

भी की ू नतम आयु 21 वष होनी चािहये न िक 25 वष।

सं िवधान के अनु े द-243(ण) म उपबं ध है िक पं चायतों के िनवाचन े ों के प रसीमन या

ऐसे िनवाचन े ों म थानों के आवं टन से सं बंिधत वै धता को भारत के िकसी ायालय म

चु नौती नहीं दी जा सकती।

[12]िन िल खत कथनों पर िवचार कीिजयेः

1. भारत म पं चायतों का चु नाव रा िनवाचन आयोग के ारा कराया जाता


है ।
2. रा िनवाचन आयु की िनयु रा के रा पाल ारा की जाती है
तथा उसे उसके पद से रा पाल उसी रीित से हटा सकता है ,िजस रीित से
उ ायालय के ायाधीश को हटाते ह।

उपयु म से कौन-सा/से कथन सही है /ह?

A) केवल 1

B) केवल 2

C) 1 और 2 दोनों

D) न तो 1 और न ही 2

Hide Answer -
उ रः (c)

ा ाः उपयु दोनों कथन सही ह।

सं िवधान के अनु े द-243(ट) म पं चायतों के चु नाव के सं बंध म ावधान है । पं चायतों का

चु नाव रा िनवाचन आयोग ारा कराया जाता है ।

रा िनवाचन आयु की िनयु रा के रा पाल के ारा की जाती है । रा िनवाचन

आयु की से वा-शत और पदाविध ऐसी होगी, िजसे रा पाल िनयम ारा िनधा रत कर।

रा िनवाचन आयु को उसके पदाविध से पू व उसी तरह से हटाया जा सकता है , िजस

तरह से उ ायालय के ायाधीश को हटाते ह।

[13]िन िल खत म से कौन-सा कथन सही नहीं है ?


रा िव आयोग सरकार के ारा लगाए गए कुल कर एवं एकि त शु ों के
A) रा एवं पं चायतों के बीच बँ टवारे के िवषय म रा पाल को िसफा रश करता
है ।

रा िव आयोग, रा पाल को ऐसे करों के िवषय म िसफा रश करता है ,


B)
जो पं चायतों को सौंपा गया है ।

रा ों की सं िचत िनिध से पं चायतों को दी जाने वाली अनु दान सहायता के


C)
िवषय म रा िव आयोग, रा पाल को िसफा रश करता है ।

पं चायतों को िव ीय सहायता दान करने हे तु उपायों पर िवचार करने के


D)
सं बंध म क ीय िव आयोग की कोई भू िमका नहीं है ।

Hide Answer -
उ रः (d)

ा ाः सं िवधान के अनु े द-243(झ) म उपबं ध है िक रा पाल 73व सं िवधान सं शोधन

अिधिनयम, 1992 के ारं भ से एक वष के अं दर और इसके प ात् े क 5 वष की समा

पर एक रा िव आयोग का गठन करे गा। रा िव आयोग रा पाल को िन मामलों म

िसफा रश करे गाः

रा सरकार ारा लगाए गए कुल करों, चुं गी, माग कर एवं एकि त शु ों का रा

पं चायतों के बीच बँ टवारा।

करों, चुं गी, माग कर और शु ों का िनधा रण, जो पं चायत को सौंपे गए ह।

रा की सं िचत िनिध से पं चायतों को दी जाने वाली अनु दान सहायता।

पं चायतों को िव ीय थित के सु धार के िलये आव क उपाय।

रा पाल ारा आयोग को सौंपे जाने वाला कोई भी मामला, जो पं चायतों के मज़बू त िव

के िलये हो।

रा िव आयोग की िसफा रश पर क ीय िव आयोग पं चायतों की िव ीय सहायता हे तु

रा की सं िचत िनिध की मज़बू ती के िलये सलाह दे ता है ।

रा पाल, रा िव आयोग ारा सौंपी गए िसफा रशों को रा िवधानमं डल के सम

ु त करता है ।

[14]िन िल खत म से िकस रा म पं चायती रा सं था लागू नहीं है ?

A) असम

B) नागालै

C) िस म

D) ि पु रा

Hide Answer -
उ रः (b)

ा ाः भारत म पं चायती राज सं था नागालै , मे घालय, िमज़ोरम, िद ी एवं ज ू -क ीर

रा ों म नहीं है ।
[15]73व सं िवधान सं शोधन अिधिनयम, 1992 पर िवचार कीिजयेः

1. ाम पं चायत
2. े पं चायत
3. िज़ला पं चायत

उपयु म से कौन-सा/से कथन सही है /ह?

A) केवल 1

B) केवल 1 और 3

C) केवल 1 और 2

D) 1, 2 और 3

Hide Answer -
उ रः (d)

ा ाः उपयु तीनों 73व सं िवधान सं शोधन अिधिनयम, 1992 म शािमल ह।

73वां सं िवधान सं शोधन, 1992 ि - रीय पं चायती राज व था का ावधान करता है :

1. ाम पं चायत - ाम र पर

2. पं चायत सिमित - म वत र पर

3. िजला पं चायत - िज़ला र पर

पं चायत सिमितः यह म वत र की सिमित है । इसे ख पं चायत/ ॉक पं चायत भी कहते

ह। इसके सद जनता ारा प से चु ने जाते ह। उ र दे श म इसे े पं चायत के नाम

से जानते ह।

[16]पं चायती राज सं था के िवषय े के सं दभ म िवचार कीिजयेः

1. सामािजक वािनकी
2. ाथिमक एवं मा िमक िश ा
3. खादी, ाम एवं कुटीर उ ोग
4. अि शमन से वाएँ

उपयु म से कौन-सा/से कथन सही है /ह?

A) केवल 1

B) केवल 2 और 3

C) केवल 1, 2 और 3

D) 1, 2, 3 और 4
Hide Answer -
उ रः (c)

ा ाः अि शमन से वाएँ 12वीं अनु सूची से सं बंिधत ह, जबिक सामािजक वािनकी, कृिष,

पशु पालन, म उ ोग, ाथिमक एवं मा िमक िश ा, ामीण िवकास, खादी, ाम और कुटीर

उ ोग, बाज़ार एवं मे ले, आिद िवषय 11वीं अनु सूची के अं तगत आत ह।

[17]74व सं िवधान सं शोधन अिधिनयम के ारा सं िवधान का भाग-9(क) जोड़ा गया


है , िजसम नगरपािलका के सं बंध म उपबं ध है । िन िल खत म से कौन-से इसम
शािमल नहीं ह?

1. िजला पं चायत
2. नगर पं चायत
3. नगरपािलका प रषद
4. नगर िनगम

नीचे िदये गए कूट का योग कर सही उ र का चयन कीिजयेः

A) केवल 2 और 3

B) केवल 1, 2 और 3

C) केवल 2, 3 और 4

D) 1, 2, 3 और 4

Hide Answer -
उ रः (c)

ा ाः उपयु म से िज़ला पं चायत नगरपािलका का भाग नहीं है । यह पं चायती राज सं था का

भाग है । े क रा म तीन कार की नगरपािलकाओं की सं रचना का उपबं ध िकया गया है ः

1. नगर पं चायत (Nagar Panchayat)

2. नगरपािलका प रषद (Municipal Council)

3. नगर िनगम (Municipal Corporation)

नगर पं चायतः वह े जो ामीण े से शहरी े म प रवितत हो रहा है ।

नगरपािलका प रषदः छोटे शहरी े ों के िलये ।

नगर िनगमः बड़े शहरी े ों के िलये ।

नगर िनगमः इसम तीन ािधकरण है - प रषद, थायी सिमित एवं आयु ।

प रषदः यह िनगम की िवचारा क एवं िवधायी शाखा है । इसम जनता ारा प से

िनवािचत ‘पाषद’ शाखा है तथा इसम कुछ नािमत सद भी होते ह। नगर प रषद का चु नाव

लड़ने के िलये नगर की मतदाता सू ची म उसका नाम होना चािहये । प रषद का मु ख महापौर

(मे यर) होता है ।

[18]74वां सं िवधान सं शोधन अिधिनयम, 1993 नगरपािलकाओं के िलये ‘वाड सिमित’

का ावधान करता है । एक वाड सिमित के गठन के िलये कम-से -कम िकतनी


जनसं ा का होना आव क है ?
A) एक लाख

B) दो लाख

C) तीन लाख

D) चार लाख

Hide Answer -
उ रः (c)

ा ाः ऐसी नगरपािलका, िजसकी जनसं ा तीन लाख या उससे अिधक है , एक या अिधक

वाड को िमलाकर ादे िशक े के भीतर वाड सिमितयों का गठन िकया जाएगा।

नगरपािलका को सं वैधािनक दज़ा दे ने के िलये 1989 म (राजीव गां धी के कायकाल म)

65वां सं िवधान सं शोधन अिधिनयम पे श िकया गया। लोकसभा म इसे पा रत कर िदया

गया, िकंतु रा सभा म पा रत न हो सका। इस िवधे यक को 1990 म धानमं ी वी.पी.

िसं ह के ने तृ म पु नः पे श िकया गया, ले िकन लोकसभा िवघटन हो जाने से यह िवधे यक

पा रत न हो सका।

[19]िन िल खत म से नगरपािलका का सद माना जा सकता है :

1. ऐसे , िजसे नगरपािलका के शासन का िवशे ष ान अथवा अनु भव


हो।
2. िनवाचन े का ितिनिध करने वाले लोकसभा या रा िवधानसभा के
सद , िजसम नगरपािलका का पू ण या अं शतः े आता है ।
3. रा सभा और िवधान प रषद के सद , जो नगरपािलका े म मतदाता
के प म पं जीकृत ह।

नीचे िदये गए कूट का योग कर सही उ र का चयन कीिजयेः

A) केवल 1 और 2

B) केवल 1 और 3

C) केवल 2 और 3

D) 1, 2 और 3
Hide Answer -
उ रः (d)

ा ाः उपयु सभी कथन सही ह।

सं िवधान के अनु े द-243(R) म नगरपािलकाओं की सं रचना के सं बंध म उपबं ध िकया

गया है । रा िवधानमं डल नगरपािलका म िन िल खत यों के ितिनिध की व था

का उपबं ध कर सकेगी:

1. वह , िजसे नगरपािलका के शासन का िवशे ष ान अथवा अनु भव हो, िकंतु

उसे नगरपािलका की सभा म मत दे ने का अिधकार नहीं होगा।

2. िनवाचन े ों का ितिनिध करने वाले लोकसभा या रा िवधानसभा का सद ,

िजसम नगरपािलका का पू ण या अं शतः े आता है ।

3. रा सभा और िवधान प रषद के सद , जो नगरपािलका े म मतदाता के प म

पं जीकृत हों।

4. सिमित के अ (वाड सिमितयों के अित र ) ।

[20]िन िल खत कथनों पर िवचार कीिजयेः

1. िज़ला योजना सिमित के 2/3 सद िज़ला पं चायत और नगरपािलका के


िनवािचत सद ों ारा यं म से चु ने जाएं गे ।
2. महानगरीय योजना सिमित के 4/5 सद महानगर े म नगरपािलकाओं
के िनवािचत सद ों एवं पं चायतों के अ ों ारा यं म से चु ने जाएं गे ।

उपयु म से कौन-सा/से कथन सही है /ह?

A) केवल 1

B) केवल 2

C) 1 और 2 दोनों

D) न तो 1 और न ही 2
Hide Answer -
उ रः (d)

ा ाः उपयु दोनों कथन गलत ह।

सं िवधान के अनु े द-243DZ एवं अनु े द-243ZE म मशः िज़ला योजना सिमित

और महानगरीय योजना सिमित का उपबं ध है ।

िजला योजना सिमित: इस सिमित के 4/5 सद (न िक 2/3 सद ) िज़ला पं चायत

और नगरपािलका के िनवािचत सद ों ारा यं म से चु ने जाएं गे । सिमित के इन सद ों

की सं ा िज़ले की ामीण एवं शहरी जनसं ा के अनु पात म होनी चािहये । रा का

िवधानमं डल िज़ला योजना सिमितयों की सं रचना, सद ों के िनवाचन का तरीका, िजला

िनयोजन के सं बंध म काय, सिमितयों के अ के िनवाचन का तरीका आिद का उपबं ध

कर सकेगी। इस सिमित का अ िवकास योजनाओं को रा सरकार को े िषत करे गा।

महानगरीय योजना सिमितः इस सिमित के 2/3 सद (न िक 4/5 सद ) महानगर

े म नगरपािलका के िनवािचत सद ों एवं पं चायतों के अ ों ारा यं म से चु ने

जाएग। सिमित के इन सद ों की सं ा उस महानगरीय े म नगरपािलकाओं एवं

पं चायतों की जनसं ा के अनु पात म होनी चािहये । रा िवधानमं डल इन सिमितयों की

सं रचना के सं बंध म उपबं ध कर सकेगा। सिमित का अ िवकास योजना रा सरकार

को भे जेगा।

महानगरीय े का अथ है ः 10 लाख से अिधक जनसं ा वाला े ।


[1] भारत म महानगरीय योजना सिमित के िवषय म िन िल खत कथनों पर िवचार

कीिजयेः

1. महानगरीय े के िलये िवकास योजना का ा प तै यार करना।


2. महानगरीय े म सरकार की ायोिजत योजनाओं को लागू करने का पू ण
दािय ले ना।

उपयु म से कौन-सा/से कथन सही है /ह?

A) केवल 1

B) केवल 2

C) 1 और 2 दोनों

D) न तो 1 और न ही 2

Hide Answer -
उ रः (a)

ा ाः उपयु म से कथन (2) गलत है ।

सं िवधान के अनु े द-243ZE म महानगरीय योजना सिमित के सं बंध म उपबं ध िकया

गया। इसका काय उस महानगरीय े के िलये िवकास का ा प तै यार कर सं बंिधत

सरकार को भे जना है । सिमित का काय सरकार की ायोिजत योजनाओं को लागू करना

नहीं है ।

[2] भारत के रा िव आयोग के िवषय म िन िल खत म से कौन-सा कथन सही


नहीं है ?

भारत के सं िवधान के 74व सं शोधन म नगरपािलका की िव ीय थित के


A)
पु निवलोकन के िलये रा िव आयोग का उपबं ध िकया गया है ।

B) रा का मु मं ी रा िव आयोग के सं गठन का िनणय करता है ।

रा िव आयोग से अपे ि त है िक वह रा के भीतर एवं नगरपािलकाओं


C)
के बीच सं साधनों के बँ टवारे की िसफा रश करे ।

रा िव आयोग रा की सं िचत िनिध से पं चायतों को सहायतानु दान के


D)
िवषय म िसफा रश करने के िलये अिधकृत है ।
Hide Answer -
उ रः (b)

ा ाः सं िवधान के अनु े द-243(I) के अं तगत रा िव आयोग के सं बंध म ावधान िकया

गया है । रा का रा पाल 73व सं िवधान सं शोधन अिधिनयम के ारं भ से 1 वष के अं दर और

े क पाँ च वष की समा पर िव आयोग का गठन करे गा। रा िव आयोग सं िवधान के

अनु े द-243(I) और अनु े द-234(Y) के तहत पं चायतों एवं नगरपािलकाओं के सं बंध म

रा पाल को िसफा रश करे गा और रा पाल उसकी िसफा रशों को रा िवधानमं डल केा ुत

करे गा। क ीय िव आयोग भी पं चायतों एवं नगरपािलकाओं के िलये रा की सं िचत िनिध म

वृ के िलये उपायों के बारे म सलाह दे ता है ।

[3] िन िल खत म से िकस सिमित/आयोग के ारा थानीय िव आयोग के िनमाण के


िवषय म सु झाव िदया गया?

A) बलव राय मे हता सिमित

B) अशोक मे हता सिमित

C) शासिनक सु धार आयोग

D) सरका रया आयोग

Hide Answer -
उ रः (b)

ा ाः उपयु म से िवक (b) सही है ।

अशोक मे हता सिमित का गठन िदसं बर 1977 म िकया गया तथा अग 1978 म इसने

अपनी रपोट सौंपी। इसने अपनी रपोट म िव आयोग की िसफा रश की थी, िजसे 73व

सं िवधान सं शोधन अिधिनयम, 1992 के ारा सं वैधािनक दज़ा दान िकया गया।

[4] सू ची-I को सू ची-II से सु मेिलत कीिजयेः

सू ची-I सू ची-II

A. सरपं च 1. ू िनिसपल कॉप रे शन

B. चे यरमै न 2. ाम पं चायत

C. मे यर 3. ु िनिसपिलटी

D. काउ लर 4. वाड

नीचे िदये गए कूट का योग कर सही उ र का चयन कीिजयेः


A B C D
A) 2 1 3 4

B) 2 3 1 4

C) 4 3 1 2

D) 4 1 3 2

Hide Answer -
उ रः (a)

ा ाः उपयु म से िवक (a) सही है ।

सरपं च - ाम पं चायत

चे यरमै न - ु िनिसपिलटी/नगरपािलका

मे यर/महापौर - नगर िनगम (Municipal Corporation)

कां उ लर - वाड

[5] िन िल खत म से कौन एक थानीय सरकार के काय म नहीं आता है ?

A) लोक ा

B) सफाई

C) सावजिनक उपयोग की से वाएँ

D) सावजिनक व था बनाए रखना

Hide Answer -
उ रः (d)

ा ाः थानीय सरकार का काय सावजिनक व था को बनाए रखना नहीं है । सावजिनक

व था बनाए रखने का काय रा सरकार के पु िलस एवं िसिवल शासन के ारा िकया जाता

है ।

[6] रा िनवाचन आयोग के काय पर िवचार कीिजयेः

1. रा की पं चायतों एवं नगरपािलकाओं के चु नावों के िलये मतदाता सू ची


तै यार करता है ।
2. रा की पं चायतों एवं नगरपािलकाओं के िलये िनवाचन करवाता है ।
3. रा पाल के िनदशनों के अनु प पं चायतों एवं नगरपािलकाओं के
अित र अ िनकायों के िलये िनवाचन करवाता है ।
4. भारत के िनवाचन आयोग ारा सौंपे गए मतदाता सू ची को तै यार करने एवं
िनवाचन करवाने का काय करता है ।

उपयु कथनों म से कौन-से कथन सही ह?


A) केवल 1 और 2

B) केवल 1, 2 और 3

C) केवल 1, 3 और 4

D) केवल 2, 3 और 4

Hide Answer -
उ रः (a)

ा ाः उपयु म से कथन (3) और (4) गलत ह।

73व एवं 74व सं िवधान सं शोधन अिधिनयम, 1992 एवं 1993 के ारा सं िवधान म

भाग-9 एवं 9(क) को जोड़ा गया। भाग-9 म पं चायत एवं भाग-9(क) म नगरपािलका का

उपबं ध िकया गया। इसके साथ ही सं िवधान म 11वीं एवं 12वीं अनु सूची जोड़ी गई, िजसम

मशः 29 एवं 18 िवषयों को शािमल िकया गया। इन सं शोधनों के ारा पं चायती राज

सं था एवं नगरपािलका को सं वैधािनक दज़ा दान िकया गया।

इस सं िवधान सं शोधन के ारा पं चायतों एवं नगरपािलकाओं के िनवाचन के िलये रा

िनवाचन आयोग का उपबं ध िकया गया। रा िनवाचन आयोग म एक िनवाचन आयु

होगा, िजसकी िनयु रा पाल ारा की जाएगी।

सं िवधान के अनु े द-243(k) म उपबं ध है िक पं चायतों के िनवाचन की िनयमावली तै यार

कराने एवं उनके सभी िनवाचनों के िलये सं चालन का अधी ण, िनदशन एवं िनयं ण एक

रा िनवाचन आयोग म होगा। इसी तरह सं िवधान के अनु े द-243(ZA) म उपबं ध है

िक नगरपािलकाओं के िलये िनवाचक िनयमावली तै यार कराने एवं उन सभी िनवाचनों के

िलये सं चालन का अधी ण, िनदशन एवं िनयं ण एक रा िनवाचन आयोग म होगा।

[7] पं चायत (अनु सूिचत े ों तक िव ार) अिधिनयम (PESA), 1996 का उ े


नहीं है ः

A) जनजातीय समु दाय को शासन दान करना।

B) जनजातीय समु दायों की पर राओं एवं रवाजों की सु र ा तथा सं र ण करना।

C) जनजाितयों लोगों को शोषण से मु करना।

D) जनजातीय े ों म ाय े ों का िनमाण करना।


Hide Answer -
उ रः (d)

ा ाः जनजातीय े ों म ाय े ों का िनमाण करने के सं बंध म इस अिधिनयम म कोई

ावधान नहीं है ।

सं िवधान के भाग-9 से सं बंिधत ावधान पां चवी अनु सूची म विणत े ों पर लागू नहीं होता,

िकंतु सं सद इन ावधानों को कुछ अपवादों तथा सं शोधनों सिहत इन े ों पर लागू कर

सकती है । इस ावधान के तहत सं सद ने पं चायत (अनु सूिचत े ों तक िव ार)

अिधिनयम, 1996 (PESA, Act, 1996) पा रत िकया है ।

पे सा कानू न के िन िल खत उ े ह-

⇒ जनजातीय समु दाय को शासन दान करना।

⇒ उसके पार रक अिधकारों को मा ता दे ना।

⇒ जनजातीय समु दाय के लोगों को शोषण से मु करना।

⇒ सहयोगी लोकतं के तहत ाम शासन थािपत करना।

[8] पं चायत (अनु सूिचत े ों म िव ार) अिधिनयम, 1996 (पे सा ए , 1998) के


अं तगत ाम सभा की श यों/भू िमकाओं के िवषय म िन िल खत म से कौन-सा
कथन सही नहीं है ?

A) ाम सभा के पास छोटे र पर वन उपज का ािम होगा।

ाम सभा के पास अनु सूिचत े ों म भू िम का ह ानां तरण रोकने की श


B)
होगी।

अनु सूिचत े ों म िकसी भी खिनज के िलये खनन का प ा अथवा लाइसस


C)
दान करने के िलये ाम सभा की अनु शंसा आव क है ।

D) ाम सभा के पास ामीण हाट-बाज़ारों के बं धन की श होगी।


Hide Answer -
उ रः (c)

ा ाः सं सद ारा पे सा कानू न वष 1996 म पा रत िकया गया तथा 24 िदसं बर, 1996 को

रा पित ारा इसे ीकृित िमली। इसके अं तगत ाम सभा को िन िल खत श याँ ा ईः

अनु सूिचत े ों म छोटे र पर (न िक िकसी भी खिनज के िलये ) खिनजों का खनन

सं बंधी लाइसस अथवा खनन प ा ा करने के िलये ाम सभा की अनु शंसा ा करना

अिनवाय है ।

ाम सभा के पास छोटे र पर वन उपज का ािम होगा।

ाम सभा के पास ामीण हाट-बाज़ारों के बं धन की श होगी।

ाम सभा को िकसी नशीले पदाथ की िब ी या उपयोग को रोकने या िनयिमत करने या

ितबं िधत करने की श होगी।

ाम सभा को अनु सूिचत े ों म भू िम से अलगाव को रोकने की श होगी। साथ ही गै र-

कानू नी तरीके से बे दखली के प ात् भू िम ा करने के िलये आव क कारवाई का

अिधकार होगा।

ेक ाम सभा अपने लोगों की परं पराओं एवं थाओं, उसकी सां ृ ितक पहचान,

सामु दाियक सं साधन तथा िववाद िनवारण के परं परागत तरीकों की सु र ा एवं सं र ण के

िलये स म होगी।

ाम सभा सामािजक एवं आिथक िवकास के काय मों एवं प रयोजनाओं को ीकृित दे गी

तथा गरीबी उ ू लन एवं काय मों के लाभािथयों की पहचान के िलये िज़ े दार होगी। इन

योजनाओं एवं काय मों के िलये े क पं चायत िनिध के उपयोग सं बंधी माण-प ाम

सभा से ा करे गी।

[9] पं चायत (अनु सूिचत े ों म िव ार) अिधिनयम, 1996 की मख िवशे षताओं म


से कौन-सी एक सही नहीं है ?

अनु सूिचत े ों म पं चायतों पर रा का िवधान वहाँ के थागत कानू न,


A)
सामािजक एवं धािमक चलन अथवा प रपािटयों के अनु प होगा।

े क पं चायत म अनु सूिचत े ों म सीटों का आर ण उन समु दायों की


B) जनसं ा के अनु पात म होगा, िकंतु अनु सूिचत जनजाितयों के िलये आर ण
कुल सीटों के एक-ितहाई से कम नहीं होगा।

िजन अनु सूिचत जनजाितयों का ितिनिध म वत या िज़ला र की पं चायत


म नहीं है , उ सरकार ारा नािमत िकया जाएगा, िकंतु नािमत सद ों की
C)
सं ा पं चायत म िनवािचत कुल सद सं ा के 1/10व भाग से अिधक नहीं
होगी।

अनु सूिचत े ों म िज़ला र पर शासकीय व था बनाते समय रा


D)
िवधाियका सं िवधान की छठी अनु सूची का अनु सरण करे गी।
Hide Answer -
उ रः (b)

ा ाः े क पं चायत म अनु सुिचत े ों म सीटों का आर ण उस समु दाय की जनसं ा के

अनु पात म होगा, िकंतु अनु सूिचत जनजाितयों के िलये आर ण कुल सीटों के 1/2 से कम (न िक

1/3 से कम) नहीं होगी। इसके अित र पं चायत के ेक र पर अ की सभी सीट

अनु सूिचत जनजाितयों के िलये आरि त होंगी।

इस कानू न की एक मु ख िवशे षता है िक अनु सूिचत े ों म लघु जल ोतों के िलये

आयोजन एवं बं धन की िज़ े दारी उपयु र की पं चायतों को दी जाएगी।

[10]भारत के रा -ज ू -क ीर के िवषय म िन िल खत म से कौन -सा कथन सही


नहीं ह?

ज ू -क ीर रा भारतीय सं घ का एक सं वैधािनक रा है तथा इसकी


A)
सीमाएं भारतीय सीमाओं का एक भाग है ।

B) सं िवधान के भाग-1 तथा अनु सूची-1 म इसके सं बंध म उपबं ध िकया गया है ।

सं िवधान के भाग-19 के अनु े द-370 म ज ू -क ीर के िलये िवशे ष


C)
उपबं ध िकया गया है , िकंतु यह उपबं ध अ थायी है ।

D) सं िवधान के भाग-6 का उपबं ध ज ू -क ीर रा पर लागू नहीं होता है ।

Hide Answer -
उ रः (c)

ा ाः

सं िवधान के भाग-21 (न िक भाग-19) के अनु े द-370 म ज ू एवं क ीर के िलये

िवशे ष उपबं ध का ावधान है ,िकंतु यह उपबं ध थायी नहीं है ।

ज ू एवं क ीर भारतीय सं घ का एक सं वैधािनक रा है तथा इसकी सीमाएं भारतीय

सीमाओं का एक भाग है । भारतीय सं िवधान के भाग-1 के अनु े द-1 तथा अनु सूची-1 म

इसे रखा गया है । रा पु नगठन अिधिनयम, 1956 से पहले इसे रा ों के ‘ख’ े णी म

रखा गया था तथा रा पु नगठन अिधिनयम, 1956 के ारा गिठत 14 रा ों म से एक

रा ज ू एवं क ीर भी है ।

सं िवधान के भाग-6 का उपबं ध ज ू -क ीर रा पर लागू नहीं होता है , ोंिक सं िवधान

के अनु े द-370 के तहत इसके िलये अलग सं िवधान बनाया गया है और वहाँ की शासन

व था उसी के अनु सार चलाई जाती है ।

[11]ज ू -क ीर रा के िवषय म िन िल खत म से कौन-सा कथन सही नहीं है ?


रा के नीित-िनदे शक त ों से सं बंिधत ावधान इस रा पर लागू नहीं होते
A)
ह।

B) मू ल क ों से सं बंिधत उपबं ध इस रा पर लागू नहीं होते ह।

C) मू ल अिधकारों से सं बंिधत उपबं ध इस रा पर लागू नहीं होते ह।

D) रा पित ारा घोिषत िव ीय आपातकाल इस रा पर लागू नहीं होता है ।

Hide Answer -
उ रः (c)

ा ाः

सं िवधान के भाग-3 म मू ल अिधकारों के सं बंध म ावधान ह। कुछ शत एवं अपवादों के

साथ ज ू -क ीर रा पर लागू होते ह।

सं िवधान के भाग-4 म रा के नीित-िनदे शक त ों के सं बंध ावधान ह। इसके उपबं ध

इस रा पर लागू नहीं होते ।

सं िवधान के भाग-4(क) म मू ल क ों के सं बंध म ावधान िकया गया है । इसके ावधान

इस रा पर लागू नहीं होते ।

सं िवधान के अनु े द-360 के तहत रा पित िव ीय आपातकाल की उद् घोषणा करता है ।

इसका उपबं ध इस रा पर लागू नहीं होता।

भारत का रा पित केवल अनु े द-352 के तहत यु या बा आ मण के आधार पर ही

अ रा ों के भाँ ित ज ू -क ीर रा म भी सीधे रा ीय आपातकाल की उद् घोषणा कर

सकता है ।

पाँ चवीं अनु सूची एवं छठी अनु सूची से सं बंिधत उपबं ध इस रा पर लागू नहीं होते ह।

अं तरा ीय सं िध/सहमित को रा िवधान मं डल की सहमित के िबना इस रा म लागू नहीं

कर सकता।

भारत के सं िवधान म िकसी कार का सं शोधन इस रा पर लागू नहीं होता, जब तक

रा पित के आदे श के ारा िव ा रत न हो।

[12]ज ू -क ीर रा के िवषय म िन िल खत कथनों पर िवचार कीिजयेः

1. भारत के उ तम ायालय की िवशे ष इज़ाज़त की अिधका रता तथा चु नाव


आयोग, िनयं क एवं महाले खा परी क का अिधकार े इस रा पर
लागू नहीं होता।
2. ज ू -क ीर रा के उ ायालय को भारत के अ उ ायालयों
के समान मू ल अिधकारों के साथ-साथ अ े ों म भी रट िनकालने की
श है ।

उपयु म से कौन-सा/से कथन सही है /ह?


A) केवल 1

B) केवल 2

C) 1 और 2 दोनों

D) न तो 1 और न ही 2

Hide Answer -
उ रः (d)

ा ाः उपयु दोनों कथन गलत ह।

उ ायालय का िवशे ष अिधकार तथा चु नाव आयोग और िनयं क एवं महाले खा परी क

का अिधकार े इस रा पर लागू होता है ।

ज ू -क ीर का उ ायालय केवल मू ल अिधकारों के िवषय म रट िनकाल सकता है ।

िनयोजन, स ि के अजन और िनवास के िवशे ष अिधकार रा के केवल थायी

िनवािसयों को दान िकया गया है ।

सं िवधान आदे श, 1986 के ारा अनु े द-249 का िव ार ज ू -क ीर रा पर कर

िदया गया है ।

[13]िन िल खत कथनों पर िवचार कीिजयेः

1. सं िवधान के अनु े द-365 के अं तगत सं घ केा यह श नहीं है िक


उसके ारा िदये गए िनदशों को न मानने की थित म ज ू -क ीर की
िवधानसभा को िनलं िबत/िवघिटत कर दे ।
2. ज ू -क ीर रा म रा पित शासन के साथ-साथ रा पाल शासन का
उपबं ध है ।
3. ज ू -क ीर रा म रा पाल शासन तो लागू िकया है , िकंतु रा पित
शासन का योग अभी तक एक बार भी नहीं आ है ।

उपयु म से कौन-सा/से कथन सही है /ह?

A) केवल 1

B) केवल 1 और 2

C) केवल 2 और 3

D) 1, 2 और 3
Hide Answer -
उ रः (b)

ा ाः उपयु म से कथन (3) गलत है ।

ज ू -क ीर रा म रा पित शासन एवं रा पाल शासन दोनों का उपबं ध है । रा के

सं िवधान के अनु े द-92 के तहत रा पाल शासन एवं सं िवधान के अनु े द-356 के

तहत रा पित शासन लगाया जाता है । ज ू -क ीर रा म पहली बार रा पित शासन 7

िसतं बर, 1986 को लगाया गया था, जबिक उससे पहले रा म रा पाल शासन लगा

था।

सं सद, सं िवधान के अनु े द-3 के तहत ज ू -क ीर रा के नाम या रा े म

प रवतन रा िवधानमं डल के सहमित के िबना नहीं कर सकती।

ज ू -क ीर के सं बंध म अविश श याँ रा के िवधानमं डल के पास ह, न िक अ

रा की भां ित सं सद के पास। केवल कुछ दशाओं म सं सद को यह श ा है , जै से

आतं कवादी गितिविध, भारत के रा े की अखं डता एवं सं भु ता का , रा ीय झं डे,

रा गान और सं िवधान के स ान के सं बंध म। सं सद ारा िनवारक िनरोध सं बंधी बनाई गई

िविधयाँ इस रा पर लागू नहीं होती ह।

[14]िन िल खत म से िकस ितिथ को ज ू -क ीर रा का सं िवधान लागू आ?

A) 26 जनवरी, 1950

B) 26 जनवरी, 1956

C) 26 जनवरी, 1957

D) 26 जनवरी, 1965

Hide Answer -
उ रः (c)

ा ाः उपयु म से िवक (c) सही है ।

ज ू -क ीर म सं िवधान के अनु े द-370 के तहत एक अलग सं िवधान का गठन िकया

गया। 17 नवं बर, 1956 को रा के सं िवधान को अं गीकार िकया गया एवं 26 जनवरी,

1957 को सं िवधान भावी हो गया।

भारत के सं िवधान म िकसी कार का सं शोधन ज ू -क ीर पर लागू नहीं होता, जब तक

िक रा पित के आदे श के ारा िव ार न िकया जाए।

सं शोधन के अनु े द-356-57 का िव ार ज ू -क ीर रा पर सं शोधन आदे श,

1964 के ारा िकया गया है ।

‘उदू ’ रा की आधा रक भाषा घोिषत की गई है ।

6व सं िवधान सं शोधन अिधिनयम, 1965 के ारा कायपािलका के धान सदर-ए- रयासत

का नाम बदलकर रा पाल कर िदया गया एवं सरकार का धान वज़ीर-ए-आज़म का

नाम बदलकर मु मं ी कर िदया गया और उनकी िनयु भारत के रा पित ारा की

जाती है ।
[15]िन िल खत कथनों पर िवचार कीिजयेः

1. रा िवधानसभा के िलये ू नतम सद सं ा 60 िनधा रत की गई है ,


जबिक ज ू -क ीर रा म िवधानसभा की कुल सद सं ा इससे
अिधक है ।
2. रा िवधान प रषद की ू नतम सद सं ा 40 िनधा रत की गई है ,
जबिक ज ू -क ीर रा म िवधान प रषद की कुल सं द सं ा इससे
कम है ।

उपयु म से कौन-सा/से कथन सही है /ह?

A) केवल 1

B) केवल 2

C) 1 और 2 दोनों

D) न तो 1 और न ही 2

Hide Answer -
उ रः (c)

ा ाः उपयु दोनों कथन सही ह।

रा िवधानसभा के िलये अिधकतम 500 एवं ू नतम 60 िनधा रत की गई है , जबिक

ज ू -क ीर रा म वतमान म िवधानसभा की कुल सद सं ा 111 है । रा के मू ल

सं िवधान म 100 थी। ज ू -क ीर ए , 1988 (20व सं िवधान सं शोधन) के ारा

इसकी सं ा म वृ की गई।

रा िवधान प रषद की कुल सद सं ा उस रा िवधानसभा के सद ों की कुल

सं ा का 1/3 से अिधक नहीं होगी, परं तु इसकी ू नतम सद सं ा 40 िनधा रत की

गई है । ज ू -क ीर रा म इसकी कुल सद सं ा-36 है ।

[16]वष 2010 म ज ू -क ीर के िलये भारत सरकार ारा एक वातालाप समू ह का


गठन िकया गया। इस वातालाप समू ह का अ िन िल खत म से कौन था?

A) िदलीप प ावकर

B) राधा कुमार

C) एम.एन. अं सारी

D) ज स जे .एस. वमा
Hide Answer -
उ रः (a)

ा ाः उपयु म से िवक (a) सही है ।

अ ू बर 2010 म ज ू -क ीर के िलये भारत सरकार ने प कार िदलीप प ावकर की

अ ता म वातालाप समू ह का गठन िकया गया। इसके सद थे - राधा कुमार एवं

एम.एन.अं सारी। इसने अपनी रपोट अ ू बर 2011 म क ीय गृ हमं ी को सौंपी।

[17]सं िवधान के भाग-21 म कुछ रा ों के िलये अ थायी, सं मणकालीन और िवशे ष


ावधान िकया गया है । इस िवषय म िन िल खत रा ों पर िवचार कीिजयेः

1. म दे श एवं छ ीसगढ
2. महारा एवं गु जरात
3. आं दे श
4. कनाटक
5. गोवा

उपयु म से कौन-से रा ों को सं िवधान के इस भाग म शािमल िकया गया है ?

A) केवल 1, 2, 3 और 4

B) केवल 1, 2, 3 और 5

C) केवल 1, 3, 4 और 5

D) केवल 2, 3, 4 और 5
Hide Answer -
उ रः (d)

ा ाः उपयु रा ों म से म दे श एवं छ ीसगढ को सं िवधान के भाग-21 म शािमल नहीं

िकया गया।

सं िवधान के भाग-21 म िजन रा ों के सं बंध म ावधान िकया गया है , वे िन ह-

⇒ ज ू एवं क ीर- अनु े द-370

⇒ महारा एवं गु जरात- अनु े द-371

⇒ नागालै - अनु े द-371(क)

⇒ असम- अनु े द-371(ख)

⇒ मिणपु र- अनु े द -371(ग)

⇒ आं दे श- अनु े द-371(घ) । आं दे श पु नगठन अिधिनयम, 2014 के ारा

इसका िव ार करके ते लंगाना रा की थापना की गई।

⇒ आं दे श म के ीय िव िव ालय की थापना- अनु े द-371(ड)

⇒ िस म- अनु े द-371(च)

⇒ िमज़ोरम- अनु े द-371(छ)

⇒ अ णाचल दे श- अनु े द-371(ज)

⇒ गोवा- अनु े द-371(झ)

⇒ कनाटक- अनु े द-371( )

98व सं िवधान सं शोधन अिधिनयम, 2012 के ारा अनु े द-371( ) जोड़ा गया। इसके

ारा कनाटक के रा पाल को है दराबाद-कनाटक े के िवकस के िलये श दान की

गई।

सं िवधान के भाग-21 म ज ू और क ीर रा के िलये ‘अ थाई उपबं ध’ श का योग

िकया है ।

[18]भारत म रा ीय दलों की मा ता के सं दभ म िन िल खत म से कौन-सा कथन

सही नहीं है ?

यिद कोई राजनीितक दल लोकसभा अथवा िवधानसभा के आम चु नावों म चार


A) अथवा अिधक रा ों म वै ध मतों का 6% मत ा करता है तथा इसके साथ
िकसी रा या रा ों म लोकसभा म चार सीट ा करता है ।

यिद कोई राजनीितक दल लोकसभा अथवा िवधानसभा के आम चु नावों म वै ध


B) मतों का 6% मत ा करता है तथा इसके साथ वह िकसी रा या रा ों
म लोकसभा या िवधानसभा म चार सीट ा करता है ।

यिद कोई राजनीितक दल कम-से -कम चार रा ों म रा रीय दल के प


C)
म मा ता ा है ।

यिद कोई राजनीितक दल कम-से -कम तीन िविभ रा ों से लोकसभा म


D)
2% थान जीतता है ।
Hide Answer -
उ रः (b)

ा ाः

कोई राजनीितक दल रा ीय दल की मा ता तभी ा कर सकता है जब तीनों म से कोई

एक िनयम का पालन करता है :

यिद लोकसभा या िवधानसभा चु नावों म चार अथवा अिधक रा ों म वै ध मतों का 6% मत

ा करता है तथा उसके साथ वह िकसी रा या रा ों म लोकसभा म 4 सीट ा

करता है ; या

कोई राजनीितक दल कम-से -कम चार रा ों म रा रीय दल के प म मा ता ा

हो; या

कोई राजनीितक दल कम-से -कम 3 िविभ रा ों से लोकसभा म 2% थान जीतता है ।

[19]भारत म िन िल खत म से िकस एक राजनीितक दल को रा ीय दल के प म


मा ता ा नहीं है ?

A) रा वादी कां े स पाट

B) ऑल इं िडया तृ णमू ल कां े स

C) रा ीय जनता दल

D) ब जन समाज पाट

Hide Answer -
उ रः (c)

ा ाः रा ीय जनता दल को वतमान म रा ीय राजनीितक दल के प म मा ता ा नहीं

है ।

वतमान म 7 राजनीितक दलों को रा ीय दल के प म मा ता ा है -

1. भारतीय रा ीय कां े स

2. भारतीय जनता पाट

3. भारतीय क ु िन पाट

4. भारतीय क ु िन पाट (मा वादी)

5. ब जन समाज पाट

6. रा वादी कां े स पाट

7. ऑल इं िडया तृ णमू ल कां े स

वतमान म ऑल इं िडया तृ णमू ल कां े स को चु नाव आयोग ारा रा ीय राजनीितक दल के

प म मा ता दान की गई है , ोंिक इसे चार रा ों-पि म बं गाल, मिणपु र, ि पु रा

और अ णाचल दे श म मा ता ा है । रा ीय राजनीितक दल बनने के िलये तीन

िनयमों म से एक िनयम यह भी है िक िकसी राजनीितक दल को कम-से -कम चार रा ों

म रा रीय दल के प म मा ता ा है तो वह रा ीय दल की मा ता ा कर

सकता है ।
[20]िकसी राजनीितक दल को रा रीय/ े ीय दल के प म मा ता दी जाती है ,
यिदः

वह रा म या तो लोकसभा या िवधानसभा चु नाव म कुल वै ध मतों का चार


A)
ितशत ा करता है ।

वह रा म या तो लोकसभा या िवधानसभा चु नाव म कुल वै ध मतों का 6%


B)
मत ा करता है ।

वह रा म या तो लोकसभा या िवधानसभा चु नाव म कुल वै ध मतों का 8%


C)
मत ा करता है ।

D) उपयु म से कोई नहीं।

Hide Answer -
उ रः (c)

ा ाः उपयु म से िवक (c) सही है ।

िकसी राजनीितक दल को रा रीय/ े ीय दल के प म मा ता ा करने के िलये

िन िल खत िनयमों को पू रा करना आव क है :

⇒ रा िवधानसभा चु नाव म कुल वै ध मतों का 6% मत ा करना तथा इसके

अित र 2 थान ा करना; या

⇒ रा म ए लोकसभा चु नाव म कुल वै ध मतों का 6% ा करना और लोकसभा की

कम-से -कम एक सीट जीतना; या

⇒ रा िवधानसभा के कुल थानों का 3% या 3 सीट जो भी ादा हो ा करना या

⇒ लोकसभा या िवधानसभा चु नाव म कुल वै ध मतों का 8% मत ा करता है । यह शत

वष 2011 म जोड़ी गई है ।
[1] िन िल खत म से कौन-सा कथन सही नहीं है ?

52व सं िवधान सं शोधन अिधिनयम के ारा भारत म पहली बार राजनीितक


A)
दल को सं वैधािनक मा ता दान की गई।

ज ू -क ीर रा की िवधानसभा ारा 52व सं िवधान सं शोधन अिधिनयम से


B)
पहले ही दल-बदल कानू न पा रत कर िदया गया।

िकसी राजनीितक दल के एक-ितहाई सद अलग होकर िकसी दू सरे दल म


C)
शािमल हो जाते ह तो उन पर दल-बदल कानू न लागू नहीं होता।

कोई सं सद सद पीठासीन अिधकारी चु न िलया जाता है और अपने दल की


D) सद ता से े ा से अलग हो जाता है तो उस पर दल-बदल कानू न नहीं
लागू होगा।

Hide Answer -
उ रः (c)

ा ाः

52व सं िवधान सं शोधन अिधिनयम म ावधान है िक िकसी राजनीितक दल के 1/3 सद

े ा से अलग होकर िकसी दू सरे दल म शािमल हो जाते ह या अलग राजनीितक दल का

गठन कर ले ते ह तो उन पर दलबदल कानू न लागू नहीं होगा, िकंतु 91व सं िवधान सं शोधन

अिधिनयम, 2003 के ारा इस ावधान को हटा िदया गया है अथात् अब इस पर भी

दलबदल कानू न लागू होगा। इस अिधिनयम के ारा बदलबदल कानू न को और अिधक

सश बनाया गया।

यिद कोई सं सद सद पीठासीन अिधकारी चु न िलया जाता है एवं वह े ा से अपनी

राजनीितक दल की सद ता छोड़ दे ता है और कायकाल के प ात् पु नः अपने राजनीितक

दल म शािमल हो जाता है तो उस पर दलबदल कानू न लागू नहीं होगा।

52व सं िवधान सं शोधन अिधिनयम, 1985 के ारा पहली बार राजनीितक दल को

सं वैधािनक मा ता दान की गई।

ज ू -क ीर रा की िवधानसभा ारा वष 1979 म दलबदल को रोकने के िलये एक

िवधे यक पा रत िकया गया।

अ /सभापित के ारा दल प रवतन को सं ान म तभी िलया जाता है , जब सदन के

िकसी सद के ारा उसे िशकायत ा हो। अ या सभापित के ारा अं ितम िनणय

ले ने से पू व उस सद को अपना प रखने का मौका दे ना अिनवाय है ।

[2] िन िल खत म से िकस सिमित का सं बंध भारत म चु नाव सु धारों से नहीं रहा है ?

A) तारकुंडे सिमित

B) राजम ार सिमित

C) िदने श गो ामी सिमित

D) इं जीत गु ा सिमित
Hide Answer -
उ रः (b)

ा ाः राजम ार सिमित का सं बंध क -रा सं बंध से रहा है , िजसका गठन 1969 म

तिमलनाडु सरकार के ारा िकया गया था।

िन िल खत सिमितयों का सं बंध चु नाव सु धार से है -

1. तारकंु डे सिमितः इसका गठन वष 1974 म जय काश नारायण ारा िकया गया था।

यह एक गै र-सरकारी सिमित थी। इसने अपनी रपोट वष 1975 म दी थी। इसे ‘जे .पी.

सिमित’ के नाम से भी जाना जाता है ।

2. िदने श गो ामी सिमितः इसका गठन वी.पी. िसं ह के ने तृ वाली रा ीय मोचा की

सरकार ने वष 1990 म िकया था। इसने अपनी रपोट 1990 म ु त की तथा वष

1996 म इसकी कुछ अनु शंसाओं को लागू िकया गया।

3. वोहरा सिमितः इसका गठन अपराध और राजनीित के बीच साँ ठगाँ ठ की जाँ च के िलये

वष 1993 म की गई थी।

4. इं जीत गु ा सिमितः इस सिमित का गठन वष 1998 म चु नावी खच के सं बंध म

िकया गया था।

5. तनखा सिमित/कोर सिमितः वष 2010 म चु नाव कानू न एवं सु धारों से जु ड़े मामलों के

िलये इस सिमित का गठन िकया गया।

[3] जन ितिनिध (सं शोधन) अिधिनयम, 1996 के िवषय म िन िल खत म से कौन-


सा कथन सही नहीं है ?

रा ीय ज अथवा सं िवधान के अपमान के अपराध म दोषिस होने पर


A) दोषिस की ितिथ से छः वष के िलये सं सद और रा िवधानमं डलों के
चु नाव लड़ने के िलये अयो होगा।

लोकसभा के िलये चु नाव लड़ने हे तु अ थ ारा जमा िकये जाने वाले ितभू ित
B)
िन े प (ज़मानत रािश) म वृ की गई।

कोई अ थ अब एक से अिधक सं सदीय िनवाचन े ों से िनवाचन के िलये


C)
खड़ा नहीं हो सकता।

चु नाव लड़ने वाले िकसी अ थ की मृ ु हो जाने पर अब िकसी िनवाचन को


D)
र नहीं िकया जा सकता।
Hide Answer -
उ रः (c)

ा ाः उपयु म से िवक (c) गलत है ।

जन ितिनिध (सं शोधन) अिधिनयम, 1996 िदने श गो ामी सिमित की िसफा रश पर

आधा रत है । इस अिधिनयम म उपबं ध िकया गया है िक एक साथ हो रहे आम चु नाव या

उपचु नाव म कोई उ ीदवार लोकसभा या िवधानसभा की दो से अिधक सीटों से चु नाव

नहीं लड़ सकता। ऐसा ितबं ध रा सभा और रा ों के िवधान प रषद के ि वािषक या

उप-चु नाव पर भी लागू होता है ।

रा ीय गौरव अपमान िनरोधक अिधिनयम, 1971 के तहत जन ितिनिध (सं शोधन)

अिधिनयम, 1996 म ावधान िकया गया िक भारतीय रा ीय ज भारत के सं िवधान का

अनादर एवं रा गान गाने से रोकने सं बंधी अपराध के िलये अ थ दोषिस होने की ितिथ

से छः वष तक सं सद और रा िवधानमं डलों के चु नाव लड़ने के िलये अयो होगा।

इस सं शोधन अिधिनयम के ारा लोकसभा चु नाव लड़ने हे तु अ थ ारा ज़मानत रािश/

ितभू ित िन े प म वृ की गई।

इस अिधिनयम म ावधान िकया गया िक चु नाव लड़ने वाले िकसी अ थ की मृ ु हो

जाती है , तब भी चु नाव र नहीं होंगे। हालाँ िक, मा ता ा राजनीितक दल के अ थ

का िनधन होने पर उस दल को सात िदन के अं दर दू सरा उ ीदवार दे ने का िवक

उपल है ।

अिधिनयम म ावधान िकया गया िक मतदान समा होने की अविध के 48 घं टे पहले तक

मतदान क के इलाके म शराब व नशीला पे य नहीं बे चा या बाँ टा जा सकता है । इसका

उ ं घन करने पर छः माह की कैद या 2000 पए जु माना या दोनों हो सकते ह।

िकसी मतदान क के आसपास िकसी तरह के हिथयार के साथ जाना सं े य अपराध होगा

और ऐसा करने पर दो वष की सज़ा या जु माना या दोनों हो सकते ह।

नामां कन वापस ले ने की आ खरी ितिथ एवं मतदान की ितिथ के बीच का ू नतम अं तराल

20 िदनों से घटाकर 14 िदन कर िदया गया है ।

सं सद या रा िवधानमं डल के उप-चु नाव छः माह के अं दर कराने होंग, िकंतु ये ावधान

िन थितयों म लागू नहीं होगा;

1. यिद उसका कायकाल एक वष से कम अविध के िलये बचा हो, या

2. चु नाव आयोग क सरकार की सलाह के प ात् यह स ािपत कर दे िक िनधा रत

अविध के अं दर चु नाव कराना किठन है ।

[4] भारतीय चु नाव के िवषय म िन िल खत कथनों पर िवचार कीिजयेः

1. रा पित चु नाव के नामां कन के िलये कम-से -कम 50 ावक और 50


अनु मोदक होना आव क है ।
2. उप-रा पित के चु नाव के नामां कन के िलये कम-से -कम 20 ावक
और 20 अनु मोदक होना आव क है ।
3. रा पित या उप-रा पित का चु नाव लड़ने के िलये जमानत रािश के प म
15000 पए भारतीय रज़व बक म रखना पड़ता है ।

उपयु म से कौन-सा/से कथन सही है /ह?


A) केवल 1

B) केवल 1 और 3

C) केवल 2 और 3

D) 1, 2 और 3

Hide Answer -
उ रः (d)

ा ाः उपयु तीनों कथन सही ह।

रा पित एवं उप-रा पित िनवाचन अिधिनयम, 1952 को वष 1997 म सं शोधन िकया

गया। इसम रा पित एवं उप-रा पित के चु नाव लड़ने के िलये ावकों एवं अनु मोदकों

की सं ा म वृ की गई। रा पित के िलये 10 से बढ़ाकर 50 एवं उप-रा पित के िलए

5 से बढ़ाकर 20 कर िदये गए। रा पित या उप-रा पित का चु नाव लड़ने वाले उ ीदवार

की जमानत रािश 2500 पए से बढ़ाकर 15000 पए कर दी गई, िजसे भारतीय रज़व

बक म जमा करना पड़ता है ।

[5] िन िल खत कथनों पर िवचार कीिजये -

1. भारत का चु नाव आयोग सरकार से परामश करके िकसी भी े णी के


के िलये डाक मतप के तहत मत डालने के िलये अिधसू िचत कर
सकता है ।
2. सश से ना म कायरत मतदाताओं को ॉ ी के तहत मत दे ने का
िवक चु नने की सु िवधा ा है ।
3. रा सभा चु नाव लड़ने के िलये यह आव क नहीं है िक उ ीदवार उस
े का मतदाता हो।

उपयु म से कौन-से कथन सही ह?

A) केवल 1 और 2

B) केवल 1 और 3

C) केवल 2 और 3

D) 1, 2 और 3
Hide Answer -
उ रः (d)

ा ाः उपयु सभी कथन सही ह।

जन ितिनिध (सं शोधन) अिधिनयम, 1999 के ारा कुछ खास तरह के मतदाताओं के

िलये डाक मत प के ज रये मत दे ने की व था की गई। चु नाव आयोग सरकार से

परामश कर िकसी भी े णी के के िलये ऐसी सु िवधा के िलये अिधसू िचत कर सकता

है ।

चु नाव कानू न (सं शोधन) अिधिनयम, 2003 और चु नाव सं चालन (सं शोधन) कानू न,

2003 के तहत ावधान िकया गया िक सश से ना म कायरत मतदाता और ऐसे सश

बल म कायरत लोगों को, जहाँ से ना अिधिनयम लागू होता है , ॉ ी के तहत मत दे ने का

िवक चु नने की सु िवधा ा है ।

जन ितिनिध (सं शोधन) अिधिनयम, 2003 के तहत रा सभा चु नाव लड़ने वाले

उ ीदवार की आवासीय यो ता सं बंधी ितबं ध को हटा िदया गया है । अब उ ीदवार

भारत के िकसी भी े से चु नाव लड़ सकता है । अब दे श के िकसी भी सं सदीय े का

मतदाता होना पया है । इसके चु नाव म गु मतदान के थान पर खु ला मतदान ारं भ

िकया गया।

राजनीितक दल को 20,000 से अिधक के े क चं दे की जानकारी चु नाव आयोग को

दे नी होगी।

चु नाव आयोग के ारा ि हीन मतदाताओं के िलये EVM (Electronic Voting

Machine) म े ल फीचर को डाला गया है । चु नाव आयोग के ारा वष 2004 म आं

दे श िवधानसभा उप-चु नाव म इसके िलये यास िकया गया। आयोग के ारा 15वीं

लोकसभा (2009) चु नाव तथा कुछ रा ों के िवधानसभा चु नावों म इले ॉिनक वोिटं ग

मशीनों (ईवीएम) म े ल फीचर को डाला गया।

[6] ‘िनगम मत सव ण’ (ए ट पोल) के िवषय म िन िल खत म से कौन-सा कथन


सही है ?

ए ट पोल यु का योग मतदाताओं के उस िनवाचने र सव ण को


A) करता है , िजससे यह पता चले िक मतदाताओं ने अपने मतािधकार का
योग िकस ाशी के प म िकया है ।

ए ट पोल वह यु है , िजससे मतदान के प रणामों के िवषय म अिधकतम


B)
सही पू वानु मान िकया जा सकता है ।

ए ट पोल हाल ही म मु िनवाचन आयु ारा िनकाली गई शासिनक


C)
यु है , िजससे पर प धारण करके मतदान रोका जा सकता है ।

D) ए ट पोल और जमनत सव ण एक ही बात है ।


Hide Answer -
उ रः (a)

ा ाः उपयु म से िवक (a) सही है ।

िनगम मत सव ण (ए ट पोल) वह प ित है , िजससे मतदान के तु रंत बाद यह पता

लगाया जाता है िक मतदाताओं ने िकस ाशी या दल के प म मतदान िकया है ।

जनमत सव ण (Opinion Poll) मतदान के पहले चु नाव के सं बंध म जनता की राय

जानने के िलये िविभ एजिसयों ारा सव िकया जाता है ।

जन ितिनिध (सं शोधन) अिधिनयम, 2009 (1 फरवरी, 2010 से भावी) के तहत

लोकसभा और रा िवधानसभाओं के चु नाव के दौरान ए ट पोल करने और उसके

प रणामों को कािशत करने पर रोक लगाई गई है । चु नाव आयोग के ारा अिधसू िचत

अविध के दौरान कोई ए ट पोल नहीं करा सकता तथा ि ं ट या इले ॉिनक

मीिडया ए ट पोल के प रणामों को कािशत एवं चा रत नहीं कर सकता। इस ावधान

का उ ं घन करने वाले को 2 वष का कैद या जु माना या दोनों हो सकता है ।

[7] लोकसभा चु नाव के मामले म सामा वग के उ ीदवारों तथा अनु सूिचत जाित/
जनजाित के उ ीदवारों के ारा जमा की जाने वाली जमानत रािश मशः है -

A) 5000 एवं 2500

B) 10000 एवं 5000

C) 15000 एवं 7500

D) 25000 एवं 12500

Hide Answer -
उ रः (d)

ा ाः उपयु म से िवक (d) सही है ।

जन ितिनिध (सं शोधन) अिधिनयम, 2009 म लोकसभा चु नाव एवं रा िवधानसभा चु नाव

लड़ने वाले उ ीदवार की जमानत रािश म वृ की गई, जो इस कार है :

1. लोकसभा चु नाव म सामा वग के िलये 10,000 से बढ़ाकर 25,000 पए कर दी

गई, जबिक अनु सूिचत जाित/जनजाित के िलये 5,000 पए से बढ़ाकर 12,500 कर

दी गई।

2. िवधानसभा चु नाव म सामा वग के िलये जमानत रािश 5,000 पए से बढ़ाकर

10,000 पए कर दी गई, जबिक अनु सूिचत जाित/जनजाित के िलये ये रािश 2500

पए से बढ़ाकर 5000 पए कर दी गई ।

[8] सव ायालय के िनदशों के अनु सार चु नाव आयोग के ारा मतदान प ों/
ई.वी.एम. मशीनों म ‘उपयु म से कोई नहीं’ (नोटा) का ावधान िकया गया है ।
इस िवषय म िन िल खत कथनों पर िवचार कीिजये :

1. कुछ रा ों के िवधानसभा चु नावों म नोटा का योग िकया गया है ।


2. नोटा (NOTA) के तहत पड़े मत को वै ध मत म शािमल नहीं िकया जाता।

उपयु म से कौन-सा/से कथन सही है /ह?

A) केवल 1

B) केवल 2

C) 1 और 2 दोनों

D) न तो 1 और न ही 2

Hide Answer -
उ रः (c)

ा ाः

भारत के उ तम ायालय के ारा वष 2013 म मतप /ई.वी.एम. म उपयु म से

कोई नहीं (NOTA : None of the Above) का ावधान करने का आदे श िदया गया।

इसी आधार पर भारत के चु नाव आयोग के ारा इसका उपबं ध िकया गया।

वष 2013 म छ ीसगढ़, म दे श, िमज़ोरम, राज थान एवं रा ीय राजधानी े िद ी

के िवधानसभा चु नाव म इसका योग िकया गया। वष 2014 म 16वीं लोकसभा चु नावों के

साथ आं दे श, अ णाचल दे श, उड़ीसा और िस म रा के िवधानसभा चु नाव म

नोटा का योग िकया गया।

नोटा (NOTA) के तहत पड़े मत को वै ध मत म शािमल नहीं िकया जाता। इसिलये यिद

नोटा के प म पड़े मतों की सं ा िकसी उ ीदवार को िमले मतों की सं ा से अिधक है

तब भी िजस उ ीदवार को सबसे अिधक मत िमले ह, उसे िवजे ता घोिषत िकया जाएगा।

[9] मतदाता िनरी ण पे पर ऑिडट टायल (वी.वी.पी.ए.टी.) के िवषय म िन िल खत

म से कौन-सा कथन सही नहीं है ?

वीवीपीएटी के ारा मतदाता स ािपत कर सकता है िक उसका मत उस


A)
अ थ के प म पड़ा है या नहीं िजसके प म उसने अपना मत डाला है ।

ईवीएम के साथ वीवीपीएटी का योग मतदान णाली म सटीकता एवं


B)
पारदिशता सु िनि त करता है ।

C) वीवीपीएटी का योग रा के िवधानसभाओं के चु नावों म िकया जा रहा है ।

D) अब तक लोकसभा के चु नाव म इसका योग नहीं िकया गया है ।


Hide Answer -
उ रः (d)

ा ाः

मतदाता िनरी ण पे पर ऑिडट टायल (VVPAT-Voter Verification Paper Audit

Trial) ईवीएम से जु ड़ी एक तं णाली है िजसके तहत मतदाता स ािपत कर सकता

है िक उसके ारा डाला गया मत उस उ ीदवार को िमला है या नहीं, िजसके प म

उसने अपना मत डाला है । यह णाली मतदाता को पे पर रसीद के आधार पर अपने मत

को चु नौती दे ने की सु िवधा दान करती है ।

वीवीपीएटी का योग सव थम वष 2013 म नागालड (नोकासे न) िवधानसभा चु नाव े

म िकया गया था। इसके बाद रा के िवधानसभाओं के चु नावों म इसका योग िकया जा

रहा है । वष 2014 के लोकसभा चु नाव म चु ने ए लोकसभा चु नाव े ों म वीवीपीएटी का

योग िकया गया।

ईवीएम के साथ वीवीपीएटी मतदान णाली म सटीकता एवं पारदिशता सु िनि त करता है ।

[10]लोकसभा/रा िवधानसभाओं के चु नावों की खच की सीमा के िवषय म


िन िल खत म से कौन-सा कथन सही है ?

सभी सं घ-शािसत दे शों के िलये लोकसभा चु नावों की खच सीमा 54 लाख


A)
पए िनि त की गई है ।

सभी सं घ-शािसत दे शों के िलये िवधानसभाओं के चु नावों की खच सीमा 20


B)
लाख पए िनि त की गई है ।

सभी रा ों के िलये लोकसभा के चु नावों की खच सीमा 70 लाख पए िनि त


C)
की गई है ।

अ णाचल दे श, िस म और गोवा रा ों के िलये लोकसभा के चु नावों म


D)
खच सीमा 54 लाख पए िनि त की गई है ।
Hide Answer -
उ रः (d)

ा ाः उपयु म िवक (d) सही है ।

िनवाचनों का सं चालन िनयम, 1969 (Conduct of Election Rule, 1961) म वष

2014 म सं शोधन िकया गया। इसके ारा लोकसभा और रा िवधानसभाओं के चु नावों

की खच सीमा म वृ की गई। इसके तहत बड़े रा , छोटे रा एवं क -शािसत दे शों

के आधार पर खच की सीमाएँ िनि त की गई। सभी बड़े रा ों एवं क शािसत दे श

िद ी के िलये लोकसभा चु नाव की खच सीमा 70 लाख िनि त की गई (इसके पू व खच

सीमा 40 लाख पए थी), जबिक सभी छोटे रा एवं अ क -शािसत दे शों के िलये

खच की सीमा 54 लाख पए िनि त की गई (इसके पहले यह सीमा 16-40 लाख पए

थी)। छोटे रा ों म शािमल ह-िस म, गोवा एवं अ णाचल दे श।

रा िवधानसभाओं के चु नावों के िलये सभी बड़े रा ों एवं क -शािसत दे श िद ी के

िलये खच की सीमा 28 लाख पए िनि त की गई है (इसके पहले खच की सीमा 16

लाख पए थी), जबिक छोटे रा ों एवं अ क शािसत दे शों के िलये यह सीमा 20

लाख पए िनि त की गई है (इसके पहले खच की सीमा 8-16 लाख पए थी)। छोटे

रा ों म शािमल ह-

⇒ गोवा

⇒ मिणपु र

⇒ मे घालय

⇒ िमज़ोरम

⇒ नागालड

⇒ िस म

⇒ ि पु रा

[11]भारतीय चु नाव के िवषय म िन िल खत कथनों पर िवचार कीिजये -

1. िवदे शों म रहने वाले भारतीय लोकसभा/रा िवधानसभा के चु नाव म


अपने मत का योग कर सकते ह।
2. चु नाव आयोग ने उपबं ध िकया है िक ईवीएम और मतप ों पर अ थ का
नाम, फोटो एवं पाट का चु नाव िच कािशत होगा।

उपयु म से कौन-सा/से कथन सही है /ह?

A) केवल 1

B) केवल 2

C) 1 और 2 दोनों

D) न तो 1 और न ही 2
Hide Answer -
उ रः (c)

ा ाः उपयु दोनों कथन सही ह।

जन ितिनिध (सं शोधन) अिधिनयम, 2010 के ारा ावधान िकया गया िक भारत का

े क नाग रक, जो िविभ कारणों से िवदे श म रह रहा है , उसे मतदान करने का

अिधकार होगा। वे अपना नाम उस मतदान सू ची म दज़ कर सकते ह, जो उसके पासपोट

म अं िकत है ।

चु नाव आयोग के आदे शानु सार 1 मई, 2015 के बाद होने वाले िकसी भी चु नाव म ईवीएम

एवं मतप ों पर उ ीदवारों का फोटो, नाम एवं पाट का चु नाव िच कािशत रहे गा। जू न

2015 म 5 रा ों म थम बार मतप ों पर उ ीदवार के फोटो का उपयोग िकया गया।

[12]सहकारी सिमितयों के सं दभ म िन िल खत कथनों पर िवचार कीिजयेः

1. सहकारी सिमितयाँ सं िवधान की सातवीं अनु सूची के समवत सू ची का िवषय


है ।
2. 97व सं िवधान सं शोधन अिधिनयम, 2011 के ारा सहकारी सिमितयों को
सं वैधािनक दज़ा दान िकया गया।
3. सं सद, सहकारी सिमितयों की थापना, िनयमन एवं ब करने सं बंधी
िनयम बना सकेगी।

उपयु म से कौन-सा/से कथन सही है /ह?

A) केवल 2

B) केवल 1 और 2

C) केवल 2 और 3

D) 1, 2 और 3

Hide Answer -
उ रः (a)

ा ाः उपयु म से कथन (1) और (3) गलत ह।

सहकारी सिमितयाँ रा सू ची का िवषय है । इसे सातवीं अनु सूची की रा सू ची की िवि

सं ा 32 म रखा गया है ।

97व सं िवधान सं शोधन अिधिनयम, 2011 के ारा सहकारी सिमितयों को सं वैधािनक दज़ा

दान िकया गया, जो 15 फरवरी, 2012 को लागू आ। क सरकार ने रा सरकारों

से 14 फरवरी, 2013 के पहले इस सं शोधन के अनु प अपने रा ों के सहका रता

सिमित अिधिनयम म सं शोधन करने को कहा है ।

रा का िवधानमं डल (न िक सं सद) सहकारी सिमितयों की थापना, िनयमन एवं ब

करने सं बंधी िनयम बनाएगी।

[13]
िन िल खत म से सं िवधान के िकस/िकन भाग/भागों म सहकारी सिमितयों के
िवषय म उपबं ध िकया/िकये गए है /ह?

1. सं िवधान के भाग-III
2. सं िवधान के भाग-IV
3. सं िवधान के भाग-IX (ख)
4. सं िवधान के भाग-XIV (क)

नीचे िदये गए कूटों का योग कर सही उ र का चयन कीिजयेः

A) केवल 3

B) केवल 2 और 3

C) केवल 1, 2 और 3

D) 1, 2, 3 और 4

Hide Answer -
उ रः (c)

ा ाः उपयु म से कथन (4) गलत है ।

सं िवधान के भाग XIV(क) म अिधकरण का उपबं ध।

सं िवधान के भाग-3 के अनु े द 19(1)(ग) म सहकारी सिमित बनाने के सं बंध म उपबं ध

िकया गया है । यह उपबं ध 97व सं िवधान सं शोधन अिधिनयम, 2011 के ारा िकया गया।

सं िवधान के भाग-4 के अनु े द 43(ख) म सहकारी सिमितयों के उ यन के सं बंध म

ावधान िकया गया। इसे 97व सं िवधान सं शोधन अिधिनयम ारा जोड़ा गया।

97व सं िवधान सं शोधन अिधिनयम, 2011 के ारा सं िवधान म एक नया भाग 9(ख) जोड़ा

गया और अनु े द-243 ZH से ले कर अनु े द-243ZT तक सहकारी सिमितयों के सं बंध

म उपबं ध िकया गया है ।

[14]सहकारी सिमितयों के बोड के िवषय म िन िल खत म से कौन-सा कथन सही

नहीं है ?

िकसी भी सहकारी सिमित के िनदे शकों की अिधकतम सं ा 21 से अिधक


A)
नहीं होगी।

रा िवधानमं डल अनु सूिचत जाित एवं जनजाित और मिहलाओं के िलये एक-


B)
एक थान का आर ण कर सकेगा।

बोड के िनवािचत सद ों और उसके पदािधका रयों की पदाविध िनवाचन की


C)
ितिथ से पाँ च वष की होगी।

िकसी भी बोड को छः माह से अिधक समय के िलये न तो अिध िमत िकया


D)
जाएगा और न िनलं िबत रखा जाएगा।
Hide Answer -
उ रः (b)

ा ाः सं िवधान म ावधान है िक बोड/िनदे शक मं डल/शासी िनकाय का गठन िनदे शकों/

सं चालकों की उतनी सं ा से होगा जो रा का िवधानमं डल िविध ारा उपबं ध करे गा, िकंतु

िकसी भी सहकारी सिमित के िनदे शकों की अिधकतम सं ा 21 से अिधक नहीं होगी।

रा का िवधानमं डल िविध बनाकर अनु सूिचत जाित या जनजाित के िलये एक सीट तथा

मिहलाओं के िलये दो थान/सीट े क सहकारी सिमित के बोड के िलये आरि त कर

सकेगी।

बोड के िनवािचत सद ों एवं उनके पदािधका रयों की पदाविध िनवाचन की ितिथ से 5 वष

की होगी एवं पदािधकारी बोड के कायकाल तक बने रहग।

सं िवधान म उपबं ध है िक िकसी भी बोड को छः माह से अिधक समय के िलये न तो

अिध िमत (Superseded) िकया जाएगा और न िनलं िबत रखा जाएगा।

नोटः

बोडः इसे सहकारी सिमित के कामकाज के बं धन के िनदशन एवं िनयं ण का

उ रदािय सौंपा गया है ।

पदािधकारी का अथः सहकारी सिमित के अ , उपा , सभापित, उप सभापित,

सिचव या कोषा से है तथा इसम बोड ारा िनवािचत कोई भी दू सरा शािमल है ।

बोड का चु नाव कायकाल पू रा होने से पहले कराया जाएगा।

[15]भारत के मू ल सं िवधान म िह ी भाषा म ािधकृत पाठ के िवषय म कोई उपबं ध


नहीं िकया गया था, िकंतु सं िवधान के अनु े द-394(क) म इसका उपबं ध िकया
गया है । िकस सं िवधान सं शोधन अिधिनयम के ारा िह ी भाषा म ािधकृत पाठ
को जोड़ा गया?

A) 42व सं िवधान सं शोधन अिधिनयम, 1976 के ारा

B) 44व सं िवधान सं शोधन अिधिनयम, 1978 के ारा

C) 58व सं िवधान सं शोधन अिधिनयम, 1987 के ारा

D) 98व सं िवधान सं शोधन अिधिनयम, 2013 के ारा


Hide Answer -
उ रः (c)

ा ाः उपयु म से िवक (c) सही है ।

सं िवधान के भाग-XXII म सं ि नाम, ारं भ, िह ी भाषा म ािधकृत पाठ एवं िनरसन

के सं बंध म उपबं ध िकया गया है ।

िह ी भाषा म ािधकृत पाठ को 58व सं िवधान सं शोधन अिधिनयम, 1987 के ारा

उपबं ध िकया गया। इस सं िवधान सं शोधन के ारा अनु े द-349(क) को जोड़ा गया और

इसम िह ी भाषा म ‘ ािधकृत पाठ’ को शािमल िकया गया।

भारत के सं िवधान को सं िवधान सभा ारा 26 नव र 1949 को अं गीकृत िकया गया तब

यह सं िवधान अं े ज़ी म था। वष 1950 म सं िवधानसभा के अ के ािधकार से इस

सं िवधान का िह ी भाषा म अनु वाद कराया गया, िजस पर सं िवधान सभा के सद ों ने

ह ा र िकये । िकंतु , इसके बाद सं िवधान म होने वाले सभी सं शोधनों को, जो अं े ज़ी भाषा

म होंगे, िह ी म अनु वाद कराने एवं उसे सं िवधान म स िलत करने के उ े से 58वां

सं िवधान सं शोधन िकया गया।

इस सं िवधान सं शोधन के ारा उपबं ध िकया गया िक इस िह ी पाठ का वही अथ लगाया

जाएगा, जो अं े ज़ी के मू ल पाठ म िविहत है । यिद अथ लगाने म कोई असु िवधा उ

होती है तो रा पित इसका उपयु परी ण कराएं गे ।

सं िवधान के अनु े द- 5, 6, 7, 8, 9, 60, 324, 366, 367, 379, 380, 388,

391, 392 एवं 393 26 नवं बर 1949 को वृ त ए एवं शे ष उपबं ध 26 जनवरी,

1950 को वृ त ए।

अनु े द-393 म उपबं ध है िक सं िवधान का सं ि नाम ‘भारत का सं िवधान’ है ।

[16]अ खल भारतीय से वाओं का जनक माना जाता है ।

A) पं िडत जवाहरलाल ने ह

B) िव लभाई पटे ल

C) राम मनोहर लोिहया

D) सरदार व भ भाई पटे ल


Hide Answer -
उ रः (d)

ा ाः

सं िवधानसभा म अ खल भारतीय से वाओं के मु ख समथक सरदार व भ भाई पटे ल थे ।

अतः उ अ खल भारतीय से वाओं का जनक माना जाता है ।

सं िवधान के अनु े द-312 म अ खल भारतीय से वाओं के सं बंध म उपबं ध िकया गया है ।

वतमान म भारत म तीन अ खल भारतीय से वाएं िव मान ह-

1. भारतीय शासिनक से वा (आई.ए.एस)

2. भारतीय पु िलस से वा (आई.पी.एस)

3. भारतीय वन से वा (आई.एफ.एस)

सं सद ारा अ खल भारतीय से वा अिधिनयम, 1951 पा रत िकया गया। यह अिधिनयम

क सरकार को रा सरकारों से परामश करके अ खल भारतीय से वाओं के सद ों की

भत एवं से वा शत के िलये ािधकृत करता है । इसके सद ों की भत एवं िश ण क

सरकार के ारा िकया जाता है , िकंतु उसे काय के िलये िविभ रा ों म भे ज िदया जाता

है । इनका वे तन एवं पशन रा सरकार ारा िदया जाता है । अ खल भारतीय से वाएं सं यु

प से क एवं रा सरकारों ारा िनयं ि त होती ह।

[17]िन िल खत कथनों पर िवचार कीिजयेः

1. सं िवधान के भाग-14 का उपबं ध ज ू -क ीर सिहत भारत के सभी


रा ों पर लागू होता है ।
2. सं िवधान म उपबं ध िकया गया है िक र ा से वा या सं घ की िसिवल से वा या
अ खल भारतीय से वा का सद रा पित के सादपय पद धारण करे गा।

उपयु म से कौन-सा/से कथन सही है /ह?

A) केवल 1

B) केवल 2

C) 1 और 2 दोनों

D) न तो 1 और न ही 2
Hide Answer -
उ रः (b)

ा ाः उपयु म से कथन (1) गलत है ।

सं िवधान के भाग-14 म (अनु े द-308 से ले कर अनु े द-314 तक) अ खल भारतीय

से वाएं , क ीय से वाएं एवं रा से वाओं के सं बंध म उपबं ध िकया गया है ।

सं िवधान के अनु े द-308 म उपबं ध है िक सं िवधान के भाग-14 से सं बंिधत उपबं ध

ज ू -क ीर रा को छोड़कर भारत के सभी रा ों पर लागू होगा।

सं िवधान के अनु े द-310 म ावधान है िक र ा से वा, सं घ की िसिवल से वा या अ खल

भारतीय से वा का सद अथवा र ा से सं बंिधत कोई पद या सं घ के अधीन कोई िसिवल

पद धारण करता है तो वह रा पित के सादयत पद धारण करे गा तथा िकसी रा की

िसिवल से वा सद या रा के अधीन कोई िसिवल पद धारण करता है तो वह उस रा

के रा पाल के पसादपयत पद धारण करे गा।

सं िवधान के अनु े द-311 म सं घ या रा के अधीन िसिवल है िसयत (यह ावधान सै

से वाओं एवं सै पद पर आसीन यों पर लागू नहीं होता) म िनयोिजत यों को

पद ु त िकया जाना, पद से हटाया जाना या रक म कमी िकये जाने से सं बंिधत उपबं ध

िकया गया है ।

[18]भारतीय सं िवधान की िकस अनु सूची म असम, मे घालय, ि पु रा और िमज़ोरम जै से

रा ों के जनजातीय े ों के शासन के िवषय म उपबं ध िकया गया है ?

A) चौथी अनु सूची

B) पाँ चवी अनु सूची

C) छठी अनु सूची

D) सातवीं अनु सूची


Hide Answer -
उ रः (c)
ा ाः सं िवधान के भाग-10 म अनु सूिचत एवं जनजातीय े ों के शासन के सं बंध म
उपबं ध िकया गया है ।

सं िवधान की छठी अनु सूची म असम, मे घालय, ि पु रा एवं िमज़ोरम जै से रा ों के


जनजातीय े ों के शासन के सं बंध म उपबं ध िकया गया है , जबिक पाँ चवी
अनु सूची म असम, मे घालय, ि पु रा और िमज़ोरम से िभ िकसी रा के अनु सूिचत
े ों एवं जनजाितयों के शासन एवं िनयं ण के िवषय म उपबं ध िकया गया ।
सं िवधान के 5वीं अनु सूची के तहत वतमान (वष 2016 तक) म 10 अनु सूिचत े
ह:
1. आं दे श
2. ते लंगाना
3. झारख
4. छ ीसगढ़
5. गु जरात
6. िहमाचल दे श
7. म दे श
8. महारा
9. उड़ीसा
10. राज थान
सं िवधान की 6ठी अनु सूची के तहत वतमान (वष 2016 तक) म कुल 10 जनजाित
े ह, जो िन िल खत ह:
(1) असमः
(a) उ री कछार पहाड़ी िज़ला
(b) काब आं गलां ग
(c) बोडोलै दे श े िज़ला
(2) मे घालयः
(a) खासी पहाड़ी िज़ला
(b) जयं ितया पहाड़ी
(c) गारो पहाड़ी िज़ला
(3) िमज़ोरमः
(a) चकमा िज़ला
(b) मारा िज़ला
(c) लाई िज़ला
(4) ि पु राः ि पु रा जनजातीय े िज़ला

[19]जनजातीय सलाहकार प रषद के िवषय म िन िल खत म कौन-सा कथन सही नहीं

है ?
A) सं िवधान की छठी अनु सूची म इसका उपबं ध िदया गया है ।

इस प रषद के तीन-चौथाई सद उस रा की िवधानसभा म अनु सूिचत


B)
जनजाितयों के ितिनिध होंगे।

यह प रषद अनु सूिचत जनजाितयों के क ाण एवं उ ान के िलये सलाह दे ती


C)
है ।

D) यह प रषद 20 से अनिधक सद ों से िमलकर बने गी।

Hide Answer -
उ रः (a)

ा ाः

सं िवधान की पाँ चवी अनु सूची (न की छठी अनु सूची) म एक जनजातीय सलाहकार प रषद

का उपबं ध िकया गया है । यह प रषद 20 से अनिधक सद ों से िमलकर बने गी अथात

इस प रषद म 20 से अिधक सद नही होंगे । इसम से 3/4 सद उस रा की

िवधानसभा के अनु सूिचत जनजाितयों के ितिनिध होंगे।

यह प रषद अनु सूिचत जनजाितयों के क ाण एवं उ ान से सं बंिधत ऐसे िवषयों पर सलाह

दे गी, जो रा पाल ारा िनिद िकया जाए। इसके िवषय म रा पाल िनयम बना सकता है

इस कार की प रषद् वै से रा ों म गिठत की जा सकती है , जहाँ अनु सूिचत जनजाितयाँ

तो है , िक ु अनु सूिचत े नहीं ह। ऐसा रा पित के िनदश पर िकया जाएगा।

[20]सं िवधान की पाँ चवी अनु सूची के िवषय म िन िल खत कथनों पर िवचार कीिजएः

1. भारत के िकसी भी े को अनु सूिचत े घोिषत करने की श रा पित


म िनिहत है ।
2. रा पित को यह श है िक वह सं सद या रा के िवधानमं डल के िकसी
िवशे ष अिधिनयम को अनु सूिचत े ों म लागू न करे या कुछ प रवतन या
अपवाद प लागू करे ।
3. सं िवधान म उपबं ध है िक रा पित अनु सूिचत जनजाितयों के क ाण एवं
अनु सूिचत े ों के बं धन हे तु एक आयोग का गठन करे गा।

उपयु म से कौन-सा/से कथन सही है /ह।

A) केवल 1

B) केवल 1 और 2

C) केवल 1 और 3

D) 1, 2 और 3
Hide Answer -
उ रः (c)

ा ाः उपयु म से कथन 2 गलत है ।

सं बंिधत रा के रा पाल को (न िक रा पित को) यह अिधकार है िक वह सं सद या

रा िवधानमं डल के िकसी िवशे ष अिधिनयम को अनु सूिचत े ों म लागू न करे या कुछ

सं शोधन या अपवाद के प म लागू करे । रा पाल अनु सूिचत े ों म शां ित एवं अ ी

सरकार के िलये जनजातीय सलाहकार प रषद से िवचार-िवमश कर िनयम बना सकता है ।

इसके अित र रा ापाल, रा पित की ीकृित से ऐसे े म लागू सं सद अथवा रा

िवधानमं डल के कानू न को समा या सं शोधन कर सकता है । रा पित िकसी भी े को

अनु सूिचत े घे िषत कर सकता है । रा पित, रा पाल से परामश कर िकसी अनु सूिचत

े के े फल को बढ़ा या घटा सकता है व सीमाओं या नामों को बदल सकता है ।

सं िवधान म उपबं ध है िक रा पित, अनु सूिचत जनजाितयों के क ाण एवं अनु सूिचत े ों

के बं धन हे तु एक आयोग की िनयु कर सकेगा। वष 1960 म यू .एन. ढे बर की

अ ता म रा पित ारा पहले आयोग का गठन िकया गया, िजसने वष 1961 मे अपनी

रपोट सौंपी। वष 2002 म िदलीप िसं ह भू रया की अ ता म दू सरे आयोग का गठन

िकया गया, िजसने वष 2004 म अपनी रपोट सौंपी।


[1] भारत का सं िवधान असम, मे घालय, ि पु रा एवं िमज़ोरम के ेक शासी िज़ले
के िलये एक िज़ला प रषद् का उपबं ध करता है । इस प रषद् के िवषय म
िन िल खत म कौन-सा कथन सही है ?

A) इसका उपबं ध सं िवधान की पाँ चवी अनु सूची के अं तगत िकया गया है ।

B) यह प रषद् तीस से अनिधक सद ों से िमलकर बने गी।

प रषद् के सभी सद वय मतािधकार के आधार पर िनवािचत िकये


C)
जाएं गे ।

D) प रषद् के सभी सद ों का कायकाल पाँ च वष की अविध का होगा।

Hide Answer -
उ रः (b)

ा ाः उपयु म से िवक (b) सही है ।

सं िवधान की छठी अनु सूची (न िक 5वीं अनु सूची) के अं तगत असम, मे घालय, ि पु रा एवं

िमज़ोरम के जनजातीय े ों के शासन के सं बंध म उपबं ध है । वतमान म इन रा ों म

10 जनजातीय े ह। इन जनजातीय े ों म शासी िज़लों का गठन िकया गया है ।

ेक शासी िज़ले के िलये एक िज़ला प रषद् का उपबं ध िकया गया है ।

िज़ला प रषद् तीस से अनिधक सद ों से िमलकर बने गी, िजसके 4 से अनिधक सद

रा पाल ारा नाम िनदिशत िकये जाएं गे एवं शे ष सद वय मतािधकार के आधार पर

िनवािचत िकये जाएं गे ।

िनवािचत सद ों का कायकाल पाँ च वष का होगा, जबिक रा पाल ारा नाम िनदिशत

सद रा पाल के सादपयत पद धारण करगे ।

शासी दे श के प म गिठत ेक े के िलये एक पृ थक ादे प रषद् होगी ।

िज़ला एवं ादे िशक प रषद् रा पाल की ीकृित के प ात् अपने अधीन े ों के िलये

िविध बना सकती है ।

िज़ला या ादे िशक प रषद् अपने अधीन े ों म जनजाितयों के आपसी मामलों के िनपटारे

के िलये ाम प रषद् या ायालय का गठन कर सकती ह। इस मामले म उ ायालय

के े ािधकार का िनधारण रा पाल ारा िकया जाएगा।

[2] सं िवधान की छठी अनु सूची के िवषय म िन िल खत कथनों पर िवचार कीिजएः

1. शासी िज़लों की थापना या पु न थापना करने , इसके े ों को घटाने या


बढाने तथा नाम प रवतन करने या सीमाएँ िनधारण करने की श
रा पाल को है ।
2. रा पाल, शासी िज़लों एवं शासी दे शों के शासन की जाँ च करने
और रपोट दे ने के िलये िकसी भी समय आयोग की िनयु कर सकेगा।
3. रा पाल उपयु आयोग की िसफा रश पर िज़ला प रषद् या ादे िशक
प रषद् का िवघटन कर सकता है ।

उपयु म से कौन-से कथन सही ह?


A) केवल 1 और 2

B) केवल 1 और 3

C) केवल 2 और 3

D) 1, 2 और 3

Hide Answer -
उ रः (d)

ा ाः उपयु तीनों कथन सही ह।

सं िवधान की छठी अनु सूची म उपबं ध है िक रा पाल असम, मे घालय, ि पु रा एवं िमज़ोरम

के जनजातीय े ों म शासी िज़लों की थापना या पु न थापना, इनके े ों को घटा या

बढा सकता है तथा इनके नाम म प रवतन या सीमाओं का िनधारण कर सकता है ।

रा पाल दे शों के शासन की जाँ च करने एवं ितवे दन दे ने के िलये िकसी भी समय

आयोग की िनयु कर सकेगा । रा पाल इस आयोग की िसफा रश पर िज़ला या

ादे िशक प रषदों को िवघिटत कर सकता है ।

सं सद या रा िवधानमं डल का अिधिनयम शासी िज़ला या शासी दे शों म लागू नही

होता या कुछ सं शोधन या अपवाद प लागू होता है । असम के मामले म इस कार के

िनदश दे ने की श (सं सदीय या रा िवधानमं डल दोनों की मामलों म) रा पाल को है ,

िक ु मे घालय, ि पु रा एवं िमज़ोरम के मामले म सं सदीय अिधिनयम के सं बंध म िनदश दे ने

की श रा पित को एवं रा िवधानमं डल के सं बंध म िनदश दे ने की श रा पाल

को ा है ।

[3] के ीय शासिनक अिधकरण (कैट) के िवषय म िन िल खत म से कौन-सा


कथन सही नहीं है ?

के ीय शासिनक अिधकरण का उपबं ध मू ल सं िवधान म नहीं िकया गया है


A)

के ीय शासिनक अिधकरण का उपबं ध सं िवधान के अनु े द-323(क)


B)
और अनु े द-323(ख) म िकया गया है ।

शासिनक अिधकरण अिधिनयम, 1985 के सरकार को एक के ीय


C)
शासिनक अिधकरण गिठत करने का अिधकार दे ता है ।

42व सं िवधान सं शोधन अिधिनयम, 1976 के ारा सं िवधान म भाग-14(क)


D) जोड़ा गया और यह भाग सं घ के िलये एक शासिनक अिधकरण का उपबं ध
करता है ।
Hide Answer -
उ रः (b)

ा ाः

42व सं िवधान सं शोधन अिधिनयम, 1976 के ारा सं िवधान म भाग-14(क) एवं

अनु े द-323(क) एवं 323(ख) जोड़ा गया, परं तु के ीय शासिनक अिधकरण (कैट)

का गठन सं िवधान के अनु े द-323(क) के तहत िकया गया है । अनु े द-323(ख) म

अ मामलों से सं बंिधत अिधकरणों (Tribunals) का उपबं ध िकया गया है । अतः के ीय

शासिनक अिधकरण का उपबं ध मू ल सं िवधान म नहीं िकया गया है ।

सं िवधान के अनु े द-323(क) सं सद को यह अिधकार दे ता है िक वह के व रा ों के

िलये शासिनक अिधकरण की थापना करे । इसी आधार पर सं सद ारा शासिनक

अिधकरण अिधिनयम, 1985 पा रत िकया गया। यह अिधिनयम के सरकार को के ीय

एवं रा शासिनक अिधकरण गिठत करने का अिधकार दे ता है । इस अिधिनयम म दो

या दो से अिधक रा ों के िलये सं यु शासिनक अिधकरण की थापना का भी उपबं ध

िकया गया है ।

[4] के ीय शासिनक अिधकरण (कैट) के अिधकार े के िवषय म िन िल खत


पर िवचार कीिजयेः

1. अ खल भारतीय से वाओं के कमचारी।


2. सै से वाओं के सद एवं अिधकारी।
3. उ - ायालय के कमचारी।
4. सं सद के सिचवालय के कमचारी।

उपयु म से कौन इसके अिधकार े म आते ह?

A) केवल 1

B) केवल 1 और 3

C) केवल 1, 3 और 4

D) केवल 1, 2 और 4
Hide Answer -
उ रः (a)

ा ाः उपयु म से 2, 3 और 4 गलत ह।

कैट के अिधकार े म िन िल खत आते ह:-

1. अ खल भारतीय से वाओं के कमचारी

2. के ीय लोक से वाओं के कमचारी

3. के के अधीन नाग रक पदों एवं सै से वाओं के िसिवल कमचारी

कैट के अिधकार े म िन िल खत नहीं आते ह:-

1. सै से वाओं के सद एवं अिधकारी

2. उ तम ायालय के कमचारी

3. सं सद सिचवालय के कमचारी

[5] िन िल खत कथनों पर िवचार कीिजयेः

1. के ीय शासिनक अिधकरण के अ एवं सद ों की िनयु रा पित


के ारा की जाती है ।
2. रा शासिनक अिधकरण के अ एवं सद ों की िनयु सं बंिधत
रा के रा पाल ारा रा पित की सलाह से की जाती है ।
3. सं यु रा अिधकरण के अ एवं सद ों की िनयु रा पित ारा
सं बंिधत रा के रा पालों की िसफा रश पर की जाती है ।

उपयु म से कौन-से कथन सहीं ह?

A) केवल 1 और 2

B) केवल 1 और 3

C) केवल 2 और 3

D) 1, 2 और 3
Hide Answer -
उ रः (b)

ा ाः उपयु म से कथन-2 गलत है ।

के ीय शासिनक अिधकरण (कैट) के अ और सद ों की िनयु रा पित के ारा

की जाती है ।

रा शासिनक अिधकरण (सै ट) के अ एवं सद ों की िनयु सं बंिधत रा के

रा पालों की सलाह पर रा पित ारा की जाती है ।

सं यु रा अिधकरण के अ एवं सद ों की िनयु रा पित के ारा सं बंिधत रा

के रा पाल की िसफा रश पर की जाती है ।

शासिनक अिधकरण अिधिनयम, 1985 म वष 2001 म सं शोधन िकया गया तथा कैट

के सद ों की है िसयत उ ायालय के ायाधीशों के समान कर दी गई है । ारं भ म

कैट म एक अ , एक उपा एवं सद होते थे । वतमान म कैट म अ के िलये

एक पद तथा सद ों के िलये 65 पद ीकृत ह।

[6] िन िल खत कथनों पर िवचार कीिजये ।

1. के ीय शासिनक अिधकरण (कैट) के अ एवं सद ों का कायकाल


6 वष या 65 वष की आयु तक, जो भी पहले हो, होता है ।
2. के ीय शासिनक अिधकरण (कैट) िसिवल ि या सं िहता कानू न की
ि याओं से बं धा आ है ।
3. के ीय शासिनक अिधकरण (कैट) के आदे शों की चु नौती भारत के
िकसी ायालय म नहीं की जा सकती।

उपयु म से कौन-सा/से कथन सही है /ह?

A) केवल 1

B) केवल 1 और 2

C) 1, 2 और 3

D) उपयु म से कोई नहीं।


Hide Answer -
उ रः (d)

ा ाः उपयु सभी कथन गलत ह।

कैट के अ का कायकाल 5 वष या 65 वष की आयु तक, जो भी पहले हो, होता है ,

जबिक इसके सद ों का कायकाल 5 वष या 62 वष की आयु तक, जो भी पहले हो,

होता है ।

कैट, 1908 की िसिवल ि या सं िहता कानू न की ि याओं से बं धा नहीं है । यह

अिधकरण ाकृितक ाय के िस ां तों ारा िनदिशत होता है ।

मू ल प से कैट के आदे शों की चु नौती केवल सव ायालय म की जा सकती है , िक ु

उ तम ायालय ने च कुमार मामले , 1992 म िनणय िदया िक कैट के आदे शों की

चु नौती उ ायालयों म दी जा सकती है । अब पीिड़त लोक से वक सीधे उ तम

ायालय म नहीं जा सकते ।

शासिनक अिधकरण अिधिनयम, 1985 पीिड़त लोक से वक को शी एवं कम खच पर

ाय िदलाता है । कैट म वादी यं अथवा अपने वकील के मा म से उप थत हो सकता

है । अ थ को इसम नाममा (केवल 50 पए) शु दे ना पड़ता है ।

You might also like